Modern Engineering mathematics-CRC Press (2018) PDF
Modern Engineering mathematics-CRC Press (2018) PDF
Modern Engineering mathematics-CRC Press (2018) PDF
CRC Press
Taylor & Francis Group
6000 Broken Sound Parkway NW, Suite 300
Boca Raton, FL 33487-2742
This book contains information obtained from authentic and highly regarded sources. Reasonable
efforts have been made to publish reliable data and information, but the author and publisher cannot
assume responsibility for the validity of all materials or the consequences of their use. The authors and
publishers have attempted to trace the copyright holders of all material reproduced in this publication
and apologize to copyright holders if permission to publish in this form has not been obtained. If any
copyright material has not been acknowledged please write and let us know so we may rectify in any
future reprint.
Except as permitted under U.S. Copyright Law, no part of this book may be reprinted, reproduced,
transmitted, or utilized in any form by any electronic, mechanical, or other means, now known or
hereafter invented, including photocopying, microfilming, and recording, or in any information
storage or retrieval system, without written permission from the publishers.
For permission to photocopy or use material electronically from this work, please access
www.copyright.com (http://www.copyright.com/) or contact the Copyright Clearance Center, Inc.
(CCC), 222 Rosewood Drive, Danvers, MA 01923, 978-750-8400. CCC is a not-for-profit organization
that provides licenses and registration for a variety of users. For organizations that have been granted
a photocopy license by the CCC, a separate system of payment has been arranged.
Trademark Notice: Product or corporate names may be trademarks or registered trademarks, and
are used only for identification and explanation without intent to infringe.
Visit the Taylor & Francis Web site at
http://www.taylorandfrancis.com
and the CRC Press Web site at
http://www.crcpress.com
Prof. A. H. Siddiqi would like to dedicate the book to
his wife, Prof. Azra H.Siddiqi
Prof. M. Al-Lawati dedicates to his spouse.
Prof. M. Messaoud dedicates to his family.
Contents
Preface xvii
Acknowledgments xxiii
vii
viii Contents
Bibliography 113
Bibliography 249
Bibliography 335
Bibliography 387
Bibliography 443
Bibliography 493
Bibliography 521
Bibliography 597
Bibliography 623
10 Wavelets 627
Bibliography 651
Bibliography 727
Bibliography 801
Index 823
Preface
xvii
xviii Preface
The idea of writing this book was proposed during the 2014 Spring Semester
while A. H. Siddiqi served as a consultant in the Department of Mathematics
and Statistics at Sultan Qaboos University, Muscat, Oman. The proposal
was finalized in a series of meetings in the office of Prof. M. Al-Lawati who
headed the department. Taylor & Francis Group then accepted the proposal
and agreed to publish the book. We appreciate the efforts of Aastha Sharma,
the publisher’s acquisition editor who arranged the contract and provided
support throughout the writing process.
We take this opportunity to thank the Government of Oman’s Sultan Qa-
boos University and Sharda University, Greater Noida, India, specially its
Chancellor, Mr. P. K. Gupta for providing immense support for this book
writing project. We also want to thank Prof. P. Manchanda who heads the
Department of Mathematics at Guru Nanak Dev University, Amritsar, India
for her valuable help in enabling us to complete this book.
A.H.Siddiqi
xxiii
Chapter 1
Matrices for Engineers
1
2 Modern Engineering Mathematics
~D
c
............
AD =AB+AC
−−→ → − →
−
If k = 0, 0AB = 0 , then 0 is called a zero vector that can be assigned any
−−→ −−→
direction. A vector AB in R2 is often written as AB = a = (a1 , a2 ), where
2
a1 and a2 are real numbers that is, a vector in R is represented by ordered
pairs of real numbers.
(ii) a − b = (a1 − b1 , a2 − b2 )
(iii) a + b = b + a for all vectors a and b
(iv) a + (b + c) =(a + b + c) for all vectors a, b and c
(v) a + 0 = a
(vi) a + (−a) = 0
(vii) k(a + b) = (ka + kb)
(viii) (k1 + k2 )a = k1 a + k2 a
a = a1 i + a2 j
R.H.S = a1 (1, 0) + a2 (0, 1)
= (a1 , 0) + (0, a2 )
= (a1 , a2 )
= a
= L.H.S.
Example 1. (a) Find the magnitude of vectors 2i + 4j, and (2, 0).
Vectors in R3
A vector in R3 is represented by an ordered triple of numbers a1 , a2 , a3 . As in
1
R2 , the magnitude is defined as kak = (|a1 |2 + |a2 |2 + |a3 |2 ) 2 .
Sum of two vectors a and b in R3 is defined as
a + b = (a1 + b1 , a2 + b2 , a3 + b3 )
a.b = a1 b1 + a2 b2 + a3 b3 . (1.1)
and
1
ka × bk = ((a2 b3 − a3 b2 )2 + (a1 b3 − a3 b1 )2 + (a1 b2 − a2 b1 )2 ) 2 .
We know that a.b = kakkbk cos θ where θ is the angle between vectors a and
b and so ka × bk2 = kak2 kbk2 − kak2 kbk2 cos2 θ or
or
(g) a.(a × b) = 0
(h) b.(a × b) = 0.
The dot product is also known as inner product or scalar product. It can be
also defined as a.b = kak kbk cos θ where θ is the angle between the vectors.
Example 3. (a) Let i = (1, 0, 0), j = (0, 1, 0) and k = (0, 0, 1) then i.i =
1, j.j = 1, k.k = 1, kik = kjk = kkk = 1 and in each case cos θ = 1.
√ 1
(b) kak = a.a = (a21 + a22 + a23 ) 2 .
(c) i.j = j.i, j.k = k.j = 0, k.i = i.k = 0.
Solution:
√ (a) i.i = (1, 0, 0).(1, 0, 0) = 1 + 0 + 0 = 1. Similarly j.j = 1, k.k = 1.
kik = 12 + 02 + 02 = 1.
1
(b) kak = (a21 + a22 + a23 ) 2 .
√ p p
a.a = (a1√ , a2 , a3 )(a1 , a2 , a3 ) = a21 + a22 + a23
Hence kak = a.a.
(c) i.j = (1, 0, 0).(0, 1, 0) = 0 + 0 + 0 = 0. j.i = (0, 1, 0).1, 0, 0) = 0.
Similarly j.k = k.j = 0, k.i = i.k = 0.
Definition 2. Vectors a and b are said to be orthogonal (perpendicular) if
a.b = 0 (dot product of a and b is zero).
Definition 3. (a) Angle between two vectors a = a1 i + a2 j + a3 k and b =
b1 i + b2 j + b3 k is defined as
a1 b1 + a2 b2 + a3 b3
θ = cos−1 .
kakkbk
(b) For non-zero vector a = a1 i + a2 j + a3 k in R3 , the angle α, β, γ between
a and the unit vectors i, j and k, respectively are called direction angles of a
and defined as
a.i a1 a2 a3
cos α = = , cos β = , cos γ =
kakkik ka|| ||a|| ||a||
and cos α, cos β and cos γ are called direction cosines of a. The direction
cosines
of a non-zero vector a are similarly the components of the unit vector
1
a
||a||
1 a1 a2 a3
a= i+ j+ k = cos αi + cos βj + cos γk.
kak kak kak kak
Definition 4. The component of a on an arbitrary vector b denoted by
compb a is defined as
1 a.b
compb a = a. b = .
kbk kbk
8 Modern Engineering Mathematics
a1 1 1
(b) cos α = =√ =√
||a|| 1+4+9 14
a2 2
cos β = =√
||a|| 14
a3 3
cos γ = =√
||a|| 14
a.b 4 + 6 − 16 −6
(c) compb a = =√ =√ .
kbk 1 + 1 + 16 18
It may be observed that the physical interpretation of the dot product is the
work done when a constant force of magnitude F moves an object a distance
d in the same direction of the force, that is, W = F.d .
A
b B
a.(b × c) = 0.
α + β = 0, α + 2β = 0
independent.
(c) Let αa + βb + γc = 0. Then
or
(α + 2β + 5γ, α − β + 2γ, α + 4β + 7γ) = 0
α + 2β + 5γ = 0 ⇒ α = −2β − 5γ
α − β + 2γ = 0 ⇒ −2β − 5γ − β + 2γ = 0
or
−3β − 3γ = 0 or β = −γ
α + 4β + 7γ = 0
α = 3β.
Hence these three vectors are linearly dependent.
Definition 7. (Dimensions) A set of elements in R3 is called basis if these ele-
ments are linearly independent and every element of R3 is a linear combination
of basis elements. The number of elements in a basis is called the dimension.
Dimension of R3 is three as i, j, k are linearly independent and every element
of R3 is a linear combination of i, j, k (for each x ∈ R3 , x = αi + βj + γk).
Definition 8. (Orthogonal set of vectors) A set of vectors in R3 , {a, b, c} is
called orthogonal if a.b = 0, b.c = 0, a.c = 0.
v1 = u1
hu2 , v1 i
v2 = u2 − v1
hv1 , v1 i
hu3 , v2 i
v3 = u 3 − v2
hv2 , v2 i
.. ..
. = .
hum , v1 i hum , v2 i hum , vm−1 i
v m = um v1 − v2 − · · · − vm−1 ,
hv1 , v1 i hv2 , v2 i hvm−1 , vm−1 i
Matrices for Engineers 11
1 1 1
Bn = { v1 , v2 , . . . , vm }
kv1 k kv2 k kvm k
is an orthogonal basis in Rm .
Remark 3. Gram-Schmidt orthogonalization can be extended for any vector
space.
Example 8. Let S = {u1 , u2 }, where u1 = (3, 1), u2 = (1, 1) is a basis for
R2 . Obtain an orthonormal basis, say {w1 , w2 } of R2 .
Solution: Let v1 = u1 = (3, 1). Then
hu2 , v1 i
v2 = u2 − v1
hv1 , v1 i
4
= (1, 1) − (3, 1)
10
(as hu2 , v1 i = 4 and hu2 , v1 i = 10)
1 3
= − , .
5 5
1 3 1
w1 = v1 , w1 = √ , √
kv1 k 10 10
1 −1 3
w2 = v2 , w2 = √ , √
kv2 k 10 10
is an orthonormal basis.
real entries but matrices with complex numbers and functions as entries have
been studied.
a11
a21
An n × 1 matrix is also called an n− vector, namely a31 is called an n−
..
.
an1
vector
or column
vector.
1
3 2
u= −2 is a 3− vector. The n− vector
−1 is a 4− vector and v =
3
0
all of whose entries are zero is denoted by 0.
If A is an n × n matrix, then rows of A are 1 × n matrices and the columns
of A are n × 1 matrices. The set of all n− vectors with real entries is denoted
R1n 1 × n matrix and also called a row vector.
Example 10. (Matrix of distances between cities given in kilometers)
London Madrid New York Tokyo Delhi
London 0 785 3469 5959 6704
Madrid 785 0 3593 6706 7255
New York 3469 3593 0 6757 11766
Tokyo 5959 6706 6757 0 5840
Delhi 6704 7255 11766 5840 0
TABLE 1.1
kmph 17 12 7 2 −3 −8
5 12 7 0 −5 −10 −15
10 −3 −9 −15 −22 −27 −34
18 −11 −20 −25 −31 −38 −45
20 −17 −24 −31 −39 −46 −53
1 2 −1 1 2 −1
Example 13. A = 3 3 2 B = 3 3 2 .
0 −4 5 0 −4 5
A and B are equal.
Definition 11. Let A = (aij ) and B = (bij ) be two matrices, then the sum
A+B is an m×n matrix C = (cij ) denoted by cij = aij +bij , i = 1, 2, 3, . . . , m
and j = 1, 2, 3, . . . , n.
112 200
The matrix F1 + F2 = 200 260 provides the total manufacturing
400 80
and shipping costs.
Matrices for Engineers 15
Remark 4. (a) Sum A and B, that is, A + B is defined if they have the same
number of rows and the same number of columns, in other words if they are
of the same
size.
2 3 −1 1 1
If A = and B = then A + B cannot be defined.
1 3 4 −1 2
(b) Let a = (a1 , a2 , a3 , . . . , an ) then a + 0 = a.
Definition 12. (Scalar multiplication) Let A = (aij ) be an m × n matrix
and α be a real number, then the scalar multiplication of A by α denoted by
αA is the m × n matrix C = (cij ), where cij = αaij , i = 1, 2, 3, . . . , m and
j = 1, 2, 3, . . . , n, that is C is obtained by multiplying each entry of A by α:
αa11 αa12 · · · αa1n
αa21 αa22 · · · αa2n
C = (αaij ) = .
.. .. .. ..
. . . .
αam1 αam2 · · · αamn
2 3 −1 8 12 −4
Example 15. Let A = 1 1 1 and 4A = 4 4 4 .
−1 6 5 −4 24 20
2 0 3 4 −1 2 3 1
Example 16. Let A = 1 3 4 2 and B = 4 6 8 −1 .
2 −2 1 1 1 1 2 1
Find (i) A + αB (ii) A − αB (iii) A − B.
−α 2α 3α α
Solution: (i) αB = 4α 6α 8α −α .
α α 2α α
2−α 2α 3 + 3α 4+α
A + αB = 1 + 4α 4 + 6α 4 + 8α 2−α
2+α −2 + α 1 + 2α 1+α
for α = 3,
−1 6 12 7
= 13 22 28 −1 .
5 −1 7 4
2+α −2α 3 − 3α 4 − α
A − αB = 1 − 4α 4 − 6α 4 − 8α 2 + α
2 − α −2 − α 1 − 2α 1 − α
5 −6 −6 1
= −10 −14 −20 5 .
−1 −5 −5 −2
16 Modern Engineering Mathematics
3 −2 0 3
A − B = −3 −2 −4 1 .
1 −3 −1 0
Σni=1 ai = a1 + a2 + a3 + · · · + an
Σnj=1 ai = a1 + a2 + a3 + · · · + an .
This summation notation satisfies the following properties:
Remark 5. (i) The right hand side matrix is obtained by adding all the
entries in each row then adding all the resulting numbers.
(ii) The left hand side is obtained by adding all the entries in each column
and then adding all the resulting numbers.
(iii) Let A1 , A2 , A3 , . . . , Ak be m×n matrices and c1 , c2 , . . . , ck are real num-
bers then Σki=1 ci Ai = c1 A1 + c2 A2 + c3 A3 + · · · + ck Ak is called a linear
combination of m × n matrices while c1 , c2 , c3 , . . . , ck are called coeffi-
cients.
20
Example 18. Let p = 100 be a 3− vector that represents the current
200
prices of three items at a general merchant store. Suppose that the store
announces a sale so that the price of each item is reduced by 20%.
(a) Determine a 3− vector that gives the price changes for three items.
Matrices for Engineers 17
(b) Determine a 3− vector that gives new price changes of the items.
Solution: (a) Since the prices of three item are reduced by 20%, the 3− vector
−54 4
−20%p = −20 = − 20 .
−40 40
(b) The new prices of the items are given by the expression
20 20 4
p − 20% = 100 − 20%p = 100 − 20
200 200 40
16 20
= 18 = 0.80 100 = −0.80 p. (1.5)
160 200
Definition 13. Let A = (aij ) be an m × n matrix; then the transpose A,
denoted by AT , is the n × m matrix defined by aTij = aji .
It is clear that transpose matrix AT is obtained from A by interchanging the
rows and columns of A.
4 3 2 4 −5 2
Example 19. (i) Let A = −5 6 8 , then AT = 3 6 1 .
2 1 −1 2 8 −1
4 0
4 3 1
(ii) Let B = , then B T = 3 6 .
0 6 −3
1 −3
2
(iii) Let D = −3 , then DT = 2 −3 −1 .
−1
2
(iv) Let E = 2 5 7 , then E T = 5 .
7
The concept of Rn can be extended for any finite number n. This means
the set of all set of n− vectors is denoted by Rn . It is also called the column
matrices.
a1 b1
a2 b2
If a = . and b = . , then the dot product or inner product of a
.. ..
an bn
and b denoted by a.b is defined as
a.b = a1 b1 + a2 b2 + · · · + an bn
Xn
= ai bi .
i=1
18 Modern Engineering Mathematics
1 2
3 −3
Example 20. (i) Let a = 1 and b = 2 then a.b = (2 − 9 +
−1 −1
2 + 1) = −4.
x 8
(ii) Let a = 2 and b = 2 . If a.b = −8, find x.
3 4
Solution:
a.b = 8x + 4 + 12
8x + 16 = −8
8x = −24
x = −3.
0.50 85
by the dot product u.v, that is
Remark 6. The product of two matrices A and B is defined only when the
number of rows of B is exactly the same as the number of columns of A.
−2 4
1 1 −1
Example 22. (i) Let A = and B = 4 −3 . Find
3 2 4
1 2
Matrices for Engineers 19
AB.
4
1 x 3 24
(ii) Let A = and B = 8 . If AB = , find x and
2 −1 1 12
y
y.
1 2 2 1
(iii) Let A = and B = . Show that AB 6= BA.
−1 3 0 0
A + D = O.
we can write D = −A so as
A + (−A) = O.
Properties of Transpose
If A and B be m × n matrices and α be a scalar (real number), then
(a) (AT )T = A.
(b) (A + B)T = AT + B T .
(c) (AB)T = B T AT .
Let
a11 a12 ··· a1n x1 b1
a21 a22 ··· a2n x2 b2
A= , x = , b = .
.. .. .. .. ..
. . ··· . . .
am1 am2 ··· amn xn bm
Then
a11 a12 ··· a1n x1
a21 a22 ··· a2n x2
Ax =
.. .. .. ..
. . ··· . .
am1 am2 ··· amn xn
Matrices for Engineers 21
a11 x1 + a12 x2 + · · · + a1n xn
a21 x1 + a22 x2 + · · · + a2n xn
= (1.7)
.. .. .. ..
. . . .
am1 x1 + am2 x2 + · · · + amn xn
b1
b2
= .. .
.
bm
Thus, we have
Ax = b.
The matrix
A is called the coefficient matrix
of the linear system (1.6), and the
a11 a12 · · · a1n | b1
a21 a22 · · · a2n | b2
matrix . .. obtained by adjoining column b to
.. ..
.. . . a2n | .
am1 am2 · · · amn | bm
A, is called the augmented matrix of the linear system (1.6). The augmented
matrix of the linear system is denoted by (A|b). If b1 = b2 = · · ·m = 0 in (1.7)
then the linear system is called a homogeneous system. A homogeneous
system is written as Ax = 0, where A is a coefficient matrix.
Example 23. (a) Write the augmented matrix of the linear system:
−2x + z = 7
2x + 3y − 4z = 9
3x + 2y + 2z = 5.
−1 2 0 2
Hence Ax = b is consistent.
0 −1 0 1
BT = =− = B.
1 0 −1 0
If we begin by an m × n matrix A = (aij ) and then cross out some, but not
all, of its rows or columns, we obtain a sub-matrix.
2 4 6 8
2 4 6 8
Let A = −4 8 −6 10 .B= is a sub-matrix of
6 0 10 −6
6 0 10 −6
2 4 6
A as well as C = is a sub-matrix of A.
6 0 10
24 Modern Engineering Mathematics
Non-singular Matrices
AB = BA = In , AC = CA = In .
2a − c = 1
a + 2c = 0
b + 2d = 0
b + 2d = 1.
T1 + 100 + 40 + T4
T2 = or − T1 + 4T2 − T4 = 140.
4
60 + T1 + T4 + 0
T3 = or − T1 + 4T3 − T4 = 60.
4
T3 + T2 + 40 + 0
T4 = or − T2 − T3 + 4T4 = 40.
4
Matrices for Engineers 27
T1 T2
T3 T4
! t
---+ Nehru Street ---+
~ ~
E E
U5 A U5 8
;;,
~ ~
Co
Akbar Street
i
+-- +--
! D c
1. Corner A
700 cars an hour come down Nehru street to intersection A.
1300 cars an hour come down 8th to intersection A.
2. Corner B
200 cars an hour leave intersection B on Nehru Street.
900 cars an hour leave intersection B on 9th Street.
3. Corner C
400 cars an hour enter on Akbar street to intersection C.
400 cars an hour come down 10th street to intersection C.
4. Corner D
200 cars an hour leave intersection D on Akbar Street.
400 cars an hour leave intersection D on 8th Street to intersection A.
Let x1 denote the number of cars leaving corner A on Nehru Street towards
corner B.
Let x2 denote the number of cars arriving to corner B on 9th Street from
corner C.
Let x3 denote the number of cars leaving corner C on Akbar Street towards
Matrices for Engineers 29
corner D.
Let x4 denote the number of cars arriving to corner D on 8th Street from
corner A.
Assumptions. To solve this problem we assume the following:
1. To speed the traffic flow every car that arrives to a given corner must
also leave; hence at any corner, the number of cars arriving is equal to
number of cars leaving.
These four equations form a system of linear equations that can be solved
using the Gauss-Jordan method (row reduction of the augmented matrix).
x1 + x4 = 1000
x1 + x2 = 1100
x2 + x3 = 700
x3 + x4 = 600
1 0 0 1
1 1 0 0
A=
0
1 1 0
0 0 1 1
1000
1100
y=
700 .
600
where
1 0 −2 0
1 4 −4 −1
A=
1 2 0 −2
0 1 −1 0
or this matrix equation can
be written as augmented matrix
.
1 0 −2 0 .. 0
.
4 −4 −1 .. 0
1
.
..
1
2 0 −2 . 0
.
0 1 −1 0 .. 0
The reduced row echelon form of this augmented matrix is
.
1 0 0 −1 .. 0
0 1 −1 ..
0 . 0
2 .
−1 ..
0 0 1 . 0
2
..
0 0 0 0 . 0
Matrices for Engineers 31
1 1
This gives us x = w, y = w, and z = w. We choose w = 2 so that all
2 2
unknowns x, y, z are positive (x = 2, y = 1 and z = 1). Thus the balanced
equation is
2N aOH + H2 SO4 → N a2 SO4 + 2H2 O.
TABLE 1.2
Steel H1 H2 H3
A1 2 3 4
A2 1 1 2
A3 3 2 1
Find the number of houses of each type of steel which can be produced using
29, 13 and 16 tons of steel.
Solution: Let x, y and z denote the number of houses that can be constructed
of each type. Then we have three equations:
2x + 3y + 4z = 29.
x + y + 2z = 13.
3x + 2y + z = 16.
This system of
three equations
can be
written
as a
matrix equation
Ax= y
2 3 4 29 2 3 4 x
where A = 1 1 2 and y = 13 or 1 1 2 y =
3 2 1 16 3 2 1 z
29
13 . This equation can be solved by the Gauss Jordan elimination
16
method. By applying operation R1 ↔ R2 (Row R1 is interchanged by R2 )
the above system takes the form:
1 1 2 x 13
2 3 4 y = 29 .
3 2 1 z 16
32 Modern Engineering Mathematics
x + y + 2z = 13
y = 3
−5z = −20.
x + y + z = 45
z =x+8 or −x + 0y + z = 8
x + z = 2y or x − 2y + z = 0.
30 B so
.---
12
A
_)
E=15V
20
c
I,
Network analysis is based on Kirchhoff’s two laws that apply to the closed
voltage loops and nodes in the the network. A node is a junction of three or
more conductors. The network in Figure 1.7 has two nodes B and D and three
closed voltage loops defined as
A→D→B→A (Loop ADBA)
C→B→D→C (Loop CBDC)
A→D→C→B→A (Loop ADCBA).
If R11 , R12 , . . . , Rmm and V1 , V2 , . . . , Vm are constants the electric network can
be written as a matrix equation by Kirchhoff’s law for each loop.
Ax = y
where
R11 R12 ··· R1m
R21 R22 ··· R2m
A=
···
··· ··· ···
Rm1 Rm2 ··· Rmm
34 Modern Engineering Mathematics
I1
v1 I2
y = v2 , x T = . .
..
v3
Im
Example 31. Set up and solve the system of equations (matrix equations)
of the networks given in Figure 1.8 and Figure 1.9.
,11
10v 27v
+ - + -
II
I, II
30
1 ~ so
5
1
60
b
I,
40
1 1
20
-'-
-~ 60
~ 30
I
FIGURE 1.9: Electrical Network
−i1 + i2 − i3 = 0
10 − 3i + 5i3 = 0
27 − 6i2 − 5i3 = 0
Matrices for Engineers 35
or
−i1 + i2 − i3 = 0
3i − 5i3 = 10
6i2 + 5i3 = 27.
1 −1 1 | 0
−1 1 −1 | 0 −8 10
−−−−−−−−−−→
0 1 |
3 0 −5 | 10 row operations 3 3 .
1
0 6 5 | 27 0 0 51 |
3
35 38 1
The solution is i1 = , i2 = , i3 = .
9 9 3
The system of equations is
i1 − i2 − i3 = 0
5i2 − i1 − 5i2 = 0
−10i3 + 5i2 = 0
or
i1 − i2 − i3 = 0
5i2 i1 + 5i2 = 5i2
5i2 − 10i3 = 0.
−1 −1 |
1 −1 −1 | 0
1 0
−−−−−−−−−− −→ 1 26
1 5 0 | 52 row operations 0 1 | .
0 5 −10 | 0 6 3
0 0 1 | 4
1.4 Determinants
1.4.1 Introduction
Let A be an n × n matrix, Associated with A is a number called the
determinant of A denoted by detA. Symbolically, we distinguish a matrix A
36 Modern Engineering Mathematics
where Mij is the determinant of the sub-matrix obtained by deleting the ith
row and jth column of A.
Example 32. Find detA and detB if
−5 0 0 0 7 0
A= 0 7 0 and B = 4 0 0 .
0 0 3 0 0 −2
Solution:
−5 0 0
detA = 0 7 0
0 0 3
= −5(21) − 0(−15 − 0)
= −105
detB = 0(0 − 0) + 7(−8)
= −56.
det(AB) = det(A)det(B).
38 Modern Engineering Mathematics
where a11 , a22 . . . and ann are the entries on the main diagonal of A.
Proofs of Theorems, 6, 7, 9 and 11 can be found in Appendix C.
Remark 11. Physical meaning
of determinant in R2 and R3
a a
In R2 determinant 1 2
is an indicator of whether the vectors (a1 , a2 )
b1 b2
and (b1 , b2 ) are co-linear. If the determinant is zero, then vectors (a1 , a2 ) and
(b1 , b2 ) are co-linear.
10 0
Solution: det(A + B) =
0 −3
= −30
detA = 6+4
= 10
detB = −35 + 4
= −31
detA + detB = −21.
Ax = y
where
a11 a12 ··· a1n
a21 a22 ··· a2n
.. .. ..
A =
. . ··· .
.. .. ..
. . ··· .
an1 an2 ··· ann
xT = (x1 , x2 , . . . , xn ),
b1
b2
y = .
..
bn
or
Example 42. Solve the following system of equations applying Cramer’s rule
(ii) x1 + x2 = 4
2x1 − x2 = 2.
(iii) x1 − x2 + 6x3 = −2
−x1 + 2x2 + 4x3 = 9
1
2x1 + 3x2 − x3 = .
2
Solution: (i)
a11 x1 + a12 x2 = b1
a21 x1 + a22 x2 = b2
⇔
Ax = y
a11 a12 x1 b1
where A = , x= , y=
a21 a22 x2 b2
a a12
detA = 11 = a11 a22 − a12 a21 .
a21 a22
b1 a12 b1 a12
= b1 a22 − a12 b2
A1 = , detA1 =
b2 a22 b2 a22
a11 b1 a11 b1
= a11 b2 − b1 a21
A2 = , detA2 =
a21 b2 a21 b2
4(−1) − 1 × 2 −6
x1 = = =2
−1 − 1 × 2 −3
and
−6
x2 = = 2.
−3
Matrices for Engineers 43
1 −1 6 −2
(iii) A = −1 2 4 and b = 9
3
2 3 −1 2
−2 −1 6
detA = −63, detA1 = 9 2 4 = 173, detA2 = −136,
3
2 3 −1
−61 detA1 −173 136 61
detA3 = , x1 = = , x2 = , x3 = .
2 detA 63 63 126
Example 43. The magnitudes T1 and T2 of the tensions are depicted by the
support wires shown in Figure 1.10, satisfying the equations:
300Kg
cos(25o ) − cos(15o )
0
Solution: A = , y = b = .
sin(25o ) cos(15o ) 300
cos(25o ) − cos(15o )
detA = ≈ 0.6428
sin(25o ) sin(15o )
0 − cos(15o )
detA1 = o ≈ 289.8
35 sin(15o )
cos(25o ) 0
detA2 = ≈ 271.9
sin(25o ) 300
44 Modern Engineering Mathematics
detA1 289.8
T1 = ≈ ≈ 450.8
detA 0.6428
detA2 271.9
T2 = ≈ ≈ 423.
detA o.6428
or
detA.detA−1 = 1 6= 0.
So B is the inverse of A.
46 Modern Engineering Mathematics
Example 45. Examine whether the following matrices are invertible (non-
singular) or non-invertible (singular). If invertible, compute the inverses.
6 0
(i) .
−3 2
−2π −π
(ii) .
−π π
0 2 0
(iii) 0 0 1 .
8 0 0
5 3
(iv) .
5 3
1 2 3
(v) 0 1 4 .
0 0 8
6 0
Solution: (i) Since det = 12 6= 0., this matrix is invertible by
−3 2
Theorem 13. By Lemma 1
1 6 −3
A−1 =
12 0 2
1 −1
= 2 4 .
1
0
6
−2π −π
(ii) = −3π 2 6= 0. By Lemma 1 it is invertible and its inverse
−π π
−1 −1
is 3π 3π .
−1 2
3π 3π
0 2 0
(iii) Let A = 0 0 1 .
8 0 0
0 1 0 1 0 0
c11 = = 0, c12 = = −8, c13 = .
0 0 8 0 8 0
Matrices for Engineers 47
Au = λu. (1.9)
48 Modern Engineering Mathematics
Remark 13. (i) Non-zero scalar multiples of eigenvectors are also eigen-
vectors.
(ii) The geometrical meaning of eigenvectors is that the image of eigenvector
u under A is parallel to u.
Au = λu or Au − λu = 0.
(Au − λIu) = 0.
(A − λI)u = 0.
(A − λI) = 0. (1.11)
For λ2 = i we have
−1 0 −1 | 0 1 0 −i | 0
1 −i 0 | 0 ⇒ 0 1 −1 | 0
1 1 −1 − i | 0 0 0 0 | 0
so that u1 = iu3 and
u2 =
u3 .
1 −1
If u3 = 1 then v = 1 and w = v = 1 .
1 1
Definition 23. Let A be n × n matrix and AT denote its adjoint matrix. A
is called symmetric if A = AT . A is called orthogonal matrix if AT A = I.
Remark 14. (i) It is clear that an n × n invertible matrix A is orthogonal
if A−1 = AT .
a11 a12 · · · a1n
..
a21 a22 . a21
(ii) Let A = .
.. .. .. .
. . . . .
an1 an2 · · · ann
A is orthogonal matrix if and only if its columns
a11 a12 a1n
a21 a22 a2n
.. = X1 , .. = X2 , . . . , .. = Xn
. . .
an1 an2 ann
form an orthogonal set.
(iii) Let A be a symmetric matrix with real entries. Then eigenvalues of A
are real.
(iv) The eigenvectors corresponding to distinct eigenvalues of an n × n sym-
metric matrix are orthogonal.
Example 47. (i) Show that the identity matrix is orthogonal.
0 −1 0
(ii) Let A = −1 −1 1 . Show that it is symmetric and its eigen-
0 1 0
vectors associated with three distinct eigenvalues are orthogonal and
construct an orthogonal matrix related to A.
1 0 0
Solution: (i) I = 0 1 0
0 0 1
1 0 0
IT = 0 1 0
0 0 1
52 Modern Engineering Mathematics
−1
1
uT2 u3 = −1 1 1 2 = (−1).1 + 1.2 + 1.(−1) = 0
−1
1 −1 1
u1 = 0 , u2 = 1 , u3 = 2 .
1 1 −1
They are eigenvectors of the given matrix which are orthogonal. Now norms
of u1 , u2 and u3 are respectively
q √ q √ q √
||u1 || = uT1 u1 = 2, ||u2 || = uT2 u2 = 3, ||u3 || = uT3 u3 = 6.
Thus, an orthogonal set of vectors is
1 1
1
−√ √
√ 3 6
2 1 2
0
, √ ,
√ .
3 6
1 1
1
√ −√
3 6
1 1 1
√ √ √
2 − 3 6
1 2
Hence
0 √ √ is an orthogonal matrix. We can check that
3 6
1 1
1 √ −√
3 6
T −1
P =P .
Example 48. Determine whether the given matrix is orthogonal:
0 1 0 0 0 0
(i) 1 0 0 (ii) 0 0 0 .
0 0 1 0 0 0
Matrices for Engineers 53
Properties
of diagonal matrix:
α1 0 0 ··· 0 β1 0 0 ··· 0
0 α 2 0 · · · 0 0 β2 0 ··· 0
Let A =
· · · · · · · · · · · · · · · and B = · · ·
.
··· ··· ··· ···
0 0 0 0 αn 0 0 0 0 βn
Then
α1 β1 0 0 ··· 0
0 α2 β2 0 · · · 0
(i) AB = BA = ···
.
··· ··· ··· ···
0 0 0 0 αn βn
(ii) |A| = α1 α2 . . . αn .
(iii) A is invertible if and only if each main diagonal element is non-zero.
54 Modern Engineering Mathematics
A3 = P D3 P −1
and in general
Ak = P Dk P −1 , k = 1, 2, 3, 4, · · · .
Matrices for Engineers 55
Verification:
A3 = (P DP −1 )3 = (P DP −1 )(P DP −1 )(P DP −1 )
= P DP −1 P DP −1 P DP −1 = P D(P P −1 )D(P D−1 )DP −1
= P DDDP −1 = P D3 P −1 .
3
7 0 0 7 0 0
(ii) If D = 0 −2 0 then D3 = 0 (−2)3 0 .
0 0 3 0 0 33
P = (v1 , v2 , . . . , vn )
and
λ1 0 ··· 0
0 λ2 ··· 0
D= .
.. .. .. ..
. . . .
0 0 ··· λn
Using sin 2θ = 2 sin θ cos θ and cos 2θ = cos2 θ − sin2 θ, we can rewrite the
above equation as
1
(b − a) sin 2θ + c cos 2θ = 0.
2
It follows that:
2c
tan 2θ = (1.21)
a−b
after writing tan 2θ = sin 2θ/ cos 2θ.
Let us now consider the range of the angle θ. You might think that 0 ≤ θ < 2π.
However, since
T rA = λ1 + λ2 = a + b.
2c 2c cos 2θ
a−b= = ,
tan 2θ sin 2θ
and inserting this on the left hand side of Equation (1.23), the latter reduces
to:
cos2 θ
(a − b) cos 2θ + 2c sin 2θ = 2c + 2c sin 2θ
sin 2θ
2c 2c
= (cos2 2θ + sin2 θ) = .
sin 2θ sin 2θ
Substituting this result back into Equation (1.23) and solving for sin 2θ, we
find:
2c
sin 2θ = p . (1.24)
(a − b)2 + 4c2
Matrices for Engineers 61
(i) Let A = (aij ) represent the image in Figure 1.11 and the matrix adjoint
of A, that is AT = aji , represents the image in Figure 1.12.
Matrices for Engineers 63
construct
Input _____. Forward
.
1mage
nxn r--+ transform ~ Quanti- ~ Symbol r--+ Compressed
images zation encoder images
is called error free compression. Let E = (n, n) denote the error between the
original digitized image A and decompressed image B. Then
n X
X n
E(n, n) = [aij − bij ] .
i=1 j=1
n n
1 XX
The mean absolute = |aij − bij | = ems .
n2 i=1 j=1
n n
1 XX
The mean square = [aij − bij ] .
n2 i=1 j=1
p
The root mean square = Mean square.
The mean square signal to noise ratio of the output image denoted by
SN Rms is defined as
Pn Pn 2
i=1 j=1 (bij )
SN Rms = Pn Pn 2.
i=1 j=1 [aij − bij ]
√
SN Rrms is called the root-mean square signal to noise ratio.
The peak signal-to-noise ration P SN R is defined as
255 × 255
P SN R = 10 log10 .
ems
The digital image processing main task is to find methods which give minimum
value of mean absolute error ems and SN Mrms .
For more details, see Neunzert and Siddiqi [9], Gonzalez and Woods [3], Gomes
and Velho [2] and Lay [7].
Matrices for Engineers 65
TABLE 1.3
A B C D E F G H I J K L M
2 1 4 3 6 5 5 7 10 9 12 11 14
N O P Q R S T U V W X Y Z
13 16 15 18 17 20 19 22 21 24 23 26 25
Here B is 5 × 3. Now
C = BA
7 10 0
19 7 6 3 2 3
= 17 6 0 −6 −5 −4
1 22 3 9 8 7
3 26 0
−39 −36 −19
69 51 71
=
15 4 27 .
−102 −84 −64
−147 −124 −95
We convert matrix C into a string that becomes the coded message, see Table
1.4.
It may be observed that H has been sent to two different values. The first H
TABLE 1.4
has been encoded as −39, and the second H has been encoded as 51. Likewise
the three blanks assigned different values as are the two E s and two D s.
The person at the other end receives only the coded message:
−39 −36 −19 69 51 71 15 4 27 −102 −84 −64 −147 −124 −95.
The receiver knows the alpha-numeric code and the coding matrix A. Thus,
the coded script is converted back into an m × 3 matrix C and then multiplied
by A−1 to undo the original matrix multiplication. Since C = BA, we get
B = CA−1 , and then we convert the string back to the alphabet.
Example 51. Encode and decode the following message using the given ma-
trix:
1 2
(i) Message: SEND HELP, A = .
1 1
(ii) Message: MADAME X HAS THE PLANS
1 2 3
A = 1 1 2 .
0 1 2
TABLE 1.5
0 1 2 3 4 5 6 7 8 9 10 11 12 13
space a b c d e f g h i j k l m
0 14 15 16 17 18 19 20 21 22 23 24 25 26
space n o p q r s t u v w x y z
19 5 14 4 0
Solution: (i) B =
8 5 12 16 0
C = AB
1 2 19 5 14 4 0
=
1 1 8 5 12 16 0
35 15 38 36 0
= .
27 10 26 20 0
represents
the decoded message.
7 15 0 14 15 18 20
(ii) B = 8 0 15 14 0 13 1 .
9 14 0 19 20 0 0
C = AB
1 2 3 7 15 0 14 15 18 20
= 1 1 2 8 0 15 14 0 13 1
0 1 2 9 14 0 19 20 0 0
50 57 30 99 75 44 22
= 33 43 15 66 55 31 21 .
26 28 15 52 4 13 1
y = Ax (1.27)
where
x1 y1
x2 y2
x= ,y =
.. ..
. .
xn yn
a11 a12 ··· a1n
a21 a22 ··· a2n
A= .
.. .. .. ..
. . . .
an1 an2 ··· ann
is invertible.
If A is a unitary matrix, then A−1 exists and A−1 = At , whereas if At is the
transpose of A, then (1.27) can be written as
x = At y. (1.29)
for k = 1, 2, 3, . . . , n.
Similarly, by Equation (1.29), each pixel xi is a linear combination of all the
pixels
for i = 1, 2, 3, . . . , n.
Equations (1.30) and (1.31) are similar to the expressions defining the forward
and the inverse transformation kernels, respectively, whereas aki is the forward
formation kernel and bik is the inverse transformation kernel.
For the two-dimensional case, (1.30) and (1.31) take the forms
and
Here, aijkl and bijkl are forward and inverse transformation kernels, respec-
tively.
In equations (1.30) and (1.31) the Fourier, Walsh and Hadmard transforms
are commonly used for encoding in this setting and they produce fairly good
results. For example, the Fourier kernel is given by
1 −j2π(ik+jl)/N
aijkl = e .
N
Further interpretation of (1.33) is possible. Let us write (1.33) in the form
y = Akl X, (1.36)
where Akl is formed in the same manner as Bkl except that the forward kernel
is used. Let us consider the number of all possible values of the coefficients in
the Equation (1.27) where
yi = Σnj=1 aij xj .
Each element xj can have any of 2m different values, each aij xj term can also
have any of 2m different values and the sum of n such terms could have any
of 2mn different values. Therefore, a natural binary representation would give
mn bit words to assign a unique word to each of the possible 2mn values of yi .
Since only m bit words would be required to code any xj , and our objective is
to use fewer bits to code yi , we must round off yi to a fewer number of allowed
levels.
A quantizer is a device whose output can have only a limited number of
possible values. Each input is forced to one of the permissible output values.
For more details, we refer to Gonzales and Woods [3].
Definition 28. Let there be a physical and mathematical system that has
n possible states. At any one time, the system is in one and only one of its
n states. Also assume that at a given observation period, say the kth period,
the probability of the system being in a particular state depends only on its
status at k − 1th period. Such a system is called Markov Chain or Markov
process.
0 ≤ pij ≤ 1, 1 ≤ i, j ≤ n.
Definition 29. A Markov matrix is called regular if, for some integer r, all
entries of T r are positive. A Markov chain or process is called regular if its
Markov matrix is regular.
Important properties of Markov matrices and chains are summarized below:
(i) If T is a regular Markov matrix, then as n approaches infinity, T n → S,
where S is a matrix of the form (v1 , v2 , . . . , vn ) with v being a constant
vector.
72 Modern Engineering Mathematics
N R S
N 0 0.25 0.25
!
R 0.5 0.5 0.25
S 0.5 0.25 0.5
Matrices for Engineers 73
Solution: Let state D be a dry day and state R be rainy day. Then the
Markov matrix of this Markov process is
D R
2 1
D
(pij ) = 3 2 .
1 1
R
3 2
Since all entries in pij are positive, the system is a regular Markov chain.
Let us begin our observations (day 0). The weather is dry so the initial state
vector is
1
X0 = ,
0
a probability vector. The the state vector on day 1 (the day after we begin
our observations) is
0.67 0.5 1 0.67
X1 = T X0 = =
0.33 0.5 0 0.33
2 1
where is written as 0.67 and as 0.33 respectively for the sake of conve-
3 3
nience. The probability of no rain on day 1 is 0.67, and the probability of rain
on that day is 0.33. Similarly
0.67 0.5 0.67 0.614
X2 = T X1 = = ,
0.33 0.5 0.33 0.386
0.67 0.5 0.614 0.604
X3 = T X2 = = ,
0.33 0.5 0.386 0.396
0.67 0.5 0.604 0.603
X4 = T X3 = = .
0.33 0.5 0.396 0.397
0.603
From the fourth day onward the state vector remains the same: .
0.397
This is the steady state vector which means that from fourth day onward, it
is dry about 60% of the time, and it rains about 40% of the time.
(d) A Markov
matrix
may have zero eigenvalues and determinant :
0 0
A= .
1 1
1 0
(e) A Markov matrix can have several eigenvalues 1: A = .
0 1
9 −8 7
ans = −6 5 −4
11 −12 0
76 Modern Engineering Mathematics
No brackets are displayed and MATLAB has assigned this matrix the name
ans. Every matrix in MATLAB must have a name. If name is not assigned,
then MATLAB assigns ans, which is known as a default variable. To assign a
matrix name, we use the assignment operator, for example
A = (4 5 8; 0 − 1 6)
is displayed as
4 5 8
B=
0 −1 6
p code and p instructions for use at run time. However, these steps
cause slow execution of programs and a MATLAB compiler is available
to compile program executable (exe) files that run faster.
How to Run MATLAB?
Let us start with the MATLAB environment. We can open the window by
double clicking the MATLAB icon on the desktop. The main window (1.8.1)
that opens will be divided into three parts: current directory, command his-
tory, and command window.
The workspace shows the variables in use and their maximum and mini-
mum values. The functions of the current directory and command history are
self-explanatory. The command window is the function that allows a user to
write MATLAB commands.
MATLAB File Formats:
MATLAB can read and write various types of files, However, there are five
main types of files for storing data and/or programs that one will use fre-
quently. They are:
(a) M Files: These are standard ASCII text files with .m extensions written
in MATLAB editor. Our main program files are written in M formats.
We will later show an example of an M file.
(b) MAT Files: These are binary files with .mat extensions created when
we variables of MATLAB workspace.
(c) Fig Files: These are binary files with .f ig extensions used to store
graphics (i.e., curves obtained by plotting data).
(d) P Files: These are M files with .p extensions. They are used mainly for
distribution of MATLAB programs with hidden MATLAB codes.
(e) MEX Files:These are callable programs with .mex extensions and are
used to interface MATLAB with C and FORTRAN.
Starting MATLAB:
Variables, Operators and Matrices
Let us start with simple commands related to matrices. To enter a matrix,
simply enter the following when the command prompt appears in the com-
mand window: >> a = [1 2 3; 5 7 4; 9 8 6], then press Enter. The following
will appear in the command window: >> a =
1 2 3
5 7 4 .
9 8 6
matrix. Simply type a− on the command prompt (>> −a) and press enter.
We get
1 5 9
ans = 2 7 8 .
3 4 6
x + 2y − z = 10.
4x + 6y + z = 20.
x − 8y + 3z = 8.
The solution can be obtained by [x] = [A]−1 [B]. (In MATLAB syntax it is
x = inv(A) ∗ B.)
The solution can be found by writing an M file in the editor and executing it.
To open editor window, go to the File menu of MATLAB window and select
New > M File and write following code.
% Solution of linear algebraic equations.
The first line which starts with % is a comment line and it is not executed.
Introduction to MATHEMATICA
Advantages and Disadvantages
• Powerful
• Easy to learn and use
• Powerful symbolic capabilities
• Coherent and unified
• Symbolic and uniform representation
80 Modern Engineering Mathematics
1223
124
FIGURE 1.15
If we want a numerical value of the last expression with, say 20 digits, then
in the cell we can type
N [% , 20]
- 9 . 8629032258064516129
FIGURE 1.16
%2
-9 . 8629032258064516 129
FIGURE 1.17
Algebraic Calculations
Mathematica can perform algebraic calculations too using several functions.
Matrices for Engineers 81
27 - 4 2 x + 1 3 x 2 + 2 x 3
FIGURE 1.18
( - 2 + x ) ( l + x ) ( 3+ x )
FIGURE 1.19
Example 57. (Figure 1.20 though Figure 1.24) To simplify [(x2 − 3x)(6x −
7) − (3x2 − 2x − 1)(2x − 3)].
- 3 + 17x - 1 2x2
FIGURE 1.20
Togethe r [ 2 x - 3 + ( 3 x - 5 ) / ( x 2 + x + 1 ) )
FIGURE 1.21
Apart[%8]
- 5+3x
- 3 + 2X + - - - -
l + x + x2
FIGURE 1.22
Ix = 5 I 4 - 3 I 7; y = 9 I 5 + 4 I 9 - 9 x; z = 2 x' - 3 y '
FIGURE 1.23
FIGURE 1.24
Built-in Functions
All built-in Mathematical functions and constants have full names that begin
with a capital letter. The arguments of a function are enclosed by brackets.
f[x_ ] : = x·2+2x;
FIGURE 1.25
f[a + b] + f[a-b]
FIGURE 1.26
FIGURE 1.27
x if − π ≤ x < 0
h(x) = x2 0≤x<π
0 elsewhere
Graphics
FIGURE 1.28
Basic Plotting
Mathematica is exceptionally good at creating two− and three−dimensional
graphs.
• The general syntax is Plot[f, {x, xmin , xmax }].
• The plot of the function h(x) above on the interval [−π, π] is obtained
by typing the following (Figure 1.28 through Figure 1.30:
10
-3 -2
-2
FIGURE 1.29
84 Modern Engineering Mathematics
1.0
-1.0
FIGURE 1.30
Multiple Plotting
MATHEMATICA can handle multiple plots.
FIGURE 1.31
Plotting Graphs
Plot a dotted curve on the plane joining the points (Figure 1.31 and 1.32):
Plot both graphs together(Figure 1.33).
Plot p1 next to p2 (Figure 1.34).
Plot a Network/Graph Plot (Figure 1.35).
Matrices for Engineers 85
. ·.
:
0.5
-6 -4 -2
-0.5
·-
.·.....,.... ·.
-10
FIGURE 1.32
p2 = ListLinePlot[list]
10
-10
FIGURE 1.33
Show[p1, p2]
FIGURE 1.34
Show[GraphicsGrid[{{p1, p2 }}]]
LO 1\
if\..
. . 0.5! \
: !
-6 -4 ·~ -2 •
~ -oi
\_{o
FIGURE 1.35
______---; !
2
FIGURE 1.36
Matrices for Engineers 87
lo'
10'
10'
100
FIGURE 1.37
2
LogLinearPlot [ E" , {x, l, 5) ]
35x 10'
30xl0'
2.5xl0'
2.0x 10'
1.5x 10'
LOx 10'
500000
1.5 20 30
) 50
FIGURE 1.38
1010
lo'
10'
10'
100
2.0 30 50
FIGURE 1.39
88 Modern Engineering Mathematics
0.5
-0.5 0.5
-0.5
FIGURE 1.40
FIGURE 1.41
Matrices for Engineers 89
Plot Data
Plotting data by making table and customizing plot (Figure 1.43).
Fun with Plots
FIGURE 1.42
FIGURE 1.43
90 Modern Engineering Mathematics
•••
• ••
1.5
• • •••
• ••
•
••
1.0
•
0.5
•
~
1 2 3
~
FIGURE 1.44
Differentiation Operations
Single Variable Differentiation
nth derivative of a function f (x): D[f (x), {x, n}] (Figure 1.44).
d
Example 58. Find [sin(1 + 4x2 )] (Figure 1.45).
dx
D[ Sin[l+4x2 ) , x )
FIGURE 1.45
d2
Example 59. Find [sin(1 + 4x2 )] (Figure 1.46).
dx2
Partial Differentiation
d3
Example 60. Find [x3 y 2 + x sin[y 2 ]] (Figure 1.47).
dxdy 2
Integration Operations
Single Variable Integration
Matrices for Engineers 91
I 8 Cos [ 1 + 4 x
2
] -
2
64 x S i n [ 1 + 4 x 2 ]
FIGURE 1.46
2 2
6 x + 2 Cos (¥" ] - 4 y Sin[¥"]
FIGURE 1.47
Integrate[Sin[l + 4 x], x]
1
- - Cos [ 1 + 4 x ]
4
FIGURE 1.48
Rπ
Example 62. Evaluate x=0
sin(1 + 4x)dx.
FIGURE 1.49
Multivariable Integration
Integrate [f [x, y], {x, a, b}, {y, c, d}]
RR 2
Example 63. Evaluate (x sin y + xy)dxdy (Figure 1.50 and 1.51)
Rπ R2
Example 64. Evaluate y=0 x=1
(x2 sin y + xy)dxdy.
Solving Single and Quadratic Equations
1 2
1 x (3 y2 - 4 xCos[y] )
12
FIGURE 1.50
1 ~ .~
3 4
FIGURE 1.51
I•·>-{:··-,..,_, •I
{{x -. 2 }' {x -> 11}
FIGURE 1.52
[ 2
{x ->- 3 (- 9 + ..J93)
l
l /3
+
(2' ( ·r;;;;;-))"'
- 9 + v 93
32/3 }
FIGURE 1.53
cubic
Example 66. cubic (Solve x3 + x + 1 = 0 for x (Figure 1.53).
ax − 2y = −1
bx − y = 1
FIGURE 1.54
Matrices for Engineers 93
FIGURE 1.55
{x -> 2 .8 0119 x l 0- 8 }
FIGURE 1.56
{x -> -0 . 5671 4 3}
FIGURE 1.57
Differential Equations
DSolve[eqn,y,{x1,x2..}] solves equations for the function y = y(x1, x2..) (See
Figures 1.58 through 1.60)
l
{{y[x ) ->C[ 1] Cos[x ) + C[2] Sin[x ] •2 e• (cos[x ) 2 +Sin [ xJ 2 ) } }
FIGURE 1.58
FIGURE 1.59
FIGURE 1.60
FIGURE 1.61
FIGURE 1.62
FIGURE 1.63
X = ({1 , 3, 4), (3, 0 , 2), (1, 1, 1}}; Y = ((2, 1, 4 ) , (1, 1, 3), (1 , 2, 1}};
X+Y
FIGURE 1.64
X.Y
FIGURE 1.65
Matrix Customization
Determinant of matrix (Figure 1.65)
Inverse of a Matrix (Figure 1.66)
Det[A]
FIGURE 1.66
Inverse[A]
236 417112
{{-.
555
-, --}. {--,
555
- -, - }. {-. - -, - }}
555
FIGURE 1.67
Example 73. Solve 3x1 −2x2 +x3 = −1 2x1 +x2 −x3 = 0 3x1 −2x2 −3x3 = 5
(Figure 1.68).
96 Modern Engineering Mathematics
I '·-~··lA]
{{1. 3, 1 } , {0, 2, 1}. {3, 2, 2 }}
FIGURE 1.68
A = {(3, -2,1), {2,1, -1), (3, -2, -3}}; b = {{-1), {0), {5));
MatrixForm[b]
FIGURE 1.69
B = Inverse [A]
Solution = B. b
FIGURE 1.70
I LinearSolve[A, b)
I {{-14
5
}. {-14}•
11
{-;3 }}
FIGURE 1.71
-5
FIGURE 1.72
-5
FIGURE 1.73
Fourier Coefficients
Calculating nth partial sums
Plot of first and third partial sums of Fourier series of f , along with plot of
function f .
Working with spreadsheets
98 Modern Engineering Mathematics
s[n_ , x _ ] := Sin[nx] ;
c[n_ , x _ ] : = Cos[nx];
IP[f_ , g_ ) : = 1/Pi!ntegrate[f•g, (x, -Pi, Pi}];
FIGURE 1.74
FIGURE 1.75
fourierSeries(func_ , N_ , x _ ] : =
aO[func] /2+Sum[aFC[func, n] c[n, x] +bFC[func, n] s[n, x] , (n, 1, N}) ;
fsl[x_ ] = fourierSeries[ab, 1, x];
fs3 [x_ ] = fourierSeries [ab, 3, x];
FIGURE 1.76
Plot [ (periodicextension, fs1 [x ), fs3 [x )}, ( x, -2 Pi, 2 Pi}, P1otRange ... All )
FIGURE 1.77
FIGURE 1.78
a ( (1]] II Grid
FIGURE 1.79
FIGURE 1.80
FIGURE 1.81
Data Manipulation
Plot data with explicit date values:
100 Modern Engineering Mathematics
~;2006, 10, 1}, 10), ((2006, 10, 15), 12), ((2006, 10, 30) , 15), ((2006 , 11, 20), 20))1
e L1atPlot(
ta)
2oco-----.-----.-----.-----.-----.-----.----oo
15
10 •
I o~Oct~O,~--~--~Oct~l6----~---~~.,~o----~--~~
~-~7,--~~
FIGURE 1.82
10
-
J~
FIGURE 1.83
Matrices for Engineers 101
I data1 ::
{{(2006, 10, 1), 10), ((2006, 10, 15), 12) , ((2006, 10, 30), 15), ((2006, 11, 20), 20))1
data2 = {({2006, 10, 5), 15), ((2006, 10, 20), B),
((2006, 11, 10), 5), ((2006, 11, 15), 1))1
DateL1 o tPlot [(datal, data2), Joined~ True)
FIGURE 1.84
50
FIGURE 1.85
Scope
2o cr-----.-----,-----,-----,-----,-----,----~
15
10 .
lui Jm
FIGURE 1.86
a = {{{2006, 6), 10), {{2006, 8), 12), {{2006, 11), 1 5), {{2007, 1), 20))1
eL1• tPlot (data)
15
10 •
lui S.p Jm
FIGURE 1.87
1.0 cr----c
,..:;o
•••- - . - - - - - - - , - - -..r.: ••---.---,------;:
.~:- ••-.------,
.~;:-
0.5
0.0
-05
FIGURE 1.88
0.5
00
-0.5
..
-l.~2.":'.toc
s_oo
;:;----'------'-----;c,;;'~:';·,_-:o,o;--'----'---,,::;:2_-,.;;-;_oo:;;-'•-,.·--c_-:-:12;-;_,7._::30:-"
FIGURE 1.89
FIGURE 1.90
data[ [1]]
{{ 2000, 1, 3 }. 33.14 )
FIGURE 1.91
FIGURE 1.92
.--
data = {56.1, 60.7, 51.6, 52. , 57.5, 56.7, 67.4, 69.9, 72.9, 69.7, 70.3, 72.1)1
DateL1•tPlot(data, {{2006, 6, 1, 8), Automatic, •&our•) ]
10
65
60
"
09:00 12:00 15:00 18:00
FIGURE 1.93
1.9 Exercises
1.1. Let a = (1, −3, 2), b = (−1, 1, 1) and c = (2, 6, 9).
Find (i) a + (b + c) (ii) 4a − (b + c) (iii) c + 2(a − 3b)
Matrices for Engineers 105
80000
75000
70000
65000
1990 2000
FIGURE 1.94
1.9. Let a and b be two vectors and α and β be scalars, then show that
1.10. Find the angle between two vectors a = (−1, 3, 1) and b = (0, 2, −4).
1.11. Let a = (5, 2, −1), b = (2, 0, −7). Compute a.(a × b) and b.(a × b).
1.12. Examine whether the following expression are scalar or vector
(i) i × (−3k) (ii) i × (j × k) (iii) (2i − j + 5k) × i
(iv) k.(j × k) (v) ||4j − 5(i × j)|| (vi) (i × k) × (j × i).
1.13. Determine whether the following vectors are linearly independent or
linearly dependent:
(i) (4,-8), (-6,12) in R2 .
(ii) (1,1), (0,1), (2,5) in R2 .
(iii) 1, (x + 1), (x + 1)2 in P2 .
2x1 + 6x2 + x3 = 7
x1 + 2x2 − x3 = −1
5x1 + 7x2 − 4x3 = 9
(ii) x1 − x2 − x3 = 8.
x1 − x2 + x3 = 3.
−x1 + x2 + x3 = 4.
(i) N a + H2 O → N aOH + H2
(ii) C5 H8 + O2 → CO2 + H2 O
10v 27v
+ - + -
50 60
30
1.29. Examine whether the given set of vectors is linearly dependent or linearly
independent.
(i) u1 = (1, −1, 3, −1), u2 = (1, −1, 4, 2), u3 = (1, −1, 5, 7).
(ii) u1 = (1, 2, 3), u2 = (1, 0, 1), u3 = (1, −1, 5).
(iii) u1 = (2, 1, 1, 5), u2 = (2, 2, 1, 1), u3 = (3, −1, 6, 1), u4 = (1, 1, 1−1).
2 3 4
1.30. Let A = 1 −1 2 . Find the indicated minor, determinant or
−1 3 5
cofactor: M12 , 2M32 , c13 , c22 , c23 .
Matrices for Engineers 109
0 2 0 1 −1 −1
1.31. Let A = 3 0 1 , B = 2 2 −2 . Find detA and detB.
0 5 8 1 1 9
a11 a12 a21 a22
1.32. Let A = and B = Show that detA =
a21 a22 a11 a12
−detB.
a11 a12
1.33. Let A = . Show that detA = detAT .
a21 a22
2 −1 1 2 1 5
1.34. Let A = 3 1 −1 and B = 4 3 8 Verify that
0 2 2 0 −1 0
detAB = detAdetB.
1.35. Suppose that A is n × n matrix such that A2 = I where A2 = AA. Show
that detA = ±1.
1.36. Check whether the following matrices are invertible, if invertible find the
inverse.
5 −1 6 −2
(i) (ii)
4 1 0 4
1 0 0 0
1 2 3 0 0 1 0
(iii) 0 −4 2 (iv)
0 0 0 1 .
−1 5 1
0 1 0 0
4 3
1.37. If A−1 = , then find A.
3 2
4 −3
1.38. Find a value of α such that the matrix A = is its own
α −4
inverse.
sin θ cos θ
1.39. Find the inverse of A = .
− cos θ sin θ
1.40. Solve the following systems applying Cramer’s rule or indicate that the
rule does not apply
(iii) x1 + x2 − 3x3 = 0
x2 − 4x3 = 0
x1 − x2 − x3 = 5
1.43. For each matrix below, produce a matrix that diagonalizes it or show
that this matrix is not diagonalizable.
5 0 0
5 3 1 0
(i) (ii) (iii) 1 0 3 .
1 3 −4 1
0 0 −2
1.44. Find the eigenvalues of the matrices below and for each value, a corre-
sponding eigenvector. Verify that eigenvectors associated with distinct
eigenvalues are orthogonal. Find an orthogonal matrix that diagonalizes
each matrix.
5 0 2 1 3 0
4 −2 −13 1
(i) (ii) (iii) 0 0 0 (iv) 3 0 1 .
−2 1 1 4
2 0 0 0 1 1
1.44. Discuss the use of matrices in rating world cities based on a number
of variables such as cost of living, quality of education, availability and
security of jobs, extent of cultural and recreation opportunities, trans-
portation, health facilities, air quality, safety and crime, and climate.
1.45. Show that Markov matrix
0 1 0
A= 0 0 1
1 0 0
1.47. Let A be Markov matrix with all positive elements, then show that λ is
the only eigenvalue of modulus 1 and (A − In ) = 1.
1.48. Describe application of Markov chain in A. S. Pushkin’s “Yevgeniy One-
gin” poem.
1.49. Discuss relevance of Markov matrix and Markov chain to Shannon’s
communication theory.
1.50. Write an essay on application of Markov chain and Markov matrices to
compute performance evaluation, page rank and Web search.
[2] J. Gomes, L. Velho, Image Processing for Computer Graphics and Vision,
Springer, 2008.
[3] R. C. Gonzalaez, R.E. Woods, Digital Image Processing, Third Edition,
Prentice Hall, 2007.
[4] K. Hardy, Liner Algebra for Engineers and Scientists Using MATLAB,
Pearson, Prentice Hall, 2005.
[5] M. T. Heath, Scientific Computing: An Introductory Survey, Mcgraw Hill,
Second Edition, 2002.
113
Chapter 2
Differential Equations
115
116 Modern Engineering Mathematics
2.1 Introduction
Mathematics has been considered the language of nature since the days of
Galileo and in modern times it is considered the mother of all technologies.
Differential equations were invented in the course of investigations of the laws
that govern the physical world. Sir Isaac Newton (1642-1727) first solved them
in the 17th century and called them fluxional equations. Gottfried Leibnitz,
a contemporary of Newton credited with the invention of calculus, renamed
fluxional equations as differential equations, a name that remains in use today.
Modern methods for solving differential equations and their practical ap-
plications constitute major portions of mathematics curricula for engineering
and science students. Leibnitz invented some of the methods but the gen-
eral theory for differential equations was developed by an electrical engineer,
Augustin-Louis Cauchy (1789–1857).
In the 20th century, several Nobel Prizes were awarded for research that
Differential Equations 117
involved differential equations. For example, William Libby received the prize
for chemistry in 1960 for the invention of carbon dating. The 1952 physics
prize was awarded to Felix Block and Edward Purcell. They calculated the
rates of change over time of magnetic moments of nuclei at each point in a
sample. The 1979 prize for medicine and physiology went to Allan Cormack
and Godfrey Hounsfield for their work on computerized tomography (CT).
Robert Merton and Myron Scholes received the 1997 economics prize for their
work with Fisher Black on the Black-Scholes option pricing model.
Differential equations play a central role in describing phenomena that
involve changes. The main goal of this chapter is to introduce basic methods
for solving ordinary differential equations and developing modeling techniques
for application to real world problems. Methods for dealing with separation
of variables, undetermined coefficients, parameter variations, Cauchy-Euler
and Laplace transform methods, and series solutions are some of the classical
methods covered in this chapter. MATLAB and MATHEMATICA solutions
of differential equations are also discussed.
These methods are more efficient than analytical approximations based
on power series. Therefore Section 2.3 (MATLAB and MATHEMATICA for
Differential Equations) may be preferred over Section 2.8 (Series Solution of
Differential Equations) in some situations. The applications of modeling tech-
niques in various fields such as environmental problems, nuclear physics, elec-
tricity and electronics, mechanics, and biomedical engineering are presented
in Section 2.6.
Example 74.
dy d2 y
+ 20y = ex , + 10y = cos x
dx dx2
d2 y dy dx dy
3 −9 + 6y = 0, + = 2x + y
dx2 dx dt dt
are examples of differential equations.
Example 75.
d3 y d2 y
3
+ 2 − 4y = e−x is 3.
dx dx
Definition 34. The degree of a differential equation is the degree of the
highest order derivative in the equation.
Example 77. (i) The degree of ODE
3
dy √
dy
y − x − 5 = 0 is 3.
dx dx
Remark 17. (i) Very often notations y 0 , y 00 , y 000 , . . . , y n are respectively used
for
dy d2 y d3 y dn y
, 2, 3,..., n.
dx dx dx dx
(ii) In symbols we can express an nth order ordinary differential equation in
one dependent variable by general form:
dn y
F (x, y, y 0 , y 00 , . . . , y (n) ) = 0, where y (n) = (2.1)
dxn
where F is a real valued function of n + 2 variables.
Definition 35. Linear and Non-linear Differential Equations
An nth order ordinary differential equation is said to be linear in y if it can
be written in the form
dy
a1 (x) + a0 (x)y = g(x).
dx
d2 y dy
a2 (x) 2
+ a1 (x) + a0 (x)y = g(x).
dx dx
Remark 18. An ordinary differential equation is linear if the following con-
ditions are satisfied.
(i) The unknown function and its derivative occur in the first degree only.
(ii) There are no products involving either the unknown function and its
derivatives or two or more derivatives.
120 Modern Engineering Mathematics
The left hand side of the given ODE y 00 − y 0 − 2y = 0 is = (4c1 e2x + c2 e−x ) −
(2c1 e2x − c2 e−x ) = −2(c1 e2x + c2 e−x ) = 0.
Example 80. (i) Choosing c = 1 we get a particular solution of the differential
equation considered in Example 79(iii).
(ii) For c1 = 1 we get a particular solution of the differential equation in
Example 79(i) that is, y = ex+c2 is a particular solution of y 00 − y = 0.
1
Example 81. (i) y = − x2 is a singular solution of the given differential
2
1
equation in Example 79(iii). y = − x2 is not obtainable from the general
2
1 2
solution y = cx+ c . However it is a solution of the given differential equation,
2
can be checked. For y = −x, by putting values of y and y 0 into the right hand
side of the equation we get
1 1
(−x)2 + x(−x) − − x2 = x2 − x2 = 0.
2 2
dy x
=− .
dx y
It is clear that
1 1 x
y1 = √ (−2x) = −
4 − x2 2 y1
and
1 1 x
y20 = − √ (−2x) = − .
4−x 2 2 y2
(ii) The relation y 2 + x − 4 = 0 is an implicit solution of 2yy 0 + 1 = 0 on the
interval (−∞, 4).
Verification: Differentiating y 2 + x − 4 = 0 with respect to x, we obtain
dy
2y + 1 = 0 or 2yy 0 + 1 = 0, which is the given differential equation. Hence
dx
y 2 + x + 4 = 0 is an implicit solution if it defines a real function
√ on (−∞, 4).
2
Solving
√ the equation√ y + x − 4 = 0 for y, we get y = ± 4 − x. Both y1 =
4 − x and y2 = − 4 − x and their derivatives are functions defined for all x
in the interval (−∞, 4). We conclude that y 2 + x − 4 = 0 is an implicit solution
on this interval.
Remark 19. Note that a relation g(x, y) = 0 can reduce a differential to an
identity without constituting an implicit solution of the differential equation.
For example x2 + y 2 + 1 = 0 satisfies yy 0 + x = 0, but it is not an implicit
solution as it does not define a real-valued function.
√ This is clear from the
solution of the equation x2 + y 2 + 1 = 0 or y = ± 1 − x2 , imaginary number.
The relation x2 + y 2 = 1 is called a formal solution of yy 0 + x = 0. That
is, it appears to be a solution. Very often we look for a formal solution rather
than an implicit solution.
Example 84. Find the differential equation of the family curves y = ce2x .
Solution: Consider
y = ce2x . (2.2)
Differentiating Equation (2.2) we get
y 0 = 2ce2x = 2y
or
y 0 − 2y = 0. (2.3)
Thus, arbitrary constant c is eliminated and Equation (2.3) is the required
differential equation of the family of curves given by Equation (2.2).
Example 85. Find the differential equation of the family of curves
y = c1 cos x + c2 sin x. (2.4)
Solution: Differentiating (2.4) twice we get
y 0 = −c1 sin x + c2 cos x (2.5)
y 00 = −c1 cos x − c2 sin x (2.6)
c1 and c2 can be eliminated from (2.4) and (2.6) and we obtain the differential
equation
y 00 + y = 0. (2.7)
(2.7) is the differential equation of the family of curves given by (2.4).
(Xo, yo)
v-\- /
y =0(x)
R
Xo
X
+-- I----+
dy
Example 88. = xy 1/2 , y(0) = 0 has at least two solutions, namely y = 0
dx
and y = x4 /16.
Example 89. (i) Does a solution of the boundary value problem y 00 + y =
0, y(0) = 0, y(π) = 2 exist?
(ii) Show that the boundary value problem
y 00 + y = 0, y(0) = 0, y(π) = 0
0 = c1 cos 0 + c2 sin 0
and 2 = c1 cos π + c2 sin π.
The first equation yields c1 = 0 and the second yields c1 = −2 which is absurd,
hence no solution exists.
(ii) The boundary values yield
0 = c1 cos 0 + c2 sin 0
and
0 = c1 cos π + c2 sin π.
126 Modern Engineering Mathematics
Both of these equations lead to the fact that c1 = 0. The constant c2 is not
assigned a value and therefore takes arbitrary values. Thus there are infinitely
many solutions represented by y = c2 sin x.
Example 90. Examine the existence and uniqueness of a solution of the
following initial value problems:
(i) y 0 = y/x, y(2) = 1.
(ii) y 0 = y/x, y(0) = 3.
(iii) y 0 = −x, y(0) = 2.
Solution: We examine whether conditions of Theorem 15 are satisfied. To
check (i), we observe that
y ∂f 1
f (x, y) = and (x, y) = .
x ∂y x
∂f
Both functions are continuous except at x = 0. Hence f and satisfy the
∂y
condition of the theorem in any rectangle R that does not contain any part
of the y-axis (x = 0). Since the point (2, 1) is not on y-axis, there is a unique
1
solution. One can check that y = x is the only solution. (ii) In this problem
2
∂f
neither f nor is continuous at x = 0. Hence Theorem 15 is not applicable.
∂y
dy
Definition 39. A first order differential equation of the form = g(x)h(y),
dx
where g(x) and h(y) are functions of x and y only, respectively is called sep-
arable or said to have separable variables.
process or system often involves a rate of change of one or more of the vari-
ables, the mathematical representation of all these assumptions may be one
or more equations containing derivatives that is, mathematical model may be
a differential equation or system of differential equations. Once a model is
obtained (Figure 2.2) we look for existence and uniqueness of its solution. If
we can solve it, we claim that the model is reasonable if its solution is con-
sistent with experimental data or known facts about the process or system.
If the prediction by the solution is of poor quality, we may increase number
of variables or make alternative assumptions in the system. The steps of the
modelling are shown in the following diagram:
Express
Assumptions assumptions in Mathematical
terms of differential formulation
l
equations
If necessary alter
assumptions or Solve differential
increase resolution of equations
model
where λ is constant of proportionality. This simple model does not take into
account many factors such a immigration and emigration that can influence
populations to grow or decline. However, it predicts correctly the population
in many cases, for example the population of the United States of America
during the period 1790 to 1860. Around 1840 the Belgium mathematician cum
biologist P. F. Verhulst discovered a mathematical model to predict the human
population of a country or city. This model is a generalization of Malthusian
model and is known as the logistic equation and its solution is called logistic
function. The graph of a logistic function is called a logistic curve. Let P (t)
denote the population at time t and let a > 0, b > 0 be constants whose
values depend on the population. The logistic model of the population growth
is given as
dP (t)
= aP (t) − bP 2 (t). (2.9)
dt
Verhulst had argued that the rate of change of the population P (t) with re-
spect to t should be influenced by growth factors such as population itself and
also factors retarding the population, namely limitations of food and space.
First term aP (t) represents growth factor while −bp2 (t) represents retarda-
tion factors. In applications, very often a is much larger than b. It is clear that
(2.9) reduces to (2.8) if b is neglected.
dA dA
∝ A(t) or = λA, (2.10)
dt dt
(2.10) is similar to (2.8) which tells us a single differential equation can serve
as a mathematical model for different physical phenomena. In physics the half
life is simply the time it takes for one-half of the atoms in an initial amount
to disintegrate or transmute into atoms of another element. For example the
half life of highly radioactive radium, Ra−226, is about 1700 years. The half
life of uranium isotope U−238 is approximately 4,500,000,000 years. λ > 0 is
called growth constant and λ < 0 is called decay constant.
Differential Equations 129
Model (2.14) can be used for spread of a rumor; see Brenan [5, p.717].
E (t)
Let E(t) denote impressed voltage on a closed loop, θ(t) denote the change on
the given capacitance and I(t) denote the amount of current after the switch
is closed. By Kirchhoff’s second law, the impressed voltage E(t), on a closed
loop must equal the sum of the voltage drops in the loop. Current I(t) is
related θ(t) on the capacitor by
dθ(t)
I(t) = .
dt
dI d2 θ(t)
By adding the three complicated drops L = L (inductor), IR =
dt dt
dθ(t) 1
R (resistor) and θ(t) (capacitor) and equating the sum to the im-
dt C
pressed voltage E(t), we get
d2 θ dθ 1
L + R + θ = E(t). (2.16)
dt2 dt C
We shall discuss the solution of the model given by (2.16) in Section 2.6.5. If
E(t) = 0, the electric vibrations of the circuit are said to be free vibrations.
A circuit is called over damped if R2 − 4LIC >0, critically damped if
Differential Equations 131
dI
L + RI = E(t).
dT
The above equation is the model for current flow where L and R are constants
known as the inductor and resistance, respectively. The current I(t) is also
called the response of the system.
−A∆h = B∆s.
132 Modern Engineering Mathematics
dh Bp
=− 2gh. (2.19)
dt A
Differential Equation (2.18) represents the rate at which water level is drop-
ping.
d2 x dx
or + 2λ + ω 2 x = F (t) (2.20)
dt2 dt
f (t) B K
where F (t) = , 2λ = and ω 2 = , represents forced motion. It may
m m m
be
observed that (2.20) is a general form of the equation
d2 x
+ ω2 x = 0 (2.21)
dt2
describing simple harmonic motion or free undamped motion. x(t) =
2π
c1 cos ωt + c2 sin ωt is the general solution of (2.20). The period T = and
ω
ω
the frequency f = .
2π
Differential Equations 133
~~~ l+s
unstretched _j
Equilibrium position motion
mg-ks =0
a b c
dA(t)
= (Rate of substance entering in)-(Rate of flowing out )
dt
= Rin − Rout . (2.22)
The rate at which the salt is entering into the tank in kg per minute = np.
The rate at which the salt is flowing out from the tank in kg per minute =
134 Modern Engineering Mathematics
n
A(t). Thus we obtain
m
dA(t) n
= np − A(t). (2.23)
dt m
Differential Equation (2.23) models concentration of a chemical mixture and
its solution provides the amount of salt at any time t. Concrete examples and
their solution will be presented in Section 2.6.9.
Remark 20. The fundamental balance Equation (2.22) or differential Equa-
tion (2.23) can be applied in different situations. This model can be used to
determine pollutant in a lake or a drug in an organ of the body, monthly
payment of a loan, see Section 2.6.9.
there is a supply of food and so foxes are added to the system at a rate
βxy, β > 0. Adding this rate to (2.24), we get a model for the fox population:
dx
= −αx + βxy. (2.25)
dt
On the other hand, if there are no foxes, the rabbit population will grow at a
rate proportional to the number of rabbits present at time t:
dy
= µy, µ > 0. (2.26)
dt
If foxes are present, a model for the rabbit population is (2.26) decreased by
λxy, λ > 0, the rate at which the rabbits are eaten during their encounters
with the foxes:
dy
= µy − λxy. (2.27)
dt
Equations (2.25) and (2.27) constitute a system of non-linear differential equa-
tions
dx
= −αx + βxy
dt
dy
= µy − λxy (2.28)
dx
where α, β, µ and λ are positive constants and (2.28) is called the Lotka-
Volterra predator-prey model.
Remark 21. Except for two constant solutions, x(t) = 0, y(t) = 0 and
µ α
x(t) = , y(t) = , the non-linear system (2.28) cannot be solved in terms
λ β
of elementary functions.
The equation describing the rate of DNAPL mass discharge from the source
region
dm
= −Asvd cs (t) (2.29)
dt
while an algebraic relationship between cs (t) and m(t) is assumed in the form
of a power law,
γ
cs (t) m(t)
= (2.30)
c0 m0
where γ is empirically determined. (2.29) and (2.30) yield a differential equa-
tion
dm
= −αmγ (2.31)
dt
that models the dissolution of DNAPL into the ground water flowing through
the source region. For more details see Falta, Rao and Basu in [5].
(t1 − t)
E(t) = E0 e RC ,
where E0 = E(0).
heart
Q !J
switch
p s c
where the intensity of the X-ray beam I(x) = E.N (x), E is the energy level of
each photon, N (x) denotes the number of photons per second passing through
a point x and A(x) is attenuation coefficient of the medium through which
X-ray is passing.
We assume that an X-ray beam is composed of photons and that each
photon has the same energy level E. Furthermore every substance through
which an X-ray passes has the property that each millimeter of the substance
absorbs a certain portion of the photons through it. This proportion which is
specific to each substance, is called the attenuation coefficient of that material.
Therefore
dN k
= N (M − N )
dt M
is a model for propagation of information among a population of size M (t).
Symbolic Solution:
The function dsolve computes symbolic solutions to ordinary differential
equations (Figure 2.8 and 2.9). The equations are specified by symbolic
expressions containing the letter D to denote differentiation. The symbols
D2, D3, . . . , DN , correspond to the second, third,. . . , N th derivatives respec-
tively. Thus, D2y is equivalent to d2 y/dt2 . We can specify the initial conditions
dx
also. For example, let us solve = x + t with initial condition x(0) = 0. We
dt
can then write following code
-1-t+exp(t)
200
180
160
140
120
100
80
60
40
20
-6 -4 -2
d2 y
tion to be solved is = cos 2t − y, with initial conditions y(0) = 1 and
dt2
dy(0)
= 0. To solve it write the following code
dt
>> y = dsolve(‘D2y=\cos(2*t)-y’, ‘y(0)=1’,‘Dy(0)=0’)
y =
4/3*\cos(t)-1/3*\cos(2*t)
140 Modern Engineering Mathematics
TABLE 2.1
Numerical Solution:
MATLAB has several built-in functions to solve various types of ODEs nu-
merically. The detail is given below.
We will use one simple example of solve ode45(). The general syntax is given
as
>>[time, solution] = ode 45(’myfunction’, tspan, xo),
where myfunction is the ODE. In the syntax, tspan is the time interval for
which we want a solution and xo is the initial condition. Let us use same ODE,
which we used above for a symbolic solution. This ODE can be solved in two
ways by writing a function file describing an ODE or by using inline function.
The code is given below:
>> myode=inline(‘x+t’);
>> [t,x]=ode45(myode, [0 2], 0);
>> plot (t,x)
We can code in alternate form by writing a function file. The code is given
below.
% function file saved as myode.m
function dxdt= myode (t,x)
dx dt=x+t;
On command prompt, we will write the following:
>> [t,x]=ode45(‘myode’, [0 2], 0);
>> plot (t,x)
We can solve higher order ODEs also. We must represent them as a set of
d2 y dy
first order ODEs. For example, let us solve 2 − (1 − y 2 ) + y = 0 with
dt dt
dy(0)
initial condition y(0) = 2 and = 0. Before writing, we have to express
dt
second order ode into set of first order ode. The ode can also be written as
ÿ = (1 − y 2 )ẏ − y . . . (1). Let y = x1 and x2 = x1.˙ . . (2). Then equation in y
˙ 2
can be written as x2 = (1 − x1 )x2 − x1 . . . (3).
The equations (2) and (3) can be written as
Differential Equations 141
4.5
3.5
2.5
1.5
05
0
0 0.2 0.8 1.2 1.4 1.6 1.8
ẋ1 x2
= . . . (4),
ẋ2 (1 − x12 )x2 − x1
and (4) is of the form ẋ = f (x).
Now we write a function file which will describe Equation (4). The code is
given below:
function xdot = myode(t,x)
xdot = [x(2); (1-x(1)^2)*x(2)-x(1)];
Then write the following commands on the command prompt:
>> [t,x]=ode45(‘myode’,[0 20], [2; 0]);
>> plot(t,x)
·1
10 12 14 16 18 20
I
Example 92. Find the general solution of y 00 (x) + y(x) = ex subject to the
boundary conditions y(0) = 0, y(0) = 1.
In[ ]:=DSolve[{y[x] + y[x] == Exp[x], y[0] == 1, y[0] == 1},
y, x]
Out[ ] = {{y - > Function[{x}; \frac{1}{2}(cos x + e^x cos^2 x
+ sin x + e^x sin^2 x)]}}
Example 93. Solve numerically the initial value problem (ordinary differen-
tial equation
y 0 (t) = y cos(t2 + y 2 ), y(0) = 1.
In[ ]:=nsol+NDSolve[{y’[t]==y[t]cos[t^2+y[t]^2],
y(0)==1},y,{t,0,20}]
Out[ ]={{y -> InterpolatingFunction [{{0,20}},<>]}}
We plot this equation with
In[ ]:= Plot [Evaluate[y[t]/.nsol],{t,0,20},
Differential Equations 143
!IIV.VAWM
5 10 15 20 X
1
where ρ(y) = or H(y) = G(x) + C, where H(y) and G(x) are anti-
h(y)
1
derivatives of p(y) = and g(x), respectively.
h(y)
Example 94. Solve the differential equation
y 0 = y/x.
1 1
Solution: Here g(x) = , h(y) = y and p(y) = and G(x) = ln x and
x y
H(y) = ln y.
Hence H(y) = G(x) + C
or ln y = ln x + ln c
ln y − ln x = ln c (See Appendix A for properties of ln x)
y
= c
x
y = cx.
Differential Equations 145
sin 7x
y= + c.
7
Example 97. Find the general solution of the following differential equations:
dy dy dy
(a) = 9y (b) x +2y = 3 (c) x +(3x+1)y = e−3x
dx dx dx
dy
Solution: (a) − 9y = 0.
dx
P (x) = −9.
R
Integrating function = I(x) = e 9dx
= e−9x ,
Z
y.e−9x = 0.e−9x dx + c
3
where I(x) = x2 , f (x) = . Thus
x
Z
3 2
yx2 = x dx + c
x
Z
3
= 3xdx + c = x2 + c
2
3 c
or y = + , 0 < x < ∞.
2 x2
Differential Equations 147
e−3x
dy 1
= 3+ y=
dx x x
1 e−3x
P (x) = 3 + , f (x) = .
x x
R
P (x)dx
Integrating factor = I(x) = e
1
!
R
3+
=e x dx = xe3x
e−3x 3x
Z
3x
y.xe = .xe dx + c
x
Z
= e0 dx + c = x + c
c −3x
or y = e−3x + e , for 0 < x < ∞.
x
One can find a particular solution of Equation (2.34) by a procedure known
as variation of parameter (for higher order linear differential equations, this
method is discussedRin Section 2.5.7). The basic idea is to find a function u
so that yp = u(x)e− P (x)dx is a solution of (2.34). Substituting yp into (2.34)
yields
dy1 du
u + P (x)y1 + y1 = f (x),
dx dx
R
where y1 = e− P (x)dx
, so that
du dy1
y1 = f (x) as + P (x)y1 = 0.
dx dx
This gives us
Z Z
f (x) f (x) R
du = dx and u= dx = f (x)e P (x)dx .
y1 (x) y1 (x)
R R
Therefore, yp = e− P (x)dx e P (x)dx f (x)dx.
R
R dy
yc = ce− P (x)dx is a general solution of + P (x)y = 0.
dx
Hence
R
Z R
y = yc + yp = ce−P (x)dx + e− P (x)dx e P (x)dx f (x)dx. (2.36)
R My − Nx
dy
I(y) = e M .
Procedure of Solution 2.38:
Step 1: Check whether a differential equation written in the form (2.38)
satisfies (2.39).
Step 2: If Equation (2.39) is satisfied then there exists a function f for which
∂f
= M (x, y). (2.40)
∂x
Integrating (2.40) with respect to x, while holding y constant, we get
Z
f (x, y) = M (x, y)dx + g(y) (2.41)
Differential Equations 149
Remark 23. (a) Right hand side of (2.42) is independent of variable x, be-
cause
Z Z
∂ ∂ ∂N ∂ ∂
N (x, y) − M (x, y)dx = − M (x, y)dx
∂x ∂y ∂x ∂y ∂x
∂N ∂M
= − = 0.
∂x ∂y
(b) We could just start the above mentioned procedure with the assumption
that
∂f
= N (x, y)
∂y
By integrating N (x, y) with respect to y and differentiating the resultant
expression, we would find the analogues of (2.41) and (2.42) to be, respectively,
Z
f (x, y) = N (x, y)dy + h(x) and
Z
∂
h(x) = M (x, y) − N (x, y)dy.
∂x
Example 98. Check whether x2 y 3 dx + x3 y 2 dy = 0 is exact.
∂M ∂N
Solution: In view of Remark 22(b) we must check whether = , where
∂y ∂x
M (x, y) = x2 y 3 , N (x, y) = x3 y 2
∂M ∂N
= 3x2 y 2 , = 3x2 y 2 .
∂y ∂x
∂M ∂N
This shows that = 3x2 y 2 = . Hence the given equation is exact.
∂y ∂x
Example 99. Determine whether the following differential equations are ex-
act. If they are exact solve them by the procedure given in this section.
150 Modern Engineering Mathematics
∂M ∂N
= 0, = 0.
∂y ∂x
∂M ∂N
Thus = and so the given equation is exact.
∂y ∂x
Apply procedure of (2.38) for finding solution.
∂f
Put = 2x − 1. Integrating and choosing h(y) as a constant of integration
∂x
we get Z
∂f
= f (x, y) = x2 − x + h(y),
∂x
h0 (y) = N (x, y) = 3y + 7, and by integrating with respect to y we obtain
3 2
h(y) = y + 7y.
2
The solution is
3
f (x, y) = x2 − x + y 2 + 7y = c.
2
(b) It is not exact as
M (x, y) = 2x + y, N (x, y) = −x − 6y
∂M ∂N
and = 1 6= = −1.
∂y ∂x
(c) M (x, y) = 3x2 y + ey , N (x, y) = x3 + xey − 2y,
∂M
= 3x2 + ey
∂y
∂N
= 3x2 + ey .
∂x
∂M ∂N
Thus = , the equation is exact.
∂y ∂x
∂f
Applying procedure for solving 2.38 and let = 3x2 y + ey . Integrating with
∂x
respect to x, we obtain
d3 y d2 y
6 + 10 − 8y = 0
dx3 dx2
y(1) = 0, y 0 (1) = 0, y 00 (1) = 0
possesses the trivial solution y = 0. Since the third order equation is linear
with constant coefficients, it follows that all conditions of Theorem 16 are
satisfied. Hence y = 0 is the only solution on any interval containing x = 1.
Example 101. Check whether the function y = 3e2x + e−2x − 3x is a solution
of the initial value problem
d2 y
− 4y − 12x = 0
dx2
y(0) = 4, y 0 (0) = 1.
Here a2 (x) = 1 6= 0, a1 (x) = −4 6= 0 for every interval containing
x = 0, g(x) = 12x. a2 (x), a1 (x) and g(x) are continuous on any interval I
containing x = 0. y = 3e2x + e−2x − 3x is a solution of the initial value prob-
lem on any interval I containing x = 0 by Theorem 16. It is also a unique
solution by the same theorem.
d2 y dy
a2 (x) + a1 (x) + a0 (x)y = g(x) (2.45)
dx2 dx
subject to
y(a) = y0 , y(b) = y1 . (2.46)
Conditions in (2.46) are called boundary conditions.
Remark 24. (i) A solution of BVP [(2.45) and (2.46)] is a function φ(x)
satisfying the differential Equation (2.45) on some interval I, containing a
and b, whose graph passes through the two points (a, y0 ) and (b, y1 ).
(ii) For a second order differential equation, other pairs of boundary conditions
could be
y 0 (a) = y0 , y(b) = y1 ,
y(a) = y0 , y 0 (b) = y1 ,
y 0 (a) = y0 , y 0 (b) = y1 .
Differential Equations 153
d2 y
+ 16y = 0
dx2
π
y(0) = 0, y( ) = 0
2
has infinitely many solutions.
Solution: It can be checked that
y = c1 cos 4x + c2 sin 4x
d2 y
+ 16y = 0.
dx2
y(0) = 0 = c1 cos 4.0 + c2 sin 4.0 or c1 = 0.
π π
y = 0 = c2 sin 4 or c2 sin 2π = 0, for any choice c2 .
2 2
Hence the boundary value problem
d2 y
+ 16y = 0
dx2
π
y(0) = 0, y( ) = 0
2
has infinitely many solutions.
dn y dn−1 y dy
an (x) + a n−1 (x) + · · · + a1 (x) + a0 (x)y = g(x) (2.47)
dxn dxn−1 dx
where g(x) 6= 0, (g(x) is not identically zero) is called a non-homogeneous
nth order differential equation. If g(x) = 0, then
dn y dn−1 y dy
an (x) + an−1 (x) + · · · + a1 (x) + a0 (x)y = 0 (2.48)
dxn dxn−1 dx
is called a homogeneous linear differential equation of nth order. We shall
see in the latter part of this chapter that in order to solve a non homoge-
neous differential Equation (2.47), we must be able to solve the associated
homogeneous Equation (2.48). Here we discuss the general solution of (2.48).
Throughout the discussion we assume that
• ai (x), i = 0, 1, 2, . . . , n are continuous.
154 Modern Engineering Mathematics
Superposition Principle
The following theorem tells us that the sum or superposition of two or more
solutions of (2.48) is also a solution of (2.48).
Theorem 17. Let y1 , y2 , y3 , . . . , yn be a solution of (2.48) on an interval I.
Then the linear combination
y = α1 y1 + α2 y2 + · · · + αn yn ,
where αi , i = 1, 2, 3 . . . , n, are arbitrary constants, is also a solution of (2.48)
on I.
Differential Equations 155
Example 104. The set consisting of e−3x and e4x is a fundamental set of
solutions of the differential equation y 00 − y 0 − 12y = 0 on (−∞, ∞). y = e−3x
is a solution of the given differential equation, that is, y 00 − y 0 − 12y = 9e−3x +
3e−3x − 12e−3x = 0.
y = e4x is also a solution of the given differential equation, that is,
e−3x
The set of {e−3x , e4x } is linearly independent as 4x = e−7x is a function
e
and not a constant. In other words neither is a constant multiple of the other
and {e−3x , e4x } is independent. Also,
e−3x
e4x
W (y1 , y2 ) = = 4ex + 3ex = 7ex 6= 0.
−3e−3x 4e4x
Example 105. y = c1 e2x + c2 e5x + 6ex is the general solution of the non-
homogeneous differential equation
y 00 − 7y 0 + 10y = 24ex .
We have
yc0 (x) = 2c1 e2x + 5c2 e5x
yc00 (x) = 4c1 e2x + 25c2 e5x
y0 = 6e2x
y 00 = 6ex , so
y 00 − 7y 0 + 10y = 6ex − 42ex + 60ex = 24ex
u00 + 2u0 = 0.
w0 + 2w = 0.
This
R is a linear first order differential equation. Applying integrating factor
e 2dx = e2 x, we can write
d 2x
e w = 0.
dx
By integrating we obtain
e2x w = c1 or w = u0 = c1 e−2x .
We can check that y1 and y2 are linearly independent on (−∞, ∞) and form
a fundamental set;
y = c1 e m 1 x + c2 e m 2 x (2.60)
is the general solution of (2.58).
Case (ii) Repeated Roots
If m1 = m2 , we obtain only one exponential solution y1 = em1 x . A second
solution is
Z 2m1 x
e
y2 = em1 x dx
e2m1 x
Z
= em1 x dx = xem1 x .
In this equation we have used −b/a = 2m1 . The general solution in this case
is
y = c1 em1 x + c2 xem2 x . (2.61)
y = c1 ex + c2 e−6x .
y 00 + 3y 0 − 2y = 0.
y(0) = 1, y 0 (0) = 2.
Solution: The auxiliary equation is
m2 + 3m + 2 = 0.
m1 = −1 and m2 = −2.
162 Modern Engineering Mathematics
y = c1 e−x + c2 e−2x .
an mn + an−1 mn−1 + · · · + a2 m2 + a1 m + a0 = 0.
If all roots (say m1 , m2 , . . . , mn ) of this equation are real and distinct, then
the general solution of (2.57) is
It is difficult to summarize other two cases because the roots of any auxiliary
equation of degree greater than 2 can occur in many combinations.
g(x) = a0 + a1 x + a2 x2 + · · · + an xn .
y 00 − y 0 + y = 2 sin 3x.
Solution: A natural first guess for a particular solution would be A sin 3x.
Since successive differentiation of sin 3x produces cos 3x and sin 3x, we are
prompted instead to assume a particular solution that includes both of these
terms:
yp = A cos 3x + B sin 3x.
Differentiating yp and substituting the results into the differential equation
gives:
yp0 = −3A sin 3x + 3B cos 3x.
yp00 = −9A cos 3x − 9B sin 3x.
164 Modern Engineering Mathematics
−8A − 3B = 0. (2.68)
3A − 8B = 2.
6 −16
yp = cos 3x − sin 3x.
73 73
Example 111. Find a particular solution of y 00 + 3y 0 + 2y = 5x2 .
Solution: We guess that yp is of the form
yp = A + Bx + Cx2 (2.69)
yp0 = B + 2Cx (2.70)
yp00 = 2C (2.71)
yp00 + 3yp0 + 2yp = 2
2C + 3B + 6Cx + 2A + 2Bx + 2Cx (2.72)
2
= 5x (2.73)
2 2
or (2C + 3B + 2A) + (6C + 2B)x + 2Cx = 0 + 0x + 5x . (2.74)
2A + 3B + 2C = 0 (2.75)
2B + 6C = 0 (2.76)
2C = 5. (2.77)
5 −15 −3 45 5 35
Thus C = , B = , A= B−C = − = .
2 2 2 4 2 4
Therefore
35 15 5
yp = − x + x2 .
4 2 2
Example 112. Solve the differential equation y 00 − 10y 0 + 25y = 30x + 3 by
undetermined coefficients.
Differential Equations 165
m2 − 10m + 25 = 0.
(m − 5)2 = 0.
m1 = 5, m2 = 5.
Solution is of form (2.61), that is,
y = c1 e5x + c2 xe5x .
Step 2. Let yp = Ax + B
yp0 = A.
yp00 = 0.
0 − 10A + 25(Ax + B) = 30x + 3.
(−10A + 25B) + 25Ax = 30x + 3.
This implies
−10A + 25B = 3, 25A = 30.
6
Thus A =
5
6 3
−10. + 25B = 3 gives B = .
5 5
6 3
yp = x + .
5 5
The general solution is
6 3
y = c1 e5x + c2 xe5x + x + .
5 5
Example 113. Solve the differential equation y 00 + 4y = 3 sin 2x by undeter-
mined coefficients.
Solution: Step 1. Find the complementary function. Auxiliary equation is
m2 + 4 = 0
m = ±2i.
Solution is of form (2.63), that is,
−3
−4b = 3 or B=
4
A = 0.
−3
yp = x cos 2x
4
3
y = yc + yp = c1 cos 2x + c2 sin 2x − x cos 2x.
4
Undetermined Coefficients− Annihilator Approach: Differential Equa-
tion (2.63) can be written in terms of operators D, D2 , D3 , . . . , Dn as
Ly = g(x) (2.78)
L(f (x)) = 0
1, x, x2 , . . . , xn−1 . (2.80)
4e2x − 10xe2x .
Differential Equations 167
m3 (m2 + 3m + 2) = 0
or m3 (m + 1)(m + 2) = 0,
has roots m1 = m2 = m3 = 0, m4 = −1. and m5 = −2.
Thus its general solution must be
The terms in the box in (2.84) constitute the complementary function of the
given equation. We can very well argue that a particular solution yp of the
given equation should also satisfy (2.83). This means that the terms remaining
(2.84) must be the basic form, of yp :
yp = A + Bx + Cx2 (2.85)
y 00 + 3y 0 + 2y = 4x2 ,
we get
2C + 3B + 2a = 0, 6C + 2B = 0, and 2C = 4.
(Aex cos x − Aex sin x) − (Aex sin x + Aex cos x) + (Bex sin x + Bex cos x)
+(Bex cos x − Bex sin x) − 2Aex cos x − 2Aex sin x + 2Bex sin x
+2Bex cos x+5(Aex cos x+Bex sin x) = ex sin x+0.ex cos x+0.constant term
or 3Aex cos x + 3Bex sin x = ex sin x.
1
Equating coefficients gives A = 0 and B = .
3
The general solution is
1
y = ex (c1 cos 2x + c2 sin 2x) + ex sin x.
3
170 Modern Engineering Mathematics
In (2.86) we assume that P (x), Q(x) and f (x) are continuous on some interval
I. As we have seen earlier in Sections 2.5.3 through 2.5.5 there is no difficulty
in finding the complementary function yc of (2.86) when P (x) and Q(x) are
constant functions.
Step 1. Find complementary function yc of (2.86) of the form yc = c1 y1 +c2 y2 .
Step 2. Find Wronksian W of y1 and y2 , that is,
y1 y2
W (y1 , y2 ) = 0
y1 y20
0 y2 y1 0
Step 3. Write W1 = , W2 = 0
f (x) y20
y1 f (x)
and find u1 and u2 by integrating
W1 W2
u01 = , u02 = .
W W
Step 4. Find a particular solution which is of form yp = u1 y1 + u2 y2 .
Step 5. The general solution of the equation is y = yc + yp .
Example 118. Solve the differential equation y 00 − y = xex by applying the
method of variation of parameters.
Solution: Corresponding homogeneous equation is y 00 − y = 0.
The auxiliary equation is m2 − 1 = 0.
Roots are m1 = 1, m2 = −1.
The complementary function is
y = c1 ex + c2 e−x
e−x
x
e
W = x = −e0 − e−0 = −2
−e−x
e
e−x
0
W1 = = −x
xex −e−x
x
e 0
W2 = x = xe2x
e xex
−x x
u01 = =
−2 2
xe2x −1 2x
u02 = = xe .
−2 2
Differential Equations 171
x2
u1 =
4
u2 = −(xe2x /4) + (e2x /8).
y = yc + yp
where yc = c1 ex + c2 e−x
x4 x
yp = u1 y1 + u2 y2 = e − (xe2x /4)e−x + (e2x /8)e−x .
4
1 1 1
Thus y = c1 ex + c2 e−x + x4 ex − xex + ex .
4 4 8
Example 119. Apply the method of variation of parameters to solve the
differential equation
y 00 − y = cosh x.
Solution: The auxiliary equation is
m2 − 1 = 0, so m1 = 1 and m2 = −1
yc = c1 ex + c2 e−x = c1 y1 + c2 y2
e−x
x
e
W (y1 , y2 ) = x = −2
−e−x
e
1
f (x) = cosh x = (ex + e−x )
2
e−x
0
= −e−x cosh x
W1 =
cosh x −e−x
x
e 0 = ex cosh x
W2 = x
e cosh x
W1 1 1
u01 = = cosh xe−x = (e−2x + 1)
W 2 4
x
W2 cosh xe −1
u02 = = = (1 + e2x )
W −2 4
−1 −2x 1
u1 = e + x
8 4
−1 1 2x
u2 = x− e
4 8
1 1 −1 1
yp = − e−2x + x ex + x − e2x e−x .
8 4 4 8
172 Modern Engineering Mathematics
d2 y dy
ax2 + bx + cy = g(x) (2.87)
dx2 dx
where a, b and c are constants, a 6= 0, and g(x) is continuous on a given interval
are called Cauchy-Euler equations. By putting y = xm , y 0 = mxm−1 , y 00 =
m(m − 1)xm−2 in (2.87) we get
d2 y dy
ax2 + bx + cy = am(m − 1)xm + bmxm + cxm
dx2 dx
= (am(m − 1) + bm + c)xm .
Thus y = xm is a solution of
d2 y dy
ax2 + bx + cy = 0 (2.88)
dx2 dx
where m is solution of the auxiliary equation :
Verification: xeiβ = (elnx )iβ = eiβlnx which, by Euler’s formula, is the same
as
Similarly,
x−iβ = cos(β ln x) − i sin(β ln x).
Adding and substracting the last two results yields
y1 = xα (xiβ + x−iβ )
Remark 30. The method described above holds true for similar equations of
order n.
Example 120. Solve the differential equations
(a) x2 y 00 − 2y = 0.
(b) x2 y 00 − 3xy 0 − 2y = 0.
(c) x2 y 00 + xy 0 + y = 0 subject to initial conditions y(1) = 1, y 0 (1) = 2.
Solution (a) The auxiliary equation is
m2 − m − 2 = 0 or (m + 1)(m − 2) = 0
so m1 = −1, m2 = 2.
The general solution is
y = c1 x−1 + c2 x2 .
174 Modern Engineering Mathematics
m2 − 4m − 2 = 0
√
4± 16 + 8
m =
2
1√
= 2± 24
2
√
= 2 ± 6.
dP
= λP (t)
dt
with P (t) > 0 and λ a constant (either negative or positive) is solved by the
separable variable method
Z Z
1
dP = λdt,
P (t)
or ln P (t) = λt + c ,
Differential Equations 175
Due to the presence of of the natural exponential function in the solution the
population model (2.8)
dP
= λP
dt
is often called the exponential or natural growth model. Figure 2.12 shows
a typical growth of P (t) in the case of λ > 0 and the case λ < 0 is given in
Figure 2.13.
P=Poe'1, 1o.70
Po
Po
as we saw earlier.
P (7) = 2P (0) = P (0)eλ−7
or
2 = eλ7 or ln 2 = 7λ
ln 2
or λ =
7
ln 2
t ln 2
3 = e 7 or ln 3 = t
7
or t = 7 ln 3 − ln 2 = ln 37 − ln 2
ln 3
= 7 ln 3 − ln 2 = 7 .
ln 2
Example 122. According to an authentic source the total population of the
world in 1999 was 6 billion persons. Assume that since then increase of about
dP (t)
212,000 persons per day according to Equation (2.8) : = λP (t).
dt
(a) What is the annual growth rate λ?
(b) Find the world population at the middle of the 21st century.
(c) In how many years the will world population be 60 billion (grow by 10
times).
Solution: (a) P0 = P (0) = 6 billion, t = 0 year corresponds to (mid) 1999.
P (t) is increasing by 212,000 or 0.000212 billion, persons per day at time t = 0
means that P 0 (0) = 0.000212(365.25) ≈ 0.07743 billions per year. From the
natural growth model
dP
= λP
dt
with t = 0 , we get
P 0 (0) 0.07743
λ= ≈ ≈ 0.0129.
P (0) 6
Differential Equations 177
Thus the world population grew at the rate of about 1.29% annually in 1999.
This value of λ gives population function
P (t) = 6e0.0129t .
that is, the world population in 2050 is 11.58 billion. Thus the world popula-
tion at the given growth has doubled in 5 years.
(c) By given information P (t) = 60 , λ = 0.0129 then by (2.93), 60 ≈ 6e0.0129t
or
ln 10
t= ≈ 178years.
0.0129
Thus, in 178 years or in 2177 the world population will be 10 times the pop-
ulation in mid 1999.
Example 123. Suppose the population of a city grows at a rate proportional
to the population at time t. The initial population of 500 increases by 15% in
10 years. What will be the population in 30 years? How fast is the population
growing at t = 30?
Solution: Let P = P (t) be the solution at time t. dPdt(t) = λP (t) and P (t) =
Aeλt .
Since P (0) = 500 , we have A = 500 and P = 500eλt .
Since 15% of 500 is 75, we get P (0) = 500e10λ = 575.
1 575 1
Solving for λ we get λ = ln = ln 1.15.
10 500 10
1
For t = 30, P (30) = 500e 10 ln 1.5∗30 = 500e3 ln 1.15 = 760years.
0 dP 1
P (30) = = λP (30) = (ln 1.15)760 = 10.62 persons per year.
dt t=30 10
Example 124. The population of an insect grows in a pond at a rate propor-
tional to the number of insects present at time t. After 3 hours it is observed
that 400 insects are present. After 10 hours 2000 insects are present. What
was the initial number of insects?
Solution: Let P = P (t) be insect population at time t and P0 be the initial
population of the insect.
dP
We have P = P0 eλt by solving = λP (t).
dt
400 1
P (3) = 400, P (10) = 2000 give us 400 = P0 e3λ or eλ = ( )3 .
P0
From P (10) = 2000, we get
400 10
2000 = P0 e10λ = P0 ( )3
P0
178 Modern Engineering Mathematics
2000 −7 2000 −3
so 10 = (P0 ) 3 and P0 = ( 10 ) 7 ≈ 201.
400 3 400 3
As discussed in Section 2.2.1 population growth is also modelled by Equation
(2.9):
dP (t)
= aP (t) − bP 2 (t).
dt
We solve it by separation of variables. Decomposing the left hand side of
dP
= dt into partial fractions and integrating yields
P (a − bP )
11 b
+ dP = dt
aP a(a − bP )
1 1
ln |P | − ln |a − bP | = t + c
a a
P
ln = at + ac
a − bP
P
= c1 eat
a − bP
P (t) = ac1 eat − bc1 P (t)eat
P (t)(1 + bc1 eat ) = ac1 eat
ac1 eat ac1
P (t) = at
= −at
.
1 + bc1 e e + bc1
a P0
Let P (0) = P0 , P0 6= , we find c1 = .
b (a − bP0 )
After substitution and simplification
aP0
P (t) = ·
bP0 + (a − bP0 )e−at
Example 125. A model for the population P (t) in a suburb of a large city
is given by the initial value problem
dP
= P (10−1 − 10−7 P ), P (0) = 500,
dt
where t is measured in months. What is the limiting value of the population?
At what time will the population be equal to one-half of the limiting value?
dP 1 1
Solution: From = P ( − 7 P ) and P (0) = 5000 we obtain, by P (t)
dt 10 10
1 1
after solution of Example 124, a = ,b= 7
10 10
500
P =
(0.0005 + 0.0995e−.1t )
so that
P −→ 1000, 000 as t −→ ∞.
If P (t) = 500, 000 then t = 52.9 months.
Differential Equations 179
dA
= λA and A(0) = 1.
dt
By solving this initial value problem we get
Example 128. Suppose a fossilized wood has 47% C−14. Assuming that
wood died at the initial time (t = 0), compute the time T it would take for 1
gram of the radioactive carbon to decay this amount.
Solution: By Equation (2.10)
dA
= λA
dt
where λ is the half life C−14
A(t) = A0 eλt ,
A(t) = represents mass of C−14 at time t and A(0) = A0 denotes the mass
at the initial time. At time T
A(T ) = A0 eλT
or
A(T )
= eλT
A(0)
or
A(T )
ln = λT
A0
or
1 A(T )
λ= ln
T A0
or
A(T )
1
A(t) = A0 e T ln .
A0
Half life of C−14 is 5600 years approximately. Hence
ln 2
T = ln(0.47).
5600
Example 129. A fossilized bone is found to contain one thousandth the
origin amount C − 14. Determine the age of the fossil.
dA
Solution: = λA(t) , where λ is a constant of proportionality of decay (let
dt
λ = −k). The solution is A(t) = A(0)e+λt by separation of variables. Half life
of C−14 is approximately 5600 years
A0
A(56000) =
2
1
A0 = A0 e5600λ
2
1
or ln = 5600λ
2
1
or λ = − ln 2 = −0.00012378
5600
Differential Equations 181
1
A0 = A0 e−0.00012378t
1000
ln 1000
t = = 55800 years.
0.00012378
Then
T − 70 = Beλt
T (t) = 70 + Beλt .
Suppose the officer arrived at 8.30 p.m. and the body temperature was 94.4o F;
8.30 p.m. is considered as t = 0 then
or B = 24.4,
giving T(t) = 70 + 24.4eλt .
Suppose the officer makes another measurement of the body’s temperature
at 10 p.m. and recorded temperature is 890 F. Taking t = 90 minutes (the
temperature at 10 p.m.) and T (90) = 89,
90λ
89 = 70 + 24.4ee
1 19
implying λ = ln Thus,
90 24.4
1 19
T (t) = 70 + 24.4e 90 ln 24.4 t
t 19
98.6 = 70 + 24.4e 90 ln 24.4
28.6 t 19
= e 90 ln 24.4
24.4
28.6 t 19
ln = ln
24.4 90 24.4
1 28.6
t = 90 19 ln 24.4
ln 24.4
= approximately − 57.07 min.
dN λ
= N (M − N ),
dt M
where N (t) is number of students who know the rumor at time t, and M is
the total number of students (population size). This equation can be written
as
dN
= µN (M − N )
dt
λ
where is constant
M
dN
= µdt
N (M − N )
Z Z
dN
= µ dt + c
N (M − N )
1 N
ln | | = µt + c
M M −N
N
so ln | | = M µt + M c
M −N
N N
ln = ln | |
M −N M −N
as N > 0 and M − N > 0.
In exponential form
N
= eM µt+M c = AeM µt , A = eM c
M −N
N = (M − N )AeM µt
N (1 + AeM µt ) = M AeM µt
M AeM µt
N =
AeM µt + 1
Differential Equations 185
M M M M
N= = = = .
1 1+ 1 −µM t 1 + be−µM t 1 + be−ct
1+ Ae
AeM µt
(b)
M = 45000.
For
45000
t = 0, N = 300 ⇒ 300 = ⇒ b = 149
1+b
t = 4, N = 900.
45000
Thus N = .
1 + 149e−ct
45000
For t = 1, N = 900, 900 =
1 + 149e−c
45000 49
1 + 49e−c = = 50 or e−c = , so
900 149
45000
N= 49 .
1 + 149( 149 )t
45000
For t = 4, N = ≈ 16000 students who know the rumor after 4
49 2
1 + 149( )
149
weeks.
I(t) = 1 + Ae−24t .
I(t) = 1 − e−24t .
186 Modern Engineering Mathematics
dh Bp
√ = − 2gdt
h A
dh B
√ = Cdt, where C = −
h A
Z Z
−1
or h 2 dt = C dt.
1
2h 2 = Ct + C1 .
1 √ ct + 2√5 2
For t = 0, h = 52 implying C1 = 2 5. Thus, h(t) = .
2
1 d2 x dx
2
= −4x − 2
2 dt dt
d2 x dx
or +4 + 8x = 0, (see Section 2.2.8)
dt2 dt
The auxiliary equation is
m2 + 4m + 8 = 0
Differential Equations 187
√
−4 ± 16 − 32
m=
2
m1 = −2 + 2i, m2 = −2 − 2i
and α = 2, β = 2, so solution is
1 d2 x
+ 8x = 0
2 dt2
d2 x
+ 16x = 0.
dt
The auxiliary equation is
m2 + 16 = 0.
m = ±4i.
Therefore,
x = c1 cos 4kt + c2 sin 4kt.
2π 2π π
Period of oscillation = = = .
w 4 2
1 4 2
Frequency = = = .
T 2π π
See Section 2.2.8.
dA A
=0−
dt 50
−t
implying A = ce 50
A(0) = 30 − ce0 then c = 30
−t
Thus A(t) = 30e 50 .
Example 140. A tank is filled to capacity with 500 gallons of water. Brine
containing 2 pounds of salt per gallon is pumped into the tank at a rate 5
gal/min. The well mixed solution is pumped out at a faster rate of 10 gal/min.
(a) Find the number A(t) of pounds of salt in the tank at the time t. (b) when
is the tank empty?
Solution: From Equation (2.23) and by given data in this problem we have
dA 10A 2A
= 10 − = 10 −
dt 500 − (10 − 5)t 100 − t
dA 2A
or + = 10.
dt 100 − t
By integrating we have
dA
= γA − 12µ
dt
where γ is the annual interest rate, µ is the monthly payment rate and µ is
multiplicative by 12 so that all the terms will have units of dollars per year.
The initial condition is A(0) = A0 , where A0 is the amount of the loan.
dA
Here γ = 0.08, A0 = 20, 000, thus − 0.08A = −12µ. The integrating factor
dt
−0.08t
is e and by Section 2.4.2, A = 150µ + ce0.08t . Since A(0) = 20, 000, c =
20, 000 − 150µ we have
(c) The defining equation for the Laplace transform is an improper integral,
which is defined as
Z ∞ Z T
−st
e f (t)dt = lim e−st f (t)dt.
0 T →∞ 0
Thus, the existence of the Laplace transform of f depends upon the existence
of the limit.
(d) A Laplace transform is rarely computed by referring directly to the def-
inition and integrating. In practice we use tables of Laplace transforms of
commonly used functions; see for example Table 2.2 later in this section.
In Section 2.7.2 we will develop methods that are used to find the Laplace
transforms of a shifted or translated function, step functions, pulses and var-
ious other functions that often arise in applications.
Differential Equations 191
(e) We shall verify that the Laplace transform is linear, that is,
by applying integration by parts. Since the first term is zero and the second
is
1
L{t} = ,
s2
by part (i).
1
Therefore L{t} = .
s2
(iii) By Definition 47, we have
Z ∞
n
L(t ) = e−st tn dt.
0
s2 1 1
or I = 2 2
= 2 .
s +1s s +1
Theorem 23. Let f1 (t) and f2 (t) have Laplace transforms and let c1 and c2
be constants, then
(i) L{f1 (t) + f2 (t)} = L{f1 (t)} + L{f2 (t)},
(ii) L{c1 f1 (t)} = c1 L{f1 (t)} and
L{c2 f2 (t)} = c2 L{f2 (t)}, equivalently,
2t 2
= lim = lim 2 at = 0,
aeat
t→∞ a e
where the last two limits are obtained by using L’Hopital’s rule. Therefore
there exists a positive constant M such that
s(s2 + 4k 2 )
1
10. eat 33. δ(t − t0 ) e−st
0 , t>0
s−a
k
11. sinh kt 2
34. eat f (t) F (s − a)
s − k2
s
12. cosh t 35. f (t − a)H(t − a) e−as F (s)
s2 − k 2
TABLE 2.2: Cont.
Sr. No. f(t) L{f(t)} Sr. No. f(t) L{f(t)}
2 2k 2
13. sinh kt 36. f (n) (t) sn F (s) − sn−1 f (0) · · · − f (n−1) (0)
s(s2 − 4k 2 )
s2 − 2k 2 dn
14. cosh2 kt 37. tn f (t) (−1)n F (s)
s(s2 − 4k 2 ) dsn
1 Rt
15. teat 38. 0
f (u)g(t − u)du F (s)G(s)
(s − a)2
n! sin at a
16. tn eat 39. arc tan
(s − a)n+1 t s
(n a positive integer) √
k 1 2 e−a s
at √
17. e sin kt 40. √ e−a /4t
(s − a)2 + k 2 πt s
s−a 1 s
18. eat cos kt 41. f (at) L {f }
(s − a)2 + k 2 a a
k Rt 1
Differential Equations
and therefore
−1 4
L = e−2t sin 4t.
(s + 2)2 + 16
−3s
−1 se
Example 146. Compute L .
s2 + 4
Solution: By Theorem 26
L−1 {c1 F1 (s) + c2 F2 (s)} = c1 L−1 {F1 (s)} + c2 L−1 {F2 (s)}
Remark 32. H(t − a) or ua (t) is another notation for the Heaviside function
defined by
0, 0 ≤ t < a
H(t − a) = .
1, t ≥ a
The graph of the Heaviside function y = H(t − 3) is shown in Figure 2.14.
Remark 33. (i) It may be observed that H(t − a)f (t − a) is simply a trans-
lation of a function f (t), that is,
0, 0≤t<a
H(t − a)f (t − a) = .
f (t − a), t ≥ a
Differential Equations 205
f(t)
06
0.4
02
2 3
Z 8 Z ∞
−
= H(t − 8)e sds + H(t − 8)e−s ds
0 8
∞
e−8s
Z
= e−s ds = .
8 s
(ii) L{t + H(t − 2)(t − 2)2 } = L{t} + L{H(t − 2)(t − 2)2 }
Z 2
1
= + H(t − 2)(t − 2)2 e−st dt
s2 0
Z ∞
+ H(t − 2)(t − 2)2 e−st dt
2
Z ∞
1
= + 0 + (t − 2)2 e−st dt
s2 2
1
= + e−2s (L{t2 })(s)
s2
1 2
= + e−2s 3
s2 s
s + e−2s
= .
s3
206 Modern Engineering Mathematics
1 e−s
F (s) = − − .
s2 s2
Example 149. Find the inverse Laplace transform of the function
1 e−5s
F (s) = + .
s4 s4
−5s
1 −1 e
Solution: f (t) = L−1 {F (s)} = L−1 { } + L { }
s4 s4
t3 (t − 5)3
= + H(t − 5) , applying Theorem 26.
3! 3!
Unit Impluse: Mechanical systems are often impacted by external forces of
large magnitude that act only for very short times. One example is electro-
magnetic force acting within an electrical circuit. A vibrating airplane wing
may be struck by lightning. A spring may receive a sharp blow from a mass.
A baseball, golf ball, or tennis ball can soar into the air when struck violently
by a bat, club, or racket. Such impacts are often called generalized functions
or distributions.
Note, however, that these impacts are not functions in the classical sense
(designated Dirac delta functions to honor P.A.M. Dirac (1902-1984, recipient
of the Nobel Prize in physics in 1933). Dirac supervised another Nobel lau-
reate, Abdus Salam who won the physics prize in 1979 and founded and led
UNESCOs International Centre of Theoretical Physics in Trieste, Italy.
The Dirac delta function introduced in 1930 contradicted the well known
Lebesgue integral theory used by engineers and physicists for 20 years. In
1950, Laurent Schwartz, a French mathematician, developed a rigorous math-
ematical theory of generalized functions. Known as the distribution theory,
it provided logical foundations for delta function techniques. Every piecewise
continuous function is a distribution. However, the delta function defined be-
low is an example of a distribution that is not a function in the classical
sense.
1
δ = [H(t) − H(t − )]
2
For any positive number , the impulse δ is defined by:
Example 150. Find the Laplace transforms of the following functions: (a)
Dirac delta function δ(t − a).
(b) δ(t − π).
π
(c) sin t.H2π (t) + δ(t − ).
2
Solution: (a) L{δ(t − a)} = L{lim→0+ δ (t − a)}
Z ∞
1
= lim+ {H(t − a) − H(t − a − )} dt
→0
0
1 1 −as 1 −(a+)s
= lim e − e
→0+ s s
1 −as 1
e − lim→0+ e−(a+)s
s s
=
e−as (1 − e−s )
= lim+ = e−as
→0 s
1 − e−s
(as lim→0+ = 1). If a = 0, L{δ(t)} = e−0.s = 1.
s
(b) We know L{δ(t − a)} = e−sa and so L{δ(t − π)} = e−sπ .
(c) Since sin t is periodic with period 2π, we have
π π
sin tH2π (t) − δ t − = sin(t − 2π)H2π (t) − δ t − .
2 2
Therefore, by the translation property Theorem 26 and by part (a) we get
desired result.
dy d2 y
equations we need to evaluate the Laplace transform of and 2 .
dt dt
Let f (t) be differentiable for t ≥ 0 and let its derivative f 0 (t) be continuous.
By applying the formula for integration by parts we find that
L{f 0 (t)} = sF (s) − f (0). (2.95)
Verification: By definition
Z ∞
L{f 0 (t)} = e−st f 0 (t)dt
0
Z ∞
= e−st f 0 (t) + s e−st f (t)dt
0
= −f (0) + sL{f (t)}
= sF (s) − f (0).
Here we have used the fact that
lim e−st f (t) = 0.
t→∞
Similarly for a twice differentiable function f (t) such that f 0 (t) is continuous
we can prove that
L{f 0 (t)} = s2 F (s) − sf (0) − f 0 (0). (2.96)
In fact we can prove the following theorem by repeatedly applying integration
by parts.
Theorem 27. Let f, f 0 , . . . , f (n−1) be continuous on [0, ∞) and of exponential
order and if f (n) (t) be piecewise continuous on [0, ∞), then
L{f (n−1) (t)} = sn F (s) − sn−1 f (0) − sn−2 f 0 (0) · · · − f n−1 (0). (2.97)
Theorem 27 can be used to generate a formula for the Laplace transform
of the indefinite integral of a function f . We then have the following theorem.
Theorem 28. Let f be piecewise continuous and of exponential order for
t ≥ 0, then Z t
1 1
L f (u)du = L{f (t)} = L{F (s)}.
0 s s
Rt
Proof. Let g(t) = 0 f (u)du. Then g 0 (t) = f (t) and g(0) = 0.
Furthermore, g(t) is of exponential order. By Theorem 27
L{g 0 (t)} = sL{g(t)} − g(0)
Z t
or L{f(t)} = sL{ f (u)du}
0
Z t
1
L{ f (u)du} = L{f (t)}.
0 s
Differential Equations 209
00
Example 151. (a)Using the
Laplace transform of f , find L{sin kt}.
1 t
(b) Show that L−1
R
F (s) = 0 f (u)du.
s
Solution: (a) Let f (t) = sin kt, then f 0 (t) = k cos kt, f 0 (t) =
−k 2 sin kt, f (0) = 0 and f 0 (0) = k. Therefore
k
L{sin kt} =
s2 + k2
(−k 2 L{sin kt} = s2 L{sin kt} − k or L{sin kt}(s2 + k2 ) = k).
(b) By Theorem 28,
Z t
1
L{ f (u)ds} = F (s).
0 s
Rt 1
This implies that 0 f (u)ds = L−1 F (s) .
s
Derivative of Laplace Transform
Theorem 29. Let f (t) be piecewise continuous and of exponential order over
each finite interval, and let
Proof. Suppose that |f (t)| ≤ M eat , t > 0 and take any s0 > a. Then consider
∂ −st
e f (t) = −tf (t)e−st .
∂s
Choose > 0 such that s0 > a + . Then we have |t|, et for all t large enough
since in fact s
lim = 0.
t−→∞ eat
210 Modern Engineering Mathematics
Thus |tf (t)| ≤ M e(a+)t , for all large t and we find that tf (t) is also of
exponential order and Z ∞
tf (t)e−st dt
0
exists by Theorem 24, that is, the integral converges uniformly. Hence F (s) is
differentiable at s0 and
Z ∞
∂ −st
F (s0 ) = e f (t)dt dt at s = s0 .
0 ∂s
Therefore
Z ∞
F 0 (s) = − tf (t)e−st dt
0
= L{−tf (t)} for all s > a.
Remark 34. It can be checked that F (s) = L{f (t)} and n = 1, 2, 3, . . . ; then
dn
L{tn f (t)} = (−1)n F (s).
dsn
Definition 52. (Convolution). Let f and g be piecewise continuous functions
for t ≥ 0. Then the convolution of f and g denoted by f ∗ g, is defined by the
integral
Z t
(f ∗ g)(t) = f (u)g(t − u)du
0
Z t
= g(u)f (t − u)du
0
= (g ∗ f )(t).
where F (s) = L{f (t)} and H(.) is the Heaviside function, see Theorem 26.
Substitute this into the integral for F (s)G(s) to get
Z ∞
F (s)G(s) = L{H(t − u)f (t − u)}g(u)du. (2.98)
0
Therefore,
Z ∞ Z ∞
F (s)G(s) = e−st g(u)H(t − u)f (t − u)dtdu.
0 0
= e−st (f ∗ g)(t)dt
0
= L{f ∗ g}.
Z t
(b) (i) (f ∗ g)(t) = f (t − x)g(x)dx
0
Z t
(f ∗ g)(t) = f (t − x)g(x)dx
0
Z t
(f ∗ g)(t) = f (t − x)g(x)dx
0
Differential Equations 213
Z t
= e−(t−x) sin xdx
0
Z t
= e−t ex sin xdx
0
t
ex
1
= e−t (sin x − cos x) + e−t
2 0 2
1 1
= (sin t − cos t) + e−t .
2 2
On other hand,
Z t
(g ∗ f )(t) = g(t − x)f (x)dx
0
Z t
= sin(t − x)ex dx
0
1 1
= (sin t − cos t) + e−t .
2 2
So, f ∗ g = g ∗ f.
1 1
(b) (ii) We know that L{e−at } = , and L{sin t} = 2 .
s+1 s +1
1 1
Thus F (s)G(s) = 2
, or
s+1s +1
−1 −1 1 1
L {F (s)G(s)} = L .
s + 1 s2 + 1
−1 1 1 1 −s + 1
=L +
2 s + 1 2 s2 + 1
1 −1 1 1 −1 s 1 −1 1
= L − L + L
2 s+1 2 s2 + 1 2 s2 + 1
1 1
= e−t − cos t + sin t
2 2
= (f ∗ g)(t)
Example 154. Apply the Laplace transform to solve the initial value problem
dy
(i) 2 = −y, y(0) = −3.
dx
(ii) y 0 − 4y = 1, y(0) = 1.
Solution : The Laplace transform of the initial value problem gives us
y(t) = 3e−t/2
By Table 2.2
−1 1
L = e4t
s−4
−1 1 1 −1 1 1
L = L −
s(s − 4) 4 s−4 s
1 −1 1 1 −1 1
= L − L
4 s−4 4 s
1 4t 1
= e − .
4 4
Thus
1 1
y(t) = e4t + eat −
4 4
5 at 1
= e −
4 4
is the solution of the given initial value problem.
Example 155. (i) y 0 + 6y = e4t , y(0) = 2.
(ii) y 0 + 7y = H(t − 2), y(0) = 3.
(iii) y 00 − 2y 0 + y = 3δ(t − 2), y(0) = 0, y 0 (0) = 1.
Solution : The Laplace transform of the initial value problem yields
1
sY (s) − y(0) + 6Y (s) = .
s−4
Solving for Y (s) we get
1 2 1 1 19 1
Y (s) = + = . + . .
(s − 4)(s + 6) s + 6 10 s − 4 10 s + 6
The inverse Laplace transform gives the solution of the initial value problem
1 4t 19 −6t
y(t) = e + e .
10 10
(ii) Taking Laplace transforms of both sides of the equation and using the
property of the Laplace transform of higher order derivative, we get
1
sY (s) − 3 + 7Y (s) = e−2s .
s
Solving for Y (s), we get
1 3
Y (s) = e−2s +
s(s + 7) s + 7
1 −2s 1 1 −2s 1 3
= e − e + .
7 s 7 s+7 s+7
Differential Equations 217
Therefore,
1
s2 Y (s) − 2 + 4sY (s) + 3Y (s) = .
s−1
Solving this for Y (s), we get
2s − 1
Y (s) = .
(s − 1)(s2 + 4s + 3)
2s − 1 A B C
Let = + + .
(s − 1)(s2 + 4s + 3) s−1 s+1 s+3
The equation can hold only if,for all s,
Thus
1 2
L{f (t) ∗ g(t)} =
s+1 s2 + 4
2
= .
(s + 1)9s2 + 4
nR o
t
Example 158. Evaluate L 0 eu sin(t − u)du .
Rt
Solution : 0 eu sin(t − u)du = f (t) ∗ g(t), where f (t) = et and g(t) = sin t
(by definition of convolution). By Theorem 30 we get
Example 159. Solve the following Volterra integral equation for f (t):
Z t
f (t) = 3t2 − e−t − f (u)et−u du
0
2
L{f (t)} = F (s), L{3t2 } = 3L{t2 } = 3
s3
1
L{e−t } =
s+1
Z t
t−u
L f (u)e du = L{f (t) ∗ h(t)}
0
= L{f (t)}L{h(t)}.
By Definition 52 and Theorem 30
Z t
L f (u)et−u du = L{f (t)}L{h(t)}
0
1
= F (s). .
s−1
Therefore,
6 1 1
F (s) = − − F (s)
s3 s+1 s+1
−s3 + 6s + 6
1
F (s) 1 + =
s−1 s3 (s + 1)
−s3 + 6s + 6
s
F (s) =
s−1 s3 (s + 1)
6 6 1 2
F (s) = − 4+ −
s3 s s s+1
by carrying out the partial fraction decomposition. The inverse problem then
gives
−1 2! −1 3! −1 1 −1 1
f (t) = 3L −L +L − 2L .
s3 s4 s s+1
= 3t2 − t3 + 1 − 2e−t
Example 160. Find f (t) such that
Z t
f (t) = 2t2 − e−t − f (t − u)e−u du.
0
220 Modern Engineering Mathematics
and by Theorem 30
Taking the Laplace transform of the both sides of the integral equation we get
By Theorem 30,
Thus
1 1
F (s) =
2
+ F (s) 2
s s +1
s2 1
or F(s) 2 =
s +1 s2
s2 + 1 1 1
or F(s) = 4
= 2 + 4.
s s s
Taking the inverse Laplace transform of this equation we get
1
f (t) = t + t3 .
6
Rt
(ii) Let F = L{f }. The equation f (t) = 4t − 3 0 f (x) sin(t − x)dx can be
written as
f (t) − 4t = −3(f (t) ∗ sin t)
Applying Laplace transforms to both sides of this equation and using the
convolution Theorem 30, we get
s2 + 1 1 1
F (s) = 8 =6 2 + 2. 2 .
s2 (s2+ 4) s +4 s
Taking the inverse Laplace transform of this equation we get
f (t) = 3 sin 2t + 2t.
Example 162. Solve y 00 − 6y 0 + 9y = t2 e3t .
Solution : Applying the Laplace transform to both sides and using linearity
we get
L{y 00 ) − 6L{y 0 } + 9L{y} = L{t2 e3t }
2
or sY(s) − sY(0) − y0 (0) − 6[sY(s) − y(0)] + 9Y(s) =
(s − 3)2
where Y (s) = L{y(t)} (Using Laplace transform of derivatives).
2
(s2 − 6s + 9)Y (s) = 2s + 5 +
(s − 3)2
2s + 5 2
Y (s) = +
(s − 3)2 (s − 3)5
2 11 2
Y (s) = + +
s − 3 (s − 3)2 (s − 3)5
(decomposing right hand side into partial fractions). Taking the inverse
Laplace transform of this equation we get
1 1 1
y(t) = 2L−1 + 11L−1 + 2L−1
s−3 (s − 3)2 (s − 3)5
1 4 3t
or y(t) = 2e3t + 11te3t + t e .
12
222 Modern Engineering Mathematics
Example 164. Solve the initial value problem y 00 −5y 0 +6y = H(t−1), y(0) =
0, y 0 (0) = 1.
e−s
s2 L{y} − sY (0)0 − y 0 (0) − 5[sL{y} − y(0)] + 6L{y} = .
s
Solving for L{y} and taking the inverse Laplace transform we get
e−s
(s2 − 5s + 6)Y (s) = +1
s
e−s 1
or Y(s) =+1+
s (s − 2)(s − 3)
1 1 1 1 1 1
L−1 − + = − e2t + e3t
6 2(s − 2) 3(s − 3) 6 2 3
(by decomposition into partial fractions)
1 1 2(t−1) 1 3(t−1)
y= − e + e H(t − 1) + e3t − e2t .
6 2 3
Example 165. Solve the initial value problem y 00 + 4y 0 = sin tH(t −
2π), y(0) = 1, y 0 (0) = 0.
Example 166. Solve the initial value problem y 00 + 4y = sin 3t, y(0) =
y 0 (0) = 0.
Differential Equations 223
Series of (2.99) is also called a power series centered at a. The power series
centered
PN at a = 0 is often referred as the power series, that is, the series
n
c
n=0 n x . A power series centered at a is called convergent
P∞ at a specified
value of x if its sequence of partial sums SN (x) = n=0 cn (x − a)n , that is,
{SN (x)} is convergent. In other words the limit of {SN (x)} exists. If the limit
does not exist the power series is called divergent. The set of points x, at
which the power series is convergent, is called the interval of convergence
of the power series. P∞
For R > 0, a power series n=0 cn (x − a)n converges if |x − a| < R and
diverges if |x − a| > R. If the series converges only at a then R = 0, and if it
converges for all x then R = ∞, |x−a| < R is equivalent to a−R < x < a+R.
A power series may or may not converge at the end points a − R and a + R
of this interval. R is called the radius of convergence. P∞
A power series is called absolutely convergent if the series n=0 |cn (x−
a)|n converges. A power series converges absolutely within its interval of con-
vergence. By the ratio test, a power series
centered at a, (2.99) is absolutely
cn+1
convergent if L = |x−a| limn=∞ is less than 1, that is, L < 1, the series
cn
divergesPif L > 1; and test fails if L = 1. A power series defines a function
∞
f (x) = n=0 cn (x − a)n whose domain is the interval of convergence of series.
If the radius of convergence R > 0, then f is continuous, differentiable and
integrable on the interval (a − R, a + R). Moreover f 0 (x) and f (x)dx can
R
We observe that the first term in y 0 and first two terms in y 00 are zero. Keeping
this in mind we can write
∞
X
y0 = ncn xn−1 .
n=1
X∞
y 00 = n(n − 1)cn xn−2 . (2.100)
n=2
P∞
Identity property: If n=0 cn (x − a)n = 0, R > 0 for all x in the interval
of convergence, then cn = 0 for all n.
Analytic at a point: A function f is analytic at a point a if it can be
represented by a power series in x − a with a positive or infinite radius of
f (n) (a)
convergence. A power series where cn = , that is, the series of the
n!
P f (a)(n)
type cn (x − a)n , is called the Taylor series. If a = 0 the Taylor
n!
series is called a Maclaurin series. In calculus ex , cos x, sin x, ln(x − 1)
can be written in the form of a power series, more precisely in the form of a
Maclaurin series. For example
x2
ex = 1+x+ + ...
2!
x3 x5
sin x = x− +
3! 5!
x2 x4 x6
cos x = 1− + −
2! 4! 6!
for |x| ≤ ∞.
Arithmetic of power series: Two power series can be combined through
operation of addition, multiplication and division. The procedures for power
series are similar to those by which two polynomial are added, multiplied and
Differential Equations 225
x2 x3 x4 x3 x5 x7
x
e sin x = 1+x+ + + + ... x− + −
2 6 24 6 120 5040
1 1 1 1
= (1)x + x2 + − + x3 + − + x4 + . . .
6 2 6 6
1 1 1
... − + x5 + . . .
120 12 24
x3 x5
= x + x2 + − .
3 30
Since the power series for ex and sin x converges for |x| < ∞, the product
series converges on the same interval.
Shifting summation index: In order to discuss power series solutions of
differential equation it is advisable to combine two or more summations as a
single summation.
P∞ P∞
Example 167. Express n=2 n(n − 1)cn xn−2 + n=0 cn xn+1 as one power
series.
Solution: In order to add the two given series, it is necessary that both
summation indices start with the same number and the powers of x in each
series be such that if one series starts with a multiple of x to the first power,
we want the other series to start with same power. In this problem the first
series starts with the x1 . By writing the first term of the first series outside
the summation notation,
∞
X ∞
X ∞
X ∞
X
n(n−1)cn xn−2 + cn xn+1 = 2.1c−2x0 + n(n−1)cn xn−2 + cn xn+1 .
n=2 n=0 n=3 n=0
Both series on the right hand side start with same power of x, namely x1 . Let
k = n − 2 or k = n + 1 respectively in the first and second series. Then the
right hand side becomes
∞
X ∞
X
2c2 + (k + 2)(k + 1)ck+2 xk + ck−1 xk . (2.101)
k=1 k=1
Keep in mind that the value of the summation index is important, not the
summation index which is a dummy variable, say k = n − 1 or k = n + 1. Now
we are in position to add series in () term by term and we have
∞
X ∞
X ∞
X
n−2 n+1
n(n − 1)cn x + cn x = 2c2 + [(k + 2)(k + 1)ck+2 x + ck1 ]xk .
n=2 n=0 k=1
226 Modern Engineering Mathematics
d2 y dy
a2 (x) 2
+ a1 (x) + a0 (x)y = 0 (2.102)
dx dx
where a2 (x) 6= 0.
This can be put into standard form
d2 y a1 (x) dy a0 (x)
+ + y=0
dx2 a2 (x) dx a2 (x)
d2 y dy
or 2
+ P (x) + Q(x)y = 0. (2.103)
dx dx
Note that x0 is said to be an ordinary point of the differential Equation
(2.102) if P (x) and Q(x) represented by a power series. A point that is not
ordinary is called a singular point.
P∞
A solution of the form y = n=0 cn (x − x0 )n is said to be a solution
about the ordinary point x0 .
Remark 35. It has been proved that if x = x0 is an ordinary point of (2.102)
then there exist two linearly independent
P∞ solutions in the form of a power series
centered at x0 , that is, y = n=0 cn (x − x0 )n . A series solution converges at
least on some interval defined by |x − X0 | < R, where R is the distance from
x0 to the closest singular point.
Power series solution about ordinary point:
P∞ dy d2 y
Let y = n=0 cn xn and substitute values of y, = y0 , = y 00 in (2.103).
dx dx2
Combine series as in Example 167, and then equate all coefficients to the right
hand side of the equation to determine the coefficients cn . We illustrate the
method by the following examples. P∞We also see through these examples how
the single assumption that y = n=0 cn xn leads to two sets of coefficients,
so we have two distinct power series y1 (x) and y2 (x) both expanded about
the ordinary point x = 0. The general solution of the differential equation is
y = C1 y1 (x) + C2 y2 (x), and it can be shown that C1 = c0 and C2 = c1 .
d2 y
The differential equation + xy = 0 is known as Airy’s equation and is
dx2
used in the studies of the diffraction of light, diffraction of radio waves around
the surface of the earth, aerodynamics and other physical phenomena. We
discuss here power series solution of this equation around its ordinary point
x = 0.
Example 168. Write the general solution of Airy’s equation y 0 + xy = 0.
c0
For k = 1, c3 = − .
2.3
c1
For k = 2, c4 = − .
3.4
c2
For k = 3, c5 = − = 0 as c2 = 0.
4.5
c3 1
For k = 4, c6 = − = c0 .
5.6 2.3.5.6
c4 1
For k = 5, c7 = − =− c1 .
6.7 3.4.6.7
c5
For k = 6, c8 = − = 0 as c5 = 0.
7.8
c6 1
For k = 7, c9 = − =− c0 .
8.9 2.3.5.6.8.9
c7 1
For k = 8, c10 = − =− c1 .
9.10 3.4.6.7.8.10
c8
For k = 9, c11 = − = 0 as c8 = 0 and so on.
10.11
228 Modern Engineering Mathematics
we get
c0 3 c1 4 c0
y = c0 + c1 x + 0 − x − x +0+ x6
2.3 3.4 2.3.5.6
c1 c0 c1
+ x7 + 0 − − x9 − x10 + 0 + . . .
3.4.6.7 2.3.5.6.8.9 3.4.6.7.8.10
After grouping the terms containing c0 and the terms containing c1 , we obtain
y = c0 y1 (x) + c1 y2 (x),
1 3 1 1
where y1 (x) = 1− x + x6 − x9 + . . .
2.3 2.3.5.6 2.3.5.6.8.9
∞
X (−1)k
= 1+ x3k
2.3 . . . (3k − 1)(3k)
k=1
1 4 1 1
y2 (x) = x− x + x7 − x1 0 + . . .
3.4 3.4.6.7 3.4.6.7.9.10
∞
X (−1)k
= x+ x3k+1 .
3.4 . . . (3k)(3k + 1)
k=1
Thus c2 = 0,
(k + 2)(k + 1)ck+2 − ck−1 = 0
Differential Equations 229
and
1
ck+2 = ck−1 , k = 1, 2, 3, . . .
(k + 2)(k + 1)
Choosing c0 = 1 and c1 = 0 we find
1 1
c3 = , c4 = c5 = 0, c6 =
6 180
and so on. For c0 = 1 and c1 = 0 we obtain
1 1
c3 = 0, c4 = , c5 = c6 = 0, c7 =
12 504
and so on. Thus two solutions are
1 1 6
y1 = 1 + x3 + x + . . . and
6 180
1 4 1 7
y2 = x + x + x + ...
12 504
Shifting the index in the third series and combining the first two yields
∞
X ∞
X
cn (n + r)(2n + 2r + 1)xn+r−1 − cn−1 xn+r−1 = 0.
n=0 n=0
c0 r(2r + 1) = 0.
1
Since c0 6= 0, either r = 0 or r = − .
2
Hence two linearly independent solutions of the given differential equation
have the forms
∞
X
y1 = F0 (x) = cn xn and
n=0
∞
X
−1/2
y2 = F−1/2 (x) = x c∗n xn .
n=0
Differential Equations 231
4
y = C1 (1 + 2x − 2x2 + x3 + . . . )
9
3/2 2 2 2 4 3
+ C2 x (1 − + x − x + . . . ).
5 35 945
x2 y 00 + xy 0 + (x2 − v 2 )y = 0 (2.108)
is called Bessel’s equation.
Solution of Bessel’s equation:
Because x = 0 is a regular singular point of Bessel’s
P∞equation we know that
there exists at least one solution of the form y = n=0 cn xn+r . Substituting
the last expression into (2.108) gives
∞
X
x2 y 00 + xy 0 + (x2 − v 2 )y = cn (n + r)(n + r − 1)xn+r
n=0
∞
X ∞
X ∞
X
n+r n+r+2 2
+ cn (n + r)x + cn x −v cn xn+r
n=0 n=0 n=0
Differential Equations 233
∞
X
= c0 (r2 − r + r − v 2 )xr + xr cn xn+2
n=0
∞
X
+xr cn [(n + r)(n + r − 1) + (n + r) − v 2 ]xn
n=1
∞
X
= c0 (r2 − v 2 )xr + xr cn [(n + r)2 − v 2 ]xn
n=1
∞
X
+xr cn xn+2 . (2.109)
n=0
If v ≥ 0, the series converges at least on the interval [0, ∞). Also, for the
second exponent r2 = −v we obtain, in exactly the same manner,
∞
X (−1)n x 2n−v
J−v (x) = . (2.114)
n=0
n!Γ(1 − v + n) 2
The functions Jv (x) and J−v (x) are called Bessel functions of the first
kind of order v and −v, respectively. Depending on the values of v, (2.114)
may contain negative powers of x and hence converge on (0, ∞).
n(n + 1)
c2 = −
2!
(n − 1)(n + 2)
c3 = −
3!
(n − j)(n + j + 1)
cj+2 = − cj , j = 2, 3, 4, . . . (2.116)
(j + 2)(j + 1)
If we let j take on the values 2,3,4,. . . , the recurrence relation (2.116) yields
(n − 2)(n + 3)
c4 = − c2
4.3
(n − 2)n(n + 1)(n + 3)
= c0
4!
(n − 3)(n + 4)
c5 = − c3
5.4
(n − 3)(n − 1)(n + 2)(n + 4)
= − c1
5!
(n − 4)(n + 5)
c6 = − c4
6.5
(n − 4)(n − 2)n(n + 1)(n + 3)(n + 5)
= − c1
5!
(n − 5)(n + 6)
c7 = − c5
7.6
(n − 5)(n − 3)(n − 1)(n + 2)(n + 4)(n + 6)
= − c1
7!
and so on. Thus for at least |x| < 1 we obtain two linearly independent power
series solutions:
n(n + 1) 2 (n − 2)n(n + 1)(n + 3) 4
y1 (x) = c0 1 − x + x
2 4!
(n − 4)(n − 2)n(n + 1)(n + 3)(n + 5) 6
− x + ... . (2.117)
6!
(n − 1)(n + 2) 3 (n − 3)(n − 1)(n + 2)(n + 4) 5
y2 (x) = c1 x − x + x
3! 5!
(n − 5)(n − 3)(n − 1)(n + 2)(n + 4)(n + 6) 7
− x + ... .
7!
236 Modern Engineering Mathematics
Similarly, when n is an odd integer, the series for y2 (x) terminates with xn ,
that is, when n is a non-negative integer, we obtain an nth degree polynomial
solution of Legendre’s equation.
Since we know that a constant multiple of a solution of Legendre’s equa-
tion is also a solution, it is traditional to choose specific values for c0 or c1 ,
depending on whether n is an even or odd positive integer, respectively. For
n = 0 we choose c0 = 1, and for n = 2, 4, 6 . . .
1.3 . . . (n − 1)
c0 = (−1)n/2 ;
2.4 . . . n
whereas for n = 1 we choose c1 = 1, and for n = 3, 5, 7, . . . ,
1.3 . . . n
c1 = (−1)(n−1)/2 .
2.4 . . . (n − 1)
P0 (x) = 1
P1 (x) = x
1
P2 (x) = (3x2 − 1)
2
1
P3 (x) = (5x3 − 3x)
2
1
P4 (x) = (35x4 − 30x2 + 3)
8
1
P5 (x) = (63x5 − 70x3 + 15x). (2.118)
8
Differential Equations 237
n = 0 : (1 − x2 )y 00 − 2xy 0 = 0
n = 1 : (1 − x2 )y 00 − 2xy 0 + 2y = 0
n = 2 : (1 − x2 )y 00 − 2xy 0 + 6y = 0
n = 3 : (1 − x2 )y 00 − 2xy 0 + 12y = 0.
2.9 Exercises
2.1. Classify the given differential equations by order, and tell whether it is
linear or non linear.
(a) y 0 + 2xy = x2
(b) y 0 (y + x) = 6
(c) y cos y = y 00
(d) y 2 sin y = y 00
(e) y 00 − 4y 0 + 3y = x4
(f) y 00 = ezy
2.2. State whether the following differential equations are linear or non-
linear. Write the order of each equation.
(a) (1 − x2 )y 00 − 6xy 0 + 9y = sin x
2
xd3 y dy
(b) 3
−2 +y =0
dx dx
(c) yy 0 + 2y = 2 + x2
d2 y
(d) + 9y = sin y
dx2
2 2 !1/2
dy d y
(e) = 1+
dx dx2
d2 r k
(f) =− 2
dt2 r
Verify that in Exercises 2.3 to 2.8 the indicated function is a solution of the
given differential equation. In some cases assume an appropriate interval.
2.3. 2y 0 + y = 0; y = e−x/2 .
238 Modern Engineering Mathematics
In Exercises 2.9 through 2.14 determine a region of the xy plane for which
the given differential equation would have a unique solution through a point
(x0 , y0 ) in the region.
dy √
2.9. = xy
dx
dy y
2.10. =
dx x
dy
2.11. =y+x
dx
y2
2.12. y 0 =
x2 + y 2
dy
2.13. = x2 cos y
dx
dy y+x
2.14. =
dx y−x
2.15. Solve the following differential equations by the separation of variables
method:
(i) dx + e3x dy
dy
(ii) = (1 + x)3
dx
dy
(iii) = e3x−2y
dx
dP
(iv) = P − P2
dt
dN
(v) + N = N + et+2
dt
dy y2 − 1
(vi) = 2
dx x −1
2.16. Solve the following linear differential equations :
dy
(i) (x2 − 9) + xy = 0
dx
Differential Equations 239
dy
(ii) x + 2y = 3
dx
dy
(iii) cos x + (sin x)y = 1
dx
2.17. Solve the following initial value problems:
dy
(i) y − x = 2y 2 , y(1) = 5
dx
dy
(ii) (x + 1) + y = ln x, y(1) = 10
dx
2.18. Determine whether the given differential equation is exact. If it is exact,
solve it.
2.23. Solve the following differential equations with or without initial values.
d2 y dy
(i) x2 − 7x + 41y = 0
dx2 dx
2
d y dy
(ii) x 2 − 3 =0
dx dx
d2 y dy
(iii) x2 2 + 3x =0
dx dx
dy
y(1) = 0, =4
dx x=1
2.24. Derive a population growth model that takes deaths into account.
2.25. A drug is infused into a patient’s blood system at a constant rate of r
grams per second. Simultaneously the drug is removed at a rate propor-
tional to the amount x(t) of the drug present at any time t. Determine
a differential equation governing the amount x(t).
2.26. Find the relation between doubling and tripling times for a population.
2.27. In an archaeological wooden specimen, only 25% of original radiocarbon
12 is present. Write a mathematical model, the solution of which will
give time of its manufacturing.
2.28. Write a mathematical model whose solution will provide the rate of
interest compounded continuously if a bank’s rate of interest is 10% per
annum.
2.29. The number of field mice in a certain pasture is given by the function
200−10t, where t is measured in years. Determine a differential equation
governing a population of owls that feed on the mice if the rate at which
the owl population grows is proportional to the difference between the
number of owls at the time t and the number of field mice at time t.
2.30. Let a dog start running to pursue a rabbit at time t0 when the dog sights
the rabbit. Determine a differential Equation (mathematical model)
whose solution will give the path of pursuit assuming that the rabbit
runs in a straight line at a constant speed so that its line of sight is
always directed at the rabbit.
2.31. To save money the manager of a manufacturing firm decides to eliminate
the advertising budget. In the absence of advertising, the sales manager
finds that sale in Indian rupees, decline at a rate that is directly propor-
tional to the volume of sales. Write a differential equation that describes
the rates of declining sales.
2.32. Suppose you deposited 10,000 Indian rupees in a bank account at an
interest rate of 5% compounded continuously. Write a mathematical
model in terms of a differential equation whose solution will give the
amount of money in your account after a year and a half.
Differential Equations 241
t ≥ 0, N (0) = N0 .
2.38. It was estimated that the earth’s human population in 1961 was 3.060
billion. Assume the population in 1996 using a model of population
growth (2.8), check this number with the actual population of the earth
based on available from authentic sources.
2.39. A breeder reactor converts relatively stable uranium 238 into the iso-
tope plutonium 239. After 30 years 0.022% of the initial amount N0 of
plutonium has disintegrated. Find the half life of this isotope if the rate
of disintegration is proportional to amount remaining.
2.40. The radioactive Pb−209 isotope of lead decays at a rate proportional to
the amount present at time t and has a half life of 4 hours. If 1 gram
of lead is present initially, how long will it take for 80% of the lead to
decay?
2.41. In the 1950 excavation at Nippur in Babylonia, charcoal from a roof
beam gave a count of 4.09 disintegrations per minute per gram. Liv-
ing wood gave 6.68 disintegrations. Assuming that this charcoal was
formed during Hammurabi’s reign, find an estimate for the likely time
Hammurabi’s succession.
242 Modern Engineering Mathematics
2.42. Suppose a large mixing tank initially holds 300 gallons of water in which
50 pounds of salt has been dissolved. Pure water is pumped into the tank
at a rate of 3 gal/min, and when the solution is well stirred it is pumped
out at the same rate. Write a differential equation for the amount A(t)
of salt in the tank at any time t.
2.43. A spherical rain drop evaporates at a rate proportional to its surface
area. Write a differential equation for its volume V as a function of
time.
2.44. A chemical A in a solution breaks down to form chemical B at a rate pro-
portional to the concentration of unconverted A. Half of A is converted
in 20 minutes. Write a differential equation describing this physical sit-
uation.
2.45. A tank with a capacity of 600 liters initially contains 200 liters of pure
water. A solution containing 3 kilograms of salt per liter is allowed to
run into the tank at a rate of 16 liters per minute. The mixture is then
removed at a rate of 12 liters per minute. Find the expression for the
number of kilograms of salt in the tank at any time t.
2.46. A large tank is filled with 600 liters of pure water. Brine containing 2
kilograms of salt per liter is pumped into the tank at a rate of 5 liters
per minute. The well mixed solution is pumped out at the same rate.
Find the number P (t) of kilograms of salt in the tank at time t. What
is the concentration of the solution in the tank at t = 10 minutes?
2.47. A 250 liter tank contains 100 liters of pure water. Brine containing 4
kilograms of salt per liter flows into the tank at a rate of 5 liters per
minute. If the well stirred mixture flows out at a rate of 3 liters per hour,
find the concentration of salt in the tank at the instant it is filled to the
top.
2.48. A thermometer reading 100◦ F is placed in a pan of oil maintained at
10◦ F . What is the temperature of the thermometer when t = 20 seconds,
if its temperature is 60◦ F when t = 8 seconds?
2.49. A thermometer is removed from a room where the air temperature is
60◦ F and is taken outside, where the temperature is 55◦ F . After 1
minute the thermometer reads 50◦ F . What is the reading of the ther-
mometer at t = 2 minutes? How long will it take for the thermometer
to reach 20◦ F .
2.50. Water is heated at 120◦ F . It is then removed from the burner and kept
in a room of 30◦ F temperature. Assuming that there is no change in the
temperature of the room and the temperature of the hot water is 110◦ F
after 3 minutes:
(a) Find the temperature of water after 6 minutes.
Differential Equations 243
(b) Find the duration in which water will cool down to the room tem-
perature.
2.51. The diagram in Figure 2.3 represents an electric circuit in which voltage
of V volts is applied to a resistance of R ohms and an inductance of
L henrys connected in series.When the switch is closed, a current will
vary with time, and it can be shown that a mathematical model for this
circuit is the first order differential equation
dI
L + IR = V.
dt
Verify that the current in the circuit is given by
V
I= (1 − e−Rt/L ).
R
(j) t2 e−2t
(k) et cos t
t, 0 ≤ t < 1
(l) f (t) =
1, t ≥ 1
(m) t(t − 2)e3t
(n) t3 − sinh 2t
(o) e−2t + 4et
1, 0 ≤ t < 2
(p) f (t) =
2, t ≥ 2
sin t, 0 ≤ t ≤ π
(q) g(t) =
0, t≥π
t
e, 0≤t<2
(r) f (t) =
0, t ≥ 2
1, 0 ≤ t < 2
(s) L(t) = 2, 2 ≤ t < 4
0, t ≥ 4
2.59. Use the first shifting theorem to solve the initial value problems.
y 00 − 6y 0 + 9y = t2 e3t .
dn
L{tn f (t)} = (−1)n F (s).
dsn
Power series
2.70. Write ex cos x in the form of a power series. Examine whether this power
series is convergent.
Solution about ordinary point
Find the general solution of the following differential equations about an or-
dinary point in terms of two power series.
2.71. y 00 − (1 + x)y = 0.
2.72. y 00 + x2 y 0 .
2.73. y 00 + y = ex .
249
250 Bibliography
3.1 Introduction
In the calculus course (Appendix B) we study properties of functions de-
fined on R (the line), R2 (the plane) or R3 (the space) with values in R, which
are called real valued or scalar functions, or scalar fields. Here, we study the
calculus of functions taking values in R2 or R3 , instead of R. Those functions
are called vector valued functions or vector fields. In Section 3.2 the concept
of a vector will be introduced along with its basic algebraic properties. Vec-
tor fields and their continuity and differentiation properties are discussed in
Section 3.3 along with the notions of gradient, divergence and curl. Moreover,
251
252 Modern Engineering Mathematics
we explain how curves and surfaces are described by such functions. Integrals
of vector fields are introduced in Section 3.4, first the line integral for scalar
and vector fields, and then the surface integrals for scalar fields.
In Section 3.5, we present three fundamental theorems of vector calculus,
namely the Green-Ostrogradski theorem, the Gauss divergence theorem and
the theorem of Stokes. Section 3.6 is devoted to certain applications of vec-
tor calculus to science and engineering, with an emphasis on problems from
mechanics.
The concept of a vector can be traced to the development of affine and
analytic geometry in the 17th century and, later, the invention of complex
numbers and quaternions. Vector calculus was developed mainly during the
second half of the 19th century; contributions stem from, among others, Sir
William Rowan Hamilton (1805-1865), William Kingdon Clifford (1845-1879),
James Clerk Maxewell (1831-1879), Hermann Gunter Grassman (1809-1877)
and Josiah Willard Gibbs (1839-1903). However, the fundamental results men-
tioned above were already discovered by George Green (1793-1841), Carl
Friedrich Gauss (1777-1855), Gabriel Stokes (1819-1903) and Mikhail Ostro-
gradski (1801-1862).
Vector calculus now serves as a basic mathematical tool in all areas of
science and engineering where mechanical, electromagnetic and thermody-
namics forces determine the behaviours of solids, fluids, electric conductors,
semiconductors and magnetic materials. Many ideas of this chapter are based
on references [7, 8]: 7 is out of print.
3.2 Vectors
Vectors and their basic properties have been introduced in Section 3.1.
In this section we discuss the concepts of magnitude of a vector and angle
between two vectors. The magnitude of a vector is called the norm. The
angle between two vectors is defined with the help of the inner product of two
vectors.
Definition 53. The inner product of two vectors x = (x1 , x2 , x3 ) and
y = (y1 , y2 , y3 ) denoted by x.y or (x, y) is defined by x.y = x1 y1 + x2 y2 + x3 y3 .
Position vector (2-space and 3-space)
The vector shown in Figure 3.1b with initial point the origin O and terminal
P (x1 , x2 ) is called the position vector of the point P and is written
−−→
OP = (x1 , x2 ).
Figure 3.2 depicts the position velocity in 3-space with the origin O and
terminal point P (x1 , x2 , x3 ) of the point and is written (x1 , x2 , x3 ). Addition
Vector Calculus 253
X
•
x + y = x + y, (x + y) + z = x + (y + z)
Definition 54. The norm has the following properties, which are analogous
to the properties of the absolute value of a real number.
Properties (3.1) and (3.2) can be checked immediately from the definition.
It will be seen below that property (3.3) is a consequence of the Schwarz
inequality. Setting α = −1 in property (3.2), we obtain
k −x k=k x k (3.4)
(they obviously satisfy k i k=k j k=k k k= 1) are also called unit coordinate
vectors or standard unit vectors, see Figure 3.3. Every vector can be
expressed as a linear combination of the unit coordinate vectors. For x =
(x1 , x2 , x3 ), we have
x = x1 i + x2 j + x3 k.
Vector Calculus 255
z axis
y axis
j
x axis
Indeed, from the rules of addition and scalar multiplication we see that
x +y
|x.y|
k Py (x) k= , (3.12)
kyk
since
x.y x.y |x.y| |x.y|
k Py (x) k=k y k= | | k y k= k y k= .
y.y y.y k y k2 kyk
In the special case y is a unit vector, k y k= 1 and (3.11) becomes
A A
A
x.y (x.y)2
= (y.x) − (y.y) = 0.
y.y (y.y)2
We now relate the projection vector to the angle θ in Figure3.5. There θ is an
acute angle (less than 90 degrees), and we have by (3.12)
k Py (x) k |x.y|
cos θ = = .
kxk k y kk x k
If the angle θ is larger than 90 degrees, then the cosine and the scalar product
x.y become negative. (The reader is urged to draw a picture similar to Figure
3.5 for this situation.) The general formula relating the angle θ between x
and y to the scalar product is
x.y
cos θ = . (3.15)
k x kk y k
Example 176. Find the angle between the vectors x = (2,3,2) and y =
(1,2,-1).
Solution: We determine the angle from formula (3.15) above. We have x.y =
2.1 + 3.2 + 2.(−1) = 6 and
√ √ p √
k x k= 4 + 9 + 14 = 17, k y k= 1 + 22 + (−1)2 = 6.
Thus
x.y 6 1√ 10.1
cos θ = =√ √ = 102 ≈ ≈ 0.594,
k x kk y k 17 6 17 17
and θ ≈ 0.935 radians, which is about 54 degrees. Since the cosine ranges
between −1 and 1, we immediately get from (3.15) the Schwarz inequality
(or Cauchy-Schwarz or Cauchy-Bunyakovski-Schwarz inequality)
|x.y| ≤k x kk y k, (3.16)
which is valid for any two vectors x and y. (Let us remark that one can
prove this inequality directly from the properties of the scalar product, with-
out recourse to the geometric construction used above.) Using the Schwarz
Vector Calculus 259
inequality, we may now prove the triangle inequality. Indeed, for two vectors
x and y we have
k x + y k≤k x k + k y k .
Actually, the expression between the two equality signs is not a true determi-
nant, since the first line consists of vectors, not of numbers, but it is a conve-
nient way to memorize (3.17). The standard rule for computing the three 2×2
determinants on the right hand side gives (3.17). Another way to memorize
(3.17) is to observe the cyclic behaviours of the indices.
260 Modern Engineering Mathematics
D
c
IIY[[
ll vll sin a
length of x = II x 11
length of y = 11y 11
A 8
ll xll
Moreover, x×y is orthogonal to both x and y. Finally, from the two remaining
possibilities the direction of x × y is chosen according to the so-called right-
hand rule: Point the index finger in the direction of x and the middle finger
in the direction of y. The thumb then points in the direction of x × y. Below
and in the exercises we will show that these geometric properties follow from
Definition 3.2.6, if the coordinate system has a right hand orientation (index
finger points to i, middle finger points to j, thumb points to k.)
Algebraic properties of cross product
In contrast to the scalar product, the cross product is anticommutative,
that is,
k y k × k x k= − k x k × k y k (3.19)
holds for all vectors x and y. As a consequence, for all vectors x we have
x × x = 0. (3.20)
On the other hand, the cross product shares some properties of the scalar
product. The distributive laws
Vector Calculus 261
x × (y + z) = x × y + x × z (3.21)
(x + y) × z = x × z + y × z (3.22)
(αx) × y = α(x × y) = x × (×y) (3.23)
hold for all vectors x, y and z and all scalars α. In particular, setting α = 0
we have for every vector x
x × 0 = 0 = 0 × x.
The properties (3.19) through (3.21) can be verified directly from the definition
of the cross product.
Example 178. (a) Calculate x × y where x = (1, −1, 3) and y = (2, 1, −1).
(b) Show that i × j = k, j × k = i and k × i = j.
(c) Compute i × (i × j) and (i × i) × j.
(d) Compute x × y where x = i − j and y = i + k.
Solution:(a) We have
i j k
−1 3 1 3 1 −1
x × y = 1 −1 3 = i − j + k = −2i + 7j + 3k.
2 1 −1 1 −1 2 −1 2 1
so we have shown that i×j = k. Similarly we can prove the other two identities.
(c) We obtain
i × (i × j) = i × k = −j, (i × i) × j = 0 × j = 0.
Alternatively we may use part (b) above with properties (3.19) through (3.21)
to compute
x × y = (i − j) × (i + k) = i × i + i × k − j × i − j × k = −j + k − i.
(x × y) × z = x × (y × z).
262 Modern Engineering Mathematics
Thus, the cross product is not associative. We come back to formula (3.18),
k x × y k=k x kk y k sin θ. From formula (3.15) we see that
k x × y k2 =k x k2 k y k2 −(x.y)2 . (3.24)
Therefore, formula (3.18) for the magnitude of the cross product follows from
(3.25).
Scalar triple product.
If x, y and z are vectors, the expression (x × y).z is called the scalar triple
product of x, y and z. Its absolute value |(x × y).z| represents the volume of
the parallelogram with edges x, y and z.
Example 179. Show that the scalar triple product satisfies
x1 x2 x3
(x × y).z = y1 y2 y3 .
z1 z2 z3
Solution:
x2 x3 x1 x3 x1 x2
(x × y).z = y2 y3 i − y1 y3 j + y1 y2 k .(z1 i + z2 j + z3 k))
x2 x3 x1 x3 x1 x2
= z1 − z2
+ z3
.
y2 y3 y1 y3 y1 y2
Scalar and vector products often appear in physics and engineering. Work is
the scalar product of force and displacement. Torque and angular momentum
are the cross products of force and displacement or force and linear momen-
tum. Maxwell’s equations, which provide the foundation of electromagnetic
theory, involve both scalar and cross products of electrical and magnetic vari-
ables.
Vector Calculus 263
Example 180. Let x = 2i−3j +4k and y = −i+2j +5k. Find (i) (2x).(x−2y)
x.y
(ii) ( y).
y.y
Solution: If these two vectors are orthogonal then their dot product (inner
product) is 0, that is,
1
(αi + j + αk)(−3i + 4j + αk) = 0
2
1
or − 3α + .4 + α2 = 0
2
or α2 − 3α + 2 = 0
or (α − 1)(α − 2) = 0.
and k d k= 100.
1
Then work done = W =k F kk d k cos θ = 20 (100) 2 = 1000 ft lb.
Example 184. Verify Lagrange’s identity
k x × y k2 =k x k2 k y k2 −(x.y)2
k x × y k2 =k x k2 k y k2 −(x.y)2 .
= x22 y32 +x23 y22 −2x2 y2 x3 y3 +x21 y32 +x23 y12 −2x1 y1 x3 y3 +x21 y22 +x22 y12 −2x1 x2 y1 y2 .
All those points (that is, the points in the range of F ) form a curve in 3-space;
if we think of t as a time variable, we may imagine an object moving along
this curve, passing the point F(t) at time t. The functions f1 , f2 , f3 are called
the component functions, or simply the components of F.
In general, let us consider a function F defined on some set S. Whenever the
values of F are vectors, the function F is called a vector field (or a vector
valued function, or a vector function). Its domain S may be an interval
as above, or it may be a subset of the plane R2 or the space R3 . In the latter
case, the vector field has the form
3.3.1 Curves
In this subsection we consider vector fields which are defined on an interval
I of the real line, as in (3.26). Such vector fields are commonly called curves,
since one imagines the points F(t) to form a curve in space as t varies in I.
F(t) = f (t)i + g(t)j, is called curves in parametric form in 2-space.
Definition 59. The vector parametric equation of a straight line in 3-space
266 Modern Engineering Mathematics
n.(γ − γ0 ) = 0 (3.31)
where P = (p1 , p2 , p3 ) has position vector γ. Thus the equation of the plane
having non-zero normal vector n = a1 i + a2 j + a3 k and passing through the
point P0 = (x1 i, x2 j, x3 k) with position vector γ0 , has equation n.(γ −γ0 ) = 0.
Its scalar form is
Example 187. (a) The equation 2p1 − 3p2 − 4p3 = 0 represents a plane that
passes through the origin and is normal to the vector (2, −3, −4) = 2i−3j−4k.
(b) Find an equation of the plane that passes through the three points P =
(1, 1, 0), Q = (0, 2, 1) and R = (3, 2, −1).
Solution: We are required to find the normal vector n to the plane in order
to write the equation of the plane (3.31). Such a vector will be perpendicular
−−→ −→
to the vectors P Q = −i + j + k and P R = 2i + j − k. Therefore, by a property,
of the cross product ( The vector cross product is orthogonal to each vector):
−−→ −→
n = P Q × P R == −2i + j − 3k
3.3.2 Distances
Definition 61. (a) (Distance from point to plane)
Let P0 = (x1 , x2 , x3 ) be a point in the plane P having equation a1 p1 + a2 p2 +
a3 p3 = d. Then the distance between P0 and the plane, denoted by s, is given
by
|a1 x1 + a2 x2 + a3 x3 − d|
s= 1 .
(a21 + a22 + a23 ) 2
Vector Calculus 267
k (γ0 − γ1 ) × v k
s= .
kvk
Example 188. (a) Find the distance from (1, −1, 2) to the plane x−y−z = 9.
(b) Find the distance from (1, 0, −1) to the line γ = i + (1 + 3t)j − (3 − 4t)k
parallel to v = 3j + 4k passing through P1 = (1, 1, −3).
|1 + 1 − 2 − 9| 9
Solution: (a) s = √ =√ .
1+1+1 3
|((1 − 1)i + (0 − 1)j) + (2 + 3)k × (3j + 4k)| |(−j + 5k) × (3j + k)|
(b) s = √ = .
2
3 +4 2 5
Example 189. (a) γ(t) = 2 cos ti + 2 sin tj + tk, t ≥ 0 is a vector equation
of circular helix.
(b) The parametric equation of the vector function γ(t) = 2 cos ti+2 sin tj +5k
is x = 2 cos t, y = 2 sin t, z = 5. Its geometrical representation is given in
Figure 3.7.
(c) The vector equation of the circle in 2-space with radius α and center (0,0)
is
(d) Graph of the curve traced by the following vector functions (vector fields)
(i) F (t) = cos ti + j + sin t + k, t ≥ 0.
(ii) F (t) = 4i + 2 cos tj + 3 sin tk.
(iii) z = x2 + y 2 , y = x, x = t.
Definition 62. Let F be a vector field defined on an interval I, let t0 ∈ I be
given. We say that the vector L is the limit of F as t tends to t0 , and we write
lim F (t) = L,
t→t0
Theorem 33. Let F (t) = f1 (t)i + f2 (t)j + f3 (t)k and L = l1i + l2j + l3k.
Then F (t) = L if and only if
0 ≤ | k F (t) k − k L k | ≤k F (t) − L k .
For the converse choose F (t) = x and L = −x, where x is any fixed non-zero
vector. We have limt→t0 k F (t) k=k x k=k L k, but limt→t0 F (t) = x 6= −x =
L. The usual properties of the limit can be extended for limits of vector fields.
Let F (t) → L, G(t) → M and α(t) → A as t → t0 . Then
carried out component by component, that is, if F (t) = f1 (t)i+f2 (t)j +f3 (t)k
is differentiable at t, then
F (t + h) − F (t)
F 0 (t) = lim
h→0 h
f1 (t + h) − f1 (t) f2 (t + h) − f2 (t) f3 (t + h) − f2 (t)
= lim [ i+ j+ k]
h→0 h h h
f1 (t + h) − f1 (t) f2 (t + h) − f2 (t) f3 (t + h) − f3 (t)
= [ lim ]i+[ lim ]j +[ lim ]k
h→0 h h→0 h h→0 h
= f10 (t)i + f20 (t)j + f30 (t)k.
to vector fields. Let F and G be two vector fields defined on an interval I, and
let α, β be scalars. We define
(F ◦ u)(t) = F (u(t))
for a scalar function u whose range is contained in I. The following rules hold
for differentiation.
(F + G)0 (t) = F 0 (t) + G0 (t) (3.42)
(αF )0 (t) = αF 0 (t) for constants α, (3.43)
(vF )0 (t) = v(t)F 0 (t) + v 0 (t)F (t), (3.44)
0 0 0
(F.G) (t) = F (t).G (t) + F (t).G(t), (3.45)
and the chain rule for vector functions
The formulas (3.42) through (3.46) are consequences of the definitions, the
componentwise formula (3.36) and the corresponding differentiation rules for
scalar functions . They can also be written in Leibniz notation, for example
d dF dG
(F + G) = + ,
dt dt dt
d dG dF
= (F. )+( .G),
dt(F.G) dt dt
d dG dF
(F × G) = (F × )+( × G).
dt dt dt
1 3
Example 190. Let F (t) = 2t2 i − 3j, G(t) = i + tj + t2 k, u(t) = t . Verify
3
(3.45)- (3.46) for these functions.
Vector Calculus 271
F 0 (t) = f10 (t)i + f20 (t)j + f30 (t)k = (t cos t + sin t)i − e−t j + k,
F 00 (t) = (−t sin t + cos t + cos t)i + e−t j + 0k = (2 cos t − t sin t)i + e−t j.
Let us note that the definition of an open set in the plane is a special case
of Definition 1.3.6.
Gradient.
Let f be a scalar function of three variables. The gradient of f at a point
(x, y, z) is defined as the vector
∂f ∂f ∂f
∇f (x, y, z) = (x, y, z)i + (x, y, z)j + (x, y, z)k. (3.47)
∂x ∂y ∂z
If f and its partial derivatives are defined on some open subset D of R3 , the
gradient thus becomes a vector field ∇f : D → R3 whose component func-
∂f ∂f ∂f
tions are just the partial derivatives , , . According to the notation
∂x ∂y ∂z
of vectors, we may also write
∂f ∂f ∂f
∇f (x, y, z) = ( (x, y, z), (x, y, z), (x, y, z), ).
∂x ∂y ∂z
∂f ∂f ∂f
(x, y, z) = 2xyz sin z, (x, y, z) = x2 sin z, (x, y, z) = x2 y cos z,
∂x ∂y ∂z
Vector Calculus 273
∂f ∂f
∇f (x, y) = (x, y)i + (x, y)j.
∂x ∂y
In dimensions, we define
r(h)
for some remainder term r(h) with → 0 as h → 0. For functions of
h
several variables, the gradient plays the corresponding role.
|f (x + h) − f (x) − ∇f (x).h|
lim = 0. (3.53)
h→0 khk
r(h)
where r(h) is a remainder term with → 0. For this reason, the function
khk
g defined by
g(z) = f (x) + ∇f (x)(z − x)
is called the linearization of f at x. Note that the difference
g(x + h) − f (x + h) = r(h)
274 Modern Engineering Mathematics
r(h) = o(k h k)
f (x + tu) − f (x)
fu0 (x) = lim ,
t→0 t
if it exists, is called the directional derivative of f at x in the direction u.
Instead of fu0 (x), we also write ∂u f (x).
Remark 37. (a) When u = ei is the ith canonical unit vector, the directional
derivative fu0 becomes the partial derivative ∂i f. In particular, for a function
f of three variables (x, y, z), we obtain
∂f
(x, y, z) = fi0 (x, y, z) = ∂x f (x, y, z), i = (1, 0, 0),
∂x
and similarly
∂f ∂f
= fj0 = ∂y f and = fk0 = ∂z f.
∂y ∂z
∂f ∂f ∂f
(b) As we know already the partial derivatives , and give the rates
∂x ∂y ∂z
of change of f in the directions i, j and k respectively. Analogously, if k u k
is a unit vector, the directional derivative fu0 (x) gives the rate of change of f
in the direction u.
(c) The geometrical interpretation of the directional derivative is essentially
the same as that of the partial derivative, except that we now look at tan-
gents to the graph of f in arbitrary directions u, not only in the direction of
the coordinate axes. The following theorem gives the connection between the
gradient of f at x and its directional derivative at x.
Vector Calculus 275
If moreover u is a unit vector and ∇f (x) is non-zero, we get from (3.55) that
Since the directional derivative gives the rate of change of the function in that
direction, it follows that the function f increases most rapidly in the direction
of the gradient and decreases most rapidly in the opposite direction.
Example 192. Let f (x, y, z) = xyz. Compute ∇f (1, 1, 1) and determine the
maximum and minimum rates of change of f at (1,1,1).
276 Modern Engineering Mathematics
Solution: We have
∂f ∂f ∂f
(x, y, z) = yz, (x, y, z) = xz, (x, y, z) = xy.
∂x ∂y ∂z
Therefore
∇f (x, y, z) = yzi + xzj + xyk = (yz, xz, xy),
and hence ∇f (x, y, z) = i + j + k = (1, 1, 1). According to (3.56) and√(3.57),
the maximum and √ minimum rates of changes are ∇f (1, 1, 1) = 3 and
−∇f (1, 1, 1) = − 3 respectively.
Example 193. Calculate the directional derivative of φ(x, y, z) = 8xy 2 − xz
at an arbitrary point (x, y, z) in the direction of the vector
1 1 1 1 1 1
u = √ i + √ j + √ k = ( √ , √ , √ ).
3 3 3 3 3 3
Solution: According to Theorem 21, we have
We compute
∂ϕ ∂ϕ ∂ϕ
(x, y, z) = 8y 2 − z, (x, y, z) = 16xy (x, y, z) = −x,
∂x ∂y ∂z
1 1 1 8y 2 + 16xy − x
∂u ϕ(x, y, z) = (8y 2 − z, 16xy, −x).( √ , √ , √ ) = √ .
3 3 3 3
Example 194. Let the temperature at a point (x, y) on a metallic plate in
xy
the xy-plane be given by T (x, y) = degrees Celsius. (a) Find the
1 + x2 + y 2
rate of change of temperature at (1,1) in the direction of u = 2i − j.
(b) An insect at (1,1) wants to walk in the direction in which the temperature
drops most rapidly. Find a unit vector in that direction.
∂T y(1 − x2 + y 2 ) ∂T x(1 + x2 − y 2 )
(x, y) = , (x, y) =
∂x (1 + x2 + y 2 )2 ∂y (1 + x2 + y 2 )2
i j 1 1
∇T (1, 1) = + = ( , ).
9 9 9 9
Vector Calculus 277
The directional derivative gives the correct rate of change only after we nor-
u (2i − j)
malize the vector u to unit length, so we set e = = √ and compute
kuk 5
the rate of change as
i j (2i − j) 1
∂e T (1, 1) = ∇T (1, 1).e = ( + ). √ = √ .
9 9 5 9 5
(b) The temperature drops most rapidly in the direction of of −∇T (1, 1).
Using part (a), we compute a unit vector e in that direction as
√
1 1 2
−∇T (1, 1) = − i − j, k −∇T (1, 1) = k,
9 9 9
so
−∇T (1, 1) 1 1
e= = − i − − j.
k −∇T (1, 1) k 2 2
Example 195. Let the temperature at each point of a metal plate be given
by the function
T (xy) = ex cos y + ey cos x.
(a) In what direction does the temperature increase most rapidly at the point
(0,0)? What is the rate of increase?
(b) In what direction does the temperature decrease most rapidly at (0,0)?
Solution: We first compute
∂T ∂T
∇T (x, y) = (x, y)i + (x, y)j
∂x ∂y
= (ex cos y − ey sin x)i + ey (cos x − ex sin y)j.
(a) At (0,0) the temperature increases most rapidly in the direction of the
∇T (0, 0) = i + j. The rate of increase is obtained as k ∇T (0, 0) k=k
gradient √
i + j k= 2.
(b) The temperature decreases most rapidly in the direction of −∇T (0, 0) =
−i − j.
We repeat that the gradient vector ∇f (x) tells us the direction of the steepest
climb of f at the point x, and its length,∇f (x), gives the steepness.
Divergence and rotation.
Here we present the definition and some properties of the differential expres-
sions termed divergence and curl or rotation.
Definition 67. Let F (x, y, z) = f1 (x, y, z)i + f2 (x, y, z)j + f3 (x, y, z)k be a
vector field with components f1 , f2 , f3 defined on some open subset D of R3
which we assume to possess partial derivatives.
(a) The divergence of F , denoted by divF or ∇.F (read as nabla dot F), is
defined as
∂f1 ∂f2 ∂f3
divF = + + . (3.58)
∂x ∂y ∂z
278 Modern Engineering Mathematics
∂f2 ∂f1
( − ) = ey − 2x,
∂x ∂y
and obtain
Example 196. (i) Let ϕ be a scalar field which possesses continuous first and
second partial derivatives. Show that ∇ × (∇ϕ) = 0 or curl(∇ϕ) = 0, that is,
the curl of the gradient of ϕ is identically equal to the zero vector field.
Vector Calculus 279
This means that the divergence of the curl of F is identically equal to the zero
scalar field.
Solution: (i) In order to compute the curl of ∇ϕ, we apply (3.61) to the
∂ϕ ∂ϕ ∂ϕ
vector field F = ∇ϕ, that is, f1 = , f2 = , f3 = . Inserting those
∂x ∂y ∂z
expressions into (3.63) we obtain
∂ 2 f3 ∂ 2 f2 ∂ 2 f1 ∂ 2 f3 ∂ 2 f2 ∂ 2 f1
= − + − + −
∂x∂y ∂x∂z ∂y∂z ∂y∂x ∂z∂x ∂z∂y
∂ 2 f3 ∂ 2 f3 ∂ 2 f2 ∂ 2 f2 ∂ 2 f1 ∂ 2 f1 ∂ 2 f1
=[ − ]+[ − ]+[ − − ]
∂x∂y ∂y∂x ∂z∂x ∂x∂z ∂y∂z ∂y∂z ∂z∂y
because for the same reason as in (i),
∂ 2 f3 ∂ 2 f3 ∂ 2 f2 ∂ 2 f2 ∂ 2 f1 ∂ 2 f1
= , = , = .
∂x∂y ∂y∂x ∂z∂x ∂x∂z ∂y∂z ∂z∂y
Example 197. Consider the vector field r(x, y, z) = xi+yj +zk (the identity
mapping on R3 ). We let r =k r k, that is, we also consider the scalar field r
given by r(x, y, z) =k r(x, y, z) k.
(a) Let n be any integer (positive or negative). Prove that ∇(rn ) = nrn−2 r.
(b) Let ϕ be a real valued function of one variable. Prove that curl(ϕ(r)r = 0).
280 Modern Engineering Mathematics
1
Solution: (a) We have r(x, y, z) =k r(x, y, z) k= (x2 + y 2 + z 2 ) 2 , so
n
rn (x, y, z) = (x2 + y 2 + z 2 ) 2 . This gives
∂(rn ) n n
(x, y, z) = (x2 + y 2 + z 2 ) 2 −1 .2x = nxrn−2 (x, y, z),
∂x 2
and similarly
∂(rn ) ∂(rn )
= nyrn−2 , = nzrn−2 .
∂y ∂z
Thus, in abbreviated notation,
∂rn ∂rn ∂rn
∇rn = i+ j+ k = nxrn−2 i + nyrn−2 j + nzrn−2 k
∂x ∂y ∂z
Example 198. Consider the vertical cylinder with height H and a circular
base of radius R.
Find the plane tangent to the side surface at the point P0 = (( √12 )R, ( √12 )R, 1).
span the tangent plane at P0 = (x0 , y0 , S(x0 , y0 )). Note that the parametriza-
tion e.g., (3.60) is regular, since the vectors in (3.61) are not parallel. The
vector
the tangent plane to Σ at P0 is equal to the set of all points (x, y, z) which
satisfy the equation
or alternatively
Indeed, a limit passage like the one expanded in calculus (based on an approxi-
mation of the rod consisting of straight pieces) shows that (3.69) is the correct
formula to compute the total mass of the rod. This motivates the following
definition.
Definition 69. (Line integral of scalar field) Let C be a smooth curve
given by (3.68) and let f be a continuous scalar field whose domain contains
284 Modern Engineering Mathematics
R
the curve C. Then the line integral of f over C is denoted by C
f ds:
Z Z b
f ds = f (r(t)) k r0 (t) k dt. (3.70)
C a
In the special case f = 1, the line integral (3.70) equals the length of the curve
C. Therefore, the function
Z tp Z t
s(t) = x02 + y 02 + z 02 dτ = k r0 (τ ) k (3.71)
0 o
Vector Calculus 285
yields the length of C from the initial point r(a) up to the point r(t) and is
called the arc length of the curve C; compare for the corresponding situation
in two dimensions. From (3.71) we see that s0 (t) =k r0 (t) k. This explains the
notation C f ds where ds stands for s0 (t)dt or k r0 (t) k dt. Accordingly, ds is
R
Example 200. Let the curve C be given by r(t) R = (cos t)i − (sin t)j + tk
on the interval 0 ≤ t ≤ π. Evaluate the integral C F.dr where F (x, y, z) =
yi − xj + 2k.
Solution: We use formula (3.73) or (3.74). In order to form F (r(t)), we have to
insert cos t for x, -sin t for y and t for z. This gives F (r(t)) = (− sin t, − cos t, 2).
Moreover, r0 (t) = (− sin t, − cos t, 1). Computing the scalar product of those
two vectors and integrating, we get
Z Z π Z π
F.dr = F (r(t)).r0 (t)dt = (sin2 t + cos2 t + 2)dt = π + 2π = 3π.
C 0 0
R
The line integral F.dr is also frequently denoted as
Z
f1 (x, y, z)dx + f2 (x, y, z)dy + f3 (x, y, z)dz, (3.75)
C
or in shorter form as Z
f1 dx + f2 dy + f3 dz.
C
The definition of those expressions is, of course, the same as above in (3.73)
or (3.74). If we formally replace x by x(t) and dx by x0 (t)dt (and analogously
for y and z) in (3.75), we arrive at (3.74).
Example 201. Evaluate xydx − y cos xdy, where C is given by x(t) = t2
R
t5
Z
1
= 25 t2 t2t − t cos t2 dt = [2 ]− 25 − [sin t2 ]− 25
− 5 2
32 1
2(625 + ) − (sin 25 − sin 4)
5 2
6314 1
− (sin 25 − sin 4).
5 2
Example 202. Find the circulation of the field F (x, y) = (−y + x)i + xj
around the circle x2 + y 2 = 1.
Solution: The parametric equation of the given circle is
We have
F (r(t)) = (− sin t + cos t)i + cos tj, r0 (t) = − sin ti + cos tj,
Vector Calculus 287
F (r(t))r02 (t) − sin t cos t + cos2 (t) + cos2 (t) = 1 − sin t cos t.
Therefore, the circulation of F around the circle (denoted as C) becomes
Z 2 Z 2
sin2 t 2
I
0
F.r = ΠF (r(t)).r (t)dt = Π(1 − sin t cos t)dt = [t − ] Π.
C 0 0 2 0
The result of the following example will be used later in the proof of the
theorem of Green and Ostrogradski.
Example 203. Let C be the graph of a function y = h(x), where h : [a, b] →
R is differentiable and let F be a vector field whose second component is zero,
that is, F (x, y) = (f (x, y), 0) for some continuous real valued function f . Show
that Z Z b
F.dr = f (x, h(x))dx. (3.76)
C a
so (3.76) is proved. (Recall that it does not determine whether the integration
variable is denoted by t or by x.)
The following properties hold for the line integral. Let F and G be continuous
vector fields defined on some region in space containing a curve C, which is
given as the range of some vector function r as above. Then
Z Z Z
(F + G).dr = F.dr + G.dr. (3.77)
C C
Z Z
αF.dr = α F.dr, α being any scalar. (3.78)
C C
This means that the line integral is linear with respect to the vector fields,
that is, the line integral of the sum of two vector fields (the scalar multiple of a
vector field) is equal to the sum of the line integrals (the scalar multiple of the
line integral). These formulas are a direct consequence of the definition of the
line integral, using the corresponding properties of the ordinary integral. For
piecewise smooth curves, the line integral is evaluated on each piece separately,
as stated in the following definition.
Definition 71. (Line integral, piecewise smooth curves) Let C be a piecewise
smooth curve given by (3.68) and let F be a continuous vector field of the
form (3.72) whose domain contains the curve C. Then the line integral of F
over C is defined by
Z Xn Z
F.dr = i − 1ti F (r(t)).r0 (t)dt, (3.79)
C 1 t
288 Modern Engineering Mathematics
The proof will be given in Appendix C. Let us take a look at formula (3.80).
Given the potential ψ , the value of the right hand side depends only on the
points A and B. This means in particular that the value of the line integral on
the left hand side does not change if we replace C by a different curve C̃, as
long as C̃ has the same initial and terminal points as C. This property is called
path independence. Theorem 36 thus states that for conservative vector fields,
line integrals are path independent. We present another related definition.
Definition 73. A continuous vector field F defined on an open region D in
R3 is called circulation free on D if we have
I
F.dr = 0 (3.81)
C
for every closed curve C in D. When the curve C is closed, its initial and termi-
nal points coincide, and the right hand side of (3.80) becomes zero. Therefore,
a conservative field is circulation free by Theorem 36. Actually, the converse
is true, too, so the following theorem holds.
Theorem 37. (Closed loop property of conservative fields) Let F be a con-
tinuous vector field defined on an open region D in R3 . Assume that D is
connected, that is, any two points in C can be connected by a smooth curve.
The following statements are equivalent:
1. F is conservative on D.
2. F is circulation free in D.
The proof will be given in Appendix C. Another way of stating Theorem
37 would be to say that a vector field F possesses a potential in D if and only
if the line integral over any closed curve in D is zero.
Vector Calculus 289
Example 204. Let ψ(x, y, z) = xyz, let C be a smooth R curve with initial
point (−1, 6, 18) and terminal point (2, 12, −8). Find C F.dr, where F = ∇ψ.
Solution: Setting A = (−1, 6, 18) and B = (2, 12, −8) in Theorem 43, we get
Z
F.dr = ψ(B) − ψ(A) = 2.12.(−8) − (−1).6.18 = −192 + 108 = 84.
C
Since r is regular, the integral on the right hand side is well-defined as the
limit of the corresponding Riemannian sums. Those sums can be interpreted as
the area of small parallelograms which approximate corresponding portions of
Σ. (We do not carry out the details, which in fact would be rather cumbersome
to do). Thus, (3.82) is a natural definition of the area of an arbitrary curved
surface. The most important special case arises when Σ is the graph of a
function S.
Theorem 38. Let Σ be a smooth surface given as z = S(x, y), where S is a
continuously differentiable function defined on some domain D of the plane.
Then we have
Z Z q
A(Σ) = 1 + ∂x S(x, y)2 ∂y S(x, y)2 dA. (3.83)
D
290 Modern Engineering Mathematics
RemarkR 38.R (i) In the special case f = 1 we obtain the area of the surface,
A(Σ) = Σ
1dσ, as we can see from the definitions.
(ii) For a given surface Σ, it is possible to choose different parametrization r
of it. One can prove that this does not change the value of the area resp. of
the surface integral.
(iii) Let a surface Σ consist of smooth components Σ1 , Σ2 , ...Σn which are
mutually disjoint or intersect each other only in a set of zero area, for exam-
ple along a curve. Such a surface is called piecewise smooth. The surface
integral over Σ is then defined as
Z Z Z Z Z Z
f dσ = f dσ + ....... + f dσ. (3.86)
Σ Σ1 Σn
For example, the boundary Σ of a cube is not smooth (the normal vector
jumps across its edges), but it is piecewise smooth, since it consists of six
square pieces which are smooth. p
(iv) The formal expression dσ = 1 + ∂x S(x, y)2 + ∂y S(x, y)2 dA. is often
called the surface element.
Vector Calculus 291
√ Z 2
128 √
Z 4
= 2 dx. 2. y 2 dy =
0 0 3
f dσ where f (x, y, z) = z 2 ,
RR
Example 206. Evaluate the surface integral Σ p
and Σ is the portion of the boundary surface of the vertical cone z = x2 + y 2
which lies between the planes z = 1 and z = 2.
Solution: We present two different approaches. For the first one, we use a
parametrization adapted to the cone, namely
We get
(∂u r × ∂v )r(u, v) = (0 − u cos v)i − (−u sin v + 0)j + (u cos2 v + u sin2 v)k
= −u cos vi + u sin vj + uk
and moreover
p √ √
k ∂u r × ∂v )r(u, v) k= u2 cos2 v + u2 sin2 v + u2 = 2u2 = u 2.
This yields Z Z Z Z √
f dσ = (x2 + y 2 ) 2dA.
D D
Since D is an annular region, it is best to evaluate this two-dimensional integral
in polar coordinates. Using well known results of polar coordinates, we get with
G = (r, θ) : 1 ≤ r ≤ 2, 0 ≤ θ ≤ 2Π that
Z Z Z Z √ √ Z Z
f dσ = 2 2
(x + y ) 2dA = 2 (r2 cosθ +r2 sin2 θ)rdA
Σ D G
√ RR 3
= 2 r drdθ which is the same as above.
f dσ, where f (x, y, z) = x2
RR
Example 207. Evaluate the surface integral
and Σ is the upper half of the sphere x + y + z = a2 .
2 2 2
p
Solution: First we apply Theorem 39. We have S(x, y) = a2 − x2 − y 2
and
∂S x ∂S y
(x, y) = − p , (x, y) = − p .
∂x 2 2
a −x −y 2 ∂y a − x2 − y 2
2
Z 2 Z a
r3
a. Π cos2 θdθ. √ dr.
0 0 a2 − r2
The rightmost integral can be evaluated with the substitution r = a sin t,
dr = a cos tdt, and (we omit the details)
Π
a
r3
Z Z 2 2 3
√ dr = a3 sin3 tdt = a .
0 a2 − r 2 0 3
Vector Calculus 293
We finally obtain
Z Z Z 2
2 2
f dσ = a Π cos2 θdθ. a3 = Πa4 .
Σ 0 3 3
Since f (x, y, z) = x2 , we have f (r(u, v)) = a2 sin2 u cos2 v, and from (3.84) we
get Z Z Z Z Z
f dσ = a2 sin2 u cos2 v.a2 sin udA
D
Π
Z Z 2
4
2
3 2 4
=a sin udu. Π cos2 vdv = Πa
0 0 3
as before.
Remark 39. (i) The theorem relates a line integral, which is one-dimensional,
to an integral over a two-dimensional region.
(ii) The theorem is used for theoretical purposes as well as for the computation
of specific integrals. In particular it may happen that one of the integrals is
much easier to calculate directly than the other one.
Example 208. (a) A particle moves counterclockwise once around the tri-
angle D with vertices (0,0), (4,0) and (1,6), under the influence of the force
F (x, y) = xyi+xj. Calculate the work done by this force, if the units of length
and force are Hmeters and Newtons, respectively.
(b) Evaluate F.dr, where F (x, y) = (x2 − y)i + (cos2 y − e3y + 4x)j and C
is (the boundary of) any square with sides of length 5. Assume C is oriented
counterclockwise.
Solution: (a) Let us denote by C the curve formed by the three sides of the
Vector Calculus 295
H
triangle. The total work done by the force F equals the line integral F.dr.
∂f ∂g
We have F = (f, g) with f (x, y) = xy, g(x, y) = x, so = x and = 1.
∂y ∂x
From (40) we know that
I Z Z Z Z
∂g ∂f
F.dr = ( − (x, y))dA = (1 − x)dA.
D ∂x ∂y D
Example H 209. (a) Use the Green-Ostrogradski theorem to evaluate the line
integral y 2 dx + x2 dy, where C is the boundary of the square with vertices
(0,0), (1,0), (0,1) and (1,1), oriented counterclockwise. Check the answer by
evaluating the line integral directly.
(b) Do the same for xydx + (y + x)dy, where C is the unit circle x2 + y 2 = 1,
H
oriented counterclockwise.
(c) Verify the validity of the Green-Ostrogradski theorem for the vector field
F = 2yi − xj and the curve C taken as the circle of radius 4 with center (1,3).
Solution: (a) Here f (x, y) = y 2 , g(x, y) = x2 , so ∂f ∂g
∂y = 2y and ∂x = 2x,
moreover D = [0, 1] × [0, 1]. Using (3.87) we get
I Z Z Z Z
∂g ∂f
f (x, y)dx + g(x, y)dy = ( − )(x, y)dA = 2x − 2ydydx
C D ∂x ∂y D
Z 1Z 1
= 2x − 2ydydx = 1 − 1 = 0. (3.88)
0 0
Analogously, the line integral along y = 1 yields the value -1, so the overall
integral gives 0 + 0 + 1 − 1 = 0, which is the same as the result in (3.88).
∂g ∂g
(b) We have f (x, y) = xy, g(x, y) = y + x, so = x and = 1, moreover
∂x ∂x
2 2
D = (x, y) : 0 ≤ x + y ≤ 1 is the unit disk. Thus with F = (f, g) we obtain
I Z Z Z Z
∂g ∂f
F.dr = ( − )(x, y)dA = 1 − xdA.
C D ∂x ∂y D
N
(Unit outer normal)
z
surface integral (see Definition 74), the integrand being the scalar function
Fn = f1 n1 + f2 n2 + f3 n3 , where fj and nj are the component functions of
the vector fields F and n. The integral on the right side is a volume integral,
∂f1 ∂f2 ∂f3
which we treat in Calculus, of the scalar function divF = + + .
∂x ∂y ∂z
(iii) Its interpretation and some applications will be discussed in Section 3.6.
Indeed, the divergence theorem is a fundamental tool in the analysis of partial
differential equations and of the phenomena described by them. Some identi-
ties useful for that purpose will be presented below, following the examples.
(iv) Equation (3.89) is sometimes helpful when one wants to compute surface
or volume integrals, because the evaluation of one side may be more conve-
nient than the evaluation of the other side.
Example 210. For each of the following data calculate the right hand side
and left side of Equation (3.89), whichever is convenient. (i) F (x, y, z) = xi +
yj − zk, Σ is the sphere of radius 4 centered at (1,1,1).
(ii)F (x, y, z) = x3 i + y 3 j + z 3 k, Σ is the sphere of radius 1 with center at the
origin.
(iii)F (x, y, z) = x2 i + y 2 j + z 2 k, Σ is the rectangular box bounded by the
coordinate planes x = 0, y = 0, z = 0 and the planes x = 6, y = 2 and z = 7.
Solution: In all cases, D denotes the region enclosed by Σ. Recall that
∂f1 ∂f2 ∂f3
divF = + +
∂x ∂y ∂z
Vector Calculus 299
where F = f1 i + f2 j + f3 k .
(i) We have f1 (x, y, z) = x, f2 (x, y, z) = y, f3 (x, y, z) = −z, so
divF(x, y, z) = 1 + 1 − 1 = 1.
We evaluate the right hand side of (3.89) as (here, vol(D) denotes the volume
of D, a ball of radius 4).
Z Z Z Z Z Z
4 256
divFdV = dV = vol(D) = Π.43 = Π.
D D 3 3
Again we evaluate the right and side of (3.89), this time with Frobenius the-
orem applied to the rectangular box,
Z Z Z Z 7Z 2Z 6
divFdV = 2 x + y + zdxdydz
D 0 0 0
Z 7Z 2 Z 7 Z 2
1
=2 [ x2 + xy + xz]x=6
x=0 dydz = 36 + 12y + 12zdydz
0 0 2 0 0
Z 7 Z 7
= [36y + 6y 2 + 12z]y=2
y=0 dz = 72 + 24 + 24zdz
0 0
Example 211. Verify the statement of Gauss divergence theorem for the
vector field F (x, y, z) = xi + yj + zk and the cone whose interior is given by
D = (x, y, z) : 0 < z < 1, 0 < x2 + y 2 < z 2 .
300 Modern Engineering Mathematics
z (0, 0, 1)
where n1 and n2 are the vector fields of outer unit normals to Σ1 and Σ2 .
According to (3.70),
∂S ∂S x y
N1 (x, y, z) = − (x, y)i − (x, y)j − k = − i − j + k
∂x ∂y z z
p
is a normal vector to Σ1 in (xy, z)), where z = x2 + y 2 . The corresponding
outer unit normal n1 is then given by
1 x y
n1 (x, y, z) = √ ( i + )j − k.
2 z z
x2 y2
Z Z Z Z Z Z
1
=√ ( + − z) + zdσ = 0 + 1dσ
2 Σ1 z z Σ2 Σ3
= Π,
2 2 2
as z = x + y on Σ1 , z = 1 on Σ2 and the area of the disk Σ2 equals Π.
The right hand side of (3.89) becomes
Z Z Z Z Z Z Z Z Z
divF dV = 1 + 1 + 1dV = 3 1dV = 3vol(D).
D D D
Since the volume of the cone of height 1 and radius 1 is equal to Π3 , the right
hand side, too, is equal to Π.
We now present Green’s identities. Here ∆ denotes the Laplace operator,
One can derive Green’s first identity from the divergence theorem by choosing
F = f ∇g in (3.89).
Line integrals of vector fields have been studied in Subsection 3.4.1 and surface
integrals in Subsection 3.4.2. The integrand of the surface integral on the right
side is a scalar field, obtained as the scalar product of the vector fields curlF
and n. From Definition 27 we recall that the notation as a vector product or
as a determinant
i j k
∂ ∂ ∂
curlF = ∇ × F = ∂x ∂y ∂z (3.94)
f1 f2 f3
is a convenient way to memorize the component definition of the rotation
curlF of the vector field F .
We assume that the surface Σ is given as z = S(x, y) with a function S defined
on the corresponding domain D in the xy-plane, and we assume that S and
its first and second partial derivatives are continuous. We have seen in (3.63)
that
1
q
n = (−∂x Si − −∂y Sj + k), ν = 1 + (∂x S)2 + (∂y S)2 (3.95)
ν
defines a unit normal in the point (x, y, S(x, y)) of Σ, if the right hand side
is evaluated at (x, y) ∈ D. We fix a suitable orientation of the boundary
curve C of Σ as follows. Let τ be the boundary of D, positively oriented by a
parametrization q : [a, b] → R2 . Then
Example 212. Verify the Stokes theorem by evaluating both sides of (3.96)
for the following data.
(a) F (x, y, z) = (x − y)i + (y − z)j + (z − x)k, the surface Σ is the portion of
the plane x + y + z = 1 which lies in the first octant.
(b) F (x, y, z) = (y − z)i + (z + x)j + (y − x)k, the surface Σ is the portion of
the paraboloid z = 9 − x2 − y 2 which lies above the xy-plane.
Solution: (a) The surface Σ forms a planar triangular region in space whose
vertices are the unit vectors i, j and k and the boundary curve C is the triangle
connecting those points and thus is piecewise smooth. The corresponding do-
main D in the xy-plane is the triangular region enclosed by x+y = 1,x = 0 and
y = 0. Thus, if we go through the corners of C in the sequence i → j → k → i,
the orientation corresponds to a positive orientation of τ , the boundary of D.
We parameterize the first piece C1 : i → j with r(t) = (1 − t)i + tj, t ∈ [0, 1].
For F (x, y, z) = (x − y)i + (y − z)j + (z − x)k, the line integral becomes
Z Z 1
F dr = F (r(t))r0 (t)dt
C1 0
Z 1
= ((1 − 2t)i + tj + (t − 1)k)(−i + j)dt
0
Z 1
1
= (3t − 1)dt = .
0 2
The other two pieces of C are parameterized by r(t) = (1 − t)j + tk and
r(t) = (1 − t)k + tj, t ∈ [0, 1], respectively. An analogous computation as
above shows that Z Z
1
F.dr = F.dr =
C2 C3 2
so I
1 1 1 3
F dr = + + = .
C 2 2 2 2
The parametrization S of Σ is given by S(x, y) = 1−x−y, so ∂x S = ∂Y S = −1
is constant. The unit normal according to (3.63) is
1
n = √ (i + j + k).
3
Moreover,
p curlF = i + j +√k. Thus, both n and curlF are constant on Σ .Since
1 + (∂x S)2 + (∂y S)2 = 3, the surface integral becomes
Z Z Z Z √ Z Z √ √
3
(curlF ).ndσ = 3dσ = 3 3dA = ,
Σ Σ D 2
center at the origin and it can be parameterized as r(t) = 3 cos ti + 3 sin tj for
0 ≤ t ≤ 2Π. We have
I Z 2 Z 2
F dr = ΠF (r(t))r0 (t)dt = Π9(− sin2 t + cos2 t)dt = 0.
C 0 0
Since the line integral equals 0, it does not matter which orientation we choose.
Moreover, we compute that curlF = 0, so
Z Z Z Z
(curlF )ndσ = 0dσ = 0.
Σ Σ
Hence, we have verified the Stokes theorem for both sets of data.
Remark 42. A vector field F defined on some open set G of space R3 is
called irrotational in G if curlF = 0 at all points of G. H This terminology is
motivated by the Stokes theorem, since the circulation C F dr equals zero for
all closed curves which arise as a boundary of a surface Σ domain G. If all
closed curves in G arise as boundaries, we then can conclude from Theorem
45 that curlF = 0 in G implies that F is conservative on G. This is the case,
for example, when G is the whole space R3 or when G is a ball.
On the other hand, if curlF(x, y, z) is not zero at some point (x, y, z), then
for smallH disks around this point whose normal is parallel to curlF(x, y, z),
we have C F dr 6= 0 and hence F cannot be conservative. However, note that
for some types of regions G there may exist closed curves C which are not
boundaries of such a surface Σ and have non-zero circulation (and hence, F
is not circulation free and not conservative on G according to Definitions 44
and 37, even though we might have curlF = 0 on G). Consider, for example,
the vector field
y x
F (x, y, z) = − 2 i+ 2 j.
x + y2 x + y2
Its domainH G is the whole space R3 except the z-axis. It satisfies curlF = 0
on G, but C F dr = 2Π 6= 0 for the circle C defined by x2 + y 2 = 1. Indeed, if
Σ is a surface with boundary C, it must intersect the z-axis, so it cannot be
contained in G.
H RR
Example 213. For the following data, evaluate C F dr or Σ
(curlF ).ndσ,
whichever is easier.
(a) F = yx2 i − xy 2 j + z 2 k, the surface Σ is the hemisphere x2 + y 2 + z 2 =
4,z ≥ 0.
(b) F = zi + xj + y 2 k, the surface Σ is the cone z = x2 + y 2 for 0 ≤ z ≤ 4.
Solution: (a) The boundary curve C of Σ can be described by r(t) = 2 cos ti+
2 sin tj for 0 ≤ t ≤ 2Π, so along C we have
F (r(t))r0 (t) = −16 cos2 t sin2 t − 16 cos2 t sin2 t = −8 sin2 (2t) = −4(1 − cos 4t),
and therefore I Z 2
F dr = Π − 4(1 − cos 4t)dt = −8Π.
C 0
Vector Calculus 305
(b) The parametric equation of C is r(t) = 4 sin ti+4 cos tj+4k for 0 ≤ t ≤ 2Π,
so
F (r(t)).r02 t,
I Z 2
F.dr = Π16 cos t − 16 sin2 t = −16Π,
C 0
p
Note that the parametrization z = S(x, y) = x2 + y 2 is not differentiable at
0; we remark that the Stokes theorem remains valid in this particular case.
vector of the object of mass m and r =k r k, and the force F (r) exerted by
the object of mass M on the object of mass m points in the direction of the
−r
unit vector krk . Thus, from (3.97)
GmM 2 r GmM 3
F (r) = − =− r, (3.98)
krk krk krk
This defines a vector field whose domain D equals the whole space R3 except
the origin. It describes the gravitational force of a point mass M located at
the origin, as a function of the position of the point mass m.
Electric Force Field
Coulomb’s law states that the electrostatic force exerted by one charged par-
ticle on another is directly proportional to the product of the charges and
inversely proportional to the square of the distance between them. Let two
particles of charge Q and q be located at the origin of R3 and at the position
given by a vector r, respectively. Then the force F (r) that the particle of
charge Q exerts on the particle of charge q equals
qQ
F (r) = r, (3.99)
4Π0 k r k3
∂ψ ∂ψ ∂ψ
∇ψ = i+ j+ k
∂x ∂y ∂z
and is called a potential of the vector field F if F = ∇ψ. In this case, F is
called a gradient field.
Vector Calculus 307
These vector fields are shown in Figures 3.11a and 3.11b respectively.
(ii) The vector field defined as F (r) = F (x, y) = −yi + xj is called the spin
field or rotation field or turning field. F (r) is perpendicular to r, since
F (r)r = −yx + xy = 0.
p F
Moreover, we have k F (r) k= x2 + y 2 =k r k. The vector fields F and kF k
are respectively shown in Figures 3.12a and 3.12b.
308 Modern Engineering Mathematics
The speed of the particle is defined as the magnitude of the velocity and it
therefore is equal to
1
k v(t) k= (x0 (t)2 + y 0 (t)2 + z 0 (t)2 ) 2 =k r0 (t) k . (3.104)
Let s(t) denote the total distance traveled up to time t, that is,
Z
s(t) = k F 0 (τ ) k dτ, see (3.101)
. Since s0 (t) = kr0 (t)k, we see from (3.104) that the speed of the particle is
equal to s0 (t). The acceleration a of the particle is defined as the rate of
change of the velocity with respect to time,
equals the total (or net) force acting upon the object. We see therefore that
p0 (t) = 0 if the net force is zero at time t.
Vector Calculus 309
Example 214. A particle in the plane moves along the circle with radius 2
centered at the origin in such a way that its x- and y-coordinates are given
by x(t) = 2 cos t, y(t) = 2 sin t.
(a) Find the velocity, the speed, and the acceleration of the particle at an
arbitrary time t.
310 Modern Engineering Mathematics
F
FIGURE 3.12b: Spin Field
kF k
(b) Sketch the path of the particle and show the position and velocity vectors
at t = Π4 .
Solution: (a) The position is described by the vector function
Its acceleration at time t is a(t) = r00 (t) = −2 cos ti − 2 sin tj. At time t = Π
4,
we have
Π Π Π √ √
r( ) = 2 cos i + 2 sin j = 2i + 2j,
4 4 4
Π Π Π Π √ √
v( ) = r0 ( ) = −2 sin i + 2 cos j = − 2i + 2j.
4 4 4 4
Example 215. A particle of charge q moving in a magnetic field B is subject
to the so-called Lorentz force:
q
F = v×B (3.105)
c
where c is the speed of light and v is the velocity of the particle. Assume
Vector Calculus 311
that the magnetic field is constant and vertically oriented, B(t) = B0 k with
B0 6= 0. Find the path
of the particle, given its initial position r(0) = r0 and velocity v(0) = v0 , as
well as its mass m.
Solution: By Newton’s law and (3.105) we have
q
mv 0 (t) = mr00 (t) = F (t) = v(t) × B(t).
c
Since B(t) = B0 k, we get
qB0
v 0 (t) = λv(t) × k, whereλ = . (3.106)
mc
Written in components v(t) = v1 (t)i + v2 (t)j + v3 (t)k becomes
v10 (t) = λv2 (t), v20 (t) = −λv1 (t), v30 (t) = 0. (3.107)
Since v30 (t) = 0 for all t, v3 is a constant function, say v3 (t) = C. From the
first two equations of (3.107) we get
or
v100 (t) + λ2 v1 (t) = o. (3.108)
A solution of (3.108) is given by
because
v10 (t) = λA cos(λt + ϕ), v1002 (t) = λA sin(λt + ϕ).
Since v10 (t) = λv2 (t), we get
v10 (t)
v2 (t) = = A cos(λt + ϕ).
λ
Therefore
T (x, y) = 10 − 8x2 − 2y 2 .
Example 216. Find an equation for the trajectory of the particle if it moves
continuously in the direction of maximum temperature increase.
Solution: Let the trajectory (a curve) be parameterized by r(t) = (x(t), y(t))
with r(0) = (2, 3). Since the direction of maximum temperature increase at
any point (x, y is given by ∇T (x, y), the velocity vector v(t) of the particle at
time t points in the direction of the gradient at its current position r(t). Thus
there is a scalar µ that may depend on t such that
are solutions of (3.110) with initial values x(0) = x0 and y(0) = y0 . Using the
initial values x0 = 2 and y0 = 3 we see that the trajectory (x(t), y(t)) satisfies,
for all values of t, the equation
3 √
y= √
4
4
x. (3.111)
2
The graph of the trajectory and Cantor plot of the temperature function are
shown in Figure 3.13.
Vector Calculus 313
v = ω × r⊥ = ω × r.
Angular velocity as the curl of linear velocity: This vector ω is called the
314 Modern Engineering Mathematics
angular velocity of the object; its length k ω k equals the angular speed. Let
ω = Ai + Bj + Ck. Then
= 2ω. (3.112)
Thus the angular velocity of a uniformly rotating body equals one-half the
curl of the linear velocity, as the latter is called in this context to emphasize
its different character.
that is,
Z
Mx = x(t)ρ(r(t)) k r0 (t) k dt, My = . . . , Mz = . . . .
Let us now assume that the wire rotates around an axis. Its moment of inertia
with respect to this axis is given by the line integral
Z
I= d2 ρds,
C
and d(x, y, z) denotes the distance of the point (x, y, z) from the axis. In
particular, we obtain the moment of inertia of the wire with respect to the
x-axis as Z
Ix = (y 2 + z 2 )ρds.
C
W = F.(r1 − r0 ) =k F kk r1 − r0 k cos α,
where α is the angle between the vectors F and r1 − r0 . Let us now parame-
terize C as r(t) = r0 + t(r1 − r0 ) with r : [0, 1] → R3 . Since r0 (t) = r1 − r0 ,
we have Z 1
F.(r1 − r0 ) = F.r0 (t)dt.
0
The latter integral is nothing more than the line integral of Definition 70, so
we can express the work W as the line integral.
Z
W = F.dr. (3.113)
C
This line integral also yields the correct value of the total work done by an
arbitrary force field Let F (x, y, z) = f1 (x, y, z)i + f2 (x, y, z)j + f3 (x, y, z)k
on a mass which has moved along an arbitrary curve C from its initial point
r(a) to its final point r(b), if C is given by r = x(t)i + y(t)j + z(t)k with
r : [a, b] → R3 . This can be seen if we approximate C by a curve consisting
of pieces of straight lines and pass to the limit. The procedure is analogous
to the one used to compute the length of a curve, we will not carry out the
details.
Example 218. Let F (x, y, z) = i − yj + xyzk be a force field. Calculate the
work done when moving a particle from (0, 0, 0) to (1, −1, 1) along the curve
x = t, y = −t2 , z = t, 0 ≤ t ≤ 1.
R
Solution: The work done is equal to the line integral C F.dr. We compute
Z Z Z 1
F.dr = dx − ydy + xyzdz = [1x0 − yy 0 + xyzz 0 ](t)dt
C C 0
316 Modern Engineering Mathematics
Z 1
1 1
= 1 + t2 (−2t) − t4 dt = [t − t4 − t5 ]10
0 2 5
1 1 3
=1− − =
2 5 10.
Example 219. Find the work done by F (x, y, z) = x2 i − 2yzj + zk in moving
an object along the line segment from (1,1,1) to (4,4,4).
Solution: First parameterize the line segment as x = y = z = 1 + 3t with
0 ≤ t ≤ 1, so x0 (t) = y 0 (t) = z 0 (t) = 3. The work done by F moving the object
along C is given by
Z Z 1
W = F.dr = [x2 x0 − 2yy 0 + zz 0 ](t)dt
C 0
Z 1
= [(1 + 3t)2 − 2(1 + 3t)2 + (1 + 3t)].3dt
0
(1 + 3t)2 (1 + 3t)3 1 27
[ − ]0 = − .
2 3 2
In Definition 72 we have called a vector field F conservative if it possesses
a potential function, that is, if F = ∇ψ for some scalar field. The following
remark explains this terminology.
Remark 45. (Conservation of mechanical energy)
Let an object of mass m move in space according to Newton’s law F = ma =
mr00 . Assume that the force field F has a potential function ψ , so F = ∇ψ
. In mechanics, one considers the potential energy −ψ and the kinetic energy
− 12 m k v k2 2, where v = r0 . The total mechanical energy E(t) of the object
at time t is given as the sum of its kinetic and its potential energy at that
time, so
1
E(t) = mr0 (t).r0 (t) − ψ(r(t)). (3.114)
2
Let us compute its time derivative E 0 (t). Using the product rule and the chain
rule we get
1 00 1
E 0 (t) = mr (t).r0 (t) + mr0 (t) − ∇ψ(r(t)).r0 (t)
2 2
= [mr00 (t) − ∇ψ(r(t))].r0 (t).
Since mr00 (t) = F (r(t)) = ∇(r(t)), we conclude that E 0 (t) = 0. This means
that the total energy remains constant (or, is conserved) along the trajectory
of the object.
Remark 46. As we have seen in Section 3.6.1, the gravitational field pos-
sesses a potential and hence it is conservative.
Vector Calculus 317
Assume for the moment that the fluid moves with constant velocity vector v
during a certain time interval [0, T ]. At time T , those particles of the fluid
which have passed through Σ during the interval [0, T ] occupy the region D.
The volume of D is given by
where n is a unit normal to Σ, and A is the area of Σ. Thus, the number Av.n
gives the rate (per unit time) of fluid volume passing through Σ. The sign of
this number depends on the direction of the flow (which is the direction of the
velocity vector v) as well as on the orientation of the normal n.
In general, the velocity field v = v(x, y, z, t) is not constant, but depends on
space and time, and the surface Σ does not lie in a plane. If Σ, however, consists
of plane parts Σj with unit normals nj and area Aj on which, moreover, v
is a constant vector v j . By the above considerations the rate of fluid volume
passing through all of Σ is equal to Σj Aj v j .nj . For the general case, the surface
integral can be viewed as the result of a limit procedure of approximating Σ
and v.n in such a piecewise fashion. Thus, the rate of fluid volume passing
through Σ at time t is equal to
Z Z
v(., t).ndσ. (3.116)
Σ
RR
Here, the notation v(., t) indicates that the surface integral Σ
f dσ has to be
evaluated with the integrand f (x, y, z) = v(x, y, z, t).n(x, y, z) for that value
of the time t for which we want to determine the rate. The integral in (3.116)
is called the flux of the vector field v across the surface Σ. In this context,
the vector field v is called the corresponding flux density. Since the rate of a
quantity is nothing else than its time derivative, we obtain the total volume
of the fluid passing through Σ during some time interval [t1 , t2 ] as the integral
of (3.116) with respect to time,
Z t2 Z Z
v(., t).ndσdt. (3.117)
t1 Σ
Note that (3.118) also applies to situations where ρ is not constant, that is, if
the fluid in question is compressible. This typically occurs when the fluid is a
gas.
Accordingly, for an arbitrary vector field F = F (x, y, z, t) with values in R3
one defines the flux of F across the surface Σ as the surface integral
Z Z Z Z
F (., t).ndσ, or F.ndσ. (3.119)
Σ Σ
The latter if F is stationary (that is, F does not depend on t) or if one just
wishes to keep the notation simple. In this context, the vector field F useful
Vector Calculus 319
Actually, when divF is continuous as we have assumed, we can prove that this
average value tends to divF (x, y, z) in the limit when D shrinks to the single
point P . From the divergence theorem (39) we obtain
Z Z Z Z Z
1 1
divF dV = F.ndσ, (3.121)
vol(D) D vol(D) Σ
For the other four faces we get corresponding expressions involving the com-
ponents f2 and f3 of F . Now let us apply the divergence theorem to the vector
320 Modern Engineering Mathematics
z Back face I
(x, y, z)
(x + !u, y, z)
∂f1
field F̃ = f1 i, instead of F . We have div F̃ = and therefore
∂x
Z Z Z Z Z Z Z Z Z Z Z Z
∂f1
dV = div F̃ dV = F̃ .ndV = If1 dσ− f1 dσ
D ∂x D Σ I I
(3.123)
since the contributions of the other four faces are zero for F̃ . Therefore,
Z Z Z Z Z Z Z
∂f1 1 ∂f1 1
(x, y, z) ≈ dV = [ If1 dσ − f1 dσ]
∂x volD D ∂x volD I I
(3.124)
∂f1
for small values of ∆x, ∆y and ∆z. Thus, the number expresses the
∂x
difference of outward flux across face II and inward flux across face I, per
∂f2 ∂f3
unit volume. The partial derivatives and can be interpreted in an
∂y ∂z
analogous manner.
Vector Calculus 321
Since m = ρvol(D) is the mass of the fluid displaced by the object, its weight
mg (the force exerted by gravity on the fluid mass) equals the buoyant force
acting on the object. In the same manner, one can show that f1tot = f2tot = 0,
322 Modern Engineering Mathematics
that is, the total force on the object is indeed vertically upward. To see this,
one replaces k by i or j in the derivation prior to (3.127). Note that the field
∂p ∂p
G in this case satisfies divG = 0, because ∂x = ∂y = 0. Thus we have derived
the principle of Archimedes from the balance of momentum which underlies
Equation (3.125) for the pressure.
Mass Balance
Consider a region G through which a fluid moves which has mass density
ρ = ρ(x, y, z, t) and velocity v = v(x, y, z, t). Let D be any subregion of G
with boundary surface Σ and unit outer normal field n. At any given time
t, the total mass M (t) contained in D is given by the volume integral of the
mass density with respect to the spatial coordinates (x,y,z),
Z Z Z
M (t) = ρ(., t)dV. (3.129)
D
On the other hand, when discussing the flux of a vector field in Subsection
3.6.3, we have seen in (3.117) that the rate of fluid mass passing through the
surface Σ in the outward direction is given by
Z Z
ρ(., t)v(., t).ndσ. (3.130)
Σ
We combine the two previous equations and merge all components under one
integral sign to obtain
Z Z Z
∂ρ
(., t) + div(ρv)(., t)dV = 0. (3.134)
D ∂t
Vector Calculus 323
Heat Flow
In this subsection we show that the heat equation in Example 8.4.15, can be
obtained as a consequence of the principle of energy conservation, using the
divergence theorem. The derivation proceeds along similar lines as in the pre-
vious subsection, so our exposition will be shorter. The total amount of heat
contained in some region D at a time t is given by
Z Z Z
Q(t) = (ρu)(., t)dV, (3.137)
D
where u denotes the specific inner energy (that is, energy per mass). Denoting
further its flow by q, energy conservation tells us that
Z Z Z Z Z
d
(ρu)(., t)dV = Q0 (t) = − q(., t).ndσ. (3.138)
dt D Σ
As in the preceding subsection we can show that the previous two equations
imply that
∂
(ρu) + divq = 0 (3.139)
∂t
holds at all points (x, y, z) and times t considered. We assume that u = cT +
α, where T is the temperature, and the specific heat capacity c as well as
the number α and the mass density ρ are constant. Fourier’s law of heat
conduction states that
q = −κ∇T, (3.140)
324 Modern Engineering Mathematics
4T = 0. (3.142)
Both preceding equations are bases for the modelling and simulation of heat
conduction and flow in many kinds of engineering problems.
since the right hand side represents the average of (curlv).n over Σr , and the
area of Σr equals Πr2 . In fact, since curlv is continuous and Σr shrinks to the
point P0 as r → 0,
Z Z
1
(curlv)(P0 ).n = lim 2 (curlv).ndσ. (3.144)
Πr Σr
Thus, the number (curlv)(P0 ).n represents the circulation of v per unit area
in the plane normal to n near P0 . In this manner, the curl of the velocity
serves as a local model for the rotation of the fluid.
Vector Calculus 325
Boundary circle Cr
Disk 2: r
}----------~ y
with a function β = β(r) < 0 yet to be fixed. We have seen in Section 3.6.4 that
we must have divv = 0 in order to satisfy the principle of mass conservation.
Using the chain rule we compute
∂v1 ∂r x2
= βr + xβ 0 (r) = β(r) + β 0 (r) p .
∂x ∂x x2 + y 2
326 Modern Engineering Mathematics
possesses those four properties. (The constant k > 0 is called the vortex
strength.) Indeed, by (3.148) we have k v(x, y) k= 2Πxk2 +y2 = 2Πr k
.
Moreover, v(x, y).(xi + yj) = 0, so v(x, y) is perpendicular to the radius
vector of the circle, and by drawing a picture one sees that the direction
is counterclockwise.
Vector Calculus 327
Streamlines and stream functions. The paths followed by the fluid par-
ticles in a fluid flow are called the streamlines of the flow. If the streamlines
can be represented as the level curves of some function ψ = ψ(x, y), then ψ is
called the stream function of the flow. In this case, every particle path must
satisfy (x(t), y(t)) = c for a suitable constant c. By the chain rule,
∂ψ ∂ψ
(x(t), y(t))xṫ + (x(t), y(t))y ṫ = 0
∂x ∂y
must hold along particle paths. Therefore, ψ and the velocity field v of the
flow are related by
∇ψ(x, y).v(x, y) = 0 (3.149)
at all points (x, y) in the domain of the flow; the velocity vectors are tangent,
while the vectors ∇ψ(x, y) are perpendicular to the streamlines.
Combined sink and vortex flow. Here we have the velocity field
1
v(x, y) = p [(−qx − ky)i + (−qy + kx)j]. (3.150)
2Π x2 + y 2
The particles in this flow rotate while moving toward the sink, so we expect
that they spiral inward. In order to find the streamlines, we will compute its
stream function, using (3.149). It is convenient to do this in polar coordinates,
using the vectors introduced in (3.60) as
1 c−qθ c −qθ
ln r = (c − qθ), r = e k = e k .e k .
k
c
Since c is an arbitrary constant, we may replace e k by c and finally obtain
that
−qθ
r = ce k , c > 0 (3.154)
holds along the streamlines. Thus, the spirals are determined by the value of
q
k.
Modelling of hurricane. Let us assume that the preceding flow model can
be used for a hurricane and that only a single measurement of velocity is
available.
Example 220. Find the strength of the parameters k and q of the flow
model (3.150) for a hurricane from the report that at 20 km distance from
the eye the wind velocity has a component of 15 km/hr towards the eye and
a counterclockwise tangential component of 45 km/hr. Estimate the size of
the hurricane by finding a radius beyond which the wind speed is less than 5
km/hr.
Solution: The velocity component toward the eye is equal to the speed of the
sink part of the flow. We have already seen that the latter is given by q/(2Πr),
so at r = 20 we have
q
15 = , so q = 600Π
2Π.20
with unit 1/hr. The tangential velocity component is equal to the speed of
the vortex part of the flow, and we have already seen that the latter is given
by k/(2Πr), so at r = 20 we have
q
45 = , so q = 1800Π,
2Π.20
again with unit 1/hr. To estimate the size, we determine r from the condition
that k v k= 5km/h. Since tangential and inward velocity are perpendicular,
we have r
1 q k
5 =k v k= ( )2 + ( )2
r 2Π 2
Therefore
1p 2 100 √ √
r= 300 + 9002 = 9 + 81 = 60 10 ≈ 189.7 km.
5 5
Vector Calculus 329
3.7 Exercises
3.1. Let x, y, z be vectors of three-dimensional space, let λ be a scalar. Show
that
(i) (x + y) + z = x + (y + z)
(ii) λ(x + y) = λx + λy
(iii) x.(y + z) = x.y + x.z
(iii) Show that (∗) holds for x = k and arbitrary vectors y, z and w.
(iv) Show that (∗) holds for for arbitrary vectors x, y, z and w.
3.3. The line in R2 that passes through the point r0 = (x0 , y0 ) and is parallel
to the non-zero vector v = (a, b) = ai + bj has parametric equations
x(t) = x0 + at, y(t) = y0 + bt,
or, in vector form r(t) = (x(t), y(t)),
r(t) = r0 + tv.
In R3 , the vector form is the same, but now r(t) = (x(t), y(t), z(t)), r0 =
(x0 , y0 , z0 ) and v = (a, b, c) and the component equations are
x(t) = x0 + at, y(t) = y0 + bt, z(t) = z0 + ct.
(a) Find the parametric equations of the line
330 Modern Engineering Mathematics
(b) Find parametric equations for the line whose vector equation is given
as
that is, the plane consists of the points r = (x, y, z) satisfying this equa-
tion. Its vector form is
N.(r − r0 ) = 0.
Find an equation of the plane that passes through the point r0 = (2, 6, 1)
having the vector N = (1, 4, 2) as a normal.
3.5. Let F and G be two vector fields defined on an interval with values in
R3 . Prove that
d dG dF
(a) [F.G] = F. + .G.
dt dt dt
d dG dF
(b) [F × G] = F × + × G.
dt dt dt
(c) Let r = r(t) be a vector valued function with values in R3 such that
k r(t) k= c for some constant c. Show that r(t).r0 (t) = r(t). dr
dt (t) = 0,
dr 0
that is, r(t) is orthogonal to (t) = r (t), for all t.
dt
RR
3.10. Evaluate the surface integral Σ
f dσ, where
p
(a) f (x, y, z) = y 2 , Σ is the part of the cone z = x2 + y 2 lying in the
first octant and between the plane z = 2 and z = 4.
(b) f (x, y, z) = xyz, Σ is the part of the surface z = 1 + y 2 for 0 ≤ x ≤
1, 0 ≤ y ≤ 1.
3.11. A particle moves once counterclockwise around the circle of radius 12
centered at the origin under the influence of the force F (x, y, z) = (ex −
3
y + x cosh x)i + (y 2 + x)j. Calculate the work done.
3.12. RApply the Green-Ostrogradski theorem to evaluate the line integrals
C
F.dr for the following data. The curves are oriented counterclockwise.
(a) F = x2 yi−xy 2 j, C is the boundary of the region defined by x2 +y 2 ≤
4, x ≥ 0 and y ≥ 0.
(b) F = (esin x − y)i + (sinh(y 3 ) − 4x)j, C is the circle of radius 4 with
center (−8, 0).
(c) F = (x2 + y 2 )i + (x2 − y 2 )j, C is the ellipse 4x2 + y 2 = 10.
RR
3.13. RLet
R RD be a region bounded by the surface Σ. Evaluate Σ
F.ndσ or
D
div(F )dV for the following data, whichever is more convenient.
(a) F = 4xi − 6yj + k, Σ is the surface of the solid cylinder x2 + y 2 ≤ 4,
0 ≤ z ≤ 6 (the surface includes both caps of the cylinder).
(b) F = 2yzi − 4xzj + xyk, Σis the sphere of radius 6 with center
(−1, 3, 1).
F.ndσ for the data F = 3xyi + z 2 k, Σ the sphere
RR
3.14. Find the value of Σ
of radius 1 centered at the origin.
3.15. Let Σ be a smooth Rsurface
R enclosing some region D and C be a constant
vector. Show that Σ
C.ndσ = 0.
RR
3.16. Evaluate Σ
(curlF ).ndσ, where F = xyi + yzj + xyk and Σ is the part
of the plane 2x + 4y + z = 8 in the first octant.
3.17. Calculate the circulation of F = (x − y)i + x2 yj + xzak counterclockwise
around the circle x2 + y 2 = 1, where a is a positive constant.
332 Modern Engineering Mathematics
3.18. Examine whether the following vector fields are conservative or not.
(a) F = cosh(x + y)(i + j − k).
(b) F = 2xi − 2yj + 2zk.
(c) F = i − 2j + k.
3.19. Let Σ be the portion of the paraboloid z = 1 − x2 − y 2 for which z ≥ 0,
and let C be the circle x2 + y 2 = 1 that forms the boundary of Σ. Verify
Stokes’theorem for the vector field
335
Chapter 4
Fourier Methods and Integral
Transforms
4.1 Introduction
Fourier methods or Fourier analysis constitute a branch of mathematics
developed formally some 150 years after Newton’s and Leibniz’ calculus and
heavily depend on integral and differential calculus. Jean Baptiste Joseph
Fourier was born in 1768 in Auxerre, a town between Paris and Dijon. He
became fascinated by mathematics at the age of 13. After the French Rev-
olution, Fourier taught in Paris, then accompanied Napoleon to Egypt and
served as permanent secretary of the Institute of Egypt. He wrote a book on
Egypt and in certain quarters he is famous more as an Egyptologist than for
his contributions to mathematics and physics. In the world of science he is
famous for, among other things, the ideas he set forth in a memoir in 1807
and published in 1822 in his book in French entitled The Analytic Theory of
Heat.
Fourier analysis shows that we can represent periodic functions, even very
jagged and irregular-looking ones, in form of a finite or infinite sum of sine
337
338 Modern Engineering Mathematics
for systems of more than two vectors, and we define orthogonality for functions
instead of vectors.
Throughout this chapter, the functions considered will be piecewise con-
tinuous. Such functions are formed by putting together continuous functions
defined on separate intervals, for example the sign function or the greatest
integer function as considered in Calculus Books, or the “sawtooth” function
defined by
1 1
f (x) = x − k , if k − ≤ x < k + . (4.1)
2 2
The formal definition runs as follows. A function f defined on an interval
I = [a, b] is called piecewise continuous, if we can decompose I into finitely
many subintervals Ik = [xk−1 , xk ] such that f is continuous in the interior of
Ik and the one-sided limits limx→xk−1 + f (x) and limx→xk − f (x) exist for all
such subintervals. If the domain of f is unbounded, we require this property
to hold for every bounded interval in the domain. As a consequence, we can
integrate piecewise continuous functions, and
Z b X Z xk
f (x) dx = f (x) dx .
a k xk−1
Example 221. (a) The functions f1 (x) = ex and f2 (x) = sin x are orthogo-
nal on the interval [π/4, 5π/4].
(b) The functions f1 (x) = x and f2 (x) = cos 2x are orthogonal on the interval
[−π/2, π/2].
In analogy to the case of vectors, for functions f : [a, b] → R we define
s
p Z b
kf k = hf, f i = f (x)2 dx (4.4)
a
and call it the norm of f . (We might also call kf k the length of f , although
its geometric meaning is no longer obvious.) We then have
Z b
kf k2 = hf, f i = f (x)2 dx .
a
340 Modern Engineering Mathematics
As in the case of vectors, we have k0k = 0 for the zero function. On the other
hand, a piecewise continuous function f satisfies kf k = 0 only if f is the zero
function; in other words, if f is nonzero, we must have kf k > 0.
If two nonzero functions f1 and f2 are orthogonal, we say that the set
{f1 , f2 } formed by those two functions is an orthogonal system. If we have
more than two functions, we require that the functions are mutually (or pair-
wise) orthogonal in the sense of the following definition.
Definition 77. (Orthogonal system, orthonormal system)
A system (or set) consisting of finitely or infinitely many nonzero functions
f1 , f2 , . . . is said to be orthogonal on the interval [a, b] if
Z b
hfm , fn i = fm (x)fn (x) = 0 ,
a
whenever m 6= n.
If moreover
hfn , fn i = 1 = kfn k , for all n,
the system is called orthonormal on [a, b].
Remark 47. An orthogonal system {f1 , f2 , . . . } can be made into an or-
thonormal system by replacing each function fn with its scalar multiple
fn
.
kfn k
Example 222. (a) The set {1, cos x, cos 2x, . . . , cos nx} is an orthogonal sys-
tem on [−π, π].
n 1 cos x cos nx o
(b) The set √ , √ , · · · , √ is an orthonormal system on [−π, π].
2π π π
(c) Examine whether the following systems are orthonormal or not on the
intervals indicated.
(i) {sin x, sin 3x, sin 5x, · · · }, I = [0, π/2],
(ii) {sin nx} : n = 1, 2, 3, . . . }, I = [0, π],
cos nπx
(iii) {1, : n = 1, 2, 3, . . . }, I = [0, l],
l
cos nπx sin mπx
(iv) {1, , } : n, m = 1, 2, 3, . . . }, I = [−l, l].
l l
Solution: We discuss here part (c).
(i) We set fn (x) = sin(2n + 1)x. For m 6= n, we obtain
Z π/2
hfn , fm i = sin(2n + 1)x sin(2m + 1)x dx
0
Z π/2
1
= [cos 2(n − m)x − cos 2(n + m + 1)x] dx
2 0
Fourier Methods and Integral Transforms 341
= 0 − 0 = 0.
For n = m we get
Z π/2 Z π/2 1 1
sin2 (2n + 1)x dx = − cos 2(2n + 1)x dx
0 0 2 2
1 π/2 1 π/2
Z Z
= dx − cos 2(2n + 1)x dx
2 0 2 0
h 1 iπ/2 1 h 1 iπ/2
= x − sin 2(2n + 1)x
2 0 2 2(2n + 1) 0
π
= .
4
Hence the given system is orthogonal
√ but not orthonormal. However, if we
multiply each function by 2/ π, the new system obtained in this way is or-
thonormal, that is,
2 sin x 2 sin 3x sin 5x 2 sin(2n + 1)x
√ , √ ,2 √ ,..., √ ,··· ,
π π π π
(by the trigonometric identity 2 sin Ax sin Bx = cos(A − B)x − cos(A + B)x)
1 π 1 π
Z Z
= [cos(n − m)x] dx − [cos(n + m)x] dx
2 0 2 0
1 h sin(n − m)x iπ 1 h sin(n + m)x iπ
= − = 0.
2 (n − m) 0 2 (n + m) 0
For m = n we have
Z π Z π 1 1
hfn , fn i = sin2 nx dx = − cos 2nx dx
0 0 2 2
1 1 h iπ π
= π− sin 2nx = ,
2 4n 0 2
342 Modern Engineering Mathematics
1 l (n − m)πx 1 l
Z Z
(n + m)πx
= cos dx + cos dx
2 0 l 2 0 l
l h (n − m)πx il l h (n + m)πx il
= sin + sin
2(n − m)π l 0 2(n + m)π l 0
= 0 + 0 = 0.
For m = n we have
Z l Z l h 1 il
nπx 1 1 2nπx l h 2nπx il
cos2 dx = + cos dx = x + sin
0 l 0 2 2 l 2 0 4nπ l 0
l
= .
2
Moreover,
Z l Z l
nπx h l nπx il
1 · cos dx = sin = 0, 12 dx = l .
0 l nπ l 0 0
The given system is orthogonal but not orthonormal. However, the system
n 1 √2 cos nπx o
√ ,√ is orthonormal.
l l l
(iv) We know already from part (iii) that
Z l Z l
nπx mπx nπx mπx
cos cos dx = 2 cos cos dx = 0 .
−l l l 0 l l
Concerning the sine, for m 6= n we get
Z l Z l
nπx mπx nπx mπx
sin sin dx = 2 sin sin dx
−l l l 0 l l
(by the identity 2 sin Ax sin Bx = cos(A − B)x − cos(A + B)x)
Z lh
(n − m)πx (n + m)πx i
= cos − cos dx
0 l l
h (n − m)πx l il h (n + m)πx l il
= sin · − sin ·
l n−m 0 l n+m 0
= 0 − 0 = 0.
Fourier Methods and Integral Transforms 343
Using the same identities as in the previous computations one derives that
Z l Z l
nπx nπx
sin2 dx = l , cos2 dx = l ,
−l l −l l
Z l
nπx mπx
sin cos dx = 0 , for n 6= m,
−l l l
moreover
Z l Z l Z l
nπx nπx
1 · cos dx = 0 , 1 · sin dx = 0 , 12 dx = 2l .
−l l −l l −l
The given system is orthogonal but not orthonormal. However, the system
n 1 1 cos nπx 1 sin nπx o
√ ,√ ,√ is orthonormal.
l l l l l
Example 223. (a) Haar systems The Haar system {hn } is defined as
follows:
Set h0 = 1. For n, k ∈ N with 0 ≤ k < 2n define hn on [0,1] by
n/2
2 , if x ∈ I(2k, n + 1)
h2n +k (x) = −2n/2 , if x ∈ I(2k + 1, n + 1)
0, otherwise
for the following reason. For an arbitrary trigonometric series, we define the
partial sums
N
1 X
sN (x) = a0 + (an cos nx + bn sin nx) . (4.13)
2 n=1
(We remind the reader that we have studied series and their partial sums in
Chapter 5.) Now let us compute, for N ≥ n ≥ 1, using the orthogonality of
346 Modern Engineering Mathematics
= 0 + an π + 0 = πan .
and if moreover
Z π Z π
f (x) cos nx dx = lim sN (x) cos nx dx ,
−π N →∞ −π
where
Z l
1
a0 = f (x) dx , (4.15)
l −l
Z l Z l
1 nπx 1 nπx
an = f (x) cos dx , bn = f (x) sin dx . (4.16)
l −l l l −l l
The following theorem gives conditions under which (4.17) holds, that is,
when the Fourier series of f on [−l, l] converges to f .
Note that in fact (i) can be viewed as a special case of (ii), since f (x) =
f (x+) = f (x−) whenever f is continuous at x.
Example 224. Expand the following functions in Fourier series.
(
0, −π < x < 0 ,
(a) f (x) = on [−π, π],
π − x, 0 ≤ x < π,
(
0 , −π < x < 0 ,
(b) f (x) = on [−π, π],
1, 0 ≤ x < π,
(
0 , −1 < x < 0 ,
(c) f (x) = on [−1, 1],
x, 0 ≤ x < 1,
(
0, −π < x < 0 ,
(d) f (x) = on [−π, π],
sin x , 0 ≤ x < π ,
π
− 4 , −π < x < 0 ,
(e) f (x) = 0 , x = 0, on [−π, π],
π
4 , 0 ≤ x < π,
(
−1 , −4 < x < 0 ,
(f) f (x) = on [−4, 4],
1, 0 ≤ x < 4,
1 π 1h 0
Z Z Z π i
bn = f (x) sin nx dx = f (x) sin nx dx + f (x) sin nx dx
π −π π −π 0
1 π
Z
1
= (π − x) sin nx dx = .
π 0 n
f(x)
7t
-+-----:_~7t~~o+-~7tr------+-~- x
3_5
- - - Fourier Approx imation
3 ---Actual Function
2
0 0 0
•••••• ••-'••••••••• • L • • • • • • • • • -'• • • • • • • • • •
=
----- --- .... --------- . .. ---------
~
1_5 0 0
~ - --------
0
~
<{
------ ---.----------
o
0
r - -------- -.-- -------
0
0
0
0
--------- -·---------- ..
0
0
0
0
0 0 0 0 0
0 0 0 0 0
0 0 0 0 0
______ _ _ _•0 _ _ _ _ _ _ _ ___ 0
!_ _ _ _ _ _ _ _ _ _ _•0 __ _____ _ 0 0
0 _5 0 0 0
0 0 0
0 0 0
0 0 0
0 0 0
0 0
-0-54L_--~-3
~---~ L__ _~_ _ _L __ _~
~----1 2---~--~4-
Time
(b) We have
π 0 π
1 π
Z Z Z Z
1 1h i
a0 = f (x) dx = f (x) dx + f (x) dx = 0 + 1 dx
π −π π −π 0 π 0
= 1,
π
1h 0
Z Z Z π
1 i
an = f (x) cos nx dx = f (x) cos nx dx + f (x) cos nx dx
π −π π −π 0
Z π h sin nx iπ
=0+ 1 cos nx dx = = 0,
0 n 0
1 π 1h 0
Z Z Z π i
bn = f (x) sin nx dx = f (x) sin nx dx + f (x) sin nx dx
π −π π −π 0
1 π
Z
1 h cos nx iπ
=0+ sin nx dx = −
π 0 π n 0
1 h cos nπ 1i
= − +
π n n
(−1)n 1
=− + , as cos nπ = (−1)n ,
nπ nπ
1
= (1 − (−1)n ) .
nπ
Since the given function is continuous except at x = 0, the Fourier series
converges to the given function for x 6= 0, and we have
∞
1 1 X 1 − (−1)n
f (x) = + sin nx .
2 π n=1 n
350 Modern Engineering Mathematics
For the graphs of f and of a partial sum of its Fourier series see Figures 4.2a
and 4.2b respectively.
f(x)
X
- lr 0
1.2 ~~~~===c====~~--r----:-----:-----:--~
- - - F ourier Approximation
- - - A c t ual F unct ion
.. ..
0 _8 -------- -·--------- .. --- --- --- ~---------
..
.. ..
---------·--------- .. --- --- ---·-- --- ---- ..
... ...
-------- ·--------- -. --- --- ---·-------- ...
. ... ...
0.2 ---------
..... .. .....
_ _ _ _ _ _ __ .,! _ _ _ _ _ _ _ _ _ ,!. _ _ _ _ _ _ _
-o_24L__ _ __~3---~-2~----~
1----o
~--~----2
~--~3~--_j4.
Time
through integration by parts. See Figures 4.3a and 4.3b for the graphs of the
function and of a partial sum of its Fourier series.
f(x)
1 ----------
~~---+----~----+-----~- X
- 7£ 0
- - - Fourier Approximation
- - - Actual Function
- --- -- ,-- -- -- --.- -- --- -- ,. -- -- -- -
' ' '
'' ''
.
'
0.8 -- - ---,----
'
'
'
. ...
'
''
-- --.- -- --- -- ,.- -- -- ---
. ' '
--- --
'
'
~ ---- ---
..
-·- -- ----- ... -------
(d) We have
Z π Z 0 Z π
1 1 1
a0 = f (x) dx = f (x) dx + f (x) dx
π −π π −π π 0
Z π
1 1 2
=0+ sin x dx = [− cos x]π0 = .
π 0 π π
Z π hZ 0 Z π
1 i
an = f (x) cos nx dx = f (x) cos nx dx + f (x) cos nx dx
π −π −π 0
Z π
1
= sin x cos nx dx
π 0
Z π
1
= [sin(n + 1)x + sin(1 − n)x]dx
2π 0
Thus,
∞
1 1 X 1 + (−1)n
f (x) = + sin x + cos nx
π 2 n=2
π(1 − n2 )
is the Fourier series for f . See the graph of the function in Figure 4.4a and
the graph of a partial sum of its Fourier series in Figure 4.4b.
f(x)
(e) We have
π
1 0 1 π
Z Z Z
1
a0 = f (x) dx =
f (x) dx + f (x) dx
π
−π π −π π −π
1 0 π 1 ππ
Z Z
π π
= − dx + dx − + = 0 .
π −π 4 π 0 4 4 4
354 Modern Engineering Mathematics
1.2 r-;=========r;---r-- -: - - : - -: - l
- - - Fourier Approximation
- - - Actual F unctio n
..... .. .
-------- ---------- .. ----------.----------
.
__ _, _________ _
...
0.6
.. ....
~
-0.2-4 -3 -2 -1 2 3 4
Tim e
Z π
1
an = f (x) cos nx dx
π −π
Z 0 Z π
1 1
= f (x) cos nx dx + f (x) cos nx dx
π −π π 0
1π 0 1π π
Z Z
=− cos nx dx + cos nx dx
π 4 −π π4 0
1 h sin nx i0 1 h sin nx iπ
=− + = 0.
4 n −π 4 n 0
Z π
1
bn = f (x) sin nx dx
π −π
Z 0
1 π
Z
1
= f (x) sin nx dx +
f (x) sin nx dx
π−π π 0
1 0 π 1 ππ
Z Z
= − sin nx dx + sin nx dx
π −π 4 π 0 4
1 h cos nx i0 1 h cos nx iπ
= −
4 n −π 4 n 0
1 cos nπ 1 1 2 2(−1)n
= − − cos nπ + = − .
4n 4n 4n 4n 4n 4n
Thus, the Fourier series for f is
∞
X 1
f (x) = sin(2n − 1)x .
n=1
2n −1
The graph of this function and of a partial sum of its Fourier series are given
Fourier Methods and Integral Transforms 355
f(x)
7ll4
X
- 7r 0 7r
- - 7ll4
- - - Fourier Approximation
.. .. ..
- --Actual Function
____ ____ .. ______ ___ __ ____ ___ .. __ _____ __
0.6
.... ... ... - -- - -- - - -J-- -- --- - -
-g 0.2
.. .. ..
---- ---- , - ----- --- . - ------ -- . -- ----- --
]f 0~--,r.---~---~----t---~.---~.~--~.~---1
-0.2
... ...
- -- - - -----:-- -- - --- --:----- - - -- -:--- --- - - -
... ..
. ---- - -
- ---- ---- ______ ___ __ ____ ___ .. ___ ____ _ - -- ----- _ _, __ ___- -- __,__ ____ - -- -·---
. ... ...
... - - ----- -- ,....-- - ----
.
...
-0.4
- ----- -- .
.. ------ --.--.. ------ --,--- . ------
-0.6 . .
- - ------ - - -.--
... ... ...
-0.8 .~,..........,,h-.-;~-.-,h~,.......-.H-- - - --- -- ~----.. - ·- --~--- .. - -- · - --:----.. - - ---
Time
(f) We have
Z 4 Z 0 Z 4
1 1 1 1 1
a0 = f (x) dx = f (x) dx + f (x) dx = [−x]0−4 + [x]40
4 −4 4 −4 4 0 4 4
= −1 + 1 = 0 .
1 0 1 4
Z Z
nπx nπx
an = − cos dx + cos dx
4 −4 4 4 0 4
1h 4 nπx i0 1h 4 nπx i4
=− sin + sin = 0.
4 nπ 4 −4 4 nπ 4 0
0
1 4
Z Z
1 nπx nπx
bn = f (x) sin
dx + f (x) sin dx
−44 4 4 0 4
1 0 1 4
Z Z
nπx nπx 1h 4 nπx i0
=− sin dx + sin dx = cos
4 −4 4 4 0 4 4 nπ 4 −4
1 4
h nπx i 4 1 1
− cos = [1 − (−1)n ] − [(−1)n − 1]
4 nπ 4 0 nπ nπ
2 2(−1)n 4 1
= − = .
nπ nπ π 2n − 1
Therefore, the Fourier series of f is
∞ h (2n − 1)πx i
4X 1
f (x) = sin .
π n=1 2n − 1 4
The graph of this function and of a partial sum of its Fourier series are given
in Figures 4.6a and 4.6b.
f(x)
X
-4 0 4
- -1
- - - Fourier Approximation
---Actual Function
... ... ...
-- --- --·- - - -- -- -·- ---- -- -'--- --- -
0
~
<{
' . . .
-0.5
... ... ... ...
--- --- - , -- --- -- y -- --- -- y - -- ---- r -- ----
0 • • •
-1 _5
-5 -4 -3 -2 -1 2 3 4 5
Time
Z π Z π
1 1 1 π
(g) a0 = f (x) dx = ex dx = (e − e−π ) ,
π −π π −π π
π π
(−1)n (eπ − e−π )
Z Z
1 1
an = f (x) cos nx dx = ex cos nx dx = ,
π −π π −π π(1 + n2 )
π π
(−1)n (e−π − eπ )
Z Z
1 1
bn = f (x) sin nx dx = ex sin nx dx = .
π −π π −π π(1 + n2 )
The Fourier series of f is
∞
eπ − e−π X h (−1)n (eπ − e−π ) (−1)n (e−π − eπ ) i
f (x) = + cos nx + sin nx .
2π n=1
π(1 + n2 ) π(1 + n2 )
See Figures 4.7a and 4.7b for the graph of the function and of a partial sum
of its Fourier series.
f(x)
- Jr 0
f (x + p) = f (x) for all x. Let us point out that often “period” is defined to
be the fundamental period.
Let f be an arbitrary function defined on (−l, l). Its Fourier series (if
convergent)
∞
1 X nπx nπx
a0 + an cos + bn sin
2 n=1
l l
Gibbs Phenomenon
Fourier Methods and Integral Transforms 359
25 rr===================,-----,-----,------,-----,-----,
- - - Fourier Approximat ion
- - - Actual Function
20 --------- ~- -------- ~ --------- :- ---------
'
..
15 --------- '~- -------- c--
...
'
------- , ---------
'' ''
.
.... ---------
'' ''
=
v
l .
~---------
' '
'' ''
3 ----~2----~1---~o~_ __ L_ _ _~2---~3---~4
- 54L_____~
Time
If we examine the graphs of the partial sums of the Fourier series in Ex-
ample 222, we observe that all of them are overshooting the true values of
the function f near its point of discontinuity. In fact, this phenomenon always
occurs when we approximate a discontinuous function with Fourier series. It
is known as the Gibbs phenomenon, in honor of Josiah William Gibbs, a
mathematical physicist working at Yale, who analyzed it prior to 1900, after
it had been discovered by Henry Wilbraham in 1848. One can show that the
overshooting amounts to approximately 9% of the size of the jump, that is,
of the difference |f (x+) − f (x−)| of the one-sided limits at the discontinuity
point x. The main point is that the amount of overshooting of the partial
sums sN does not decrease when N tends to infinity.
As an example, let us consider the Fourier series of the step function
−1 , if x ∈ [−π, 0),
f (x) = 0 , if x = 0,
1, if x ∈ (0, π],
The graph of s15 is given in Figure 4.8, and the graph of s100 is given in Figure
4.9. We observe the overshooting of the partial sum around the points where
f is discontinuous.
~ s(x)
.. .. ..
1.5 . ------ .-----,, -----.------,------.------ .------.------.
... ....
... ... ... ... ...
....----- -----.-----
.. -- ---·---------
.. .
. .
...
.. .. ..
-0.5 -------- - --- - ------ -- ------.----------.----- ----
... ..
.
:A ____ _____ ...._______ ___ .
. .,_ _______ _
-1 ... ....
..
-1.5 L __ _L __ __t__ __ l __ __ l __ __ l __ __ l __ __ l_ __ _ J . ..
-4 -3 -2 -1 0 2 4
Time
FIGURE 4.8: Fourier Approximation Plot for 15 Harmonics for Square Func-
tion
For this paragraph, we assume the reader to have some basic familiar-
ity with complex numbers. The sine and cosine are related to the complex
exponential function by
and, analogously,
Z π
f (x)einx dx = πan + iπbn , n > 0,
−π
Fourier Methods and Integral Transforms 361
,Suxfx)
1.5 , - - - - -, -- - - -, -- - - -, ------,-----,----- ,- - - -- ,- - - - - ,
----------:----------
' '
-0.5 ,--
' --- --- - -.---
' -- ---- - ',.' -- --- -- -- -- --- --- --.-----
'
' -- --- ,.'' -- ----- -- ,'' --- --- ---
'' '' '' '' '' ''
' ' ' ' '
' ' ' ' '
FIGURE 4.9: Fourier Approximation Plot for 100 Harmonics for Square Func-
tion
we see that the complex Fourier coefficients are related to the Fourier sine and
cosine coefficients by
1 1
cn = (an − ibn ) , c−n = (an + ibn ) ,
2 2 (4.19)
an = cn + c−n , bn = i(cn − c−n ) ,
cn einx + c−n e−inx = cn (cos nx + i sin nx) + c−n (cos nx − i sin nx)
= an cos nx + bn sin nx .
Therefore, the partial sums sN of the Fourier series can also be represented
as
N N
1 X X
sN (x) = a0 + (an cos nx + bn sin nx) = cn einx ,
2 n=1 n=−N
Over an interval (−l, l), l > 0, the complex form of the Fourier series is defined
as
∞
1 l
Z
X nπx inπx
cn e l , where cn = f (x)e− l dx , n = 0, ±1, ±2, ±3, . . .
n=−∞
2l −l
It can be verified that the set of functions { √12π einx }n∈Z is an orthonormal
set.
Sine and Cosine Fourier Series
We consider Fourier series of even and odd functions. Recall that if g is an
even function on (−π, π), then
Z 0 Z π
g(−x) = g(x) for x ∈ [0, π), g(x) dx = g(x) dx ,
−π 0
1 π 1 0 1 π
Z Z Z
a0 = f (x) dx = f (x) dx + f (x) dx
π −π π −π π 0
Z π
2
= f (x) dx ,
π 0
Z π
1 0 1 π
Z Z
1
an = f (x) cos nx dx = f (x) cos nx dx + f (x) cos nx dx
π −π π −π π 0
Z π
2
= f (x) cos nx dx ,
π 0
since the function h(x) = f (x) sin nx is odd. Consequently, the Fourier series
of an even function f is a cosine series,
∞
a0 X
+ an cos nx ,
2 n=1
Similarly we find that the Fourier series of an odd function f contains only
sine terms, hence it is a sine series
∞ Z π
X 2
bn sin nx , where bn = f (x) sin nx dx .
n=1
π 0
Indeed, in this case g(x) = f (x) cos nx is an odd function, while h(x) =
f (x) sin nx is an even function, so in particular
1 π 1 π 1 π
Z Z Z
a0 = f (x) dx = − f (x) dx + f (x) dx = 0 ,
π −π π 0 π 0
1 π 1 0 1 π
Z Z Z
an = f (x) cos nx dx = f (x) cos nx dx + f (x) cos nx dx
π −π π −π π 0
1 π 1 π
Z Z
=− f (x) cos nx dx + f (x) cos nx dx = 0 .
π 0 π 0
2 π 2 π 2
Z Z
2 1 π 2
a0 = f (x) dx = x dx = · x3 0 = π 2 ,
π 0 π 0 π 3 3
and
2 π 2 π 2
Z Z
an = f (x) cos nx dx = x cos nx dx
π 0 π 0
π
2 h x2 iπ Z
4
= sin nx − x sin nx dx
π n 0 nπ 0
Z π
4 π 4
= 0 + 2 x cos nx 0 − 2 cos nx dx
n π n π 0
4 4 π 4
= 2 (−1)n − 3 sin nx 0 = 2 (−1)n .
n n π n
Thus the Fourier series of f is given as the cosine series
∞
1 2 X (−1)n
π +4 cos nx .
3 n=1
n2
period l, that is, f (x+l) = f (x) for all x, and l is the smallest number satisfying
this condition. We define ω = 2π/l as the frequency corresponding to l. Let
∞
1 X
a0 + an cos(nωx) + bn sin(nωx) (4.21)
2 n=1
is called the phase angle form of the Fourier series. Indeed, if two pairs
(a, b) and (d, δ) of numbers are related by
a = d cos δ , b = −d sin δ ,
p b (4.23)
d = a2 + b2 , δ = arctan − ,
a
then
a cos(nωx) + b sin(nωx) = d cos(nωx + δ)
holds for all x, as a consequence of the trigonometric identities, so the series
(4.21) and (4.22) correspond term by term. The phase angle form of the Fourier
series is also called its harmonic form. It represents a periodic function as a
superposition of cosine waves. The term cos(nωx + δn ) is the nth harmonic,
dn is the nth harmonic amplitude and δn is the nth phase angle of f .
Note that the harmonic amplitude satisfies
dn = 2|cn | , (4.24)
where cn = (an − ibn )/2 is the nth complex Fourier coefficient of f as intro-
duced earlier.
The amplitude spectrum or frequency spectrum(Figure 4.10) of the
periodic function f is a plot of |cn | = dn /2 on the vertical versus n along the
horizontal axis. The phase spectrum of f is a plot of the points (n, δn ) for
n = 0, 1, 2, . . . , where δn = arctan(−bn /an ) is the nth phase angle of f .
Example 226. Find the complex Fourier series of f on the given intervals.
Furthermore find the frequency spectrum of the function.
(a) (
0 , −π < x < 0 ,
f (x) = on [−π, π].
x, 0 ≤ x < π,
(b) (
−1 , −2 < x < 0 ,
f (x) = on [−2, 2].
1, 0 ≤ x < 2,
Fourier Methods and Integral Transforms 365
(c) (
cos x , 0 < x < π/2 ,
f (x) = on [0, π].
0, π/2 ≤ x < π ,
1 i −inπ 1 h −inx 1 1 iπ
= e − e
2n 2π in in 0
1 i −inπ 1 −inπ 1
= e + 2
e −
2n 2πn 2πn2
1 + inπ −inπ 1
= 2
e − 2 , for n 6= 0,
2πn
Z π 2n π
1 π
c0 = x dx = .
2π 0 4
The complex Fourier representation of f becomes
∞
π 1 X 1
f (x) = + [(1 + inπ)e−inπ − 1]einx .
4 2π n2
n=−∞,n6=0
(b) We have
Z 2 Z 0 Z 2
1 −inπx/2 1h −inπx/2
i
cn = f (x)e dx = (−1)e dx + e−inπx/2 dx
4 −2 4 −2 0
i 1
= [−1 + einπ + e−inπ − 1] = [−2 + (−1)n + (−1)n ]
2nπ 2nπ
1 − (−1)n
= , for n 6= 0,
nπi
Z 2
1
c0 = f (x) dx = 0 .
4 −2
Thus
∞
X 1 − (−1)n inπx/2
f (x) = e .
n=−∞
nπi
0.7
0.6
0.5
0.4 n
1
Frequency Spectrum Co = 0 , ICnl = - ( -:)
0.3 n*pt
0..2
(1 − (−1)n )/nπ.
(c) Using cos x = (eix − e−ix )/2 we get
1 π 1 π/2
Z Z
−2inx
cn = f (x)e dx = cos xe−2inx dx
π 0 π 0
1 π/2 1 ix
Z Z π/2
−ix −2inx 1
= (e − e )e dx = (e(1−2n)ix − e−(1+2n)ix ) dx
π 0 2 2π 0
1 h 1 1 iπ/2 2ne−inπ + i
= e(1−2n)ix + e(1+2n)ix = .
2π i(1 − 2n) i(1 + 2n) 0 π(1 − 4n2 )
the function and its Fourier transform both contain all the information about
the function. One can compute the transform from the original function as
well as reconstruct the function from its transform, that is, one can invert the
transform.
In the previous section, we have studied the decomposition of a func-
tion into its Fourier series, which is a periodic function. This works well for
functions defined on a bounded interval, as we can always think of them as
periodically extended to the whole real line. In contrast to that, the Fourier
transform acts on arbitrary (non-periodic) functions.
converges.
For the definition of the Fourier transform, several variants are in common
use, see Remark 57 below. We choose the following one.
Definition 80. (Fourier transform)
Let the function f be defined on R and assume that it is integrable on R. The
function fˆ, defined on R by
Z ∞
ˆ
f (ξ) = f (t)e−iξt dt , (4.25)
−∞
that is, the real and imaginary parts of the integrand are evaluated separately
and yield the real and imaginary part of the integral, which is a complex
number (in this case, the number fˆ(ξ)).
368 Modern Engineering Mathematics
since |eix | = 1 for all real numbers x. Therefore and since f is assumed
to be integrable on R, the improper integral in (4.25) is defined. One then
infers from the properties of parameter-dependent integrals that the Fourier
transform fˆ is a continuous function. In addition, one can prove that fˆ(ξ)
tends to zero as ξ tends to ±∞. (The latter result is called the Riemann-
Lebesgue lemma.)
(iii) As it stands, the requirement that f has to be integrable on the whole
line is rather restrictive. For example, Definition 80 does not cover the case
when f is a constant function. Indeed, the Fourier transform of the constant
1 is defined, but it is no longer a function defined on R, but a more general
mathematical object (although it is called the Dirac function). This, however,
is outside the scope of this book.
Example 227. Find the Fourier transforms of the following functions.
(a) f (t) = e−|t| .
(
0, t<0
(b) f (t) = .
e−t , t≥0
(b) Let (
1, t ≥ 0
H(t) =
0, t < 0
Fourier Methods and Integral Transforms 369
be the Heaviside function (see Chapter 2). Then f (t) = H(t)e−t and
Z ∞ Z ∞ Z ∞
ˆ
f (ξ) = f (t)e −iξt
dt = −t −iξt
H(t)e e dt = e−t e−iξt dt
−∞ −∞ 0
Z ∞ it=∞
h 1 1
= e−(1+iξ)t dt = − e−(1+iξ)t = .
0 1 + iξ t=0 1 + iξ
(c) We obtain
Z ∞ Z a h −k it=a
fˆ(ξ) = f (t)e−iξt dt = ke−iξt dt = e−iξt
−∞ −a iξ t=−a
k h −iξa i 2k
=− e − eiξa = sin(aξ) ,
iξ ξ
since
eiξa − e−iξa
sin(aξ) = .
2i
Remark 51. (i) For a given function f , the Fourier transform (if defined)
yields a new function fˆ. We may thus view the Fourier transform as a mapping
whose domain and range are certain sets of functions. Such a mapping is
commonly called an operator. Let us denote it by F, so
F(f ) = fˆ . (4.26)
Although this is, in a strict sense, mathematically not correct (it confuses
the functions f and fˆ with their function values f (t) and fˆ(ξ)), it leads to a
concise way of writing formulas. In this notation, the result of Example 227
(a) becomes
2
F[e−|t| ] = .
1 + ξ2
In the next two theorems, we state some important properties of the
Fourier transform.
Theorem 46. Let f be a function which is integrable on R.
(a) Time shift. Let t0 be a real number, let g(t) = f (t − t0 ). Then
This means that the Fourier transform of the translated function f equals the
Fourier transform of f multiplied by a factor. In shorter notation, without
introducing the function g explicitly, (4.27) becomes
F[f (t − t0 )] = e−iξt0 fˆ(ξ) .
(b) Frequency shift. Let ξ0 be a real number, let g(t) = eiξ0 t f (t). Then
ĝ(ξ) = fˆ(ξ − ξ0 ) , ξ ∈ R, (4.28)
or, in shorter notation,
F[eiξ0 t f (t)] = fˆ(ξ − ξ0 ) .
(c) Scaling or dilation. Let a be a real number with a 6= 0, let g(t) = f (at).
Then
1 ˆ ξ
ĝ(ξ) = f , ξ ∈ R. (4.29)
|a| a
This states that the Fourier transform of the scaled function is obtained by re-
placing ξ by ξ/a in the Fourier transform of the original function and dividing
by the magnitude of the scaling factor.
The formulas in the theorem above are obtained from properties of the
integral. For example, (4.27) results from the computation
Z ∞ Z ∞
−iξt −iξt0
ĝ(ξ) = f (t − t0 )e dt = e f (t − t0 )e−iξ(t−t0 ) dt
−∞ −∞
Z ∞
−iξt0 −iξs −iξt0 ˆ
=e f (s)e ds = e f (ξ) ,
−∞
d2 ˆ
F[t2 f (t)] = − f (ξ) . (4.34)
dξ 2
In abstract terms, the right hand side of (4.37) defines the inverse F −1 of
the Fourier transform F. It is called the inverse Fourier transform
Z ∞
−1 1
(F [g])(t) = g(ξ)eiξt dξ . (4.38)
2π −∞
if
R ∞it exists, is called the principal value (or Cauchy principal value) of
−∞
h(x) dx. Thus, under the assumptions of Theorem 51 we also obtain the
formula Z ∞
1
f (t) = fˆ(ξ)eiξt dξ
2π −∞
at points t where f is continuous, provided we interpret the integral as its
principal value.
Remark 57. If one wants the frequency variable ξ to denote ordinary fre-
quency instead of angular frequency, one defines the Fourier transform by
Z ∞
fˆ(ξ) = f (t)e−2πitξ dt .
−∞
Fourier Methods and Integral Transforms 373
It is also possible to mix these variants. Therefore, when dealing with the
Fourier transform, one has to make sure which convention is used.
Localization and Uncertainty Principle
In this subsection we explain the fact that a function f and its Fourier
transform fˆ cannot both be concentrated on a small interval.
First, consider the dilation g(t) = f (at). For a > 1, g represents a com-
pression of f around t = 0 by the factor a. On the other hand, Theorem 46(c)
says that
1 ˆ ξ
ĝ(ξ) = f ,
|a| a
that is, we have to stretch fˆ by a factor a to obtain ĝ. If a < 1, g is a stretched
version of f while ĝ is a compressed version of fˆ.
Second, assume that fˆ(ξ) = 0 outside some interval [−l, l]. We say in
this case that f has bandwidth l. From the Fourier inversion formula which
becomes Z l
1
f (t) = fˆ(ξ)eiξt dξ
2π −l
one concludes, with the aid of a result of complex function theory which we
cannot present here, that f can be zero only at isolated points, so it spreads
out to infinity and in particular cannot have the property that f (t) = 0 outside
some interval [−M, M ].
Third, one can quantify this phenomenon. Consider the expression
Z ∞ Z ∞ −1
2 2
∆f = t |f (t)| dt · |f (t)|2 dt .
−∞ −∞
If the large values of f arise only at small values of t and f decays rapidly as t
gets large, the numerator will be small in comparison with the denominator, so
374 Modern Engineering Mathematics
4.3.2 Convolution
The convolution of two sequences a = {an } and b = {bn }, where n ranges
over all integers, is defined as
∞
X
(a ∗ b)k = aj bk−j ,
j=−∞
provided the infinite series converges. If aj and bj are non-zero only for j ≥ 0,
the convolution becomes the finite sum
k
X
(a ∗ b)k = aj bk−j .
j=0
(f + g) ∗ h = (f ∗ h) + (g ∗ h) ,
w = S[u] .
376 Modern Engineering Mathematics
∗ g(ξ) = fˆ(ξ)ĝ(ξ) .
f[ (4.43)
For convolution in the frequency domain, we have
1 ˆ
fcg(ξ) = (f ∗ ĝ)(ξ) . (4.44)
2π
Thus, under the action of the Fourier transform or its inverse, multiplica-
tion becomes convolution and vice versa. This is a major reason why convo-
lution plays a prominent role in the calculus of Fourier transforms.
∗ y(n) = x̂(n)ŷ(n) ,
xd (4.48)
where the real functions p(x), p0 (x), r(x) are continuous on [a, b], and p(x)
and r(x) are positive on [a, b].
Remark 59. Any differential equation of the form (4.49) can be written in
Sturm- Liouville form.
Indeed, first divide (4.49) by a(x) to obtain
00 b(x) 0 c(x) r(x)
y (x) + y (x) + +λ y = 0.
a(x) a(x) a(x)
α1 y(a) + β1 y 0 (a) = 0,
α1 y(b) + β1 y 0 (b) = 0, (4.51)
Remark 60. For different values of the constants α’s and β’s in the boundary
conditions (4.51) we have special types of boundary conditions.
For β1 = β2 = 0, (4.51) these are called Dirichlet boundary conditions.
For α1 = α2 = 0, (4.51) these are called Neumann boundary conditions.
Other types of boundary conditions that are often encountered are periodic
boundary conditions when
n2 π 2
λn = n = 1, 2, 3, . . . .
l2
The corresponding eigenfunctions are
nπx
yn (x) = sin , n = 1, 2, 3, ....
l
It can be proved in general that every eigenvalue of the regular Sturm-
Liouville problem is a real number.
(a) There exist an infinite number of real eigenvalues that can be arranged
in increasing order λ1 < λ2 < λ3 < · · · < λn < · · · such that λn → ∞
as n → ∞.
(b) For each eigenvalue there is only one eigenfunction (except for constant
multiples).
(c) Eigenfunctions corresponding to different eigenvalues are linearly inde-
pendent.
(d) The set of eigenfunctions corresponding to the set of eigenvalues is or-
thogonal with respect to the weight function p(x) on the interval [a, b].
The train’s whistle and the blinking of a car’s beam can be viewed as quan-
tities varying in time which contain information; they are examples of time
dependent signals. Early in human history, signals of smoke by day and of fire
by night have been used to transmit information. In recent times, telegraph,
telephone, radio, television and radar have been used as signal transmitters.
A radio signal consists of a sine or cosine wave with radio frequency, called
the carrier wave, which has been modulated with the information to be trans-
mitted.
Signals can be divided into two categories, analog signals (functions de-
fined on a continuum of numbers, for example an interval in R) and digital
signals, which are defined on a discrete set like the integers. In 1949, Claude
E. Shannon of Bell Telephone Laboratories published a mathematical result
now known as the Shannon sampling theorem. This result provided the foun-
dation for digital signal processing. It tells us that if the range of frequencies
of a signal measured in cycles per second does not exceed n, then the time-
continuous signal can be reconstructed with complete accuracy by measuring
its amplitude 2n times a second.
The study of signals is relevant not only in telecommunication, but also
in telemetry, astronomy, oceanography, optics, crystallography, geophysics,
bioengineering, bioinformatics and medicine.
382 Modern Engineering Mathematics
holds for some l > 0, and the smallest such number l is called the bandwidth
of the signal. This means that the total frequency content of the signal f lies
in the band (or interval) [−l, l]. Moreover, let us assume that f is integrable
and square integrable on R, that is, the signal f has finite energy. Let it be
recovered from its Fourier transform as
Z ∞
1
f (t) = fˆ(ξ)eiξt dξ .
2π −∞
Because of (4.54),
Z l
1
f (t) = fˆ(ξ)eiξt dξ . (4.55)
2π −l
where Z l
1
cn = fˆ(ξ)e−nπiξ/l dξ . (4.57)
2l −l
∞ nπ
X 1 1 i(lt−nπ)
= f [e − e−i(lt−nπ) ] ,
n=−∞
l lt − nπ 2i
This is the main content of Shannon’s sampling theorem which states that a
function of bandwidth l can be completely recovered by (4.60) from its values
at the points nπ/l, where n = 0, ±1, ±2, . . . . This result forms the basis for the
conversion between analog and digital signals. If we convert an analog signal f
of bandwidth l into a digital signal by evaluating it at times t = 0, ±π/l, ±2π/l,
we can convert it back to an analog signal without loss of information, at least
in principle. Note that an exact evaluation of (4.60)) involves an infinite sum
of values of f at arbitrarily large positive and negative times. Developing
suitable approximations, both from the theoretical and practical standpoints,
is one of the areas of signal analysis and signal processing.
4.7 Exercises
4.1. Discuss the relationship between linear independence and orthonormal-
ity. Can you convert an orthogonal system into an orthonormal system?
x
4.2. (a) Show
that
f (x) = e and g(x) = sin x are orthogonal on the interval
π 5π
, .
4 4
(b) Show that {cos x, cos 3x, cos 5x, . . .} is an orthogonal set on [0, π/2].
4.3. Expand the following functions into Fourier series on the given interval.
(a) f (x) = x + π, −π < x < π.
(b) f (x) = e−8x for −4 ≤ x ≤ 4.
0, −π < x < 0
(c) f (x) = .
x2 , 0 ≤ x < π
384 Modern Engineering Mathematics
4.5. Compare the graph of the function f (x) = x2 with the 3rd , 6th , 10th
and 13th partial sums of its Fourier series on the interval [−2, 2].
4.6. Write the complex Fourier series of the following functions.
4.7. Expand the following functions in an appropriate cosine and sine Fourier
series.
(a) f (x) = x3 , −π < x < π.
(
x − 1, −π < x < 0 ,
(b) f (x) =
x + 1, 0 ≤ x < π .
4.8. Let f (x) = 4 sin x, 0 < x < π, f (x + π) = f (x). Sketch this function and
its Fourier series. Find the frequency spectrum of f .
4.9. Let f be integrable on [−l, l].
Z l
if |f (x)|2 dx is finite and a0 , an and bn are its Fourier coeffi-
−l
cients.
(c) Show that an → 0 and bn → 0 as n → ∞.
4.10. Find the Fourier transforms of the following functions
Fourier Methods and Integral Transforms 385
4.11. Prove Theorem 46 (b): Let ξ0 be a real number, let g(t) = eiξ0 t f (t),
then ĝ(ξ) = fˆ(ξ − ξ0 ).
Calculate (f ∗ g)(x).
387
Chapter 5
Applied Partial Differential
Equations
389
390 Modern Engineering Mathematics
Solution:
ux = cos x cos t, uxx = − sin x cos t,
ut = − sin x sin t, utt = − sin x cos t,
and it is clear that
uxx = utt .
1 x2
Example 231. Show that u(x, t) = t− 2 e− t is a solution of the PDE
1
ut = uxx .
4
This type of equation is known as a one-dimensional heat equation or diffusion
equation.
Solution:
−1 −3 − x2 −5 x2
ut = t 2 e t + t 2 x2 e− t
2
−3 x2
ux = −2xt 2 e− t
x2 −5 x2
uxx = −2te− t + 4t 2 x2 e− t
so, clearly,
1
ut = uxx .
4
Example 232. Find the general solution of the PDE
uxx (x, y) = 0.
Solution: This equation means that ux (x, y) does not depend on x. Therefore,
ux (x, y) = A(y), where A(y) is an arbitrary function. Integrating the last
equation with respect to x (keeping y constant), it follows that the general
solution of the equation is given by
uyx + uy = 0.
Solution: In order to get the general solution, we convert this PDE into an
ordinary differential equation (ODE) by the substitution w = w(x, y) = uy .
Then the given equation becomes
wx + w = 0.
392 Modern Engineering Mathematics
w(x, y) = a(y)e−x
uy = a(y)e−x .
df
where f is any differentiable function so that = a(y) and g is any arbitrary
dy
function.
Boundary and initial conditions
Boundaries are a set of constraints that describe the nature of unknown func-
tion u(x, y) satisfying the PDE, on the boundary of the domain Ω, denoted
by ∂Ω. There are three important classes of boundary conditions:
1. Dirichlet conditions
2. Neumann conditions
∂u
(x, y) = h(x, y) for x, y ∈ ∂Ω
∂n
∂u
where denotes the derivative of u in the direction of the outward unit
∂n
normal vector to ∂Ω.
3. Robin mixed conditions
∂u
αu(x, y) + β (x, y) = l(x, y) for x, y ∈ ∂Ω
∂n
where α, β are nonzero constants or functions, and l(x, y) is a function defined
on ∂Ω.
Initial conditions
When PDEs involve the time variable t, then we have to consider initial con-
ditions called Cauchy conditions. These conditions specify the value of the
unknown function and its higher order derivative at the initial time t = t0 .
If the functions f (x, y), h(x, y), and l(x, y) are identically zero on their
domain ∂Ω, they are called homogeneous boundary conditions; otherwise they
are called non-homogeneous conditions.
A differential equation is said to be linear if the function F is algebraically
Applied Partial Differential Equations 393
linear in each of the variables u, ux , uy , uxx , uxy , uyy , ..., and if the coefficients
of u and its derivatives are functions only of the independent variables. An
equation that is not linear is said to be non-linear; a non-linear equation is
quasi-linear if it is linear in the highest order derivatives.
For example, an equation of the form
Parabolic if b2 − 4ac = 0
−(uxx + uyy ) = λu
394 Modern Engineering Mathematics
ut = a2 uxx
4. Wave equation
utt = c2 uxx
5. Schrodinger equation
6. Telegraph equation
7. Airy’s equation
ut + uxxx = 0
8. Beam equation
ut + uxxxx = 0
ut + div F (u) = 0
ut + uux = 0
ut − uxx = f (u)
ut + uux + uxxx = 0
Applied Partial Differential Equations 395
8. Maxwell’s equation
−1
div E = 0, curl E = Ht
c
1
div H = 0, curl H = Et
c
where E(x, y, z) and H = H(x, y, z) represent electric and magnetic field
in empty space, and c is the speed of the light.
9. Euler’s equations for incompressible, inviscid flow
ut + u.∇u = −∇p
div u = 0
where u(x, t) denotes the temperature distribution and k the thermal diffu-
sivity.
The equation, in its simplest form, goes back to the beginning of the 19th
century. Besides modelling temperature distribution it has been used to model
the following physical phenomena:
• Diffusion of one material within another such as smoke particles in air.
• Chemical reactions, such as the Belousov-Zhabotinsky reaction which
exhibits fascinating wave structure.
• Electrical activity in the membranes of living organisms, for example,
the Hodgkin-Huxley model.
396 Modern Engineering Mathematics
will also satisfy the heat equation and the boundary conditions. If we assume
that the above series is convergent and the initial condition (5.3) is satisfied,
we obtain
∞
X nπx
f (x) = u(x, 0) = cn sin , 0 < x < l.
n=1
l
Section 4.2.2 this shows that the above is the sine Fourier series of f (x).
Therefore, the coefficients cn are given by
Zl
2 nπx
cn = f (x) sin dx, n = 1, 2, . . . . (5.14)
l l
0
whose solution is 2
ω2 t
U (ω, t) = F (ω)e−a .
Now, from the inverse Fourier transform discussed in Chapter 4, it follows
that
Z∞ Z∞
1 1 2
ω 2 t iωx
u(x, t) = U (ω, t)e iωx
dω = F (ω)e−a e dω
2π 2π
−∞ −∞
Z∞ Z∞
1 2 2
ω 2 t iωx
= f (ξ)e−iω ξ e−a e dω
2π
−∞ −∞
Z∞ Z∞
1 2
ω 2 t iω(x−ξ)
= e−a e dω f (ξ)dξ
2π
−∞ −∞
Z∞
∞
Z
1 2 2
e−a ω t cos (ω(x − ξ)) dω f (ξ)dξ
=
π
−∞ 0
Z∞ (x−ξ)2
1
= √ e− 4a2 t f (ξ)dξ.
2c πt
−∞
In this section we solve the initial value and boundary value problems for the
heat equation using the Laplace transforms.
Applied Partial Differential Equations 399
ut = a2 uxx , 0 < x < 2, t > 0
u(0, t) = 0, u(2, t) = 1, t > 0 . (5.16)
u(x, 0) = 0, 0 < x < 2
Applying the Laplace transform to both sides of the heat equation and
using the initial condition, we have
d2 U
− sU = 0.
dx2
The general solution of this ordinary differential equation is
√ √
U (x, s) = c1 cosh( sx) + c2 sinh( sx).
we obtain √ √ #
∞
"
X e−2(n+1−x) s e−2(n+1+x) s
U (x, s) = − .
n=0
s s
Therefore, from the Laplace transform (Table 2.2) we get
∞
" √ ! √ !#
X
−∞ e−2(n+1−x) s −∞ e−2(n+1+x) s
u(x, t) = L −L
n=0
s s
∞
X 2n + 1 − x 2n + 1 + x
= erf √ − erf √
n=0
2 t 2 t
where X(x) and T (t) are functions of single variables x and t, respectively.
Differentiating (5.20) with respect to x and t, and substituting the partial
derivatives in equation (5.17), we obtain
00
X 00 (x) 1T (t)
= 2 . (5.21)
X(x) c T (t)
Equation (5.21) holds identically for every 0 < x < l. Notice that the left
side of this equation is a function which depends only on x, while the right
hand side is a function of only t.Since x and t are independent variables, this
can happen only if each function in both sides of (5.21) is equal to the same
constant λ. 00
X 00 (x) 1T (t)
= 2 = λ.
X(x) c T (t)
From the last equation we obtain the two ordinary differential equations
and
T 00 (t) − c2 λT (t) = 0. (5.23)
From the boundary conditions (5.18) it follows that
This implies
X(0) = X(l) = 0
since T (t) = 0 would imply u(x, t) = 0. Solving the eigenvalue problem (5.22)
with the above boundary conditions just as in Chapter 4, we find that the
eigenvalues λn and corresponding eigenfunctions Xn (x) are given by
nπ 2 nπx
λn = − , Xn (x) = sin , n = 1, 2, . . . .
l l
The solution of the differential equation (5.23) corresponding to the above λn
is given by
nπct nπct
Tn (t) = an cos + bn sin , n = 1, 2, ...
l l
where an and bn are constants to be determined. Therefore, we obtain a se-
quence of functions
un (x, t) = Xn (x)Tn (t)
or
nπct nπct nπx
un (x, t) = an cos + bn sin sin , n = 1, 2, . . . ,
l l l
each of each satisfies the wave equation (5.17) and the boundary conditions
(5.18). Since the wave equation and the boundary conditions are linear and
homogeneous, a function u(x, t) of the form
∞ ∞
X X nπct nπct nπx
u(x, t) = un (x, t) = an cos + bn sin sin (5.24)
n=1 n=1
l l l
will also satisfy the wave equation and the boundary conditions. If we assume
that the above series is convergent and that it can be differentiated term by
term with respect to t, from (5.24) and the initial conditions (5.19), we obtain
∞
X nπx
f (x) = u(x, 0) = an sin ,0<x<l (5.25)
n=1
l
and
∞
X nπc nπx
g(x) = ut (x, 0) = bn sin , 0 < x < l. (5.26)
n=1
l l
Using the Fourier sine series (from Chapter 4) for the functions f (x) and g(x)
from (5.25) and (5.26), we obtain
Zl Zl
2 nπx 2 nπx
an = f (x) sin dx, bn = g(x) sin dx, n = 1, 2, . . . .
l l nπc l
0 0
A formal justification that the function u(x, t) is the solution of the wave
equation is given by Theorem 45 in Chapter 4.
402 Modern Engineering Mathematics
Z∞
F (u(x, t)) = U (x, ω) = u(x, t)e−iωx dx
−∞
d
F (ut (x, t)) = (F (u(x, t)))
dt
d2
F (utt (x, t)) = (F (u(x, t))) = Utt (ω, t).
dt2
Let us now solve the wave equation with initial conditions
d2 U (ω, t)
+ c2 ω 2 U (ω, t) = 0.
dt2
The general solution is
Now,
C1 (ω) = U (ω, 0) = F (ω)
and
ωcC2 (ω) = Ut (ω, 0) = G(ω).
so
G(ω)
C2 (ω) = .
ωc
Applied Partial Differential Equations 403
Hence
1
U (ω, t) = F (ω) cos(ωct) +
G(ω) sin(ωct).
ωc
Applying the inverse Fourier transform, we get
Z∞
1 1
u(x, t) = F (ω) cos(ωct) + G(ω) sin(ωct) eiωx dω.
2π ωc
−∞
∂2φ ∂2φ
+ 2 = 0. (5.29)
∂x2 ∂y
" ∂
#
2 ∂x
The equation is also written as ∇ φ = 0, where ∇ = ∂ . This equation
∂y
has no dependence on time, just on the spatial variables x, y. The Laplace’s
equation describes steady state situations such as:
(i) Temperature distributions
. 1lX
y w = w0 szn -
b +----""---, a
w=O
w=O
a X
w= O
FIGURE 5.1: Laplace Equation with Dirichlet Equation
Applied Partial Differential Equations 405
w(x,y) is the displacement in
z-direction
w(0, y) = 0, for 0 ≤ y ≤ b.
w(x, 0) = 0, for 0 ≤ x ≤ a.
w(a, y) = 0, for 0 ≤ y ≤ b.
π
w(x, b) = w0 sin a, for 0 ≤ x ≤ a.
x
Solution by separation of variables
Let w(x, y) = X(x)Y (y) be solution of (5.29). Substituting it in (5.29), we get
X 00 Y + XY 00 = 0
X 00 Y 00
+ = 0
X Y
X 00 Y 00
=− = k
X Y
where k is a constant that is either equal to 0, or > 0, or < 0.
Case I: When k = 0, then
w(0, y) = 0 ⇒ B = 0 or C = D = 0.
If C = D = 0, then Y (y) ≡ 0, so w(x, y) ≡ 0.
Continuing with B = 0, we have
w(a, y) = 0 ⇒ ACay = 0
A = 0 or C = 0 ⇒ w(x, y) ≡ 0.
w(x, 0) = 0 ⇒ BC sinh αx = 0
B = 0 ⇒ w(x, y) ≡ 0.
w(x, 0) = 0 ⇒ BC sin αx = 0
B = 0 ⇒ w(x, y) ≡ 0.
w0 πx πy
w(x, y) = sin sinh .
sinh πb
a
a a
X 00 Y + XY 00 = 0
or
X Y 00
=− = −λ
X Y
X 00 Y 00
in which both and − must be constant because x and y are indepen-
X Y
dent. Then
X 00 + λX = 0
and
Y 00 − λY = 0.
From the boundary conditions
X(0) = X(a) = 0.
408 Modern Engineering Mathematics
X 00 + λX = 0, X(0) = X(a) = 0
with eigenvalues
n2 π 2
λn = , n = 1, 2, . . .
a2
and eigenfunctions
nπx
Xn (x) = sin , n = 1, 2, . . . .
a
For any eigenvalue, the corresponding equation for y is
n2 π 2
Y 00 − Y =0
a2
with general solutions
nπy nπy
Yn = cn e a + dn e− a ,
nπx nπy
= bn sin sinh , n = 1, 2, . . .
a a
which all satisfy the Laplace equation in the rectangle and the homogeneous
boundary conditions on the lower and vertical sides. To find a solution satis-
fying the condition on the sides y = b, we use superposition principle
∞
X nπx nπy
u(x, y) = bn sin sinh . (5.30)
n=1
a a
and thus
Za
2 nπx
bn = nπb
f (x) sin dx.
a sinh a
a
0
With this choice of bn , Equation (5.30) gives the solution of this Dirichlet
problem for a rectangle.
Example 236. Consider the following Laplace equation with mixed boundary
conditions
uxx + uyy = 0. 0 < x < a. 0 < y < b
ux (0, y) = 0, ux (a, y) = 0, 0 < y < a
u(x, 0) = 0, u(x, b) = f (x), 0 < x < a.
X Y 00
=− = −λ
X Y
X 00 + λX = 0
and
Y 00 − λY = 0.
From the boundary conditions
and
ux (a, y) = X 0 (a)Y (y) = 0
we have
X 0 (0) = X 0 (a) = 0
because Y (y) cannot be identically zero. Similarly, from
we have
Y (0) = 0.
X 00 + α2 X = 0, X 0 (0) = X 0 (a) = 0.
It follows that
Za
1
A0 = f (x)dx
ab
0
and
Za
nπb 2 nπx
An sinh = f (x) cos
a a a
0
or
Za
2 nπx
An = f (x) cos .
nπb a
a sinh 0
a
Solution: Let
Z∞
U (x, ω) = F(u(x, y)) = u(x, y)e−iωy dy
−∞
and
U (0, ω) = F({) =F (ω).
If we apply the Fourier transform to the Laplace equation, then in view of the
boundary conditions for the function u(x, y) at x = 0, we get the ordinary
differential equation
d2 U (x, ω)
− ω 2 U (x, ω) = 0, x > 0.
dx2
The general solution of of the above equation is given by
U (x, ω) = F (ω)e−|ω|x .
412 Modern Engineering Mathematics
and
d2 U
L(uxx ) =. (5.32)
dx2
From (5.31) and (5.32) and the boundary conditions with respect to y, we get
d2 U
+ s2 U = sin πx. (5.33)
dx2
The general solution of the corresponding homogeneous ordinary differential
equation (5.33) is given by
Now, using L (u(0, y)) = U (0, s) = 0 and L (u(π, y)) = U (π, s) = 0, we get
c1 = 0 and c2 can be any real number. For sake of simplicity, we take c2 = 0,
which yields
1
U (x, s) = sin πx
s2 − π 2
and, taking the inverse of both sides, we obtain
1
u(x, y) = L−∞ sin πx
s2 − π2
1 π
= sin πx L−∞
π s2 − π 2
1
= sin πx sinh πy.
π
∂2w ∂2w
+ = f (x, y).
∂x2 ∂y 2
One encounters this equation while studying electrostatic potential in the
presence of charge, gravitational potential in the presence of distributed mat-
ter, equilibrium displacement of a membrane under distributed forces, velocity
potential for an inviscid, incompressible, irrotational homogeneous fluid in the
presence of distributed sources of sinks, and state temperature in the presence
of thermal sources or sinks. For solutions we refer to Adzievski and Siddiqi
[1].
414 Modern Engineering Mathematics
dt = (365*24*60*60)/Nt;
% Length of each time step in seconds
(∼ 6.3 ∗ 10∧ 3 seconds, or ∼ 105 minutes)
K = 2*10∧ − 6;
% canonical K is 10e − 6m∧ 2/s
T = 15*ones(Nz+1,Nt+1);
% Create temperature matrix with N z + 1 rows, and N t + 1 columns
% Initial guess is that T is 15 everywhere.
time = [0:12/Nt:12];
T(1,:) = 15-10*sin(2*pi*time/12); Nt = 5000;
% Choose number of time steps
dt = (365*24*60*60)/Nt;
% Length of each time step in seconds
(∼ 6.3 ∗ 10∧ 3 seconds, or ∼ 105 minutes)
K = 2*10∧ − 6;
% canonical K is 10e − 6m∧ 2/s
T = 15*ones(Nz+1,Nt+1);
% Create temperature matrix with N z + 1 rows and N t + 1 columns
% Initial guess is that T is 15 everywhere.
time = [0:12/Nt:12];
T(1,:) = 15-10*\sin(2*pi*time/12);
% Set surface temperature
maxiter = 500
for iter = 1:maxiter
Tlast = T; % Save the last guess
T(:,1) = Tlast(:,end);
% Initialize the temp at t = 0 to the last temp
Nt = 5000;
% Choose number of time steps
dt = (365*24*60*60)/Nt;
% Length of each time step in seconds
(∼ 6.3 ∗ 10∧ 3 seconds, or ∼ 105 minutes)
K = 2 ∗ 10∧ − 6;
% canonical K is 10e − 6m∧ 2/s
T = 15*ones(Nz+1,Nt+1);
% Create temperature matrix with N z + 1 rows, and N t + 1 columns
416 Modern Engineering Mathematics
colorbar
figure(3)
depth = [0:dz:Nz*dz];
contourf(time,-depth,T);
title(‘Temperature plot (contourf)’)
colorbar
-2
-4
-6
-8
-10
0 20 60 110 1110 120
10
2D
30
40
50
60
111
10
90
100
0 2 4 6 II 10 12
-311 18
..co 16
-50
14
-QI
12
-70
-811
-911
-100
0 2 6 8 10 12
k = k+1;
Told = T;
for i = 2:n-1
for j = 2:n-1
T(i,j) = dt*((Told(i+1,j)-2*Told(i,j)+Told(i-1,j))/dx∧ 2 . . .
420 Modern Engineering Mathematics
+ (Told(i,j+1)-2*Told(i,j)+Told(i,j-1))/dy∧ 2) . . .
+ Told(i,j); end
end
error = max(max(abs(Told-T)));
end
subplot(2,1,1), contour(x,y,T)
title(‘Temperature (Steady State)’), xlabel(‘x’),
ylabel(‘y’), colorbar
subplot(2,1,2), pcolor(x,y,T), shading interp,
title(‘Temperature (Steady State)’), xlabel(‘x’),
ylabel(‘y’), colorbar
end
u(n,k+1) = 0;
end
plot(x,u), xlabel(‘x’),ylabel(‘y’)
1.-----~----~----~~----~----~
o..4 0.6
X
A~Ft>>,,,,,, ....
0 2 " 6 8 10 12 14
u(x)
~ ~I • • ~,<<'',,@1 1
>
0 2 " 6 8 10 12 14
u(x)
(b)
end
plot(x,u), xlabel(‘x’),ylabel(‘y’)
-IL6
-no
...
-·.~._.~._.----oD.3~._.~= ._'7-.---='._,=--='=._.-._"="-._j1
X
elseif(i==N-1)
A(i,i-1)=-1;
A(i,i)=2;
else
A(i,i-1)=-1;
A(i,i+1)=-1;
A(i,i)=2;
end
end
A=A/((delta_x)^2);
for i=1:N-1
b(i)=functionf(i*delta_x,k)
end
u=A^(-1)*b’;
for i=1:N+1
u_ex(i)=exact_solution((i-1)*delta_x,k);
end
for i=1:N+1
if (i==1)
u_dis(i)=0;
elseif(i==N+1)
u_dis(i)=0;
else
u_dis(i)=u(i-1,1);
end
end
for i=1:N+1
t(i)=(i-1)*delta_x;
end
norm_maxl2(j)=0;
for i=1:N+1
if (abs(u_dis(i)-u_ex(i)) > norm_maxl2(j))
norm_maxl2(j)=abs(u_dis(i)-u_ex(i));
end
end
norm_maxl2(j);
426 Modern Engineering Mathematics
norm_l2(j)=0;
for i=1:N+1
norm_l2(j)=norm_l2(j)+(u_dis(i)-u_ex(i))^2*delta_x;
end
norm_l2(j)=(norm_l2(j))^1/2);
norm_maxh1(j)=0;
for i=1:N
if (abs(((u_dis(i+1)-u_ex(i+1))
-(u_dis(i)-u_ex(i)))/delta_x) > norm_maxh1(j))
norm_maxh1(j)=abs(((u_dis(i+1)-u_ex(i+1))
-(u_dis(i)-u_ex(i)))/delta_x);
end
end
norm_maxh1(j);
norm_h1(j)=0;
for i=1:N
norm_h1(j)=norm_h1(j)+(((u_dis(i+1)-u_ex(i+1))
-(u_dis(i)-u_ex(i)))/delta_x)^2*delta_x;
end
norm_h1(j)=(norm_h1(j))^(1/2);
figure
hold on
plot(t,u_ex,’blue’, t,u_dis,‘red’)
ylim([-0.02 0.12]);
hold off
end
plot(log(M), -log(norm_maxl2),‘blue’,log(M), -log(norm_l2), ‘red’,
log(M), -log(norm_maxh1), ‘cyan’, log(M), -\log(norm_h1),
‘magenta’, log(M), 2*log(M),‘black’)
function f=functionf(x,k);
if (k==1)
f=2;
end
if (k==2)
f=-6*x+12*x^2;
end
Applied Partial Differential Equations 427
function u_ex=exact_solution(x,k);
if (k==1)
u_ex=x*(1-x);
end
if(k==2)
u_ex=x^3*(1-x);
end
••
....
0.1
D.JlB
D.Jlll
OJ)t
D.D2
0.1
insulated strip connects the two semi-infinite plates and is held at a constant
potential 100V . Find the potential and the electric field in the region bounded
by the plates.
Figures 5.17 plot the potential and show that the potential V has no local
maximum or minima; all extremes are on the boundaries. The solution of the
Laplace equation is the most featureless function possible, consistent with the
boundary conditions: no hills, no valleys, only the smoothest surface.
430 Modern Engineering Mathematics
~1
/
~
~
0 ~1 Q2 Q3 Q5 1
~·
~6 ~7 ~ ~
100
80
100
60
> 50
> 40
0
0
20
0
X [m] 4 -1 y [m)
Example 239. Find the general solution of the partial differential equation
potential
80
0.5
60
.;, 0
>.
-0.5 20
0
2 3 4
x [m]
∂2u ∂u ∂2u
% + b = T .
∂t2 ∂t ∂x2
The parameter b is normally determined from physical experiments.
External Forcing
It is easy to include an external force acting on the string. Say we have a
vertical force f˜ij acting on mass mi . This force affects the vertical component
of Newton’s law and gives rise to an extra term f˜(x, t) on the right-hand side
of (5.1). In this model we would add a term f˜(x, t).
Waves on membrane
Elastic waves in rod
434 Modern Engineering Mathematics
Consider an elastic rod subject to a hammer impact at the end. This exper-
iment will give rise to an elastic deformation pulse that travels through the
rod. A mathematical model for longitudinal waves along an elastic rod starts
with the general equation for deformations and stresses in an elastic medium,
where % is the density, u the displacement field, the stress tensor, and f body
forces. The latter has normally no impact on elastic waves. For stationary
deformation of an elastic rod, σxx = Eux , with all other stress components
being zero. Moreover, u = u(x)i. The parameter is known as Youngs modulus.
Assuming that this simple stress and deformation field, which is exact in
the stationary case, is a good approximation in the transient case with wave
motion. (5.34) simplifies to
∂2u
∂ ∂u
ρ 2 = E .
∂t ∂x ∂x
p = K∇ · u, (5.38)
Applied Partial Differential Equations 435
Taking the divergence of this equation, using ∇ · u = p/K from (5.38), gives
the acoustic approximation to elastic waves:
1
ptt = K∇ · ∇p . (5.39)
%
under the assumption that the relative spatial gradient ∇%1 = %2 ∇% is small.
This approximation results in the simplified equation
K
ptt = ∇2 p + s. (5.42)
%
The acoustic approximations to seismic waves are used for sound waves in
the ground, and the Earth’s surface is then a boundary where p equals the
atmospheric pressure po such that the boundary condition becomes p = po .
Anisotropy p
Quite often in geological materials, the effective wave velocity c = K/% is
different in different spatial directions because geological layers are compacted
such that the properties in the horizontal and vertical directions differ. With
z as the vertical coordinate, we can introduce a vertical wave velocity cz and
a horizontal wave velocity ch , and generalize (G) to
p+ + ∇ · (pu) = 0 (5.44)
put + pu.∇u = ∇p (5.45)
p = p(p). (5.46)
These equations are often referred to as the Euler equations for the motion
of a fluid. The parameters involved are the density %, the velocity u, and the
pressure p. Equation (5.45) reflects mass balance, (K) is Newton’s second law
for a fluid, with frictional and body forces omitted, and (M ) is a constitutive
law relating density to pressure by thermodynamics considerations. A typical
model for (M ) is the so-called isentropic relation, valid for adiabatic processes
where there is no heat transfer:
1/r
P
% = po
p0
p = po p 1/r .
( po )
po and p0 are reference values for p and % when the fluid is at rest, and γ is the
ratio of specific heat at constant pressure and constant volume (γ =√5/3 for
air). It is modeled by a standard linear wave ptt = c2 ∇2 p where C = rpo /po
is the speed of sound in the fluid.
5.9 Exercises
Separation of variables method for heat equation
In Exercises 5.1 through 5.7 use the separation of variables method to solve
the heat equation
ut = a2 uxx , 0 < x < l, t > 0.
√
5.1. a = 3, l = π, u(0, t) = u(π, t) = 0, u(x, 0) = x(π − x)
subject to:
Use the Fourier transform method to solve the indicated heat equation, subject
to the given initial and boundary value conditions.
3 π
x, 0<x<
2 4
π 3π
5.19. u(x, 0) = sin x, ut (x, 0) = 1, <x< .
4 4
3(π − x), 3
π<x<π
4
x, π
0<x<
5.20. u(x, 0) = π 2 , ut (x, 0) = 0, 0 < x < π.
π − x, <x<π
2
5.21. Use the Fourier transform to solve the following wave equations subject
to the initial and boundary conditions.
5.23. utt (x, t) = c2 uxx (x, t) + f (x, t), 0 < x < ∞, t > 0
u(x, 0) = 0, ut (x, 0) = 0
ux (0, t) = f (t), 0 < t < ∞
(Hint: Use the Fourier sine transform)
Use the Laplace transform to solve the initial boundary value problem for the
wave equation on the interval (0, 1), subject to the given conditions.
In the following problems, solve the Laplace equation in [0, a] × [0, b] subject
to the given boundary conditions.
5.35. u(x, 0) = f (x), u(x, b) = 0, 0 < x < a; u(0, y) = 0, u(a, y) = 0, 0 < y <
b.
5.38. ux (0, y) = u(0, y), ux (0, y) = 0, 0 < y < b; u(x, 0) = 0, u(x, b) = f (x),
0 < x < a.
5.42. D = {(x, y) : 0 < x < ∞, 0 < y < 2}; u(0, y) = 0, for 0 < y < 2,
u(x, 0) = f (x), u(x, 2) = 0 for 0 < x < π.
5.43. D = {(x, y) : 0 < x < ∞, 0 < y < ∞}; u(0, y) = e−y , for u(π, y) = e−y
for 0 < y < ∞ and u(x, 0) = e−x for 0 < x < ∞.
1, |x| < 1
5.44. D = {(x, y) : − ∞ < x < ∞, 0 < y < ∞}; u(x, 0) = .
0, |x| > 1
1
5.45. D = {(x, y) : − ∞ < x < ∞, 0 < y < ∞}; u(x, 0) = .
4 + x2
5.46. D = {(x, y) : − ∞ < x < ∞, 0 < y < ∞}; u(x, 0) = cos x.
Applied Partial Differential Equations 441
[2] N. Asmar, Partial Differential Equations with Fourier Series and Bound-
ary Value Problems, Second Edition, Pearson, 1992.
[3] W. E. Boyce, R. C. Diprima, Elementary Differential Equations and
Boundary Value Problems, Pearson, 2010.
443
Chapter 6
Algorithmic Optimization
445
446 Modern Engineering Mathematics
∂f
Gi = Gi (x) = , (1 ≤ i ≤ n). (6.1)
∂xi
∂2f
Hij = Hij (x) = . (6.2)
∂xi xj
With the gradient ad Hessian in hand, we can write the linear and quadratic
terms in the Taylor expansion of f
1
f (x + h) = f (x) + G(x)T h + hT H(x)h + . . . . (6.3)
2
The gradient vector points in the direction of steepest ascent or (increase),
and its negative points in the direction of steepest decrease.This important
fact is proved by taking any unit vector u, and considering how the function
changes locally at x in the direction of u. This can be seen by computing the
directional derivative
d
f (x + tu) |t=0 . (6.4)
dt
By using the Taylor formula (6.3), with h replaced by tu, we obtain
1
f (x + h) = f (x) + tG(x)T u + t2 uT H(x)u + . . . (6.5)
2
whence
d
f (x + tu) = G(x)T u + tuT H(x)u + . . . . (6.6)
dt
Algorithmic Optimization 447
Using t = 0, we obtain
d
f (x + tu) |t=0 = G(x)T u (6.7)
dt
as the rate of change of f at x in the direction of u. By the Cauchy-Schwarz
inequality,
That is, this rate of change cannot exceed kG(x)k. On the other hand, we can
attain this upper bound by taking u to be the unit vector in the direction of
G(x).
An obvious strategy for locating a minimum point of f is to start at
any point x and determine the direction in which f decreases at the fastest
rate, that is, in the direction of −G(x). A line search can be carried out on
{x − tG(x) : t > 0}; we start over by computing a new direction of steepest
descent, and so on.
and
H(x∗ ) positive-definite.
A common strategy in minimization problem is to assume that our function
is locally approximately a quadratic polynomial. A general quadratic function
of n variables is of the form
1 T
f (x) = x Ax − bT x + a. (6.9)
2
Here, a is scalar and b is a constant vector having n components. The matrix
A is n × n, symmetric, and constant (constant entries).The right side of the
448 Modern Engineering Mathematics
equation displays (from left to right) the term of exact degree 2, the terms of
exact degree 1, and the term of degree 0. In the case of two variables, the first
terms are the three types x21 , x22 , and x1 x2 .
Because quadratic functions should be good local models for any given
function that is twice differentiable, we will derive some basic facts about
them. First, the full form of f is
n n n
1 XX X
f (x) = xi xj Aij − bi xi + a (6.10)
2 i=1 j=1 i=1
To find the critical point of f , that is, the point where the derivative of f
vanishes, we set G(x) = 0 and note that this occurs when Ax = b, or in other
words, when
x = A−1 b (6.14)
Example 242. Write the quadratic function
in the standard vector-matrix notation, and find the minimizer and the min-
imum value for the function.
Solution:
− 32
4 −1 x1
f (x) = (x1 , x2 ) − 2 (x1 , x2 ) +1
−1 3 x2 1
whereby
− 32
4 −1
A= ,b=
−1 3 1
From (6.13) and (6.14), the minimizer can be found by solving the linear
system
Ax = b
Algorithmic Optimization 449
or
− 32
4 −1 x1
= . (6.15)
−1 3 x2 1
Applying the usual Gauss Elimination algorithm, only one row operation is
required to replace the coefficient matrix in upper triangular form. This gives
T
∗ ∗ ∗T
7 5 T
x = x1 , x2 = − , = (−0.31818, .22727) . The minimal value for
22 22
7 5 13
f is f (x∗ ) = f − , 22 = ≈ .29546.
22 44
It is instructive to compare the algebraic solution with the minimization
procedure learned in multi-variable calculus. The critical points of f (x1 , x2 )
are found by setting both partial derivatives equal to zero
∂f
= 8x1 − 2x2 + 3 = 0
∂x1
∂f
= −2x1 + 6x2 − 2 = 0
∂x2
or
4x1 − x2 = − 23
−x1 + 3x2 = 1
This is precisely the same linear system we already constructed in (7.63).
To check whether x∗ is a (local) minimum, we need to analyze the Hessian
matrix, which is the symmetric matrix of second order derivatives evaluated
5 T
at x∗1 , x∗T 7
2 = − 22 , 22
∂2f ∂2f
∂x21 ∂x2 ∂x1 8 −2
H= = = 2A
∂2f ∂2f −2 6
∂x1 ∂x2 ∂x22
The pivots, i.e., the diagonal entries of D, are all positive, and hence A is
positive definite. Therefore, f (x) has a unique minimizer x∗ = (x∗1 , x∗2 , x∗3 ),
which is found by solving the linear system Ax = b. The solution is then
quickly obtained by forward and backward substitution:
with
f (x∗ ) = f (2, −3, 2) = −11
H(xk )δ + G(xk ) = 0
that is
x1 = x∗ = A−1 b
Algorithmic Optimization 451
df (xk )
= G(xk )uk+1 < 0
dλ uk+1
to give minimizer λk .
4. Set xk = xk−1 + λk uk
5.Test
for convergence:
if
xk − xk−1
< 1 , or
G(xk )
< 2 , or f (xk ) − f (xk−1 ) < 3
then STOP and x∗ ' xk
else go to Step 6.
6.Set k = k + 1 and go to Step 2.
the direction of steepest descent is derived below. At x0 we seek the unit vector
u such that for F (λ) = f (x0 + λu), the directional derivative
df (x) dF (0)
= = G(x)u
dλ u dλ
assumes a minimum value with respect to all possible choices for the unit
G(x0)
vector u at x0 . Clearly for the particular choice u = − the directional
kG(x0)k
0
dF (0) G(x )
derivative at x0 is given by = −G(x) = − kG(x0 )k = least
dλ kG(x0 )k
value.
Thus this particular choice for the unit vector corresponds to the direction
of steepest descent. The search direction
G(x0 )
u=−
kG(x0 )k
u1 = −G x0
uk+1 = G xk + βk uk
and
1 2
F (λ) = f (x0 + λu1 ) = (10 − 5λ) + (10 − 5λ) (−5) + 25
2
For optimal descent
dF df (x)
(λ) = = −5 (10 − 5λ) + 25 = 0
dλ dλ u1
This gives
1 5 1 0
λ1 = 1, x = and G(x ) =
−5 −5
Iteration 2:
G(x1 )
2
2 1 1 0 25 −5 −5
u = −G(x ) + u =− + =
kG(x0 )k
2 −5 25 0 5
2 1 2 5 −5 5(1 − λ)
x = x + λu = +λ =
−5 5 −5(1 − λ)
and
1
F (λ) = f (x1 + λu2 ) = 25(1 − λ)2 − 50(1 − λ)2 + 50(1 − λ)2
2
An explicit and technical meaning has been assigned to the term. It means
finding the maximum value of a linear function of n variables over a convex
polyhedral set in Rn .
The goal of an LP problem is to maximize or minimize (optimize) a linear
objective function f in n decisions variables x1 , x2 , ..., xn subject to a set of
m constraints.
Let c = (c1 , c2 , ..., cn )T ∈ Rn , b = (b1 , ..., bm )T ∈ Rm , and A = (aij ) is an
m × n matrix. This is called a standard maximization problem if it has
the form: Find the maximum of
f = c1 x1 + c2 x2 + ... + cn xn (6.19)
f = cT x, x ∈ Rn , (6.21)
subject to the constraint
Ax ≤ b, and x ≥ 0
v = sup cT x : x ∈ K .
(6.24)
TABLE 6.1
The company expects net profits of $20 and $30, respectively, on each unit
of P1 and P2 sold. Write down the LP problem How should the company
schedule production in order to maximize net weekly profit?
Solution: The model translates directly into linear programming problem in
standard form with n = m = 2 in (6.19) and (6.20).Let x1 and x2 represent,
respectively, the number of units of products P1 and P2 produced per week.
The company’s objective is to maximize
such that
x1 + 2x2 ≤ 40 (hours, M1 )
x1 + x2 ≤ 30 (hours, M1 ).
f = 6x1 + 14x2
Algorithmic Optimization 457
TABLE 6.2
6 14 0 0 0 0
2 1 1 0 0 12
2 3 0 1 0 15
1 7 0 0 1 21
0 0 12 15 21
x = (x1 , x2 , x3 , x4 , x5 )T ≥ 0
Our first vector will be x = (0, 0, 12, 15, 21)T . All these data are summarized
in the first tableau
Every step of the simplex method begins wit a tableau. The top row con-
tains coefficients that pertain to the objective function F . The current value
of F (x) = cT x is displayed in the top right corner. The next m rows in the
tableau represent a system of equations embodying the quality constraints.
458 Modern Engineering Mathematics
TABLE 6.3
cT 0 F (x)
A I b
x (nonbasic) x (basic)
0 ≤ x3 = 12 − x2
0 ≤ x4 = 15 − 3x2
0 ≤ x5 = 21 − 7x2
x2 ≤ 12 x2 ≤ 15 x2 ≤ 3
TABLE 6.4
6 0 0 0 −2 42
13
7 0 0 1 − 17 9
11
7 3 0 1 − 37 6
1 7 0 0 1 21
0 3 9 6 0
The new basic variables are x2 , x3 , and x4 , and we must now determine the
next tableau in accordance with the preceding five rules. In order to satisfy
Rule 5, we note that x2 = (21 − x5 )/7. When this is substituted in F , we find
a new form for the objective functions:
The situation presented now is similar to that of the beginning. The nonbasic
variables are x1 and x5 . Ant increase in x5 will decrease F (x), and so it is x1
that is now allowed to become a basic variable. Hence, we hold x5 fixed at 0
and allow x1 to increase as much as possible. These constraints apply:
0 ≤ x3 = 9 − 13
7 x2
0 ≤ x4 = 6 − 11
7 x2
0 ≤ 7x2 = 21 − x1
These lead to
63 42
x1 ≤ x1 ≤ x1 ≤ 21
13 11
The new basic variable x1 is allowed to increase to only 42/11, and new values
x3 , x4 , and x5 are computed from tableau or from the constraints equations
immediately above. The new x-vector is
T
x = 42/11 27/11 21/2 0 0
460 Modern Engineering Mathematics
The nonbasic variables are now x4 , and x5 . To satisfy Rule 5, we use the
substitution x1 = (7/11)(6 − x4 ). then
Summary
On the basis of this example and the explanation, we can summarize the
work to be done on any given tableau as follows:
1. If all coefficients in F (that is, the top row in the tableau) are ≤ 0, then
the current x is the solution.
2. Select the nonbasic variable whose coefficient in F is positive and as
large as possible. This becomes a new basic variable. Call it xj .
3. Divide each bi by the coefficient of the new basic variable in that row,
aij . The value assigned to the new basic variable is the least of ratios. Thus,
if bk /akj , we set xj = bk /akj .
4. Using pivot element akj , create 00 s in column j of A with Gaussian
elimination steps.
or equivalently
Find w = (wj ) ∈ Rn , z = (zj ) ∈ Rn
satisfying
w − Mz = q
w, z ≥ 0 (6.26)
T
w z=0
We denote the above LCP by the pair (q, M ). The name comes from the
third condition, the complementarity condition which requires that at least
one variable in the pair (wj , zj ) should be equal to 0 in the solution of the
problem, for each j = 1 to n.
Example: The Obstacle Problem
The obstacle problem consists of finding the equilibrium position of an
elastic membrane that is held at a fixed position on its boundary and which
lies over an obstacle.
Consider stretching an elastic string fixed at the endpoint (0, 0), and (0, 4)
over an obstacle defined by a function ψ (in this example, we use ψ(x) =
1 − (x − 2.2)2 . Notice that the position of the string will be defined by ψ(x)
for x between unknown points P and Q. , and that in the intervals 0 ≤ x ≤ P ,
and Q ≤ x ≤ 4, the string lie along straight line segments connecting (0, 0) to
(P, ψ(P )) and (Q, ψ(Q)) to (4, 0), respectively. If we represent the equilibrium
position of the string by the function u, then u must satisfy the following
conditions.
This representation of the problem is complicated by the presence of the
free boundaries P and Q. The complementarity framework allows a simpler
representation, which does not require free boundaries. First, note that since
there is no downward force on the string, u00 (x) ≤ 0 for all x, except possibly at
x = P or x = Q where u00 may be discontinuous. Also, note that u(x) ≥ ψ(x)
everywhere. Finally, at each point x, either u00 (x) = 0 or u(x) = ψ(x). Thus,
if we ignore momentarily the discontinuity of u00 at P and Q, we see that u
must satisfy the conditions
u(x) ≥ ψ(x), 0 ≤ x ≤ 4
00
u (x) ≤ 0
(6.27)
(u(x) − ψ(x))u00 (x) = 0
u(0) = u(4) = 0.
u0 = un = 0
ui−1 − 2ui + ui+1
≤0 i = 1, ..., n − 1
h2
ui − ψ(xi ) ≥ 0
ui−1 − 2ui + ui+1
(ui − ψ(xi )) = 0.
h2
Using the simple change of variables zi = ui − ψ(xi ), this system is equivalent
to the linear complementarity problem (q, M ), where M is an (n − 1) × (n − 1)
tridiagonal matrix and q is an (n − 1)-vector defined by
2 −1 0 0 ... 0
−1 2 −1 0
0 −1 2 −1 0
M = ,
. . . . .
. . 2 −1
0 −1 2
−2ψ(x1 ) + ψ(x2 )
ψ(x1 ) − 2ψ(x2 ) + ψ(x3 )
q=
ψ(xn−3 ) − 2ψ(xn−2 ) + ψ(xn−1 )
ψ(xn−2 ) − 2ψ(xn−1 )
The columns z = (z1 , ..., zn−1 ) of this LCP then gives the discrete approxi-
mation to u at the interior points by the relation ui = zi +ψ(xi ), i = 1, ..., n−1.
or equivalently
F (x∗ )T , (x − x∗ ) ≥ 0 ∀x ∈ K
(6.29)
where K is a closed convex set, F is a given continuous function from K to
Rn , and h., .i denotes the inner product in n dimensional Euclidean space.
In geometric terms, the variational inequality (6.28) states that F (x∗ )T
is orthogonal to the feasible set K at the point x∗ . This formulation is par-
ticularly convenient because it allows for a unified treatment of equilibrium
problems and optimization problems. Indeed, many mathematical problems
can be formulated as variational inequality problems. Some examples are given
below.
subject to: x ∈ K
Proof. Let φ(t) = f (x∗ + t(x − x∗ )), for t → [0, 1]. Since φ(t) achieves its
minimum at t = 0, 0 ≤ φ0 (0) = ∇f (x∗ )T .(x − x∗ ), that is, x∗ is a solution of
(6.31).
f (x) ≥ f (x∗ ), ∀x ∈ K
Indeed, for instance, there was a time when airline terminal passengers
formed separate queues in front of check-in-counters. But now we see invari-
ably only one line feeding into several counters. This is the result of the real-
ization that a single line policy serves better for the passengers as well as the
airline management.
Such a conclusion emanated from analyzing the mode by which a queue is
formed and a service is provided. The analysis is based on building a math-
ematical model representing the process of arrival of passengers who join the
queue, the rules by which they are allowed into service, and the time it takes
to serve the passengers. In management science terminology a waiting line is
referred to as a queue, and the body of knowledge dealing with waiting lines is
known as queuing theory. In the early 1900s A.K. Erlang, a Danish telephone
engineer, began a study of congestion and waiting completion of telephone
calls. Since then, queuing theory has grown far more sophisticated and has
been applied to wide variety of waiting line situations.
We identify the unit demanding service, whether it is human or otherwise,
as the customer. The unit providing service is known as the server .This
terminology of customers and servers is used in a generic sense regardless of
the nature of the physical context. Some examples are given below:
(a) In communication systems, voice or data traffic queue up for lines for
transmission. A simple example is the telephone exchange.
(b) In a manufacturing system with several work stations, units completing
work in one station wait for access to the next.
(c) Vehicles requiring service wait for their turn in garage.
(d) Patients arrive at a doctor’s clinic for treatment.
Numerous examples of this type are everyday occurrence. While analyz-
ing them we can identify some basic elements of the systems. These are the
following:
• Input Process. In most cases the arrival are products of external factors.
Therefore, input is described in terms of random variables that can
represent either the number arriving during a time interval or the time
interval between successive arrivals.
• Customer waiting time. From a customer view, time spent in the queue
and in the system are two characteristics of importance.
Let Tq and T be the time a customer spends in queue and in the system,
respectively. We assume that the system operates according to a “first come,
first served” (FCFS) queue discipline. With an FCFS queue discipline, the
waiting time for service (Tq ) of an arriving customer is the amount of time
required to serve the customers already in the system. The total time T in
the system is Tq plus the service time.
The ratio of arrival rate to service rate plays a significant role in measuring
the performance of queueing systems.
arrival rate
ρ = traffic intensity = .
service rate
6.7.1 Queue
M/M/1
The M/M/1 is the simplest of the queuing models used in practice.
λ2
Lq = E(Qq ) = (6.33)
µ (µ − λ)
Now, let Tq and T be the time a customer spends in queue and in the
system respectively. We assume that the system operates according to a “first
come, first served” (FCFS) queue discipline. With an FCFS queue discipline,
the waiting time for service (Tq ) of an arriving customer is the amount of time
required to serve the customers already in the system. The total time T in the
system is Tq plus the service time. When there are n customers in the system,
since service times are exponential with parameter µ, the total service time
of n customers is Erlang with probability density
µn xn−1
fn (x) = e−µx .
(n − 1)!
ρ λ
Wq = E(Tq ) = =
µ (1 − ρ) µ (µ − λ)
And, since the total time in the system, T, is the sum of Tq and the service
time, we get
λ 1 1
W = E(T ) = + = (6.35)
µ (µ − λ) µ µ−λ
Combining the result (6.32) and (6.35), we note the relationship
L = λW (6.36)
Lq = λWq (6.37)
Example 246. An airport has a single runway. Airplanes have been found
to arrive at the rate of 15 per hour and it is estimated that each landing
takes 3 minutes. Assuming a Poisson process for arrivals and an exponential
distribution for landing times, we use the M/M/1 model to determine the
following performance measures.
60
service rate = /hour.
3
λ 3
utilization = ρ = = .
µ 4
(b) Expected number of airplanes waiting to land:
2
ρ2 (0.75)
Lq = = = 2.25.
1−ρ 0.25
(d) Probability that the waiting will be more than 5 minutes, 10 minutes, No
waiting.
6.7.2 Queue
M/M/s
The multiserver queue M/M/s is the most used model. It aims at an-
alyzing service stations with more than one server such as banks, checkout
counters in stores, check-in counters in airports. The arrival of customers is
assumed to follow a Poisson process, service times are assumed to have an
exponential distribution, and the number of servers is s. These servers are as-
sumed to provide service independently of each other. The arriving customers
are assumed to form a single queue and the one at the head of the waiting
line enters the service as soon as a server is free.
Let λ be the arrival rate and µ the service rate. Denoting by L and Lq
the mean number of customers in the system and the number in the queue,
λ λ
respectively, and setting ρ = and α = following (6.32), we get
sµ µ
ραs p0
L=α+
s!(1 − ρ)2
ραs p0
Lq = (6.38)
s!(1 − ρ)2
where "s−1 #−1
X αr αs α
p0 = + 1− .
r=0
r! s! s
The estimated waiting time in the the queue, will be
α s p0
Wq = E(Tq ) = . (6.39)
s!sµ(1 − ρ)2
Comparing (6.38) with (6.39), we have again Little’s law formula
Lq = λWq .
Let Tq be the waiting time and Fq (t) = P [Tq ≤ t] be its distribution function.
Then
α s p0
Fq (t) = 1 − e−sµ(1−ρ)t .
s!(1 − ρ)
Example 247. In the airport problem of example, how would the perfor-
mance measures change if there are two runways while assuming the same
arrival and service rates?
(a) Runway utilization:
arrival rate = 15/hour (λ),
service rate = 20/hour (µ),
number of servers = 2(s)
λ 3
utilization of each runway = ρ = =
sµ 8
470 Modern Engineering Mathematics
"
3 2
−1 #−1
3 3
= 1+ + 4 1− = 0.4545
4 2 8
h i
3 3 2
8 4 (0.4545)
Lq = 2 = 0.1227.
2 58
(c) Expected waiting time.
h i
3 2
s
α p0 4 (0.4545)
Wq = = 2
s!sµ(1 − ρ)2 2 × 2 × 20 1 − 38
= 0.00818 hour = 0.49 minute.
(d) Probability that the waiting time will be more than 5 minutes? 10
minutes? No waiting?
αs p0
P (no waiting) = Fq (0) = 1 −
s!(1 − ρ)
3 2
4(0.4545)
=1−
2(1 − 3/8)
= 0.7955.
α s p0
P (Tq > t) = e−sµ(1−ρ)t .
s!(1 − ρ)
3 2
4 (0.4545) −2( 1 )( 5 )5
P (Tq > 5 minutes) = e 3 8
2(5/8)
= 0.1245.
Definition 86. The most used norms on RN are the l2 and l∞ norms. For
x = (x1 ,x2 , ..., xn )t are defined by
n
( n )1/2
X X
kxk∞ = max |xi | , kxk1 = |xi | , kxk2 = x2i
1≤i≤n
i=1 i=1
kAk ≥ 0.
kAk = 0 if and only if A = 0, the matrix with 0 entries.
kA + Bk ≤ kAk + kBk .
kABk ≤ kAk kBk .
The only norms we consider are those that are natural consequences of the
vector norms l2 and l∞ . Let A = (aij ) be a These are n × n matrix, then the
l∞ norm of A is
n
X n
X q
kAk∞ = max |aij | , kAk1 = max |aij | , kAk2 = ρ (AT A)
1≤i≤n 1≤j≤n
j=1 i=1
472 Modern Engineering Mathematics
where ρ AT A is the largest eigenvalue of the positive and definite matrix
AT A. If A is a real symmetric and positive definite matrix, then kAk2 =
|λmax | = ρ (A)
Example 249. Compute the l∞ , l1 , and l2 norms of
1 1 0
A= 1 2 1
−1 1 2
Solution:
3
X
|a1j | = |1| + |1| = 2,
j=1
3
X
|a2j | = |1| + |2| + |1| = 4,
j=1
3
X
|a3j | = |−1| + |1| + |2| = 4
j=1
3
X
kAk∞ = max |aij | = 4
1≤i≤3
j=1
3
X
|ai1 | = |1| + |1| + |−1| = 3,
i=1
3
X
|ai2 | = |−1| + |2| + |2| = 4,
i=1
3
X
|ai3 | = |1| + |2| = 3
i=1
3
X
kAk1 = max |aij | = 4
1≤j≤3
i=1
1 1 −1 1 1 0 3 2 −1
AT A = 1 2 1 1 2 1 = 2 6 4
0 1 2 −1 1 2 −1 4 5
To calculate |λmax | we need the eigenvalues of AT A. If
det(AT A − λI) = 0
Algorithmic Optimization 473
or
3−λ 2 −1
det 2 6−λ 4
−1 4 5−λ
= −λ3 + 14λ2 − 42λ = −λ(λ2 − 14λ + 42) = 0
Then √
λ = 0 or λ = 7 ± 7,
so
√ √ o √
q r n q
kAk2 = ρ (AT A) = max 0, 7 − 7, 7 + 7 = 7 + 7 ≈ 3.108
Ax = b (6.40)
TABLE 6.5
k 0 1 2 3 4 5
(k)
x1 0.000 0.6000 1.0473 0.9326 1.0152 0.9890
(k)
x2 0.000 2.2727 1.7159 2.053 1.9537 2.0114
(k)
x3 0.000 −1.1000 −0.8052 −1.0493 −0.9681 −1.0103
(k)
x4 0.000 1.8750 0.8852 1.1309 0.9739 1.0214
k 6 7 8 9 10
(k)
x1
(k)
x2
(k)
x3
(k)
x4 0.9944 1.0036 0.9989 1.0006 0.9998
by solving for xi in each row. That is, for an initial guess x(0) = (0, 0, 0, 0)t
we compute the iterate x(1) as follows
Solution: The decision to stop after ten iterations was based on the criterion
(10)
x − x(9)
∞ 8.0 × 10−4
x(10)
= < 10−3
∞
1.9998
TABLE 6.6
k 0 1 2 3 4 5
(k)
x1 0.000 0.6000 1.030 0.0065 1.0009 1.0001
(k)
x2 0.000 2.3272 2.037 2.0036 2.0036 2.0000
(k)
x3 0.000 −0.9893 −1.014 −1.0025 −1.0003 −1.0000
(k)
x4 0.000 0.8789 0.9844 0.9983 0.9999 1.0000
becomes
(D − L − U )x = b
Then one can notice that the Jacobi iterative method is
x(k+1) = T x(k) + c (6.49)
with
T = D−1 (L + U ) and c = D−1 b
and, concretely, in the following example.
1
− 15 3
0 10 0 5
1 0 1
− 3 25
T = 11 11 11 and c = 5
−1 1
0 1 − 11
5 10 10 10
0 − 38 1
8 0 15
8
while the Gauss-Seidel method is also an iterative method of the form (6.49)
with
−1 −1
T = (D − L) U and c = (D − L) b
Theorem 57. For any x(0) ∈ Rn , the sequence x(k) k≥1 defined in (6.41)
converges to the unique solution of x = T x + c and, therefore, to the solution
of system (6.40), if and only if the spectral radius of T , ρ(T ) < 1.
Ax = b.
Given an initial guess x0 to this system, and the Krylov subspace
Km (A, r0 ) = r0 , Ar0 , A2 r0 , ..., Am−1 r0
which yields
xm = x0 + V m ym
−1
ym = Hm (βe1 ), e1 = (1, 0, ..., 0).
Algorithm
1. Compute r0 = b − Ax0 , β = kr0 k2 , and v1 = r0 /β
2. Define the m × m matrix Hm = (hij ); set Hm = 0
3.For j = 1, 2, ..., m Do:
4. Compute wj = Avj
5. For i = 1, ..., j Do:
6. hij = (wj , vi )
7. wj = wj − hij vj
8. EndDo
9. Compute hj+1 ,j = kwj k2 . If hj+1 ,j = 0 set m = j and GoTo 12
10. Compute vj+1 = wj /hj+1 ,j
11. EndDo
−1
12. Compute ym = Hm (βe1 ) and xm = x0 + Vm ym .
−∇φ(x) = b − Ax = r
480 Modern Engineering Mathematics
where r is known as the residual vector for x. Note that r = 0 if and only if x is
the solution, and so the size of r measures, in a certain sense, how accurately
x comes to solving the system. Moreover, the residual vector indicates the
direction of steepest decrease in the quadratic function, and is thus a good
choice of direction to head off in search the true minimizer.
The initial result is the gradient descent algorithm, in which each successive
approximation uk to the solution is obtained by going a certain distance in
the residual direction:
The conjugate gradient method is one of the best iterative techniques for
solving linear systems
Ax = b
with A, as an n × n Symmetric Positive Definite matrix. The method can
be viewed as an orthogonal projection technique onto the Krylov subspace
Km (r0 , A) where r0 is the initial residual. In the conjugate gradient method
the update of the iterate is obtained from (6.52) by replacing rk with a new
direction pk that is not parallel to that of the gradient. These methods were
originally proposed by Hestenes and Stielfel [20] (see also Hestenes [21]) as
direct methods for symmetric definite linear systems, as (in the absence of
round-off errors) they produce the exact solution in a finite number of steps.
Nowadays, they are implemented as iterative schemes which are capable of
producing very accurate results in a small number of iterations.
In the Conjugate Gradient (CG), method, the directions {pk } are A-
conjugate, that is, they satisfy the orthogonality property (pj , Apm ) = 0 for
m 6= j. In particular
The idea of the method is based on the following remark. Let p0 , ..., pm be lin-
early independent vectors, x0 being an initial guess, and construct the solution
x by successive approximation, with the kth iterate having the form
where αk are non-zero real numbers. The idea is not to try to specify the
conjugate basis vectors in advance, but rather to successively construct them
during the course of the algorithm. We begin, merely for convenience, with an
initial guess x0 = 0 for that solution. The residual vector r0 = b − Ax0 = b
indicates the direction of steepest decrease of φ(x) at x0 , and we update the
original guess by moving in this direction, taking p1 = r0 = b as our first
conjugate direction. The next iterate is x1 = x0 + α1 p1 , and we choose the
parameter α1 so that the corresponding residual vector
r1 = b − Ax1 = r0 − α1 p1
Algorithmic Optimization 481
is such that
r1
is as close to zero as possible. This occurs when r1 is or-
thogonal to r0 , and so we require
T
2 T
0 = r0 r1 =
r0
− α1 r0 Ap1
2
=
r0
− α1 (r0 , p1 )
2
=
r0
− α1 (p1 , p1 ).
Therefore
0
2
r
α1 = 1 1
(p , p )
and so
0
2
r
x = x + 1 1 p1
1 0
(r , r )
is our new approximation to the solution. We can assume that α1 6= 0, since
otherwise the residual r0 = 0. In this case, x0 = 0 would be the exact solution
of the system, and there would be no reason to continue the procedure.
Now let us set p2 = r1 + s1 p1 , where the scalar factor s1 is determined by
the orthogonality requirement
0 = p1 , p2 = r1 + s1 p1 , p1 = r1 , p1 + s1 p1 , p1
so
r1 , p1
s1 = − .
(p1 , p1 )
Now, using the orthogonality of r0 and r1 , we get
T r 0 − r 1
T 1
2
r1 , p1 = r1 Ap1 = r1 = −
r1
α1 α1
while
T 1
r0
2
r1 , p1 = p1 Ap1 =
α1
Therefore, the second conjugate direction is
1
2
r
2 1 1
p = r + s1 p , where s1 = 2 .
kr0 k
We then update
x2 = x1 + α2 p2 = x0 + α1 p1 + α2 p2 = α1 p1 + α2 p2
r2 = b − Ax2 = r1 − α2 Ap2
482 Modern Engineering Mathematics
and so
1
2
r
α2 = 2 2
(p , p )
Continuing in this manner, at the kth stage, we have already constructed the
conjugate vectors p1 , ..., pk , and the solution approximation xk as a suitable
linear combination of them. The next conjugate direction given by
k
2
r
k+1 k k
p = r + sk p , where sk = 2
krk−1 k
results from the A-orthogonality requirement: pi , pk = 0 for i < k. The
updated solution approximation
k
2
r
k+1 k k+1
x = x + αk+1 p , where αk+1 = k+1 k+1
(p ,p )
7. pk+1 = rk + sk pk .
Observe that the algorithm does not require solving any linear systems:
apart from multiplication of a matrix times a vector to evacuate Apk , all other
operations are rapidly evaluated Euclidean dot products. Unlike Gaussian
Algorithmic Optimization 483
Solution: Implement the conjugate gradient method, starting from the initial
guess x0 = (0, 0, 0)T . The corresponding residual vector is r0 = b − Ax0 =
b = (1, 2, −1)T . The first conugate direction is p1 = r0 = (1, 2, −1)T . Use the
formula to obtain the updated approximation to the solution
3
0
2
r
1 2
6
x1 = x0 + 1 1 p1 = 2 = 3 .
(r , r ) 4
−1 − 12
T
noting that p1 , p1 ) = p1 Ap1 = 4. In the next stage of the algorithm, we
1 5
1 1
compute the residual r = b − Ax = − 2 , −1, − 2 . The conjugate direction
is
3
− 12
1
2
r
15 1 4
p2 = r 1 + 1
2x =
−1 + 2 2 = 3
0 6 2
kr k −1
− 52 − 15
4
T
which, as designed, satisfies the conjugacy condition p1 , p2 = p1 Ap2 = 0.
Each entry of the ensuing approximation
1
2 3 3 7
r
2 15 4 3
x2 = x1 + 2 2 p2 = 3 + 27 2 3
= 14
(p , p )
2 3
− 32 4 − 15 4 − 17
3
Since we are dealing with a 3 × 3 system, we will recover the exact solution
by one more iteration of the algorithm.The new residual is r2 = b − Ax2 =
484 Modern Engineering Mathematics
which can be, easily checked, is conjugate to both p1 and p2 . Thus, the solution
is obtained from
7 10
−
2
2
r
3 20 9 2
x3 = x2 + 3 3 p3 = 14 9
3 + 15
10
9 =
5
(p , p )
− 17 2 − 10 −6
3 9
so
wk , wk
ŝk = .
(wk−1 , wk−1 )
Thus
C −1 rk−1 , C −1 rk−1
r̂k−1 , r̂k−1 wk−1 , wk−1
α̂k = = =
(C T pk , C −1 Apk )
(p̂k , p̂k ) T
C T pk , C −1 A (C −1 ) C T pk
and
k wk−1 , wk−1
α̂ = .
(pk , pk )
Further
x̂k = x̂k−1 + α̂k p̂k
so
C T x̂k = C T x̂k−1 + α̂k C T p̂k
and
xk = xk−1 + α̂k pk .
Also,
r̂k = r̂k−1 − α̂k Âp̂k
so T
C −1 rk = C −1 rk−1 − α̂k C −1 A C −1 p̂k
that is
rk = rk−1 − α̂Apk .
Finally
p̂k+1 = r̂k + ŝk p̂k
and
C T pk+1 = C −1 rk + ŝk C T pk
yields
T
pk+1 = C −1 C −1 rk + ŝk pk
or T
pk+1 = C −1 wk + ŝk pk .
Example 254. The next example illustrates the effect of preconditioning on
a poorly conditioned matrix. The linear system Ax = b with
0.2 0.1 1 1 0 1
0.1 4 −1 1 −1 2
A= 1 −1 60 0 −2 and b = 3
1 1 0 8 4 4
0 −1 −2 4 700 5
486 Modern Engineering Mathematics
0.01062616286)T .
Solution: The matrix A is symmetric and positive
definite
but is ill-
conditioned with condition number K∞ (A) = kAk∞
A−1
∞ = 13961.71.
We will use tolerance 0.01 and compare the results obtained from the Jacobi,
Gauss-Seidel iterative methods and from the conjugate gradient method with
C −1 = I. Then we precondition by choosing C −1 as D−1/2 , the diagonal ma-
trix whose diagonal entries are the reciprocal of the positive square roots of
the diagonal entries of the positive definite matrix A. The results are presented
in Table...The Conjugate method gives the most accurate approximation with
the smallest number of iterations.
Method Number of Iterations
x(k) − x∗
∞
Jacobi 49 0.00305834
Gauss-Seidel 15 0.02445559
Conjugate gradient 5 0.00629785
Conjugate gradient 4
Preconditioned 0.00009312
x = x0 + V m y
Algorithmic Optimization 487
So
b − Ax = b − A(x0 + Vm y)
= r0 − AVm y
= βv1 − Vm+1 H̄m y
= Vm+1 βe1 − H̄m y .
xm = x0 + V m ym
Algorithm (GMRES)
1.Compute r0 = b − Ax0 , , β = kr0 k2 , and v1 = r0 /β
4. Compute wj = Avj
Rjj−1 : V j → V j−1
is a restriction operator;
(iii) solve the the defect problem
x̄(j) = xn(j)
1
+ Πjj−1 x(j−1)
where
Πjj−1 : V j → V j−1
is a prolongation operator.
2. Post-smoothing on the fine grid
Set x̂0(j) = x̄(j) and do n2 times
6.11 Exercises
Analysis of a Quadratic Function
6.1. Find the minimum value of the function f (x1 , x2 , x3 ) = x21 + 2x1 x2 +
3x22 + 2x2 x3 + x23 − 2x1 + 3x3 + 2. How do you know that your answer
is really the global minimum ?
6.3 For each matrix A, vector b,and scalar c, write out the quadratic function
f (x) given by (6.9). Then either find the minimizer x∗ and minimum
value f (x∗ ), or explain why there is none.
1
4 −12 −2
a. A = ,b= ,c=3
−12 45 2
3 2 4
b. A = ,b= ,c=0
2 1 1
3 −1 1
4
c. A = −1 2 −1 ,b= , c = −3
1
1 −1 3
−1
2
d. A =
,
b=
−3 , c = 0
4x1 + x2 − x3 ≥ −3
(d) f = 2x1 + x2 + 2x3 , x1 − x2 + 2x3 ≤ 5 .
3x1 + 2x2 + x3 ≤ 10
6.5. Find the first two iterations of the Jacobi method for the following linear
system, using x(0) = 0
Algorithmic Optimization 491
a.
3x1 − x2 + x3 = 1
3x1 + 6x2 + 2x3 = 0
3x1 + 3x2 + 7x3 = 4
b.
10x1 − x2 =9
−x1 + 10x2 − 2x3 = 0
−2x2 + 10x3 = 4
c.
10x1 + 5x2 =6
5x1 + 10x2 − 4x3 = 25
−4x2 + 8x3 − x4 = − 11
− x3 + 5x4 = − 11
6.7. Solve the following linear systems Ax = b, using the conjugate gradient
method, keeping track of the residual vectors and solution approximation
as you iterate
a.
3 −1 2
A= , b=
−1 5 1
b.
6 2 1 1
A= 2 3 −1 , b = 0
1 −1 2 −2
c.
6 −1 −1 5 1
−1 7 1 −1 2
A=
−1
, b =
1 3 −3 0
5 −1 −3 6 −1
492 Modern Engineering Mathematics
493
494 Bibliography
[15] Y. Saad, Iterative Methods for Sparse Linear Systems, PWS, Publishing
Company, 1996.
[16] A. H. Siddiqi, Functional Analysis with Applications, Springer, Nature,
2017.
[17] Stephen C. Billupsa, Katta G. Murtyb, Complementarity problems, Jour-
nal of Computational and Applied Mathematics 124: 303-318, 2000.
[18] M. Rosenlicht, Introduction to Analysis, Dover Mathematics series, 1968.
[19] M. Reed and B. Simon, Functional Analysis, (Methods of Modern Math-
ematics), Academic Press, 1980.
495
496 Modern Engineering Mathematics
first derivative:
f (x + h) − f (x)
f 0 (x) = lim . (7.1)
h→0 h
This means that
f (x + h) − f (x)
(7.2)
h
can be a good candidate for an approximation of f 0 (x) for sufficiently small h.
In order to determine how good this approximation is, we use the well-known
Taylor formula.
then we write
|f (x)| = O(|g(x)|). (7.6)
If we take n = 1 in (7.3), we can obtain
f (x + h) − f (x)
f 0 (x) ≈ , forward difference approximation.
h
Replacing h by −h in (7.7), we obtain
f (x) − f (x − h)
f 0 (x) = + O(h). (7.9)
h
Computational Numerical Methods in Engineering 497
TABLE 7.1
Example 257. Use Euler’s method to approximate the solution to the initial
value problem
dy
(
= y − t2 + 1, 0 ≤ t ≤ 2,
dt (7.33)
y(0) = 0.5
with k = 0.2. The exact solution is y(t) = (t + 1)2 − 0.5et . Table 7.1 shows
comparison between the approximate values at tn and the exact values.
Note that the error grows slightly as the value of t increases. This controlled
error growth is a consequence of the stability of Euler’s method, which implies
that the error is expected to grow no worse than it would in a linear manner.
Runge-Kutta Methods
Second order Runge Kutta method
The second order Runge-Kutta method is also known as the improved
Euler’s method. Starting from the initial point (t0 , y0 ), we compute two
slopes:
With these slopes, we define the next values of the dependent variables to be
sn1 + sn2
yn = y0 + k . (7.35)
2
An analysis using Taylor’s expansion reveals an improvement in the estimate
for the truncation error. For the second order Runge Kutta method, we have
TABLE 7.2
With these slopes we define the next value of the dependent variable:
sn1 + 2sn2 + 2sn3 + sn4
yn = y n−1 +k . (7.38)
6
For the fourth order Runge-Kutta method, we have
Example 258. Use the Runge-Kutta method of order four to obtain approx-
imation to the solution of the initial value problem
dy
(
= y − t2 + 1, 0 ≤ t ≤ 2,
dt (7.40)
y(0) = 0.5
with k = 0.2. The exact solution is y(t) = (t + 1)2 − 0.5et . Table 7.2 shows
comparison between the approximate values at tn and the exact values.
502 Modern Engineering Mathematics
Multistep methods
In previous sections we have discussed numerical procedures for approximating
the solutions of the initial value problem in which the data at the point t = tn
are used to compute an approximate value of the solution y(tn+1 ) at the next
point t = tn+1 . In other words, the computed value of y at any mesh point
depends only on the data at the preceding mesh point.
These are called one-step methods. However, once approximate values
of the solution y have been obtained at a few points beyond t0 , it is natural to
ask whether we can make use of some of this information, rather than just the
value at the last point, to compute the approximate value at the next point.
More precisely, if y1 at t1 , y2 at t2 ,. . . , yn at tn are known, how can we use
this information to determine yn+1 at tn+1 ? Methods that use information
at more than the last mesh point are referred to as multistep methods. In
this section we describe two types of multistep methods: Adams methods and
backward differentiation formulas.
Adams methods
Recall that Z tn+1
y(tn+1 ) − y(tn ) = y 0 (t)dt (7.41)
tn
where y(t) is the solution of the initial value problem (7.27). The basic idea
of an Adams method is to approximate y 0 (t) by a polynomial Pk (t) of degree
k and to use the polynomial to evaluate the integral on the right side of
(7.41). The coefficients in Pk (t) are determined by using k + 1 previously
calculated data points. For example, suppose that we wish to use a first degree
polynomial P1 (t) = At + B. Then we need only the two data points (tn , yn )
and (tn−1 , yn−1 ). Since P1 is to be an approximation to y 0 , we require that
P1 (tn ) = f (tn , yn ) and that P1 (tn−1 ) = f (tn−1 , yn−1 ). Now, denoting f (tn , yn )
by fn , A and B must satisfy the equations
Atn + B = fn (7.42)
Atn−1 + B = fn−1 .
f0 = f (t0 , y0 ) = 0 + 1 − 1 = 0
f1 = f (t1 , y1 ) = 0.2 + 1.02140000 − 1 = 0.22140000
f2 = f (t2 , y2 ) = 0.4 + 1.09181796 − 1 = 0.49181796
f3 = f (t3 , y3 ) = 0.6 + 1.22210646 − 1 = 0.82210646.
and
uij+1 − 2uij + uij−1
δy uij = (7.54)
h2
to approximate the derivatives uxx and uyy in (. . . ), respectively, we obtain
the five point difference scheme
ui+1j − 2uij + ui−1j uij+1 − 2uij + uij−1
+ = −fij , 1 ≤ i, j ≤ J − 1 (7.55)
h2 h2
or
ui+1j + ui−1j + uij+1 + uij−1 − 4uij = −h2 fij . (7.56)
The boundary condition (7.50) is replaced by
ui0 = g(xi , 0) = gi0 , 0 ≤ i ≤ J (7.57)
and the right hand side by
fij = f (xi , yj ), 1 ≤ i, j ≤ J − 1. (7.58)
The local truncation error
1
Tij = [u(xi+1 , yj ) + u(xi−1 , yj ) + u(xi , yj+1 ) + u(xi , yj−1 )
h2
−4u(xi , yj )] + f (xi , yj ). (7.59)
Assuming that the derivatives of u are continuous up to order 4 in both x
and y, we find, by Taylor expansion, that
h2 ∂ 4 u ∂ 4 u
Tij = (uxx + uyy )(xi , yj ) + + 4 (xi , yj ) + f (xi , yj ) (7.60)
12 ∂x4 ∂y
from which we obtain
h2
4 4
∂ u ∂ u
|Tij | ≤ max 2 4 + 4 (7.61)
12 (x,y)∈(0,1) ∂x ∂y
that is, the truncation error is O(h2 ).
Denote the (J − 1)2 dimensional vector
T
U = [u1,1 , . . . , uJ−1,1 ; u1,2 , . . . , uJ−1,2 ; . . . ; u1,J−1 , . . . , uJ−1,J−1 ] (7.62)
where T means the transpose. With this notation, the scheme (7.56) leads to
a linear system
AU = F (7.63)
where A is a matrix of order (J − 1)2 given by
B −I
−I B −I
. . .
. . .
(7.64)
. . −.
−I B −I
−I B
506 Modern Engineering Mathematics
with I being the identity matrix of order J − 1, and B being a matrix of order
J − 1 given by
4 −1
−1 4 −1
. . .
A= . . . .
. . −.
−1 4 −1
−1 4
2
Furthermore, F is a (J − 1) dimensional vector whose elements are
un+1
j − unj (unj+1 − 2unj + unj−1 )
= (7.71)
4t h2
un+1
j = unj + µ(unj+1 − 2unj + unj−1 ), 1 ≤ j ≤ J − 1, 0 ≤ n ≤ N − 1, (7.72)
or
un+1
j = µunj−1 + (1 − 2µ) unj + unj+1 ), 1 ≤ j ≤ J − 1, 0 ≤ n ≤ N − 1, (7.73)
where we denote
4t
µ=
. (7.74)
h2
The boundary condition can be approximated directly as
Note that with the scheme (7.73), the approximate solution un+1
j at any in-
terior point can be obtained by a simple marching in time.
Example 260. Consider the problem
ut = uxx , 0 < x < 1, 0 < t < 0.5
u(0, t) = u(1, t) = 0, 0 < t < 0.5
u(x, 0) = sin πx, 0≤x≤1
with
0 ≤ x ≤ 12
10x ,
f (x) = 1 .
10(1 − x), 2 <x<1
508 Modern Engineering Mathematics
TABLE 7.3
TABLE 7.4
Exact Approximation
u(0.4, 4.005) = 0.5806 u52 = 0.5758
u(0.6,0.06) = 0.5261 u63 = 0.5208
u(0.2, 0.010) = 0.2191 u10
1 = 0.2154
u(0.8, 0.14) = 0.1476 u14
4 = 0.1442
For h = 0.1, 4t = 0.4, µ = 0.4 < 0.5. Solving the system at t = 1.2, t = 4.8,
t = 14.4, we obtain approximations that are presented in Table 7.3.
We compare in Table 7.4 a sample of exact values with their corresponding
approximations.
Implicit scheme
Computational Numerical Methods in Engineering 509
Similarly, by substituting
(unj − un−1
j )
ut (xj , tn ) ≈ (7.77)
4t
and
(unj+1 − 2unj + unj−1 )
uxx (xj , tn ) ≈ 2 . (7.78)
(4x)
into (7.66), another difference scheme can be constructed as
(unj − un−1
j ) (unj+1 − 2unj + unj−1 )
= 2 . (7.79)
4t (4x)
In which case, we obtain an implicit scheme
Note that we have to solve a linear system at each time step to obtain the
approximate solution unj at all interior points. That is why the scheme (7.80)
is called implicit in order to distinguish it from the explicit scheme (7.73).
Crank-Nicolson scheme
A more rewarding scheme can be derived by averaging the explicit scheme
(7.73) at the point (xj , tn ) and the implicit scheme (7.80) at the point
(xj , tn+1 ) to obtain
" #
(un+1
j − unj ) 1 unj+1 − 2unj + unj−1 un+1 n+1
j+1 − 2uj + un+1
j−1
= + (7.81)
4t 2 h2 h2
or
µun+1 n+1
j+1 − (2 + µ)uj + un+1 n n n
j−1 = µuj+1 − (2 + µ)uj + uj−1 . (7.82)
The advantage of the Crank-Nicolson scheme (Table 7.5) over the explicit
one is that it doesn’t depend on the parameter µ.
Example 261. Use the Crank-Nicolson method to approximate the solution
of the problem
ut = 14 uxx , 0 < x < 2, 0 < t < 0.3
u(0, t) = u(2, t) = 0, 0 < t < 0.3
u(x, 0) = sin πx, 0 ≤ x ≤ 2
TABLE 7.5
TABLE 7.6
Exact Approximation
u(0.75, 0.005) = 0.6250 u53 = 0.6289
u(0.50, 0.20) = 0.5261 u20
2 = 0.5208
u(0.25, 0.10) = 0.5525 u10
1 = 0.5594
Let unj ≈ u(xj , tn ) be the approximation to the exact solution u(x, t) at the
point (xj , tn ). Now, using the central differences for both time and space
partial derivatives in the wave equation, we obtain
un+1
j − 2unj + un−1
j unj+1 − 2unj + unj−1
2 = (7.86)
(4t) h2
or
un+1
j = −un−1
j + 2(1 − δ 2 )unj + δ 2 (unj+1 + unj−1 ), 1 ≤ j ≤ J − 1 (7.87)
where
4t
δ= . (7.88)
h
Computational Numerical Methods in Engineering 511
To start the approximation, we use Taylor’s formula for u(x, t) with respect
to t to obtain
2
(4t)
u(xj , t1 ) ≈ u(xj , 0) + ut (xj , 0)4t + utt (xj , 0) . (7.89)
2
From the wave equation and the initial condition u(x, 0) = f (x) we have
Using the other initial condition ut (xj , 0) = g(x), Equation (7.89) becomes
2
(4t)
u(xj , t1 ) ≈ u(xj , 0) + g(xj )4t + f 00 (xj ) . (7.91)
2
Now if we use the central finite difference approximation for f 00 (xj ), then
(7.91) takes the form
2
(4t)
u(xj , t1 ) ≈ u(xj , 0) + g(xj )4t + (f (xj+1 ) − 2f (xj ) + f (xj−1 ) (7.92)
2h2
and since f (xj ) = u(xj , 0) = uj,0 we have
δ2
u(xj , t1 ) ≈ 1 − δ 2 uj,0 + (uj−1,0 + uj+1,0 ) + g(xj )4t.
(7.93)
2
The last approximation allows us to have the required first step
δ2
u1j = 1 − δ 2 u0j + (u0j−1 + u0j+1 ) + g(xj )4t.
(7.94)
2
The boundary condition (7.84) can be approximated directly as
where
2(1 − 2δ 2 ) δ2
0 0 0
δ2 2(1 − 2δ 2 ) δ2
0 δ2 . . 0 .
(7.98)
0 0 δ2 2(1 − 2δ 2 ) δ2
0 0 ...0 δ2 2(1 − 2δ 2 )
The technique defined by (7.87) and (7.94) is known as the explicit finite
difference approximation of the wave equation.
512 Modern Engineering Mathematics
———————
xi 0.0000
−u00 + αu = f
(7.99)
u(0) = 0, u(1) = 0
where
dv
H 1 (0, 1) = v ∈ L2 (0, 1) such that ∈ L2 (0, 1) (7.101)
dx
Let us develop a finite element method for the problem. For a natural
number N , we partition the set Ω̄ = [0, 1] into N parts:
Ω̄ = ∪N
i=1 Ki (7.103)
For the basis functions of the space Vh , we introduce the ”hat” functions
associated with the nodes x1 , . . . , xN −1 . For i = 1, . . . , N − 1, let
[c]l(x − xi−1 )/hi , xi−1 ≤ x ≤ xi
φi (x) = (xi+1 − x)/hi+1 , xi ≤ x ≤ xi+1 (7.104)
0, otherwise.
These functions are continuous and piecewise linear. The first order weak
derivatives of the basis functions are piecewise constants:
[c]l1/hi , xi−1 ≤ x ≤ xi
φ0i (x) = −1/hi+1 , xi ≤ x ≤ xi+1 (7.105)
0, otherwise.
Write
N
X −1
uh = uj φj .
j=1
Let us find the coefficients matrix of the system (7.105) in the case of a uniform
514 Modern Engineering Mathematics
We see that in matrix and vector notation, in the case of a uniform partition,
the finite element system (7.107) can be written as
Au = b (7.108)
where
u = (u1 , . . . , uN −1 )T (7.109)
2h 2 h 1
is the unknown vector, a = + , b=c= − ,
3 h 6 h
a b
c a b .
c . .
A=
. . .
(7.110)
. a b
c a b
c a
is the load vector. The matrix A is sparse, thanks to the small supports of
the basis functions. One distinguished feature of the finite element method is
that the basis functions are constructed in such a way that their supports are
as small as possible, so that the corresponding stiffness matrix is as sparse as
possible.
Spectral method
The spectral element method represents a special case of Galerkin methods
in which the finite dimensional space of test functions is made of continuous
piecewise algebraic polynomials of high degree on each element of a partition
Computational Numerical Methods in Engineering 515
of the computational domain. For ease of exposition, we will focus only on the
one-dimensional problem: Find u(x) such that:
−u00 + u = f
(7.112)
u(0) = u(1) = 0.
(S + αM )u = f . (7.116)
516 Modern Engineering Mathematics
Next we determine the entries of the matrices S and M for two special
−1/2
cases: ω(x) ≡ 1 and ω(x) ≡ 1 − x2 .
We set ω(x) ≡ 1 and fN = IN f , the Legendre interpolation polynomial of
f with respect to the Legendre-Gauss-Lobatto points. Then problem (7.115)
becomes
Z 1 Z 1 Z 1
00
− uN vN dx + uN vN dx = IN f vN dx ∀vN ∈ XN (7.117)
0 0 0
which is called the Legendre-Galerkin method.
The linear system for (7.117) depends on the choice of the basis functions of
XN , which can be constructed with Legendre polynomials as
φk (x) = Lk (x) + αk Lk+1 (x) + βk Lk+2 (x), k ≥ 0
where the constants αk and βk are uniquely determined by the boundary
conditions of the continuous problem.
7.3 Exercises
Use the Runge-Kutta of order 4 (RK4) method with k = 0.1 to compute
a four-decimal approximation to the following initial value problems:
7.1. y 0 = 2x − 3y + 1, y(1) = 5.
7.3. y 0 = 1 + y 2 , y(0) = 0.
7.4. y 0 = x2 + y 2 , y(0) = 1.
7.6. Use RK4 method with k = 0.1 to approximate y(0.5) where y(x) is
2
the solution of the initial value problem y 0 = (x + y − 1) , y(0) = 2.
Compare this approximate value with the actual value.
(a) Use the RK4 method with k = 1 to approximate the velocity ν(5).
(b) Use a numerical solver to graph the solution of the IVP on the
interval [0, 6].
(c) Use separation of variables to solve the IVP and then find the actual
value ν(5).
7.12. Use the numerical scheme (7.53) to solve the following boundary value
problems:
(a)
(b)
(c)
(d)
7.13. Use the explicit finite difference scheme (7.68 to 7.74) to approximate
the solution of the initial boundary value problem
ut = uxx , 0 < x < 2, 0 < t < 1
u(0, t) = u(2, t) = 0, 0 ≤ t ≤ 1
u(x, 0) = f (x)
with
1, 0 ≤ x ≤ 1
f (x) = , h = 0.25 and 4t = 0.025.
0, 1 ≤ x ≤ 2
7.15 Solve problem 7.13 by the Crank-Nicolson Method with h = 0.25 and
∆t = 0 : 025.
7.17 Use the explicit finite difference for the wave equation to approximate
the solution of the initial boundary value problem
when
521
522 Bibliography
523
524 Modern Engineering Mathematics
of complex
√ numbers z = x + iy, x ∈ R, y ∈ R) and the imaginary number
i = −1 is the root of the algebraic equation x2 + 1 = 0. It is well known
that complex analysis was developed as a result of mathematical curiosity but
subsequently it was found very useful in signal and image processing, fluid
flow, quantum mechanics and many other areas of engineering.
It is common belief that real analysis is more useful than complex analysis
but in practice the reverse is true. One strange application of complex anal-
ysis is in quantum mechanics, where complex numbers are used to represent
approximately the probabilities of different states of a system. Complex anal-
ysis can be thought of as a subject that deals with the study of calculus of
functions of imaginary numbers.
The story of imaginary numbers began in 1545 with the work of Italian
mathematician Girolamo Cardano (1501-1576). The historical development of
imaginary numbers is covered in John Stilwell’s Mathematics and Its History
(Springer, 2010). Cardano commented that arithmetic dealing with quanti-
ties involved mental tortures and noted that computations sometimes seemed
useless. Many mathematicians of that time agreed with him.
In 1572, Rafael Bombelli, another Italian mathematician, showed the util-
ity of roots of negative numbers. Interested readers√are referred to Paul Nahin’s
book titled An Imaginary Tale: The Story of −1 (Princeton University
Press, 1998). See also the November 1999 issue of Review Notices of AMS,
pages 1233 through 1236. History indicates that the seeds of imaginary num-
bers were planted in the 12th century when Arabian algebra was translated
and introduced to Italy.
Rene Descartes (1596-1650), the French mathematician and philosopher,
devised imaginary terms in complex numbers. One of the most distinguished
mathematicians in history, Carl Friedrich Gauss (1777-1855), suggested the
use of complex numbers instead of imaginary numbers. Euler was the first
mathematician to propose the square root of 1 symbolized by i.
Complex numbers are ordered pairs of real numbers. They may be rep-
resented by points in a plane. The xy plane is known as a complex plane or
Argand’s plane (for Jean Robert Argand, 1768-1822) when displaying com-
plex numbers. Gauss published a work on geometric representation of com-
plex numbers as points in a plane. He also published the first proof of the
fundamental theorem of algebra: every polynomial equation of degree n with
complex coefficients different from zero has n roots in complex numbers.
Great controversy surrounded the acceptance of complex numbers in al-
gebra and analysis in 1770, by 1830, the geometry of complex numbers was
accepted by Gauss and then by the rest of the mathematics world. Gauss and
Simon Denis Poisson (1781-1840) initiated studies of complex functions and
their integrals. August Louis Cauchy (1789-1857) contributed to the field in
a series of papers written between 1814 and 1845. He formulated his inte-
gral theorem and related concepts such as independence of path and integral
representations of functions and their derivatives.
Pierre Alphonse Laurent (1813-1854) developed his Laurent series around
Complex Analysis 525
(a) z + u (b) z − u
z
(c) zu (d)
u
Solution: (a) z + u = (6 + 4) + (5 + 3)i = 10 + 8i.
(b) z − u = (6 − 4) + (5 − 3)i = 2 + 2i.
(c) z.u = (6 + 5i)(4 + 3i) = (24 − 15) + (18 + 20)i = 9 + 38i.
z 6 + 5i 4 − 3i 24 + 20i − 18i + 15 39 + 2i 39 2
(d) = = = = + i. The set
u 4 + 3i 4 − 3i 16 + 9 25 25 25
of all complex numbers is often denoted by C.
Imaginary axis
(y-axis)
•
x+iy
(x,y)
Real axis
(x-axis)
FIGURE 8.1: The Complex Plane
(x, y)
x(l, O)+y(l ,0)
x+iy
The first component of this vector is Re(z) and the second component is
Im(z). In this interpretation, the definition of addition of complex numbers
is equivalent to the parallelogram law for vector addition because we add two
vectors by adding their respective components, see Figure 8.3.
528 Modern Engineering Mathematics
. .(x+u)+i(y+v)
y
x = r cos θ, y = r sin θ.
Thus
=x+iy
z=x-iy
r
I xsm
.8
1
I
8
I
X
z |z|
|zw| = |z||w|, | | = (8.3)
w |w|
Complex Analysis 531
and
z
arg(zw) = arg z + arg w, arg( ) = arg z − arg w. (8.4)
w
(iii) − 1 + 5i = x + iy ⇒ x = −1, y = 5
√ 5
⇒r= 6 , θ = tan−1
1
√ i tan−1 (−5)
−1 + 5i = 6e .
√ √
12 12( 3 − i) 12 3 − i12 √
(iv) √ = √ √ = = 3 3−3i
3+i ( 3 + i)( 3 − i) 3+1
√
⇒ x = 3 3, y = −3
√ −3 1
r = 27 + 9 = 6, θ = tan−1 √ = tan−1 (− √ )
3 3 3
12 1 1
√ = 6(cos tan−1 (− √ ) + i sin tan−1 (− √ )
3+i 3 3
π π −iπ
= 6(cos(− ) + i sin(− )) = 6e 6 .
6 6
p
(b) r2 = x2 + y 2
100 = x2 + y 2
π
100 = x2 + x2 tan
4
π
100 = x2 sec2
4
y
tan θ =
x
π y
tan =
4 x
√
100
or x =
sec π4
z = reiθ
where θ is the angle from the positive real axis to the line from the origin
through z (Figure 8.8).
Definition 88. (a) A point a is said to be an interior point of a set S of
the complex plane y entirely within S. A subset S of a complex plane is called
an open set if every point z of S is an interior point.
(b) A point a is said to be a boundary point of a set of complex numbers
S if every open disk about a contains at least one point in S and at least one
point not in S.
S is called closed if it contains all of its boundary points.
(c) A complex number a is a limit point of a set S of complex numbers if
every open disk about a contains atleast one point of S different from a.
534 Modern Engineering Mathematics
lz-al=p
Remark 63. (a) A boundary point may or may not be in S. No point can
Complex Analysis 535
lzl=r
Example 269. Let B consist of all points satisfying |z − a| < ρ. Every point
of B in an interior point because about any point is B we can draw a disk of
small enough radius to contain only points in B. Thus B is an open set which
does not include any points on its bounding circle |z − a| = ρ.
536 Modern Engineering Mathematics
·23i
|zn − z| < ,
(ll)::·plliM (b)w·plliM
dw
.
dz
Complex Analysis 539
f 0 (z) = 2z if f (z) = z 2 .
x2 + y 2 + x y
f (z) = + .
(x + 1)2 + y 2 (x + 1)2 + y 2
z x + iy x + iy
(iii) f (z) = = =
z+1 x + iy + 1 (x + 1) + iy
x + iy (x + 1) − iy
=
(x + 1) + iy (x + 1) − iy
x2 + y 2 + x (y + y 2 )
= + i
(x + 1)2 + y 2 (x + 1)2 + y 2
x2 + y 2 + x y + y2
⇒ u(x, y) = 2 2
, v(x, y) = .
(x + 1) + y (x + 1)2 + y 2
in (8.17), we get
Remark 66. The Cauchy-Reimann equations are not sufficient for ensuring
analyticity of f (z) unless derivatives
∂u ∂v ∂u ∂v
, , and
∂x ∂x ∂y ∂y
are continuous. In fact the following result holds.
Suppose real valued functions u(x, y) and v(x, y) are continuous and have
∂u ∂v ∂u ∂v
continuous derivatives , , and in a domain D. If u and v satisfy the
∂x ∂x ∂y ∂y
Cauchy-Riemann equation then the complex function f (z) = u(x, y) + iv(x, y)
is analytic in D.
Analyticity implies differentiability but not vice versa. We have an analogue of
the above result that gives sufficient conditions for differentiability, namely if
u(x, y) and v(x, y) are continuous and have continuous first order derivatives
in a neighborhood of z, and if u and v satisfy the Cauchy-Reimann equations
at the point z, then f (z) = u(z, y) + iv(x, y) is differentiable at z and f 0 (z) is
given by
∂u ∂v ∂v ∂u
f 0 (z) = +i = −i .
∂x ∂x ∂y ∂y
Remark 67. It may be noted that if a complex function f (z) = u(x, y) +
iv(x, y) is analytic through a domain D, then the real functions u(x, y) and
v(x, y) must satisfy Cauchy-Riemann equations at every point of the domain.
Example 273. (a) Find u(x, y) and v(x, y) if
(i) f (z) = z 2 .
Complex Analysis 543
1
(ii) f (z) = , z 6= 0.
z
(b) Show that f (z) = (2x + y) + i(y 2 − x) is not analytic at any point.
(c) Show that f (z) = Re(z) is not analytic for z 6= 0.
(d) Show that the following functions are analytic in an appropriate domain.
1 1 1 x − iy x − iy
(ii) f (z) = = = = 2
z x + iy x + iy x − iy x + y2
x y
= −i 2
x2 + y 2 x + y2
x y
⇒ u(x, y) = 2 2
, v(x, y) = − 2 .
x +y x + y2
∂u ∂v
⇒ = 1, = 0,
∂x ∂y
⇒ Cauchy-Riemann equation is not satisfied, for z 6= 0. Hence f (z) is not
analytic.
∂u ∂y 2 − (x − 1)2 ∂v
= ,
∂x ∂[(x − 1)2 + y 2 ] ∂y
∂u 2y(x − 1) ∂v
=− = −
∂y [(x − 1)2 + y 2 ]2 ∂x
and f is analytic in any domain not containing z = −1.
Harmonic functions: A real valued ϕ(x, y) is called harmonic in the do-
∂2ϕ ∂2ϕ ∂2ϕ
main D if 2
, 2 and exist and are continuous and
∂x ∂y ∂x∂y
∂2ϕ ∂2ϕ
+ = 0.
∂x2 ∂y 2
Theorem 59. Let f (z) = u(x, y) + v(x, y) be analytic in a domain D. Then
the functions u(x, y) and v(x, y) are harmonic.
Proof. Let u(x, y) and v(x, y) have continuous second partial derivatives.
Since f is analytic, the Cauchy-Riemann equations are satisfied. Differen-
tiating both sides of
∂u ∂v
=
∂x ∂y
with respect to x and both sides of
∂u ∂v
=−
∂y ∂x
with respect to y we get
∂2u ∂2v ∂2u ∂2v
= , = − .
∂x2 ∂x∂y ∂y 2 ∂x∂y
Under the assumption of continuity, the mixed partials are equal. By adding
these equations we get
∂2u ∂2u ∂2v ∂2v
2
+ 2 = − = 0.
∂x ∂y ∂x∂y ∂x∂y
Complex Analysis 545
This implies that u(x, y)is harmonic. Similarly we can show that
∂2v ∂2v
+ = 0.
∂x2 ∂y 2
∂u
Solution: (i) u(x, y) = x ⇒ =1
∂x
and
∂2u ∂2u
=0 and =0
∂x2 ∂y 2
∂2u ∂2u
+ 2 = 0,
∂x2 ∂y
⇒ u(x, y) is harmonic.
∂2u
(ii) = 2ex cos y+ex (x cos y−y sin y)
∂x2
∂2u
= ex (−x cos y + y sin y − 2 cos y)
∂y 2
∂2u ∂2u
⇒ + 2 = 0.
∂x2 ∂y
Thus u is harmonic.
Example 275. Let u(x, y) = x2 − y 2 , Find a conjugate harmonic function of
u.
Solution: u(x, y) = x2 − y 2 is harmonic.
Now
∂u ∂v
= 2x =
∂x ∂y
⇒ v = 2xy + h(x)
546 Modern Engineering Mathematics
∂u ∂v
= −2y = − = −2y − h0 (x)
∂y ∂x
implies h0 (x) = 0 or h(x) = c. Therefore
f (z) = x2 − y 2 + i(2xy + c)
⇒ v(x) = (2xy + c)
is a conjugate harmonic function of u.
Exponential and logarithmic functions
t ≤ 4.
R
(ii) Evaluate C (z + 3)dz, where C is x = 2t, y = 4t − 1, 1 ≤ t ≤ 3.
(iii) Evaluate C z 2 dz, where C is z(t) = 3t + 2it, −2 ≤ t ≤ 2.
R
x = f (t), y = g(t), a ≤ t ≤ b
and A and B are points (f (a), g(a)) and (f (b), g(b)) respectively.
A curve C is smooth if f 0 and g 0 are continuous on the closed interval
[a, b] and not simultaneously on the open interval (a, b), see Figure 8.11a.
C is piecewise smooth if it consists of a finite number of smooth curves
C1 , C2 , . . . , Cn joined end to end, namely C = C1 ∪ C2 ∪ C3 ∪ . . . , Cn (see
Figure 8.11b). C is closed if A = B, see Figure 8.12a.
C is a simple closed curve if A = B and the curve does not cross itself,
see Figure 8.12b.
If C is not a closed curve, then the positive direction on C is the direction
corresponding to increasing values of t.
c.
Let
4zk = zk − zk−1 , k = 1, 2, . . . , n.
Assume that kP k denotes the norm of the partition, that is, kP k = maximum
R
of |4zk |. The contour integral or integral of f (z) on C, denoted by C f (z)dz
is defined by
Z Xn
f (z)dz = lim f (zk∗ )4zk .
C kP k→0
k=1
Theorem 63. Let f (z) be analytic in any domain D1 that contains D and its
boundary curves, where D is a doubly connected domain with outer boundary
C1 and inner boundary C2 . Then
I I
f (z)dz = f (z)dz
C1 C2
Proof. The proof is based on Green’s theorem and the Cauchy-Reimann equa-
tion. Since f 0 is continuous throughout D, the real and imaginary parts of
f (z) = u(x, y) + iv(x, y) and their first derivatives are continuous on D. We
have
I X
f (z)dz = lim (u + iv)(4x + i4y)
C
X X
= lim (u4x − v4y) + i (v4x + u4y)
I I
= u(x, y)dx − v(x, y)dy + i v(x, y)dx + u(x, y)dy
C C
Z Z Z Z
∂v ∂u ∂u ∂v
= (− − )dA + i ( − )dA (8.23)
D ∂x ∂y D ∂x ∂y
by applying Green’s theorem on each line integral. Since f is analytic, we have
∂u ∂v
=
∂x ∂y
and
∂u ∂v
=−
∂y ∂x
that clearly
H imply that both integrals on the right hand side of (8.23) are zero.
Hence C f (z)dz = 0.
2
(b) C ez dz; where C is a simple closed curve.
H
Then
(z − z0 )n = rn einθ
and
dz = ireiθ dθ.
We get I Z 2π
(z − z0 )n dz = rn einθ reiθ dt
C 0
Z 2π
= irn+1 ei(n+1)θ dθ.
0
If we apply the Euler formula, the right hand side takes the form
Z 2π Z 2π
irn+1 cos(n + 1)θdθ + i sin(n + 1)θdθ .
0 0
f (z) − f (z0 ) ε
| |<
z − z0 ρ
at each point of the circle. The length of the circle is 2πρ. Hence by Theorem
60(e) I
f (z) − f (z0 ) ε
dz < 2πρ = 2πε.
z − z0 ρ
|z−z0 |=r
This implies the second term of (8.27) is zero and the result is proved.
Complex Analysis 555
Assume that I
(n) n f (z)
f (z0 ) = dz.
2πi C (z − z0 )n+1
556 Modern Engineering Mathematics
Then
I
(n + 1) f (z)
f (n+1) (z0 ) = n+2
dz
2πi C (z − z0 )
I
n f (z) 1
= (n + 1) n+1
dz.
2πi C (z − z0 ) (z − z0 )
nM
| f (n) (z0 ) |≤ .
rn
(d) Suppose an entire function f (z) is bounded, say | f (z) |< k for all z. Using
the inequality of the part (c), we get
k
| f 0 (z0 |≤ .
r
Since f (z) is entire, this time for all r, normally we can choose r as large as
we please so we conclude that f 0 (z) = 0 for all z. This means that f (z) is
constant.
1
(e) Let P (z) 6= 0 for all z. This implies that Q(z) = is an entire function.
P (z)
| Q(z) |→ 0 as | z |→ ∞ implies Q(z) is bounded for all finite z. Thus by
part(d), Q(z) is constant and so P (z) is constant, thus our assumption leads
to contradiction that P (z) is non-constant. Hence P (z) = 0 for atleast one z.
(f ) Let Z
F (z) = f (z 0 )dz 0 .
D
It can be checked that F (z) is analytic and by the part (b) F 0 (z) is analytic
and so f (z) = F 0 (z) is analytic in D. This proves Morera’s theorem.
Example 280. Evaluate the following integrals applying Cauchy’s integral
formula for derivatives and functions:
H z+1
(a) C z4
dz, where C is the circle | z |= 1.
+ 4z 3
H 1 + 2ez
(b) C
dz; | z |= 1.
z
3
H ez
(c) C (z − i)3
dz, where C is closed curve not passing through i.
Complex Analysis 557
Solution: (a)
z+1
f (z) =
+ 4z 3 z4
is not analytic at z = 0 and z = −4 and only z = 0 lies in the contour | z |= 1.
f (z) can be written as
z+1
f (z) = .
z+4
z3
We can identify z0 = 0, n = 2 and
z+1
f (z) = .
z+4
By the quotient rule,
−6
f 00 (z) =
(z + 4)3
and so by Cauchy’s integral formula for derivatives we have
I
z+1 2πi 00 3π
4 + 4z 3
dz = f (0) = − i.
c z 2 32
(b) By Cauchy’s integral formula for
f (z) = 1 + 2ez ,
1 + 2ez
I
dz = 2πi(1 + 2e0 ) = 6πi.
c z
(c) If C does not enclose i then this integral is zero by Cauchy’s theorem since
the only point at which
3
ez
f (z) =
(z − i)3
fails to be differentiable is i. Assume that C encloses i because the factor
z − i occurs to the third power in the denominator and use n = 2 in Cauchy’s
3
integral formula for derivatives with f (z) = ez to obtain
3
ez
I
2πi(n)
dz = f (i) = πif 00 (i).
(z − 1)3 2
We have 3
f 0 (z) = 3z 2 ez
and 3 3
f 00 (z) = 6zez + 9z 4 ez
so 3
ez
I
dz = πi[6ie−i + 9e−i ] = (−6 + 9i)πe−i .
c (z − i)3
558 Modern Engineering Mathematics
ities.
r <| z − z0 |< R,
where r is the radius of the inner circle and R is the radius of the outer circle.
If r = 0 the annulus is a punctured disk (open disk with the center removed).
0 <| z − z0 |< ∞, represents the entire complex plane except z0 and r <|
z − z0 |< R, contains all points outside the inner circle of radius r. The notion
of Laurent series leads to concept of a residue that in turn provides another
way to evaluate complex and real integrals.
Taylor’s theorem states that
∞
X f k (z0 )
f (z) = ak (z − z0 )k ,
k
k=−∞
Laurent expansion
Let f be analytic within an annulus D defined by r <| z − z0 |< R. Then f is
represented by the series
∞
X
f (z) = ak (z − z0 )k (8.33)
k=−∞
or for
1
| z − z0 |> = r.
rn
P∞
The second term, k=0 ak (z − z0 )k in (8.35) is called the analytic part of
the Laurent expansion and it converges for | z − z0 |< R. Thus, the sum of
these two parts converges, if r <| z − z0 | and | z − z0 |< R, that is, the
Laurent expansion converges in the annulus: r <| z − z0 |< R.
FIGURE 8.18: Closed Simple Curve Enclosing z0 and Lying in Annulus r <|
z − z0 |< R
sin z
Example 282. (a) (i) Show that has a removal singularity at z = 0.
z
sin z
(ii) Show that f (z) = 2 has a simple pole at z = 0.
z
(b) Show that z = 0 is a removal singularity of
e2z − 1
f (z) = .
z
(c) Determine the zeros and their order for the functions
(i) f (z) = (z + 2 − i)2
9
(ii) f (z) = z +
z
(iii) f (z) = e2z − ez
Solutions: (a) (i) We know that
z3 z5
sin z = z − + + ...
3! 5!
or
sin z z2 z4
=1− + ...
z 3! 5!
The right hand side is of the form
a0 + a1 (z − z0 ) + a2 (z − z0 ) + · · · · · · for z0 = 0.
562 Modern Engineering Mathematics
sin z
Hence z = 0 is a removal singularity of .
z
(ii) We have
sin z 1 z z3
= − + ...
z2 z 3! 5!
1
Since 0 <| z |, we see that a−1 = 6= 0, and so z = 0 is a simple pole of the
z
sin z
function f (z) = 2 .
z
(b) Since
2z 22 z 2 23 z 3
e2z = 1 + + + + ...,
1! 2! 3!
we get
2 22 2 23 3
2z
e −1 (1 + z + z + z + ..) − 1
= 1 2 3
z z
2 22 23
= + z + z2 + . . .
1 2 3
e2z − 1
z = 0 is a removal singularity of the function f (z) = as it is of the
z
form
a0 + a1 (z − z0 ) + a2 (z − z0 ) + . . .
for z = 0.
(c) (i) f (z) = (z + 2 − i)2 has a zero at z = −2 + i. It is of order 2 as
f 0 (z) = 2(z + 2 − i)
=) = 0 at z = −2 + i
is a zero of order 1.
1
Definition 95. (residue) The coefficient a−1 of in the Laurent series
(z − z0 )
expansion of f (z)
∞
X
f (z) = ak (z − z0 )k
k=−∞
a−2 a−1
= ··· + + + a0 + a1 (z − z0 ) + a2 (z − z0 )2 + . . . .
(z − z0 )2 (z − z0 )
where f has removable singularities at the point z0 is called the residue of f
at z0 and is often denoted by Res(f (z), z0 ).
The following theorems provide techniques for computing residues for a
simple pole and pole of order n.
Theorem 65. (a) If f has a simple pole at z = z0 then
g(z)
(b) If f (z) = where g and h are analytic at z = z0 if g(z0 ) 6= 0 and if
h(z)
the function h has a zero of order 1, at z0 , then f has a simple pole at z = z0
and
g(z0 )
Res(f (z), z0 ) = 0 .
h (z0 )
Theorem 66. If f has a pole of order n at z = z0 , then
1 dn−1
Res(f (z), z0 ) = lim (z − z0 )n f (z). (8.37)
(n − 1) z→z0 dz n−1
Proof. (a) For a simple pole z = z0 the Laurent expansion of f (z) about z0
is given by
a−1
f (z) = + a0 + a1 (z − z0 ) + a2 (z − z0 )2 + · · ·
(z − z0 )
Multiplying both sides by (z − z0 ) and taking the limit
h(z) − h(z0 )
lim = h0 (z0 )
z→z0 z − z0
g(z) g(z) g(z0 )
Res(f (z), z0 ) = lim (z − z0 ) = lim = 0 .
z→z0 h(z) z→z0 h(z) − h(z0 ) h (z0 )
z − z0
Since f has a pole of order n, its Laurent expansion for 0 <| z − z0 |< R is of
the form
a−n a−2 a−1
f (z) = n
+ ··· + 2
+ + a0 + a1 (z − z0 ) + . . . .
(z − z0 ) (z − z0 ) z − z0
Multiplying the last expression by (z−z0 )n and differentiating the result (n−1)
times we get
dn−1
(z − z0 )n f (z) = (n − 1)a−1 + na0 (z − z0 ) + . . . .
dz n−1
By taking the limit as z → z0 of both sides we get
dn−1
lim (z − z0 )n f (z) = (n − 1)a−1
z→z0 dz n−1
as all terms on the right hand side will be zero except the first term. This gives
us
1 dn−1
a−1 = Res(f (z), z0 ) = lim (z − z0 )n f (z).
(n − 1) z→z0 dz n−1
This proves the theorem.
Theorem 67. (Residue theorem) Let f (z) be analytic inside a simple closed
curve C and on C, except for finitely many singularities z1 , z2 , z3 . . . , zn inside
C. Then the integrals f (z) taken counterclockwise around C equal 2πi times
the sum of the residues of f(z) at z1 , z2 , z3 . . . , zn , namely
I n
X
f (z)dz = 2πi Res(f (z), zk ).
C k=1
By the Laurent expansion (8.35) and Cauchy’s integral formula (8.34), residue
f (z) I
1
z = z1 = a−1 = f (z)dz = Res(f (z), z1 )
2πi C1
or I
f (z)dz = 2πiRes(f (z), zk ), k = 1, 2, . . . , n. (8.39)
C
Remark 71. The evaluation of integrals using the residue theorem depends
on the determination of residues at singular points. The residue theorem
should be used to evaluate integrals of those functions and finding those
residues at singular points is not tedious.
566 Modern Engineering Mathematics
1
Example 283. (a) Find the residue of f (z) = at z = 1.
2(z − 1)2
−1
Res(f (z), 1) = .
4
(b) The Laurent expansion of
3
f (z) = e z
is
3 3 32 33
ez = 1 + + 2 + 3 + ...
z 2z 3z
a−1 = Residue(f (z), 0) = 3.
(c) The given function has a simple pole at z = −4i. By Theorem 65(a)
3 − 6i −4i − 6i 2 3
Res(f (z), −4i) = lim (z + 4i) = lim = = − + i.
z→−4i z→−4i (z − 2)2 (−4i − 2)2 5 10
2 1 2 z − 1 (z − 1)2 (z − 1)3
f (z) = = (1 − + − + ...
5(z − 1) z−1 5(z − 1) 5 52 53
1+
5
2 1 2 2(z − 1) 2(z − 1)2
= − + − .
5 (z − 1) 25 53 54
2
Res(f (z), 1) = a−1 = .
5
(b) Laurent expansion of f (z) at z = 0 is
23 22 2
f (z) = · · · − 6
+ 4 − 2 + 1....
3z 2z 1z
implying Res(f (z), 0) = 0.
Example 285. Evaluate by the residue theorem
ez
I
4 3
dz,
C z + 5z
1 d2 e3
Res(f (z), 0) = lim 2 z 3 4
2 z→0 dz z + 5z 3
or
1 (z 2 + 8z + 17)ez 1 17π
Res(f (z), 0) = lim = .
2 z→0 (z + 5)3 2 125
Therefore, I
11 17π 17
f (z)dz = 2π = i.
C 2 125 125
Example 286. Find the contour integral of terms over the circle | z |= 2.
568 Modern Engineering Mathematics
sin z
(i) tan z = .
cos z
1
(ii) .
(z − 1)(z + 2)2
Solution: (i) The integrand
sin z
tan z =
cos z
has simple poles at the points where cos z = 0. The only zeroes for cos z are
real numbers
π
z = (2n + 1) , n = 0, ±1, ±2, ±3, . . . .
2
π π
and − and are in the given circle | z |= 2. Thus
2 2
I
π π
tan zdz = 2πi(Res(f (z), − ) + Res(f (z), )).
|z|=2 2 2
By Theorem 65(b), for h(z) = cos z, g(z) = sin z,and h0 (z) = − sin z,
π
π sin
Res(f (z), ) = 2 = −1
2 π
− sin
2
and
−π
−π sin
Res(f (z), )= 2 = −1.
2 −π
− sin
2
Hence I
tan zdz = 2πi(−1 − 1) = −4πi.
|z|=2
(ii) I
1
dz = 2πi[Res(f (z), 1) + Res(f (z), −2)]
C (z − 1)(z + 2)2
by the residue theorem
1 1
Res(f (z), 1) = , Res(f (z), −2) = − .
9 9
Thus I
1 1 1
dz = 2πi( − ) = 0.
(z − 1)(z + 2)2 9 9
Example 287. Evaluate I
z+1
dz
C z 2 (z + 2)2
using the residue theorem; C in the circle | z |= 3.
Complex Analysis 569
I
z+1
Solution: dz = 2πiRes((f (z), 2i) + 2πiRes(f (z), 0)
z 2 (z
+ 2)2
1 1 1 1
= 2πi[(− − i) + ( + i)] = 0.
4 2 4 2
Example 288. Evaluate I
z+1
dz
C z 2 (z − 2i)
by applying the residue theorem over the circle C: | z − 2i |= 1.
Solution: z = 0 and z = 2i are singular points but z = 0 does not lie inside
| z − 2i |. z = 2i is a simple pole.
z+1
Res(f (z), 2i) = L lim (z − 2i)
z→2i z 2 (z
− 2i)
2i + 1 1 1
= =− − i
−4 4 2
I
1 1
f (z)dz = 2πi(− − i)
4 2
1
= π(1 − i).
2
First we convert the formula into a complex integral over the unit circle center
at the origin. The unit circle can be expressed in the form
The sum on the right hand side is taken over all of the poles p of f (z) enclosed
by the unit circle.
Example 289. Evaluate
Z 2π
1
dθ
0 10 − 6 cos θ
1
Note that z = and z = 3 are singular points but z = 3 does not lie inside
3
the unit circle. Therefore by the residue theorem
I
dz 1
= 2πiRes(f (z), )
|z|=1 (3z − 1)(z − 3) 3
1 1
= 2πi lim (z − )
z→1/3 3 (3z − 1)(z − 3)
1 1
= 2πi lim
3 z→1/3 (z − 3)
1 3 π
= 2πi (− ) = − i.
3 8 4
Therefore, Z 2π
1 π
dθ = .
0 10 − 6 cos θ 4
Example 290. Evaluate
Z π
1
dθ
0 λ + µ cos θ
where 0 < µ < λ.
Z 2π Z π Z 2π
1 1 1
Solution: dθ = dθ+ dθ.
0 λ + µ cos θ 0 λ + µ cos θ π λ + µ cos θ
Let w = 2π − θ in the second integral on the right hand side to obtain
Z 2π Z 0
1 1
dθ = (−1)dw
π λ + µ cos θ λ + µ cos(2π − w)
Zππ
1
= dw.
0 λ + µ cos w
Therefore, Z 2π Z π
1 1
dθ = 2 dθ
0 λ + µ cos θ 0 λ + µ cos θ
572 Modern Engineering Mathematics
Z π
1 2π
Z
1 1
dθ = dθ
0 λ + µ cos θ 2 0 λ + µ cos θ
or Z 2π Z
1
dθ = f (z)dz,
0 λ + µ cos θ |z|=1
where
1
1 2i
f (z) = =− 2 .
β 1 iz µz + 2λz + µ
λ + (z + )
2 2
f (z) has simple poles at
p
−λ ± λ2 − µ2
z= .
µ
Since λ > µ, these numbers are real. Only one of them,
p
−λ + λ2 − µ2
z1 = ,
µ
is enclosed by the unit circle | z |= 1. The other is outside the unit circle and
is irrelevant for our purpose. Then
Z π
1 2π
Z
1 1
dθ = dθ
0 λ + µ cos θ 2 0 λ + µ cos θ
1
= 2πi(Res(f, z1 ))
2
−2i π
= πi =p .
2µz1 + 2λ λ − µ2
2
Z ∞ Z R
P.V. f (z)dz = lim f (z)dz
−∞ R→∞ −R
n
X
= 2πi Res(f (z), zk ).
k=1
Z ∞
dx 1 πi 1 πi
= 2πi(− e 4 + e− 4 )
−∞ 1 + x4 4 4
2πi πi πi
= − (e 4 − e− 4 )
4
−2πi π π π
= 2i sin = π sin = √
4 4 4 2
as
eiθ − e−iθ π
= sin θ ( Here θ = ).
2i 4
Example 293. Find the principal value of
Z ∞
dx
2 2
.
−∞ (x − 3x + 2)(x + 1)
x2 + 1 = (x − i)(x + i).
By Theorem 65, z = −i is in the lower half plane, which is not relevant here.
The final result is obtained by applying Theorem 67.
Example 294. Evaluate the Cauchy principal value of the improper integral
Z ∞
x2
2 2
dx.
−∞ (x + 1)
Z ∞
x2 π
Solution: 2 2
dx = 2πiRes(f (z), i) = .
−∞ (x + 1) 2
rllngeoff
(ll)::·pi~M (b)w·pi~M
Rez = 2, u(x, y) = 4 − y 2
and
v(x, y) = 4y.
1
There are parametric equations of a curve in the w-plane. Substituting y = v
4
into the first equation eliminates the parameter y to give
1 2
u=4− v .
16
Complex Analysis 577
In other words, the image of the vertical line x = 2 in Figure 8.22(a) is the
parabola shown in Figure 8.22(b)
x:Re:::2
(ll)::·plliM (b)w·plliM
x:Re:::O
(>) (b)
f (z) = z 2 = x2 − y 2 + 2ixy = −y 2
as Rez = x = 0.
u(x, y) = −y 2 , v(x, y) = 0.
In short the complex function
Remark 72. (a) If z(t) = x(t) + iy(t), describes a curve C in the region, then
w = f (z(t)), a≤t≤b
u(x, y) = a
will be mapped to a point that lies on the vertical line u = a, and a point z
on the level curve
v(x, y) = b
will be mapped to a point w that lies on the horizontal line
v = b.
T (z) = w = z + b,
f (z) = ez .
w = u(x, y) + iv((x, y)
1 1 x − iy
= =
x + iy x + iy x − iy
x − iy x −y
= = 2 +i 2
x2 − y 2 x − y2 x − y2
x −y
u(x, y) = and v(x, y) = 2 .
x2 − y 2 x + y2
For a 6= 0 the level curve u(x, y) = a can be written as
1
x2 − x + y 2 = 0
a
Complex Analysis 579
or
1 2 1
(x − ) + y 2 = ( )2 .
2a 2a
The level curve is therefore a circle with its center on the x-axis and passing
through the origin. A point z on this circle other than zero is mapped to a
point w on the line u = a. Similarly, the level curve v(x, y) = b, b 6= 0 can be
written as
1 1
x2 + (y + )2 = ( )2 ,
2b 2b
and a point z on this circle is mapped to a point w on the line v = b; it is
clear that
1
f −1 (w) =
w
and
1
w = f (z) =
z
or
1
z=
w
−1
and so f = f . Therefore we conclude that it maps the horizontal line y = b
to the circle
1 1
u2 + (v + b2 )2 = ( b)2
2 2
and it also maps the vertical line to x = a to the circle
1 1
(u − a)2 + v 2 = ( a)2 .
2 2
Example 298. (a) Find the image curve in the w-plane of the circle| z |= 1
under the complex function
1
w = f (z) = z + .
z
(b) Find the image region in the w-plane of the rectangle 0 ≤ x ≤ 1, 0 ≤ y ≤ 1
under the complex function
w = f (z) = ez .
0
~ Zo
X 0
'W Wo
Remark 73. Let z11 and z21 denote tangent vectors to curves C1 and C2 ,
respectively, then
by applying the law of cosines to the triangle determined by z10 and z20 . Thus
f 0 (z0 ) 6= 0.
and
w20 = f 0 (z0 )z20 .
Since f 0 (z0 ) 6= 0, we obtain
0
| f (z0 )z10 |2 + | f 0 (z0 )z20 |2 − | f 0 (z0 )z10 − f 0 (z0 )z20 |2
Φ = cos−1
2 | f 0 (z0 )z10 || f 0 (z0 )z20 |
0 2 0
| z1 | | f (z0 ) |2 + | z20 || f 0 (z0 ) |2 − | f 0 (z0 ) |2 | z10 − z20 |2
= cos−1
2 | f 0 (z0 ) || z10 | f 0 (z0 ) || z20 |
0 2
| z1 | + | z20 |2 − | z10 − z20 |2
= cos−1 = θ.
2 | z1 | z2 |
(d) w = f (z) = z 3 − 3z + 1
1
(e) w = f (z) = z +
z
a b
Remark 74. (a) For c = 0, w = T (z) = z + ; Möbius transform is linear.
d d
(b) If c 6= 0 and a = 0 then the Möbius transform
b
T (z) =
cz + d
582 Modern Engineering Mathematics
is an inversion.
(c) In case ac 6= 0, we can write
a bc − ad
w = T (z) = 1+
c a(cs + d)
and so the Möbius transform is a decomposition of a linear transformation
and inversion of mappping of (8.47) is obtained by solving for z, namely:
cwz + wd = az + b
z(cw − a) = b − wd
or
b − wd dw − b
z= =
wc − a −cw + a
a a
when c 6= 0, then −cw + a = 0 for w = , and let be the image of z = ∞.
c c
The complex together with the point ∞ is called the extended complex plane.
(d) Since
a(cz + d)
T (z) = (cz + d)2
−(az + b)c
ad − ac −d
= 6= 0, for z 6= .
(cz + d)2 c
Every Möbius transform is a conformal mapping. We can summarize the above
properties of the Möbius transform in the form of theorems.
Fixed points of complex mappings
Fixed points of a complex mapping w = f (z) are mapped, that is, w = f (z) =
z.
8.7 Applications
In this section we briefly indicate how the results of complex analysis can
be used in areas such as Electrostatics potential, Heat Flow, Fluid Flow and
Tomography.
∂2φ ∂2φ
∇2 φ = + 2 =0 (8.48)
∂x2 ∂y
584 Modern Engineering Mathematics
F = φ + iψ
one can simultaneously handle equipotential lines φ = constant and their or-
thogonal trajectories, namely the lines or flow ψ = constant.
Furthermore, in solving the Dirichlet problem of finding a potential with given
boundary values the conformal mapping can be used. The electrical force of at-
traction or repulsion between charged particles is governed by Coulomb’s law.
This force is the gradient of a function φ called the electrostatic potential.
At any points free of charges φ in a solution of Laplace equation
∇2 φ = 0.
The surfaces φ(x, y) = constant are called equipotential surfaces. At each point
P the gradient of φ is perpendicular to the surface φ = constant through P ;
that is, the electrical force has the direction perpendicular to the equipoten-
tial surface. We consider here two-dimensional problems, that is, we consider
physical systems that lie in three-dimensional space, but are such that the po-
tential φ is independent of one of the space coordinates, so that φ depends
only on two coordinates, which are x and y and equipotential surfaces appear
as equipotential lines (curves) in the xy plane.
Complex potential: Let φ(x, y) be harmonic in some domain D and ψ(x, y)
a conjugate harmonic of φ in D. Then
∂T ∂2T ∂2T
= α2 ( 2 + ) (8.50)
∂t ∂x ∂y 2
represents heat conduction in a body of homogeneous material, where T de-
notes temperature, t stands for time and α2 is a positive constant depending
on the material of the body. Therefore, if a problem is steady, so that,
∂T
= 0,
∂t
the heat equation reduces to Laplace’s equation.
∂2T ∂2T
∇2 T = + = 0. (8.51)
∂x2 ∂y 2
T (x, y) is called the heat potential. It is the real part of the complex heat
potential.
F (z) = T (x, y) + iψ(x, y) (8.52)
The curves T (x, y) = constant are called isotherms (lines of constant tem-
perature) and the curves ψ(x, y) = constant heat flow lines, because heat flows
from higher to lower temperatures along them. Discussions in Section 8.7.1
can be reinterpreted as problems on heat flow.
It may be observed that to have a steady flow, the boundary of the domain of
586 Modern Engineering Mathematics
the heat flow must be kept at constant temperature by heating or cooling. Use-
fulness of conformal mappings and complex potentials may be demonstrated
in study of temperature distribution in a region comprising a cross section of
a solid cylinder and heat conduction in the upper half plane; for details see
for example Section 16.3 of [7].
V2
ArgV =
V1
at each point z = x + iy. Here V1 and V2 are components of V in the direction
of the x and y axes respectively.
Yo - - - - - - - - - -
"
is called the complex potential of the flow, and ψ(x, y) = constant are the
streamlines. The velocity V is given by
V = V1 + iV2 = F 0 (z).
ψ is called the stream function and φ is called the velocity potential. The
curves φ(x, y) = constant are called equipotential lines. V is the gradient
φ,
∂φ ∂φ
V1 = , V2 = .
∂x ∂y
In fact, for F = φ + iψ,
∂φ ∂ψ
F0 = +i
∂x ∂x
with
∂ψ ∂φ
=−
∂x ∂y
by the Cauchy-Riemann equation. These together yield
∂φ ∂ψ ∂ψ ∂φ
F 0 (z) = −i = +i = V1 + iV2 = V.
∂x ∂x ∂x ∂y
Let us consider any smooth curve C in the z-plane, given by z(z) = x(s)+iy(s),
where s is the arc length of C. Let
∂V
∂t
denote
R the component of V tangent to C, see Figure 8.26.
V
C t
ds is called the circulation of the fluid along C.
∂V2 ∂V1
−
∂x ∂y
is called velocity of the flow and
1 ∂V2 ∂V1
( − ) = w(x, y)
2 ∂x ∂y
a
v
If C is taken as a circle of radius r then the mean velocity of the fluid along
C is equal to
∂V ∂V1
( − )
1 ∂x ∂y
.
2 2πr
A fluid is called incompressible if
∂V1 ∂V2
+ =0
∂x ∂y
in region that is free of sources or sinks V that is, points at which fluid is
produced or disappears.
Theorem 73. Let u and v be continuous with continuous first and second
partial derivatives in a simply connected domain D.
Let ui + vj be irrotational and solenoidal in D. Then u and −v satisfy the
Cauchy-Riemann equation in D, and
curl(ui + vj) = 0
and
div(ui + vj) = 0
for solenoid a flow.
The following theorem implies that any differentiable function
8.7.4 Tomography
This section discusses the role of complex analysis in tomography. The word
tomography comes from the Greek word for slice and the discipline deals with
finding internal structures of non-transparent objects by transmitting signals
such as electromagnetic waves at various frequencies. Examples of such fre-
quencies are radio transmissions, microwaves, light waves in the visual spec-
trum, x-rays, and acoustic waves. Tomography deals with inverse problems
in which the unknown parameters of a system are estimated from its known
reactions to specific signals.
Austrian mathematician Johann Radon studied the integral transform and
inversion of an integral of a real valued function over straight lines in 1917. The
technique became known as the Radon transform and it plays an important
role in tomography, specifically for computed axial tomography (CAT) scans,
barcode scanners, electron microscopy of structures like viruses and proteins,
590 Modern Engineering Mathematics
8.8 Exercises
8. 1 Write the following numbers in the form x + iy:
(a) (5 − 9i) + (2 − 4i)
(b) (1 − i)3
10 − 5i
(c)
6 + 2i
(4 + 5i) + 2i3
(d)
(2 + i)2
8.2. Let z = x + iy. Find the indicated expressions:
(a) Rez(z 2 )
(b) Im(z 2 + z 2 )
(c) | z + 5z |
(a) f (z) = z 3 − 3z + 1
1
(b) f (z) = (z 2 − 1) 2
1 1
8.36. Show that the mapping w = (z + ) maps the circle | z |= r onto an
2 z
ellipse with foci 1 and −1 in the w-plane.
Complex Analysis 595
8.37. Find the image of the given circle =(z) = 2 under the linear fractional
transformation
2z + i
w= .
2−i
8.38. Prove Theorem 65.
8.39. Prove Theorem 66.
8.40. Analyze the flow given by the complex potential f (z) = αz where α is
a non-zero complex constant.
8.41. Solve Example 298.
8.42. Solve Example 299.
8.43. Solve Example 300.
8.44. Solve Example 301.
2π
8.45. Find the temperature T in the sector 0 ≤ arg z ≤ , | z |≤ 1 if T =
√ 3
10◦ C on the x-axis, 80◦ C on y = − 3x, and the curved portion is
insulated.
8.46. Find the complex potential of a parallel flow in the upward direction of
y = x.
597
Chapter 9
Inverse Problems
599
600 Modern Engineering Mathematics
9.1 Introduction
In this chapter we explain the concept of inverse problems through concrete
examples. A mathematical model enables us to understand a mathematical
representation of a person, a building, a vehicle, a tree or a process such as
weather pattern, traffic flow or air flow over a wing of an aeroplane. Models
are created from a mass of data, equations and computations that mimic the
actions of objects or processes presented.
Models usually include graphic displays that translate number crunching
into animations that one can see on a computer screen or other visual device.
Models can be simple images they can be complex, carrying all the character-
istics of the object or process they represent. A complex model will simulate
the action and reaction of the real world phenomenon. To make these models
behave as they would be in real life, accurate, real time simulations require
fast computers with a lot of number crunching power. Generally modeling is
the process of representing a real world object or phenomenon by a set of
mathematical equations. More specifically, the term is often used to describe
the process of representing three-dimensional objects on a computer.
For proper understanding of any real world system, a mathematical for-
mulation (mathematical modeling) of that system is essential. Usually mathe-
matical formulation is an ordinary differential equation or a partial differential
equation or in the form of an optimization problem (finding maxima or min-
ima of a function) or matrix equation. Finding solutions of these equations
(models) for given parameters and boundary or/and initial conditions is the
response to a direct problem. Finding parameter or initial conditions or
boundary conditions of a model with given solution is known as an inverse
problem.
There exists vast literature on direct problems. But in spite of the impor-
tance of inverse problems, less attention has been paid to the topic in many
parts of the world. The main objective of this chapter is to illustrate the con-
cepts of inverse problems with the help of concrete examples from pre-calculus,
calculus, matrix analysis, differential equations, partial differential equations,
image processing and medical imaging.
Inverse Problems 601
h v
R
x
vy
θ
O vx P
the horizontal range. The direct problem is to find OP if θ is given while the
inverse problem is to find θ given OP (horizontal range).
Example 303. State the physical law that governs the relationship between
the depth of the wall and the time it takes a dropped stone to reach the
bottom of a well, neglecting air resistance.
This equation provides relationship between the drain time function T and
(f (u))2
the shape function f provided f is continuous and √ is integrable on
u
[0, T ]. Computing the drain time function T for given f is known as the direct
problem. The inverse problem consists of determining the shape f if the drain
time T is given.
We neglect the weight of the cable. Assume that weight of the load is
distributed nonuniformly along the interval [−1, 1] of the x-axis. LetRus assume
x
that cable has a unique lowest point say, x = 0 in a simple case. −1 w(u)du
is the total weight of the segment [−1, x] for each x ∈ [−1, 1] for a weight
distribution w(x).
A given weight distribution w gives a shape y = y(x) for the hanging cable.
The direct problem is to find shape y caused by a given weight distribution w
while its inverse problem is to determine a distribution w from observing the
shape y.
h v
x such that Ax = y for a given A and vector y. This problem gets a lot of
attention in linear algebra but rarely seen as example of an inverse problem.
Another interesting inverse problem, called the identification problem is
the one in which we find matrix A such that Ax = y for given vectors x and
y. In this subsection we confine discussion to the inverse causation problem.
A solution x ∈ Rn of the inverse causation problem, where A is m × n matrix
and y ∈ Rm is a given vector, exists if and only if y belongs to the range of
A, that is, in the subspace
R(A) = {Ax|x ∈ Rn }.
Determining whether y ∈ R(A), that is, whether a solution exists, and finding
all solutions can be accomplished by the method of Gaussian elimination. The
uniqueness issue is addressed by the null space of A, that is
N (A) = {x ∈ Rn |Ax = 0}.
The Gaussian elimination method is an elegant algorithm for characterizing
the null space and settling the uniqueness question. Let us assume that Ax = y
has a unique solution for each y, that is, A−1 exists. We are interested in know-
ing the error in the solution x for small errors in y. Let x be the unique solution
of the system of equation Ax = y for known y, and let x̃ be the unique so-
lution of the system Ax = ỹ, that is, Ax̃ = ỹ where ỹ is perturbation of y, then
ky − ỹk
kx − x̃k ≤ kA−1 kkyk
kyk
ky − ỹk
= kA−1 kkAkkxk .
kyk
Hence,
kx − x̃k ky − ỹk
≤ cond(A)
kxk kyk
where cond(A) = kAk−1 kAk.
This is called the condition number of the matrix A with respect to the norm
k.k. This condition number gives an upper bound for the relative error in the
solution caused by a relative error in y. For matrices with large condition
numbers, that is, ill-conditioned matrices, relatively small changes in y yield
relatively large changes in the solution x.
For each such pair we call x the input and y the output. An m × n matrix A
is called identifiable from the matrix pair (x, y) if Ax = y.
(Av, Au − b) = 0
where c0 is the initial concentration of solute in the vessel. The direct problem
is to find c(t), concentration at time t for given parameters a, c0 , V and r.
There are several related inverse problems like finding the initial concentration
or finding parameters for given concentration at time t.
θ x
g
y
t (min) u(o F )
5 72
16 62
15 54
Find the ambient temperature, the initial surface temperature, and heat
transfer coefficient (S, T (0), λ).
Inverse Problems 611
f (t) = f (0)ert .
In the variable interest rate, the interest rate is a function of time, say r(t).
This implies
d
ln f = r(t)
dt
which gives Rt
f (t) = f (0)e 0 r(s)ds .
Given f (0) and r(t), finding f (t) is regarded as a direct problem while finding
r(t) given f (t) for all t is an inverse problem. f (t) is called value history and
r(t) is known as variable interest rate.
Example 309. Find the interest rate r(t) so that the number of Rs 50,000
invested yields Rs 50,000 in 10 years.
∆u + k 2 u = 0 in RN \D (N = 2, 3)
This solution is assumed to be the sum of the plane incident wave ui and a
scattered wave us due to the presence of an obstacle, that is,
u = u0 + us .
9.6.3 Examples
These and related inverse scattering problems have various applications in
computer tomography, seismic and electromagnetic exploration in geophysics,
and nondestructive testing of materials.
9.6.5 Examples
The inverse problem for the following well known equations can be posed
on similar lines.
(a) Laplace equation in R2 (two-dimensional)
∂2u ∂2u
∇2 u = ∆u = + 2 = 0,
∂x2 ∂y
where ∇2 = ∇.∇ denotes the Laplacian
∂ ∂
∇= , .
∂x ∂y
The equation is satisfied by the electrostatic potential in absence of charges,
by the gravitational potential in the absence of mass, by the equilibrium dis-
placement of a membrane with a given displacement of its boundary and by
the velocity potential for an inviscid, incompressible, irrotational homogeneous
fluid in the absence of sources and sinks and many other real world systems.
(b) Poisson equation (non-homogeneous Laplace equation)
∇2 u = −f (x, y).
Inverse Problems 615
One encounters this equation while studying the electrostatic potential in the
presence of charge, the gravitational potential in the presence of distributed
matter, the equilibrium displacement of a membrane under distributed forces,
the velocity potential for an inviscid, incompressible, irrotational homogeneous
fluid in the presence of distributed sources and sinks, and the steady state
temperature in the presence of thermal sources or sinks.
(c) Transport equation in R (one-dimensional)
∂u ∂u
+c = 0,
∂t ∂x
where c is a constant and u(x, t) denotes the location of a car at time t and
position x.
(d) Traffic flow
∂u ∂u
+ a(u) = 0,
∂t ∂x
where u(x, t) denotes the density of cars per unit kilometer of expressway at
location x and time t and a(u) is a function of u, say the local velocity of
traffic at location x at time t.
(e) Burger’s equation in one dimension
∂u ∂u
+u = 0.
∂t ∂x
This equation arises in the study of streams of particles or fluid flow with zero
viscosity.
(f ) Eikonal equation in R2 (two dimensional)
2 2
∂u ∂u
+ =0
∂x ∂y
models problems of geometric optics.
(g) Helmholtz equation
(∇2 + k 2 )u = 0
h2 ∂ 2 u ∂ 2 u
+ 2 + (E − V )u = 0.
2m ∂x2 ∂y
Note that m is the mass of the particle whose wave function is u(x, y), h is
the universal Planck’s constant, V is the potential energy and E is a constant.
This equation arises in quantum mechanics. If V = 0 then it reduces to the
Helmholtz equation.
(k) Korteweg de Vries (KDV) equation in one dimension
∂u ∂u ∂ 3 u
+ cu + = 0.
∂t ∂x ∂x3
This equation arises in shallow water waves.
(l) Euler equation in R3
∂u 1
+ (u.∇)u + ∇p = 0.
∂t ρ
The u denotes the velocity field, and p the pressure.
(m) Navier-Stokes equation in R3
1
ut + (u.∇)u + ∇p = v∇2 u.
ρ
The v denotes the kinematic viscosity, p stands for pressure and ρ for density
of the fluid, and u = (u1 , u2 , u3 ) represents velocity of the fluid.
(n) Maxwell equations in R3
∂E
− ∇ × H = 0.
∂t
∂H
− ∇ × E = 0.
∂t
E and H denote the electric and the magnetic fields, respectively.
traded on all major exchanges. Strike price or exercise price is the price at
which the underlying asset is bought in options. Underlying assets are com-
modities, shares, foreign currencies, government or private enterprise bonds,
stocks and stock indices. Equity is a share in the ownership of a company
which usually guarantees the right to vote at meeting and share in dividends
(payments to shareholders as return for investment). A derivative (financial
derivative) is a contract or security whose payoffs are final (asset). In many
cases a derivative is the price of the underlying equity. The termination time
of a derivative (contract), usually the time when payoff value is calculated and
paid, is called expiry (time). Volatility is a measure of standard deviations of
returns. It is a function of underlying asset and time.
Let u denote the value or premium of the options which depends on a
number of factors such as stock price say x, current time t, the maturity date
T , the exercise price K, the risk free interest r and local volatility (infinitesimal
standard deviation) σ (may be constant only time dependent or may satisfy
relation of type σ(x, t) = σ(x)σ(t). It has been proved that u(x, t) is a solution
of the partial differential
∂u 1 2 2 ∂2u ∂u
+ x σ (x, t) 2 + rx − ru = 0. (9.4)
∂t 2 ∂x ∂x
This is the Black-Scholes equation.
Direct problem of option pricing: Given the local volatility function
σ(x, t) find the solution of (9.4) with boundary conditions:
Inverse problem: Find the local volatility function σ(x, t) such that the
solution to (9.4) with different strikes K and maturities T satisfy
where the right hand side denotes the current market price of the option with
the corresponding strike and maturity at time t∗ when the underlying price
(asset) is x∗ . Practitioners refer to this problem as model calibration.
9.7.3 Tomography
Scientists who developed the field of tomography have received several No-
bel prizes. The word tomography comes from the Greek word for slice and the
discipline deals with exposing internal structures of non-transparent objects
by transmitting signals such as electromagnetic waves (such as radio waves,
microwaves and X-rays) and acoustic waves. In computed tomography, scan-
ner images do not provide ready images as X-ray systems do. Rather, the
images result from intricate data measurements and mathematical processes.
The scientists who have studied the relevance of wavelets and their variants
to tomography include Mark Bottema, Bill Moran and Sofia Suvorova [5],
Glenn Easley, Flavia Colonna, Demetrio Labate, and Kanghui Guo. Several
publications focus on variants of wavelets such as curvelets, ridgelets, complex
wavelets and vector valued wavelets.
travel time tomography reflection travel time tomography and wave form to-
mography have been developed to estimate seismic tomography.
Seismic tomography is normally formulated as an inverse problem. A beau-
tiful account of this special theme and inverse problems in general can be
found in Iske and Randen [17] and Vogel [42]. An application of wavelets in
tomography has been discussed in Bottema et al. [5].
9.9 Exercises
9.1. Solve the inverse problem of determining the depth D of a well from the
elapsed time t between the dropping of a stone and hearing of a splash.
Does the equation determining D have a unique solution? Is there a
unique feasible solution?
9.2. A stone is dropped into a well and 4.2 seconds later a splash is heard.
How deep is well?
9.3. Find the weight distribution with total weight 1 that gives shape y(x) =
π
1 − cos .
x
9.4. Show that if a shape is a parabola its weight distribution is constant.
9.5. Find the solution of Exercises 304 through 307.
9.6. Find matrix A that satisfies AX = B, where
1 1 1 −2 6 3
X = −1 −1 0 and B = −1 0 0 .
0 1 1 3 0
9.10. Show that the two value histories have the same interest rate if and only
if their ratio is a positive constant.
622 Modern Engineering Mathematics
623
624 Bibliography
[31] P. Kuchment, The Radon Transform and Medical Imaging, Vol.85, Re-
gional Conference Series in Applied Mathematics, SIAM, 2014.
[32] P. Monk, Finite Element Methods for Maxwell’s Equations, Oxford Sci-
ence Publications, Oxford, 2003.
[33] F. Natterer, The Mathematics of Computerized Tomography, Wiley, New
York, 1986, reprinted in 2001 by SIAM.
10.1 Introduction
Wavelet theory is the outcome of a multidisciplinary endeavor that brought
together mathematicians, physicists and engineers. This interaction created a
flow of ideas that goes well beyond the construction of new bases and trans-
forms.
In fact, wavelet theory is a refinement and extension of Fourier analysis.
Short-comings of Fourier analysis were realized as early as 1946. To remove
these deficiencies, Nobel Laureate of Physics, Dennis Gabor, introduced the
windowed Fourier transform (short-time transform). This transform suffered
from some algorithmic difficulties and to eliminate them wavelet theory was
introduced.
During the 1980’s, French geophysicist Morlet developed a new approach
the wavelet transform, while studying problems in oil and gas exploration. The
term “wavelet” arose because the function from which the wavelet transform
is constructed can be thought of as a localized wave.
In the last 35 years wavelet methods have been applied to diverse fields of
science, engineering and technology [1] through [5], [9] through [11], [14], [17]
through [40]. Typical applications are presented in these references to: physics,
geophysics, function approximation, signal processing, harmonic analysis, dif-
ferential equations, electrical engineering, biomedical engineering, EEG, ECG,
627
628 Modern Engineering Mathematics
MRI, finance and financial data analysis, mechanical engineering, civil engi-
neering, remote sensing, forestry and historical document retrieval.
A leading worker in this field, Prof. Yves Meyer, has received the Abel
Prize of 2017 for his significant contributions in wavelet methods.
In this chapter we give an overview of wavelet methods and their appli-
cations to several areas of engineering and technology, namely biometrics,
computed axial tomography, seismic tomography, medical imaging and power
systems.
It can be checked that {ψj,k } is an orthonormal basis for the square summable
functions.
630 Modern Engineering Mathematics
1.5
1.0 t- -
I
I
0 .5 I- I
-
.,
I
I
0.0
I
I I
- 0 .5 I- I I
I
-
I
I
•
I
- 1.0 I- 6 -
- 1.5
0
- 1 ·_
0 2 4
Db4 and Db10 (Figures 10.5 and 10.6) are from the Daubechies family of
wavelets. Db wavelets have no analytical expression, they are constructed
numerically.
Definition 100. Let ψ be a wavelet. The wavelet coefficients dj,k of a
square integrable function f are defined as
Z ∞
dj,k = hf, ψj,k i = f (t)ψj,k (t) dt . (10.4)
−∞
Since the wavelet family is an orthonormal basis, one can show that for
632 Modern Engineering Mathematics
1
I
1 ~,
"' 1.'
0
~ I
-1
0 1Q 15
FIGURE 10.6: Db10
The right-hand side is called the wavelet series or the wavelet represen-
tation of f . The limit in the double sum is understood as convergence in the
quadratic mean, as in (10.2),
Z ∞ N
X N
X 2
lim f (t) − dj,k ψj,k (t) dt = 0 .
N →∞ −∞ j=−N k=−N
Remark 76. The wavelet coefficients dj,k measure the frequency content of
f near the point t = 2−j k. Small values of j correspond to low frequency
components and high values of j correspond to high frequency components.
According to the above,
N
X N
X
f (t) ≈ dj,k ψj,k (t) (10.6)
j=−N k=−N
In Fourier analysis we studied the Fourier series the Fourier transform. For
wavelets, have wavelet coefficients and wavelet transforms.
Definition 101. (Wavelet transform) Let ψ be a square integrable real valued
function on R. For a, b ∈ R with a 6= 0 we set
1 t − b
ψa,b (t) = p ψ .
|a| a
While the formulas defining the wavelet transform make sense for any
square integrable function ψ, as in the definition above, the wavelet transform
is of good use only if it is invertible, like the Fourier transform, because only in
this case we can retrieve the original function from its wavelet transform. As
an extreme case, setting ψ = 0 we have Wψ [f ] = 0 for all f , so all information
is lost when we apply Wψ . A sufficient condition for invertibility is given in
the following definition.
Definition 102. Let ψ beR integrable as well as square integrable on R. We
∞
say that ψ is admissible if −∞ ψ(t)2 dt = 1 and
∞
|ψ̂(ξ)|2
Z
cψ = dξ < ∞ . (10.8)
−∞ |ξ|
Remark 78. A function ψ can be admissible according to this definition only
if Z ∞
ψ(t) dt = 0 .
−∞
Remark 79. The function displays the time information and hides the in-
formation about frequencies while the Fourier transform displays information
about frequencies and hides the time information. The energy of two signals
represented by ψ ψ̂ is the same, that is,
Z ∞ Z ∞
2 2
kψ̂k = |ψ̂(ξ)| dξ = |ψ̂(t)|2 dt = kψk2 .
−∞ −∞
It may be seen that the Haar wavelet has only one vanishing moment. It can
be proved that if ψ has large vanishing moments then few wavelet coefficients
hf, ψj,k i will be large.
· · · ⊂ V−2 ⊂ V−1 ⊂ V0 ⊂ V1 ⊂ V2 ⊂ . . .
(b) We have ∩j Vj = {0}, that is, the zero function is the only function which
belongs to the intersection of all spaces Vj . In other words, no nontrivial
function belongs to all Vj .
(c) Every
P square integrable function can be expanded in a (finite or infinite)
series j aj gj of functions gj ∈ Vj , where convergence is understood in the
mean quadratic sense. Equivalently, every square integrable function can be
approximated to arbitrary precision by elements of Vj .
636 Modern Engineering Mathematics
"
"
"
"
...
"
... .. •
FIGURE 10.7: The Mexican Hat Wavelet ψ(t)
wavelet satisfies the basic condition for any wavelet, namely, that its integral
over (−∞, ∞) vanishes. This is possible from the following expression:
x2 d2 − x2
(1 − x2 )e− 2 =− e 2,
dx2
638 Modern Engineering Mathematics
d2 x2 d −2x x2
− 2 e− 2 = − e 2
dx dx 2
2x − x2 x2
− 2
= x(− )e 2 +e
2
x2
= (1 − x2 )e− 2 .
sin πt
φ(t) = = sincπt, − ∞ < t < ∞,
πt
and its associated basic wavelets
1 1
sin π t − 2 − sin 2π t − 2
ψ(t) = (10.15)
π t − 12
These wavelets are continuous. However, they are infinitely many times
differentiable. Also, in a certain sense they are better localized than Haar
ones. However, even though they tend to zero as t → ±∞, they nevertheless
do not die out to identically zero beyond a finite interval. Hence, they are
denied a compact support.
Wavelets 639
OB
0.6
0.4
02
-02
-0.4
-OB
-OB
-1
- 10 -6 -1! _. -2 10
08
0.6
- 0.4
-0.6
-0.8
~,·~ ....~----~.----_.~---~~--~----~--~----~--~----~.0
o----
FIGURE 10.10
Figures 10.12 through 10.14 indicate how wavelets are used in image com-
pression.
FIGURE 10.11
0011010110110
0101110100010
1010101011110
0101011101101
1101101100101
Biometric Capture & Feature Template Storage
Presentation Preprocessing Extraction Creation
Threshold: 3.5
1(11)
Zeros: 42%
Retained energy:
1M
99.95%
2(oo)
)~•)
passes through a medium, such as muscle or brain tissue, some of the energy in
the beam is absorbed by the medium and some passes through. The intensity
of the beam as it emerges from the medium can be measured by a detector.
The initial and final intensities tells us about the ability of the medium to
absorb energy.
For the sake of clarity we make some assumptions that present an idealized
view of what an X-ray is and how it behaves. Assume X-ray beam composed
of photons and is monochromatic. Each photon has the same energy level of
E and the beam propagates at a constant frequency, with the same number
of photons per second passing through every centimeter of the path of the
beam. Let us denote N (x) the number of photons per second passing through
a point x. Then the intensity of the beam at the point x is
I(x) = E.N (x).
Every substance through which an X-ray passes has the property that each
millimeter of the substance absorbs a certain proportion of the photons that
pass through it. This proportion which is specific to the substance, is called the
attenuation coefficient of the material. A Hounsfield unit of the medium is
denoted by H medium and defined by
A denotes the true attenuation coefficient Beer’s law:
dI
= −A(x)I(x)
dx
dI
= −A(x)dx.
I
If the beam starts at a location with initial intensity I and is detected, after
passing through the medium, at the location x, with final intensity, then we
get Z x1 Z x1
dI
=− A(x)dx.
x0 I x0
Thus Z x1
ln[I(x1 )] − ln[I(x0 )] = − A(x)dx.
x0
Wavelets 643
Here we know the initial and final values of intensity I(x), and A(x) providing
special property of medium being sampled by the X-ray is unknown.
Thus from the measured intensity of X-ray we are able to determine not the
values of A itself, but the values of the integral of A along the line of X-ray.
What we can measure: we can design an X-ray emission/detection machine
that can measure the values of I(x). Hence we can compute from the equation
that is integral of the (unknown) attenuation coefficient function along the
path of X-ray.
What we want to know: The value of A(x) at each location depends on
the nature of matter located at the point x. We wish to know A(x).
Tomography is from a Greek word meaning slice. In this field we want to
find internal structure of a non-transparent object by sending signals like
electromagnetic waves of different frequencies and acoustic waves.
The current excitement in tomographic imaging originated with Hounsfield
and Cormak’s invention of X-ray computed tomography for which they jointly
received 1979 Noble prize for physiology and medicine.
Radon transform invented by the Austrian mathematician Johann
Radon is the integral transform consisting of a scalar valued function over
the straight line. By computerized (computed) tomography (CT) we mean
the reconstruction of a function from its line or plane integrals. Essentially it
amounts to inverting the Radon transform. Wavelet methods have been used
in the studies of Radon Transform [42].
(a) Suppon of the Fourier transfonn of a (b) Fourier domain suppon of several el-
classical shearlet. enJenls of the shearlet system, for differ-
ent values of a and s.
FIGURE 10.15
datasets and the results evaluated with compression ratio, PSNR, average
difference, cross correlation and normalized absolute error calculations.
••• j••'
... .... ····:·· 7!XIl
·-.
7tm .... .... ·····
•• ; •••• 0
':·· . 600')
600')
.... ·····:·· ..... ·: .
..
.... ...
~·
Sill)
:nxJ
1!XIl
200)
1000
7120
Wave-Length, ft Depth, ft
yses to determine locations, quantities, and other parameters of oil and gas
reserves worldwide.
10.4 Exercises
1, 0 <≤ t < 1/2
10.1 Let ψ(t) = −1, 1/2 ≤ t < 1
0, otherwise
1, 0 <≤ t < 1
and φ(t) =
0, otherwise.
ψ(t) is known as the Haar mother wavelet and φ(t) is the called Haar
scaling function or Haar father wavelet.
(a) Write the support of ψ and φ. Draw the graphs of
(b) ψ(4t − 3), ψ(4t + 3), ψ(23 t)
(c) φ(2t), φ(2t − 2), φ(23 t) on intervals of your choice.
Wavelets 649
10.2 Show that for ψ and φ of Exercise 10.1 the following relation holds:
10.5 For the Haar scaling function (Haar father wavelet) φ show whether or
not the following translated sets re orthogonal on (−∞, ∞):
(a) {φ(2t − k)}, k ∈ Z and (b) {ψ(2t − k)}, k ∈ Z.
10.6 Examine the question of Exercise 10.5 for the Haar mother wavelet.
10.7 Introduce the concept of energy of the signal and discuss its relationship
with the wavelet transform of the signal.
651
652 Bibliography
[32] R. H. Shumway and D. S Stoffer, Time Series Analysis and Its Appli-
cations, Springer, 2003.
Bibliography 653
[34] A. H. Siddiqi, G. Korvin, et al. (Eds.), Theme Issue on Wavelet & Frac-
tals in Science and Engineering, Arab. J. Sci. Eng., 28–29, 2003–2004.
[35] A.H. Siddiqi, I. S. Duff and O. Christensen (Eds.), Modern Mathematical
Methods and Algorithms for Real Word Systems, Anamaya and Anshan,
2006.
http://www.math.poly.edu/courses/matlab.html
http://faculty.olin.edu/bstorey/notes/Fourier.pdf
http://www.mathworks.com/products/signal
http://www.bearcave.com/misl/misl tech/
http://oceanz.tamu.edu/˜baum/wavelets.html
http://www-stat.stanford.edu/˜wavelab/Wavelab 850/AboutWaveLab.pdf
http://www-stat.stanford.edu/˜donoho/Reports/1995/wavelab.pdf
Chapter 11
Miscellaneous Topics Used for
Engineering Problems
655
656 Modern Engineering Mathematics
Hausdorff Measure
Let U be any non-empty subset of n-dimensional Euclidean space, Rn ; the
diameter of U is denoted by |U | and defined as
|U | = sup{|x − y| : x, y ∈ U },
that is, the greatest distance apart of any pair of points in U . If {Ui } is a
countable (or finite) collection of sets of diameter almost δ that cover F , that
∞
is F ⊂ U Ui with 0 < |Ui | ≤ δ for each I, we say that {Ui } as a δ-cover of F .
i=1
Suppose that F is a subsetP of Rn and s is a non-negative Pnumber. For any
∞ s ∞ s
δ > 0 we define Hδ (F ) = inf{ i=1 |Ui | : {Ui }Hδs (F ) = inf{ i=1 |Ui | : {Ui }
s
is a δ-cover of F .
Thus, one looks at all covers of F by sets of diameter at most δ and seek
to minimize the sum of the sth powers of the diameter. As δ decreases, the
class of permissible covers of F is reduced. Therefore, the infinimum Hδs (F )
increases, and so approaches a limit as δ → 0. We write,
This limit exists for any subset F of Rn , though the limit value can be 0 or ∞.
Hδs (F ) is called the s-dimensional Hausdorff measure of F . The Hausdorff
dimension of F , denoted by dimH F , is defined as
dim H F = inf{s : H s (F ) = 0}
= sup{s : H s (F ) = ∞}.
0 < H s (F ) < ∞.
Definition 106. (Spectrum) Let Sα be the set of all points t ∈ R, where the
pointwise Lipschitz regularity of f is equal to α. The spectrum of singularity,
D(α), of f is the fractal dimension of Sα . The support of D(α) is the set of α
658 Modern Engineering Mathematics
such that S α is not empty. The singularity spectrum gives the proportion of
Lipschitz singularities that appear at any scale a. Fractal dimension is a first
order parameter of complexity which can degenerate. Very different structures
share the same fractal dimension. Therefore, Nekka and Li [34], [35] studied
the concept of the Hausdorff measure spectrum functions (HMSFs).
They introduce HMSF as a new way to distinguish sets having the same
fractal dimension. HMSF is based on a Hausdorff measure of the translation
of the set through itself in a continuous manner. Translation is made contin-
uously on each point (local) and the Hausdorff measure (global) is estimated
implying HMSF extracts the whole information set. The indicator function of
the intersection of a set with its translate can be viewed as a two-point joint
moment (autocovariance). This explains in a way why HMSF completes the
information obtained from pointwise descriptors like fractal dimension. From
the definition of capacity dimension as a special form of Hausdorff dimension,
it follows that if we make a disjoint cover of the support of f with intervals of
size s then the number of intervals that intersect is:
Na (s) ≈ s−D(α) .
BH (0) = 0
2
E{|BH (t) − BH (t − ∆)| } = σ 2 |∆|2H .
Partition Function
One cannot compute the Lipschitz regularity of a multifractal because its
singularities are not isolated and finite numerical resolution is not sufficient
to discriminate them. To overcome this difficulty Areodo, Bacry and Muzy [56]
introduced the concept of wavelet transform modulus maximum using a global
partition function. Let ψ be a wavelet with n vanishing movements. Mallat
[57] states that if f has pointwise Lipschitz regularity α0 ≤ nat v then the
wavelet transform has a sequence of modulus maxima that converges toward
v at fine scales. The set of maxima at the scale a can thus be interpreted as a
cover of the support of f with wavelets of scale a. At these maxima locations
1
|Tψ f (a, b)| ≈ aα0 + 2 .
Let {Up (a)}p∈Z be the position of all local maxima of Tψ g(a, b) at a fixed
scale a. The partition function Z measures the sum at a power q of all these
Miscellaneous Topics Used for Engineering Problems 659
For each q ∈ R, the scaling τ (q) measures the asymptotic decay of Z(q, a) at
fine scales:
log Z(q, a)
τ (q) = lim .
a→0 log a
This typically means that
Z(q, a) ≈ aτ (q) .
Theorem 76. (Areodo, Bacry, Jaffard, Muzy [56]) Let ∧ = [αmin , αmax ] be
the support of D(α). Let ψ be a wavelet with vanishing moments. If f is a
self-similar signal then
Theorem 77. The scaling exponent τ (q) is a convex and increasing function
of q. The Legendre transform is invertible if and only if D(α) is convex, in
which case
D(α) = min(q(α + 1/2) − τ (q)).
q∈R
4. Spectrum: compute:
Smooth Perturbations
Let f be a multi-fractal whose spectrum of singularity D(α) is calculated from
τ (q). If a regular signal is added to f then the singularities are not modified
∼
and the singularity spectrum of f = f + g remains unchanged. We study the
effect of this smooth perturbation on the spectrum calculation.
∼
The wavelet transform of f is T f˜(u, s) = T f (u, s) + T g(u, s).
∼
Let τ (q) and τ (q) be the scaling exponent of the partition functions Z(q, s)
and calculated from the modulus maxima respectively .
Theorem 78 (Arneodo, Bacry, Muzy). Let ψ be a wavelet with exactly n
vanishing moments. Suppose that f is a self-similar function.
∼
1. If g is a polynomial of degree p < n then τ (q) = τ (q) for all q ∈ <.
2. if g (n) is almost every where non-zero then
q ≥ qc
∼ τ (q)
τ (q) =
(n + 12 )
q ≤ qc
where qc is defined by
1
τ (qc ) = (n + )qc .
2
Hurst Exponent and Application
There are many processes that have random (stochastic) components and also
exhibit some predictability between one element and the next. In statistics,
this is sometimes described by the autocorrelation function (the correlation
of data set with shifted version of the data set). The autocorrelation is one
measure of whether a past value can be used to predict a future value.
A random process that has some degree of autocorrelation is referred to as
long memory process (or long range dependence). River flow exhibits this kind
of long-term dependence. A hydrologist, named Hurst, studied Nile River flows
and reservoir modeling [27 and website]. The Hurst exponent, to be estimated
in the sequel using wavelet method having various kinds of applications, is
named for him. In the recent past, applications of the Hurst exponent have
attracted attention of researchers working in different fields.
The Hurst exponent is also directly related to the fractal dimension, which
gives a measure of the roughness of a surface. The fractal dimensions and
their refinements have been used in diverse fields. The relationship between
the Hausdorff fractal dimension D and the Hurst exponent H is D = 2 − H.
Correlations
Let us recall concept of cross correlation and auto-correlation which are closely
related to Hurst exponents and analysis of real world systems.
Cross Correlation
Miscellaneous Topics Used for Engineering Problems 661
p(k) = Ck −α .
where C is a constant and p(k) is the auto-correlation function with lag k. For
a given X1 , X2 , . . . . . . , Xn at time t1 , t2 , . . . . . . , tn the k-lag auto-correlation
function is defined as
n−k
X X n
rk = (Xi − X̄)(Xi+k − X̄)/ (Xi − X̄)2 ,
i=1 i=1
X1 + X2 + · · · + Xn
where X̄ = .
n
It may be remarked that in the above definition, the observations are uni-
formly sampled. Unlike cross correlation, the auto-correlation results in a cor-
relation coefficient indicating the degree of similitude between two values of
same variable at times ti and ti+k .
The auto-correlation function is used to detect non-randomness in data
and to identify an appropriate time series model if the data are not random.
When auto-correlation is used to detect non-randomness, it is usually only the
1-lag auto-correlation that is of interest. When the auto-correlation is used to
identify appropriate time series model, the k -lag auto-correlation is plotted.
Relationship of Autocorrelation and Hurst Exponent
Exponent of α is related to the Hurst exponent by the equation
H = 1 − α/2. .
The value of Hurst exponent lies between 0 and 1. A value of 0.5 indicates
a true random walk (a Brownian motion time series). In a random walk,
there is no correlation between any element and future elements. A Hurst
exponent value 0.5 < H < 1 indicates persistent behavior (for example a
positive auto-correlation). If there is increase of time step there will probably
662 Modern Engineering Mathematics
where v
uR 2
u ∞ 2
u 0 ξ ψ̂(ξ) dξ
ξc = t R
u 2
∞
ψ̂(ξ) dξ
0
ξc
τ is length of a signal, and ξ = . The wavelet variance defined as
a
1 2
σ 2 (a) = |T (a, b)| db
τ
is often used in practice to determine dominant scale in the signal. We assume
that τ is of sufficient length to gain a reasonable estimate of σ 2 (a).
Brownian walks can be generated from a defined Hurst exponent. If the
Hurst exponent is 0.5 < H < 1, the random walk will be a long memory pro-
cess. Data sets like this are referred to as fractional Brownian motion (fbm).
The motion can be generated by a variety of methods including wavelet trans-
form. It may be recalled that in 1827 botanist R. Brown noticed that minute
Miscellaneous Topics Used for Engineering Problems 663
particles suspended in liquid moved on highly irregular paths. There are sev-
eral phenomena of this nature such as smoke particles in air, fluctuations in
the stock market recorded at intervals of 5 minutes during market hours. Ein-
stein published a mathematical study of this motion, which eventually led
Perrin’s Nobel Prize winning calculation of Avogadro’s number [43].
1
It may be noted that spectral density is proportional to β where β = 2H + 1,
ξ
1
PW (ξf ) ∝
ξβ
ξa
where ξ = , and a is the scale parameter. For this reason, the fbm is
a
sometimes referred to as noise. There are several methods [43] to estimate the
Hurst exponent for real world systems. We present here one of those methods
which is based on wavelet methodology. The wavelet coefficient variance scale
index j denoted by σj2 is defined as follows.
2J−j
X−1
σj2 = (dj,k )2 /2J−j .
k=0
It is known that the wavelet power spectrum PW (ξ) is related to the wavelet
coefficient variance
J−j
2−1
2j ∆t X 2 ∆t 2
PW (ξj ) = dj,k ∆t = σ
τ ln 2 ln(2) j,k
k=0
j
as τ = 2 ∆t, where ∆t is sampling time (one may choose). That is,
2
PW (ξm ) ∝ σj,k .
Combining these relations and the fact that frequency ξ is inversely propor-
tional to the wavelet scale a = 2j , we obtain the scaling relationship
2 2H+1
σj,k ∝ aj
or simply,
2H+1
σj2 ∝ aj
or
1
H+ 2
σm ∝ am .
For an orthonormal multiresolution analysis using a dyadic grid, the scale a
is proportional to 2m . We can therefore take base 2 logarithms of both sides
2 2H+1
of σj,k ∝ aj to get equation
2
log2 (σm ) = (2H + 1)m + C
664 Modern Engineering Mathematics
where the constant C depends both on the wavelet used and the Hurst ex-
ponent. By plotting this, the slope of regression line λ will give the Hurst
exponent.
λ − 1
H= .
2
This method has been applied to estimate the Hurst exponents for financial
and rainfall data [43] for different time intervals. A graphic user interface
(GUI) has been developed.
Wavelet Correlation Coefficient
As mentioned above wavelets can detect both the location and the scale of
a structure. Wavelets are parameterized both by a > 0 (dilation parameter)
and translation parameter b(−∞ < b< ∞) such
that
x−b
ψa,b = ψ .
a
The wavelet domain of one-dimensional function ψ is rather two-dimensional
in nature; one dimension corresponds to scale and other to translation. The
continuous wavelet transform for one dimension is defined as
Z ∞
∗ |x − b|
ω(a, b) = f (x)ψ
−∞ b
where a is the scale, f (x) is a one-dimensional function and ψ ∗ (* is a complex
conjecture) is an analyzing wavelet, also known as mother wavelet. We can
choose a Mexican hat wavelet as a possible analyzing wavelet:
! !
2 2
(|x − b|) 1 |x − b| |x − b|
ψ = 2− exp − .
a (2π)0.5 a a2 a2
Wavelet spectrum denoted by M (a)Z is defined as
1 ∞
M (a) = w(a, b)2 db.
a −∞
The wavelet spectrum has power law behavior
M (a) ≈ aλ .
Wavelet spectrum M (a) defines the energy of wavelet coefficient for scale a.
Here λ is exponent, the value of which is decided by the power of a.
Wavelet Cross Correlation Coefficient is defined as
If r is negative then the variations at two stations are not correlated and
for positive value the variations are correlated. In order to check the features
of temperature variations at two metrological stations one can plot wavelet
spectrum of temperatures at these stations and compare the features of the
correlation coefficients. In order to check the correlation between any two
parameters at two different stations we compute r with the aid of equation or
r given by equations.
I0 = [0, 1]
666 Modern Engineering Mathematics
I1 = [0, 13 ] [ 23 , 33 ]
S
I2 = [0, 19 ] [ 29 , 39 ] [ 69 , 79 ] [ 89 , 99 ]
S S S
1
S 2 3 S 6 7 S 8 9 S 18 19 S 20 21 S 24 25 S 26 27
I3 = [0, 27 ] [ 27 , 27 ] [ 27 , 27 ] [ 27 , 27 ] [ 27 , 27 ] [ 27 , 27 ] [ 27 , 27 ] [ 27 , 27 ].
I 4 = I3 minus the middle open third of each interval in I3 .
...
...
...
I N = I N −1 minus the middle open third of each interval in I N −1 .
The Cantor set C is defined as
∞
\
C= In ,
n=0
where I0 ⊇ I1 ⊇ I2 ⊇ I3 ⊇ IN −1 ⊇ IN ⊇ IN +1 .
C 6= ∅ as O ∈ C. C is a perfect set.
Eo A E, C
~a" F
In Figure 11.1, X1 , Y1 , Z1 are middle points of the sides AB, BC and AC,
respectively. Remove the triangle with vertices X1 , Y1 and Z1 .
X100 , X1000 , Y 0 are middle points of the triangle AX1 Z1 and so on. Next we
remove the central triangles of three left-over triangles of E1 .
Continue this process. Whatever is left is called the Sierpinski gasket or
Sierpinski triangle (Figure 11.1). For an interesting account of the Sierpinski
gasket, we refer to Stewart [45].
Iterated Function System
Miscellaneous Topics Used for Engineering Problems 667
T 00 (x) = x
E,
E,
E,
E,
...•
F
and is given by
Definition 109. The fixed point A ∈ H(X) described in the IFS theorem
is called the attractor of the IFS. The attractor is also called deterministic
fractal or fractal.
Let W : R2 → R2 be a contraction map, then A and W (A) are shown in
Figure 11.3
14'(A1
[ [ [ N
[
W (B) = W1 (B) W2 (B) .... WN (B) = Wn (B)
n=1
for each B ∈ H(X). Then W is a contraction mapping with contractivity
factor s = max{sn : n = 1, 2, . . ., N }.
xn+1 = rxn (1 − xn )
Value ol r --+
148 170 199 242 272 293 347 405 467 505 559 606 % Aug
136 158 184 209 237 259 312 355 404 404 463 508 % Sep
119 133 162 191 211 229 274 306 347 359 407 461 % Oct
104 114 146 172 180 203 237 271 305 310 362 390 % Nov
118 140 166 194 201 229 278 306 336 337 405 432 ]; % Dec
When we create a time series object, we can keep the time information along
with the data values. We have monthly data, so we create an array of dates
and use it along with the Y data to create the time series object. The Matlab
Code is:
yr = repmat((1949:1960),12,1);
mo = repmat((1:12)’,1,12);
time = datestr(datenum(yr(:),mo(:),1));
ts = timeseries(y(:),time,‘name’,‘AirlinePassengers’);
ts.TimeInfo.Format = ‘dd-mmm-yyyy’;
tscol = tscollection(ts);
plot(ts)
..
FIGURE 11.5: Time Series Plot: Airline Passenger data 1949 to 1960
Example 322. We can use above example to examine trend and seasonality
(Figure 11.6). This series seems to have a strong seasonal component, with
a trend that may be linear or quadratic. Furthermore, the magnitude of the
seasonal variation increases as the general level increases. Perhaps a log trans-
674 Modern Engineering Mathematics
formation would make the seasonal variation be more constant. First we’ll
change the axis scale.
h\_gca = gca;
h\_gca.YScale = ‘log’;
11 ....... ..,..
1. Secular trend: smooth long term direction of a time series (Figure 11.7).
2. Cyclical Variation: the rise and fall of a time series over periods longer
than one year (Figure 11.8)
3. Seasonal Variation: Patterns of change in a time series within a year
which tends to repeat each year (Figure 11.9).
4. Irregular variation: classified as episodic (unpredictable but identifi-
able)or residual (chance fluctuation and unidentifiable).
Miscellaneous Topics Used for Engineering Problems 675
..
~ Hun Cop<!t k.i.L'(Mf:C"J.
4 8 t 0 E F G I J
J Ytlr ... _.,•• ,000¥!
2
3
"')
;004
"-0
67.l
Number of Anociatesat Home
Depot, Inc. {1993 tc 2007)
.....
. • ....-.-
4 1991 111.8
•oo r
-·
5 1990
»O r-·--
-
6 1)91 1~ .•
.. -
7 1!1911 156.7 - 3110
~
,a
...,
- ••
1099 20.L.. >SO
.II
""""
l
217J
10
11
2001
lOQ2
256.1
:>lll.
no
JOO •
-
~0
2J)I)) 2lll!••
·~
13 2001 323.1
0 f- ~ ~
lA 2DOS 34U 1992 lt94 ltU ItA f()OO 1002 NICI4 1006 tOOl
lS 21106 360.4 T.,.
16 2007 )Jl.l
75
70
0
8
"C
0
65
60
\Long-term
(/)
secular trend
~Recovery1
55
(/)
-~
~ 50
co
45
40
20.0
v
'2
~
.E
!ft 10.0
"'
Q)
'iij
Cf)
0
at a2 a3 a. a, a2 a3 a. a, a2 a3 a.
2007 2008 2009
CHART 16-2 S"lcs of B"scball and Softball Equipment, Hcrchcr Sporting Goods,
2007-2009 by Qtmtcr
Electroencephalogram (EEG)
Electroencephalogram is used to study brain activity. A seizure causes a sud-
den surge of electrical activity in the brain and may be an indicator of various
types of brain disorders. Time series analysis measurements help diagnose and
treat diseases of the nervous system such as epilepsy. More than 40 types of
Miscellaneous Topics Used for Engineering Problems 677
Wavelet methods
Wavelet methods have emerged as a key time frequency analysis and coding
tool for EEG and other types of signals. Wavelet transforms and scalograms
act like microscopes by zooming into small structures to reveal time events
and into large structures to determine wave form trends. Energetic parame-
ters of signals are analyzed to extract details of specific events. The techniques
are based on the different frequenices of transient events that exhibit different
energy contents.
A paper by Magosso et al. [58] summarizes recent developments and cites
Daubechies technique and other types of wavelets such as Coeflets and Sym-
lets. Performance of various wavelets have been studied and indicate that
wavelet transforms out-perform short-time Fourier (Gabor) transform.
A wavelet is a quickly vanishing oscillation function localized as a continuous
or discrete signal decomposing into a scaled and translated version ψa,b (t) of
678 Modern Engineering Mathematics
where a and b are the scale and translation parameters, respectively, with
a, b ∈ R and a 6= 0.
The continuous wavelet transform (CWT) of a signal f ∈ L2 (R) or space of
finite energy signals or the space of the square integrable functions is defined
as:
Z ∞ s
1 ? t−b
W (a, b) = Ca,b = f (t) ψ dt = hf, ψa,b i
−∞ |a| a
where h., .i denotes the inner product (dot product) and the symbol * means
complex conjugate. CWT provides a redundant representation of the signal
and requires a heavy burden of computation.
The discrete wavelet transform (DWT) is obtained by discretizing the
parameters a and b. Choose a = 2−j , b = k2−j with j, k ∈ Z. By substituting
this we get
ψj,k (t) = 2j/2 ψ(2j t − k).
The DWT can be written as
Z ∞
dj,k = f (t)2j/2 Ψ(2j t − k)dt = hf, ψj,k i
−∞
where dj,k are known as wavelet (or detailed) coefficients at scale j and loca-
tion k.
By appropriate selection of mother wavelet ψ, the family {ψ j,k (t)}j,k∈Z forms
an orthonormal basis for L2 (R). The original signal can be constructed from
DWT. Further more, the DWT may be interpreted in terms of a multiresolu-
tion analysis, where a hierarchy of approximation and details of the signal is
constructed in nested subspaces of L2 (R).
Given a signal f (t), its multiresolution decomposition at level h defined as ψ(t)
as mother wavelet, while it is a companion function, called a scaling function:
This equation expresses the synthesis of the original signal from wavelet coef-
ficients.
Let a signal be of finite length, say N and N = 2M . The approximated
and detailed signals at scale j will have only 2M −j samples each, because of
the downsampling operation. Coefficients at each scale j are placed at instant
tj,k = k2j (k = 0, 1, 2, ...2M −j ).
Hence, the range of scales that can be investigated is 1 ≤ j ≤ M , since the
decomposition can proceed only until the individual details contains a single
coefficient.
If the decomposition is carried out over all resolution levels M , the wavelet
expansion will be
M
M 2X
X −1
f (t) = dj,k 2j/2 ψ(2jt − k).
j=1 k=0
Note that k starts from 0 since we assume, without loss of generality, that the
signal starts from t = 0.
Scalograms
Scalograms are graphical representations of the squares of the wavelet coef-
ficients for the different scales. They are isometric views of sequences of the
wavelet coefficients versus wavelength. A scalogram clearly shows more details,
identifies the exact location and time and detects low frequency cyclicity of the
signal. The scalogram surface highlights the location (depth) and scale (wave-
length) of dominant energetic features within the signal. The combination of
the various vectors of coefficients at different scales (wavelengths) forms the
scalogram. The depths (location and time) with the largest (strongest) coeffi-
cients indicate the position where the particular wavelength change is taking
place. The scalogram provides a good space-frequency representation of the
signal.
Wavelet energy
The energy contained in a one-dimensional signal f (t) is
Z d
E(t) = |f (t)|2 dt = kf k2 .
c
Wavelet transform decomposes a given signal into coefficients from which the
original function can be reconstructed. Total energy of the given signal and
its wavelet transform are identical.
A scalogram is a graphical representation of the square of the wavelet
coefficient versus wavelength. A scalogram shares more details of a signal and
detects low frequency cyclicities (Figure 11.10).
where the symbol h i denotes the average over many samples of x(t) and
H denotes the Hurst exponent taking values between 0 and 1. If H = 0.5,
we obtain the usual Brownian motion. The Hurst exponent is related to the
fractal dimension D of the time series curve by:
D = 2 − H.
If the fractal dimension D for the time series is 1.5, we again get the
usual Brownian motion. In this case there is no correlation between ampli-
tude changes corresponding two successive time intervals. Therefore, no trend
in amplitude can be discerned from the time series and the process is un-
predictable. However, as the fractal dimension decreases to 1.0, the process
becomes more predictable as it exhibits persistence. The future or the past
trend is more likely to follow an established trend [52]. As the fractal dimen-
sion increases from 1.5 to 2.0, the process exhibits anti-persistence; a decrease
in amplitude of the process is more likely to lead an increase in the future or
past trend. Hence the predictability again increases. The scale independent
unit thus gives us predictability of the acting process. References [39] through
[40], [41] through [44] and [53] deal with the prediction of meteorological be-
havior using fractal methods for India, Saudi Arabia and Mexico data.
Fractal dimension (D)
Fractal dimension is a numerical measure of the roughness of an object. We
are familiar with one dimension of a straight line, or two dimensions of a plane.
What about a fractal dimension between the two. A mathematical definition
of this concept is given in Section 11.1.1. A relationship between Hurst ex-
ponent (H) and fractal dimension (D) is given earlier. Computation of H is
described in Section 11.1.1. There are several software programs available for
682 Modern Engineering Mathematics
the computation of fractal dimensions and Hurst exponents; see for example
MATLAB and Benoit software.
Wavelet analysis is a tool for analyzing localized variations in power by
decomposing a trace into time frequency space to determine both the dom-
inant modes of variability and how those modes vary in time. This method
is appropriate for analysis of non-stationary traces, that is, where the vari-
ance does not remain constant with increasing length of the data set. Fractal
properties are present where the wavelet power spectrum is a power law func-
tion of frequency. The wavelet method is based on the property that wavelet
transforms of self-affine traces have self-affine properties. Consider n wavelet
transforms each with a different scaling coefficient ai , where S1 , S2 , . . . , Sn
are the standard deviations from zero of the respective scaling coefficients ai .
Define the ratio of the standard deviations G1 , G2 , . . . , Gn−1 as:
D = 2 − H.
If the fractal dimension D for the time series is 1.5, there is no correlation
between amplitude changes corresponding to two successive time intervals.
Therefore, no trend in amplitude can be discerned from the time series and
the process is unpredictable. However, as fractal dimension decreases, to 1,
the process becomes more predictable as it exhibits persistence. Predictably
indices (denoted by P IT , P IP and P IR ) for temperature, pressure, and pre-
cipitation are defined as follows:
Concepts of fractal and multifractal and their relevance to real world systems
were introduced by Benoit Mandelbrot, for updated references and interest-
ing introduction of the theme we refer to Mandelbrot and Hudson [27]. In
many real world systems represented by time series, the pattern of singulari-
ties shown by a graph of points changes abruptly and analysis is a challenging
task. The time series of rainfall data are usually fractal or multifractal.
Wavelet-based Hurst exponent and fractal dimensional analysis
Miscellaneous Topics Used for Engineering Problems 683
Software list
• BenoitT M software developed by B. Mandelbrot
}
___ .,.
I nput
Vertor ___ .,. Output
Vector
__ _.,.
The output function determines the neuron’s signal processing and is denoted
by o(I). Together these two functions determine the values of the outgoing
neuron signals. The total function acts in the input space of N -dimension
called the parameter space. The composition of these two functions is called
the transfer function o(I(x)). The activation and the output functions of the
input and the output layers (Figure 11.11) may be of different types than those
of the hidden layer. In general, linear functions are used for inputs, outputs
and non-linear functions for hidden layers.
Single neuron
The basic unit of computation in a neural network is the neuron (Figure 11.12)
often called a node or unit. It receives input from other nodes or from an
external source and computes an output. Each input has an associated weight
(w), which is assigned on the basis of its relative importance to other inputs.
The node applies a function f (defined below) to the weighted sum of its
inputs as shown in the figure:
The network takes numerical inputs X1 and X2 and has weights w1 and w2 as-
sociated with those inputs. Additionally, another input 1 with weight b (called
the bias) is associated with it. The output Y from the neuron is computed as
Miscellaneous Topics Used for Engineering Problems 687
(Input 1) Xl
(Input 2)
shown in the figure. The function f is non-linear and is called the activation
function (Figure 11.13). The purpose of the activation function is to introduce
non-linearity into the output of a neuron. This is important because most real
world data is non-linear and we want neurons to learn non-linear represen-
tations. Every activation function (or non-linearity) takes a single number
and performs a certain fixed mathematical operation on it. There are several
activation functions you may encounter in practice:
1. The sigmoid function takes a real valued input to a range between 0 and
1:
σ(x) = 1/(1 + exp(−x))
.
2. The tanh function squashes a real-valued input and squashes it to the
range [−1, 1]:
tanh(x) = 2σ(2x)−1
.
3. The rectified linear unit (ReLU) threshold a real valued input by replac-
ing negative values with 0:
Network topologies
Feed forward network: The data flow from input to output units is strictly
feed forward. The data processing can extend over multiple (layers of) units,
688 Modern Engineering Mathematics
Output I
Outpu12
Output nodes are collectively referred to as the output layer and are respon-
sible for computations and transferring information from the network to the
outside world.
In a feedforward network, the information moves in only one direction: forward
from the input nodes, through the hidden nodes (if any) to the output nodes.
There are no cycles or loops in the network. This property of feedforward
networks is different from recurrent neural networks in which the connections
between the nodes form a cycle.
Feedforward Neural Networks
The Single Layer Perceptron is the simplest type and has no hidden layer
and can learn only linear functions. A multilayer perceptron (MLP) contains
one or more hidden layers along with an input layer and an output layer. It
can handle linear and non-linear functions. Figure 11.15 shows a multilayer
perceptron with a single hidden layer. All connections of an MLP have asso-
ciated weights. The figure shows only three weights (w0 , w1 , w2 ).
Layer Description
The input layer has three nodes. The bias node has a value of 1. The other
two nodes have external inputs values of X1 and X2 and depend on the input
data set. No computations are performed in the input layer. The 1, X1 and
X2 outputs are fed into the hidden layer.
The hidden layer also has three nodes and the bias node has an output of 1.
The outputs of the other nodes depend on the 1, X1 , X2 outputs and weights
associated with the connections (edges). The output connection from one of
the hidden nodes is highlighted in Figure 11.15. The output from other hid-
den node can be calculated. Remember that f denotes activation function.
Outputs from the other hidden layer are fed to the output layer.
The output layer has two nodes that receive inputs from the hidden layer and
perform computations similar to those shown for the highlighted hidden node
in the figure. The Y1 and Y2 values calculated act as outputs of a multilayer
perceptron.
Given a set of features X = (x1 , x2 , . . .) and a target y, a Multi Layer
Perceptron can learn the relationship between the features and the target,for
either classification or regression.
Example 323. (XOR Problem) Some problems cannot be solved by any
perceptron. In fact there are more such problems than problems which can be
solved using perceptrons. The most often quoted example is the XOR problem
of building a perceptron which takes two Boolean inputs and outputs the XOR
of them. What we want is a perceptron which will output one if the two inputs
are different and 0 otherwise.
690 Modern Engineering Mathematics
Yl
Y2
y
1
FIGURE 11.16
So the output is the desired 0 output. If one of the inputs is 0 and the other is
Miscellaneous Topics Used for Engineering Problems 691
1, then the net input is 1. This is above the threshold, and so the output 1 is
obtained. But the given perceptron fails in that case. To see that no perceptron
can be built to solve the problem, try to build one. Now we need to cover an
important point regarding the input data and the desired output. We use the
binary OR operator as an example to explain the function of the weights and
threshold. With OR we want a binary output so a single perceptron with two
inputs is created. Now the search space for the neural network can be drawn
as shown in Figure 11.17.
Input 1
''
''
•
''
''
' ' 1;.,
"$
':0~
~ ..
''
''
''
0 '' lnput 2
''
The dark dots represent values of true and the light dot represents a value of
false; you can clearly see how the two classes are separable. We can draw a
line separating them as in the above example. This separating line is called
a hyperplane. A single neuron can create a single hyperplane and the above
function can be solved by a single neuron. Another important point is that the
hyperplane above is a straight line and that means we used a linear activa-
tion function (i.e. a step function) for our neuron. If we used a sigmoid or
similar function the hyperplane would resemble a sigmoid shape as seen in
Figure 11.18. The hyperplane generated by the image depends on the activa-
tion function used.
Remember the threshold (bias) value cited earlier? What does that
do? Simply put, it shifts the hyperplane left and right while the weights ori-
entate the hyperplane. In graphical terms the threshold translates the hyper-
plane while the weights rotate it. This threshold also needs to be updated
during the learning process. The basic procedure is as follows:
/,... ---
/
I
I
J
I
I
/
-/
-------
lnput 2
The learning rate term is very important because it greatly affects the per-
formance and accuracy of a network. For more details we refer to [20], [23] ad
[63].
Zadeh introduced fuzzy sets with flexible membership. In fuzzy sets, many
degrees of membership are allowed. The degree of membership to a set is indi-
cated by a number between 0 and 1. Fuzzy sets can be considered extensions
and generalizations of the crisp sets.
Miscellaneous Topics Used for Engineering Problems 693
~· (x) p (x)
1.0
0
20 50 80 50
(a) ( b)
Bell: A bell MF (Figure 11.20) is a function with two parameters defined by:
1
bell(x, a, b, c) = x−c 2b .
1+
a
MF
c-a c c +a X
2a
expert systems, robotics and pattern recognition. The fuzzy inference system
is also known as the fuzzy expert system, fuzzy model, fuzzy-rule-based sys-
tem, fuzzy logic controller and fuzzy system. The basic structure of a fuzzy
inference system consists of five functional components:
1. Rule base which contains a selection of fuzzy rules.
2. Database which defines the membership functions used in the fuzzy
rules.
3. Reasoning mechanism which performs the inference procedure upon the
rules and given facts to derive a reasonable conclusion.
4. Fuzzification interface which transforms the crisp inputs into degrees of
match with linguistic values.
5. Defuzzification interface which transforms the fuzzy results of the infer-
ence into a crisp output.
The steps of fuzzy reasoning (inference operations upon fuzzy if-then rules)
performed by fuzzy inference systems are:
1. Compare the input variables with the membership functions on the an-
tecedent part to obtain the membership values of each linguistic label
(fuzzification step).
2. Combine (through a specific T-norm operator, usually multiplication or
min) the membership values on the premise part to get firing strength
(weight) of each rule.
3. Generate the qualified consequents (either fuzzy or crisp) of each rule
depending on the firing strength.
4. Aggregate the qualified consequents to produce a crisp output (defuzzi-
fication step).
696 Modern Engineering Mathematics
where x is the input to the node i and Ai is the linguistic label associated
with this node function. Oi1 is the membership function of Ai and it specifies
the degree to which the given x satisfies the quantifier Ai . Assume µAi (x) is
bell-shaped with a maximum equal to 1 and minimum equal to 0 such as the
generalization bell function
1
µAi (x) = 2 bi
x−ci
1+ ai
where {ai , bi , ci } is the parameter set. As these parameters change, the bell
shaped functions vary accordingly. Any continuous and piecewise differentiable
functions such as commonly used trapezoidal or triangular-shaped member-
ship functions are also qualified candidates for node functions in this layer.
Parameters in this layer are referred as premise parameters.
Layer 2: Every node in this layer is a circle node which multiplies the incoming
signals and sends the product out. For instance
Oi4 = wi fi = wi = wi (pi x + qi y + ri )
where is the output of layer 3 and (pi , qi , ni ) is the parameter set. Parameters
in this layer will be referred as consequent parameters.
Layer 5: The single node in this layer is a circle node that computes the overall
output as the summation of all incoming signals, i.e.,
P
X wi fi
Oi5 = wi fi = Pi .
i wi
(a)
Layer 1
Lsyer4
~ Layer 2 Lsyer3 ~
~ ~ LayerS
X ~
f
y
(b)
Since we understand the fuzzy operators AND, OR and NOT, the premise
of a fuzzy rule may well be formed from a combination of fuzzy propositions.
All the rules of a fuzzy system constitute the decision matrix. Here is the
decision matrix for our tip example:
If the service is bad or the food is awful then the tip is low.
If the service is good then the tip is average.
If the service is excellent or the food is delicious then the tip is high.
es
vs
es V4
Example 327. In Figure 11.23 edges e2 , e6 and e1 are incident with vertex
v4 .
Definition 120. Two nonparallel edges are said to be adjacent if they are
incident on a common vertex.
Example 328. e2 and e7 in Figure 11.23 are adjacent. Similarly, two vertices
are said to be adjacent if they are the end vertices of the same edge. In the
figure, v4 and v5 are adjacent, but v1 , and v4 are not. The number of edges
incident on a vertex vi , with self-loops counted twice, is called the degree,
d(vi ), of vertex vi .
Theorem 81. The number of vertices of odd degree in a graph is always even.
702 Modern Engineering Mathematics
Definition 121. A graph that has neither self-loops nor parallel edges is
called a simple graph. In some graph theory literature, a graph is defined
to be simple, but in most engineering applications it is necessary that parallel
edges and self-loops be allowed; this is why our definition includes graphs with
self loops and/or parallel edges.
It should also be noted that in drawing a graph, whether the lines are drawn
straight or curved, long or short is immaterial. What is important is the
incidence between the edges and vertices. For example, the two graphs drawn
in figure below are the same, because incidence between edges and vertices is
the same in both cases.
3 3
(a) (b)
In a diagram of a graph, two edges may seem to intersect at a point that does
not represent a vertex, for example, edges e and f in Figure 11.24. Such edges
should be thought of as in different planes, thus having no common point.
(Some authors break one of the two edges at such a crossing to emphasize this
fact.)
Applications of Graphs
Graph theory has a very wide range of applications in engineering and other
areas. A graph can be used to represent almost any physical situation involv-
ing discrete objects and a relationship among them. The following are four
examples from hundreds of such applications.
Konigsberg Bridge problem
The Konigsberg bridge problem is perhaps the best-known example in graph
theory. It was a long-standing problem until solved by Leonhard Euler (1707-
1783) in 1736, by means of a graph. Euler wrote the first paper in graph
theory and thus became the originator of the theory the rest of topology. The
problem is depicted in Figure
c D
In Figure 11.28 it shows how this problem can be represented by a graph; the
conduits are shown as edges while the houses and utility supply centers are
vertices. Thus the answer to the problem is no.
Miscellaneous Topics Used for Engineering Problems 705
Since there are only a few different types of electrical elements, the varia-
tions in networks are chiefly due to the variations in topology. Thus electrical
network analysis and synthesis are mainly studies of network topology. In
the topological study of electrical networks, factor 2 is separated from l and
is studied independently. The topology of a network is studied by means of
its graph. In drawing a graph of an electrical network the junctions are rep-
resented by vertices, and branches (which consist of electrical elements) are
represented by edges, regardless of the nature and size of the electrical ele-
ments. An electrical network and its graph are shown in Figures 11.29 (a) and
11.29 (b).
Definition 123. A graph in which all vertices are of equal degree is called a
regular graph (or simply a regular).
Example 331. The graph of three utilities shown in Figure 11.29b is a regular
of degree 3.
Example 332. (Seating problem) Nine members of a new club meet each
day for lunch at a round table. They decide to sit such that every member
has different neighbors at each lunch. How many days can this arrangement
last?
706 Modern Engineering Mathematics
This situation can be represented by a graph with nine vertices such that
each vertex represents a member, and an edge joining two vertices represents
the relationship of sitting next to each other. Figure 11.31 shows two possible
seating arrangements: l 2 3 4 5 6 7 8 9 l (solid lines) and 1 3 5 2 7 4 9 6 8 I
(dashed lines). It can be shown by graph-theoretic considerations that there
are only two more arrangements possible. They are l 5 7 3 9 2 8 4 6 l and I 7
9 5 8 3 6 2 4 1 . In general it can be shown that for n people the number of
such possible arrangements is (n − 1)/2 if n is odd and (n − 2)/2 if n is even.
9
/
----r--------- 1
\
\
I \
I \
I \
' / \
7 --J---~L ------\--- 2
' f , \
'i-, /
,: ,)', , . . .
I
......
\\
1/ \ ,
, .........
, ....),,...
--
.............
'<., \ -----
,"" ......
;;:--~~.... \
5 4
set E need be finite, in most theories and most application sets are finite. A
graph with a finite number of vertices and a finite number of edges is called
a finite graph; otherwise, a graph is infinite. The graphs in Figures 11.23,
11.26, 11.27 and 11.29(b) are all examples of finite graphs. Portions of two
infinite graphs are shown in Figure 11.31
FIGURE 11.33: Graph Containing Isolated Vertices, Series Edges, and Pen-
dant Vertex
Definition 126. Two adjacent edges are said to be in series if their common
vertex is of degree 2.
Example 334. In Figure 11.32 the two edges incident on v1 are in series.
• 1.7
. £16
In the definition of a graph G = (V, E), it is possible for the edge set E to
Miscellaneous Topics Used for Engineering Problems 709
be empty. Such a graph, without any edges, is called a null graph. In other
words, every vertex in a null graph is an isolated vertex. A null graph of six
vertices is shown in Figure 11.33. Although the edge set E may be empty, the
vertex set V must not empty; otherwise, there is no graph. In other words, by
definition, a graph must have at least one vertex.
Application of graph theory to linguistic has attracted attention of fairly
good number of scholars in non-engineering fields, for example, Morgan
Sanderegger, an assistant linguistics professor at McGill University in Mon-
treal [mcgill.ca/∼morgen/rlhijmeGraphs.pdf].
x2 + 2x + 1
3y 3 − 8y 2 + 4y − 2.
The value of the largest exponent is called the degree of the polynomial.
If a set of data contains n known points, then there exists exactly one poly-
nomial of degree n − 1 or smaller that passes through all of those points. The
polynomial’s graph can be thought of as “filling in the curve” to account for
data between the known points. This methodology, known as polynomial in-
terpolation, often (but not always) provides more accurate results than linear
interpolation. The main problem with polynomial interpolation arises from the
fact that even when a certain polynomial function passes through all known
data points, the resulting graph might not reflect the actual state of affairs.
It is possible that a polynomial function, although accurate at specific
points, will differ wildly from the true values at some regions between those
points. This problem most often arises when “spikes” or “dips” occur in a
graph, reflecting unusual or unexpected events in a real-world situation. Such
anomalies are not reflected in the simple polynomial function which, even
though it might make perfect mathematical sense, cannot take into account
the chaotic nature of events in the physical universe.
Example 335. Given data points at x0 = 2, y0 = 3 and at x1 = 5, y1 = 8,
find the following: At x = 4, y =?
Miscellaneous Topics Used for Engineering Problems 711
FIGURE 11.35
TABLE 11.1
x f(x)
2.0 0.85467
2.3 0.75682
2.6 0.43126
2.9 0.22364
3.2 0.08567
f[x.. )
flxsl
FIGURE 11.36
n.
S0 (x) x ∈ [x0 , x1 ]
S1 (x) x ∈ [x1 , x2 ]
S(x) := . .
. .
Sn−1 (x) x ∈ [xn−1 , xn ].
The coefficients can be found by choosing a z0 and then using the recurrence
relation
yi+1 − yi
zi+1 = −zi + 2 .
xi+1 − xi
Cubic spline
All splines considered on this page are cubic splines; they are all piecewise
cubic functions. However, anyone who says “cubic spline” usually means a
special cubic spline with continuous first and second derivatives. The cubic
spline is given by the function values in the nodes and derivative values on the
edges of the interpolation interval (either of the first or second derivatives).
• If the exact values of the first derivative in both boundaries are known,
714 Modern Engineering Mathematics
1.2,---,---.-----.-----,------,--,---.-----.----.-----,
-0.2 '----'----'----'----'-----'--'----'----'---'---'
-5 -4 -3 -2 -1 0 2 3 4 5
an outlier. On the graph you can see a set of points having one outlier. On
the intervals which are next to the outlier, the spline noticeably deviates from
the given function because of the outlier. We can see that in contrast to the
cubic spline, the Akima spline is less affected by the outliers. An important
property of the Akima spline is its locality. Function values in [xi , xi+1 ] de-
pend on fi−2 , fi−1 , fi , fi+1 , fi+2 , fi+3 only. The second property which should
be taken into account is the non-linearity of the Akima spline interpolation:
the result of interpolation of the sum of two functions doesn’t equal the sum
of the interpolation schemes constructed on the basis of the given functions.
No fewer than five points are required to construct the Akima spline. In the
inner area (i.e. between x2 and xN −3 when the index goes from 0 to N − 1)
the interpolation error has order O(h2 ).
- Original
- cubic
- Akima
-1 L-r---------~--------~
-1 0
FIGURE 11.38
FIGURE 11.39
FIGURE 11.40
The uses and applications of Maxwell’s equations are too many to count. By
understanding electromagnetism we’re able to create images of the body using
MRI scanners; we’ve created magnetic tape, generated electricity, and built
computers. Any device that uses electricity or magnets is on a fundamental
level built upon the original Maxwell equations. To describe the weak force,
physicists drew analogies to electromagnetism, and eventually found them-
selves a step higher up the unification ladder. Their ideas suggested that the
Miscellaneous Topics Used for Engineering Problems 721
two forces were, in fact, just two sides of the same coin: the unified elec-
troweak force. The Universe would be completely different if the weak force
were not weak. The idea of unification suggests that the similarity of the two
forces, electromagnetism and the weak force, was only apparent right after the
Big Bang, when the Universe was incredibly hot. As temperatures cooled, the
forces crystallized and became different. Weird as it might seem, the concept
isn’t entirely unfamiliar: think of the dramatic change that happens to water
when it freezes to ice.
The laws of nature responsible for the behavior of water are the same
everywhere and do not favor any particular direction in space. This is why a
patch of ocean looks much like any other, and appears the same no matter
from what direction you look at it. The icebergs that form when the water
freezes, however, display none of that symmetry: no two look the same. Those
with rotational symmetry are extremely rare. As the Universe cooled, the
messenger particles of the weak force (and other particles) acquired mass,
while the messenger particles of electromagnetism remained massless.
Lightning is electrostatic discharge between electrically charged regions of
a cloud during an electrical storm. Electroweak unification was a real triumph
of theoretical physics and led to Nobel Prizes in Physics for Sheldon Glashow,
Abdus Salam, and Steven Weinberg for the unified electroweak framework,
and for François Englert and Peter Higgs for the description of the mass-
related symmetry breaking mechanism. For detailed study we refer to Neun-
zert and Siddiqi [35] and Monk [31].
uid or a gas) resulting from their collision with the fast-moving atoms or mole-
cules in the gas or liquid.
2. Random Walk
with
jt
tj = .
n
The integration is evaluated in the summation at the left-hand point tj−1. It
is important that each function evaluation does not know about the random
increment that multiplies it, i.e., anticipate integration is non-anticipatory.
or in differential form:
dXt = µ(t, Xt )dt + σ(t, Xt )dBt
where µ and σ so that the integrals make sense.
Itô’s lemma
dF 1 d2 F
dF = dX + dt,
dX 2 dX 2
where F (X) is a function of Brownian motion X(t).
We obtain the result, dF = 2XdX +dt. For a comprehensive account, see [19],
[30], [50] and [54].
724 Modern Engineering Mathematics
11.12 Exercises
11.1. (a) Write the fractal dimensions of well-known fractals like
(i) Cantor set
(ii) Sierpinski triangle
(iii) Von Koch curve
(b) Can you give examples of two physical situations which are not
necessarily similar (same) but have the same fractal dimension?
11.2. Write a note on irregular geometry.
11.3. Explain the concept of IFS with a concrete example.
11.4. Define the Hausdorff metric. Is Hausdorff metric space is a metric space.
11.5. Discuss connections between fractal, Lorenz attractor, Strange attractor,
Rossler attractor and Henón attractor.
11.6. Write an essay on relations between fractals and chaos.
11.7. Is it possible to use fractals for weather forecasting or predictions of
earthquakes and tsunamis?
11.8. Give examples of time series representing physical phenomena.
11.9. What is the Hurst exponent? What is the relation between Hurst param-
eter and fractal dimension? Who initiated the study of fractals? What
do you know about multifractals?
11.10. Give example of feedforward networks.
11.11. Prove that the number of vertices of odd degrees in a graph is always
even.
11.12. Indicate areas of science, social science, linguistics, engineering and tech-
nology where graph theory has been used. Is there any relevance of graph
theory to big data?
11.13. Draw simple graphs of one, two and three vertices.
11.14. Show that the maximum number of edges in a simple graph with n
vertices is n(n − 1)/2.
11.15. Let X(t) denote Brownian motion then show that:
Rt
(i) 0 X(τ )dX(τ ) = 12 X 2 (t) − 12 t
Rt Rt
(ii) 0 f (τ )dX(t) = f (t)X(t) − 0 X(τ )df (τ ), where f (t) is bounded and
continuous function on [0,1].
Relation (ii) is known as the integration by parts of stochastic calculus.
Miscellaneous Topics Used for Engineering Problems 725
if x2 ∈ (−∞, 0)
0,
2
if x2 ∈ (0, 1)
2x ,
f (x) = 2 − x, if x2 ∈ (1, 4) .
1
3 2
x , if x ∈ (4, ∞)
16
Find knots. Show that B− splines are splines.
11.18. Find the Fourier transform of B1 (x) (B− spline of order 1).
11.19. Explain how splines can be used in finite element methods.
11.20. Write an essay on B-splines and wavelet expansions.
[9] Z. Elhadj, Lozi Mappings: Theory and Applications, CRC Press, 2013.
[10] K. Falconer, Fractal Geometry: Mathematical Foundations and Applica-
tions, Second Edition, Wiley-Blackwell, 2003.
[11] D. P. Feldman, Chaos and Fractals: An Elementary Introduction, Oxford
University Press, 2012.
[12] E. Foufoula-Georgiou and P. Kumar (Eds), Wavelets in Geophysics, Aca-
demic Press, San Diego, 1994.
[13] K. M. Furati, Z. Nashed and A. H. Siddiqi, Mathematical Models & Meth-
ods for Real World Systems, Chapman & Hall, New York, 2006.
[14] Igor, Gorban, Randomness and Hyper-Randomness, Springer, 2017.
[15] K. Holling, Finite Element Methods with B.Splines, Applied Mathemat-
ics, Philadelphia, 2003.
727
728 Bibliography
[29] Y. Meyer (with S. Jaffard and R. D. Ryan), Wavelets: Tools for Science
and Technology, SIAM, Philadelphia, 2001.
[30] G. Mircea, Stochastic Calculus, Applications in Science and Engineering,
Springer, 2002.
[31] P. Monk, Finite Element Methods for Maxwell’s Equations, Oxford Sci-
ence Publications, 2003.
Bibliography 729
[33] F. Nekka and J. Li, Introduction of triadic Cantor sets with their trans-
lates. Fundamental properties, Chaos, Solitons Fractals, 13(9): 1807-1817,
2002.
[34] F. Nekka and J. Li, A continuous translation based method to reveal the
fine structure of fractal sets, Arabian Journal for Science and Engineer-
ing, 28, Part I, 169-188, 2003.
[35] H. Neunzert and A. H. Siddiqi, Topics is Industrial Mathematics Case
Studies and Related Mathematical Methods, Kluwer, 2000.
[36] A. M. Ovrutsky and A. S Prokhoda, Computational Materials Science:
Surfaces, Interfaces, Crystallization, Kindle Edition, Elsevier, 2013.
[37] B. Oksendal, Stochastic Differential Equations: An Introduction with Ap-
plications, Springer, reprint, 2003.
[38] A. Pelander, A Study of Smooth Functions and Differential Equations on
Fractals, Dissertation, Uppsala university, Sweden, 2007.
[39] G. Rangarajan, and D.A. Sant, Fractal dimension analysis of Indian cli-
matic dynamics, Chaos, Solitons Fractals 19: 285-291, 2004.
[40] G. Rangarajan, M. Ding (Eds.) Processes with Long-Range Correlations
Theory and Applications, Springer-Berlin, Heidelberg and New York,
2003.
[41] S. Rehman, A. H. Siddiqi, Wavelet-based correlation coefficient of time
series of Saudi meteorological data, Chaos, Solitions Fractals, 39: 1764-
1789, 2009.
[44] A. H. Siddiqi, Theme issues: wavelet and fractal methods is science and
engineering, Arab. J. Sci. Eng., 28-29, 2003-2004.
[45] I. Stewart, Four encounters with Sierpinski Gasket, Math. Intel., 17: 52-
64, 1995.
[46] R. S. Strichartz, Fratafolds based on the Sierpinski gasket and their spec-
tra, Trans. Amer. Math. Soc. 355: 4019-4043, 2003.
730 Bibliography
733
734 Modern Engineering Mathematics
it
√ has been found very useful in mathematics to introduce the number i =
−1, so that i2 = −1. i is called the imaginary unit. Numbers of the form
a + bi, where a and √ b are real numbers, are called complex numbers. For
example, 2 + 5i and 3 − 6i are complex numbers. A complex number a + bi
is called imaginary (or pure imaginary) if a = 0, so 9i is an imaginary
number. Since a = a+0i, every real number a can be interpreted as a complex
2
number, as for example , thus R ⊂ C. The algebraic operations are defined
3
for complex numbers in the natural way, for example
√ √ √
(3 + 5i) + (4 − 2i) = 3 + 4 + 5i − 2i = 7 + (5 − 2)i ,
√ √ √
(3 + 5i) − (4 − 2i) = 3 − 4 + 5i − (− 2i) = −1 + (5 + 2)i ,
√ √ √
(3 + 5i) · (4 − 2i) = 3 · 4 + 5i · 4 − 3 2i − 5i · 2i
√ √
= (12 + 5 2) + (20 − 3 2)i .
π = 3.141592653 . . . ,
√
2 = 1.414213562 . . . .
3
2
-3 -2 ·1 0 1 2 3
-1
-2
-3
x = x1 and y = y1 .
Basic Concept of Calculus 737
y = m(x − x1 ) + y1 .
Indeed, this number yields the length of the line segment which connects P1
and P2 , by the theorem of Pythagoras. As a particular case, the distance of
P1 = (x1 , 0) and P2 = (x2 , 0) equals
p
d(P1 , P2 ) = (x2 − x1 )2 = |x2 − x1 | .
(x − x0 )2 + (y − y0 )2 = r2 .
The number r equals the length of the line segment joining the origin O and
the point P , and θ equals the angle between the line OP and the x-axis. Note
that θ is usually measured in radians, that is, an angle of 90o has θ = π/2, the
full angle of 360o has θ = 2π and so on. Formula (A.1) expresses the Cartesian
coordinates of P in terms of the polar coordinates of P Conversely, we obtain
the polar coordinates of P from its Cartesian coordinates by the formula
p y
r = x2 + y 2 , θ = tan−1 . (A.2)
x
Trigonometric functions
The trigonometric functions are directly related to the geometry of the circle.
738 Modern Engineering Mathematics
........................................ p(x,y )
e
M
0 X X
Y"
Consider the point P with Cartesian coordinates (x, y) and polar coordinates
(r, θ) as in Figure A.3. The standard trigonometric functions are defined as
follows.
y
sin θ = (read as “sine of θ”),
r
x
cos θ = (read as “cosine of θ”),
r
and analogously
y x r r
tan θ = , cot θ = , sec θ = , csc θ = ,
x y x y
called the tangent, the cotangent, the secant and the cosecant, respectively.
From elementary geometry we see that the definitions above do not depend
on the chosen value of r, as long as r > 0, so it suffices to consider the case
r = 1 (the unit circle) See Figure A.4.
Recall that θ is usually measured in radians (θ = π/2 corresponds to an
angle of 90o and so on).
Trigonometric Identities
A trigonometric identity is an equation involving trigonometric functions that
is true for all angles for which both sides of the equation are defined. We state
a few useful trigonometric identities.
{0,1)
l
tan x
0 B A
!
FIGURE A.4: Trigonometric Functions and the Circle
740 Modern Engineering Mathematics
In general,
x2 x2
1− <1− < 1. (A.26)
2! 3!
If x is small enough, inequality (A.26) continues to hold even after we add
the remaining terms of the series (A.24) and (A.25), because those remaining
terms have exponents of x greater than 2, and hence the remaining sums have
the form x2 r(x) for some function r with limx→0 r(x) = 0.
TABLE A.1
arctangent y = arctan(x) x∈R −π/2 < y < π/2 −90o < y < 90o
The inverse function notations sin−1 , cos−1 etc. are natural, but one must
be aware of the following notational conflict. Since we commonly write sin2 x
instead of (sin x)2 , an unspecified use of sin−1 (x) might mean arcsin(x) as well
as 1/(sin x).
its entire domain R.) They are called area hyperbolic functions, and for
the mathematical notation the prefix “ar” precedes the name of the original
function, for example “arsinh” denotes the inverse of sinh. Table A.2 lists the
principal branches of the inverse hyperbolic functions.
TABLE A.2
TABLE A.3
√ √
+ x2 +
arsinh(x) = ln (x 1) arcosh(x) = ln (x+ x2
− 1)
1 1+x 1 x+1
artanh(x) = ln arcoth(x) = ln
2 1−x 2 x−1
√ ! √ !
1+ 1 − x2 1 1 + x2
arsech(x) = ln arcsch(x) = ln +
x x |x|
(c) ln(xr ) = r ln x
(d) ln x1 = − ln x
(e) ex+y = ex ey
(f) ln(ex ) = x for all real values of x
(g) elnx = x for x > 0
1
(h) ln 1 = 0, ln e = 1, ln e = −1 and ln(e2 ) = 2
u−1 du = du
R R
2. u = ln u + C
eu du = eu + C
R
3.
ueu du = eu (u − 1) + C
R
4.
R
5. sin udu = − cos u + C
R
6. cos udu = sin u + C
R
7. tan udu = ln sec u + C
R
8. cot udu = ln sin udu + C
eau (a sin mu−m cos mu)
eau sin mudu =
R
9. m2 +a2 +C
eau (m sin mu+a cos mu)
eau cos mudu =
R
10. m2 +a2 +C
R
11. sec udu = ln(sec u + tan u) + C
R
12. csc udu = ln(csc u − cot u) + C
sec2 udu = tan u + C
R
13.
csc2 udu = − cot u + C
R
14.
R
15. sec u tan udu = sec u + C
R
16. csc u cot udu = − csc u + C
du
= arcsin ua + C
R
17. a2 −u2
du 1
arctan ua + C
R
18. a2 +u2 = a
√
√ du
R
19. a2 −u2
= ln(u + u2 ± a2 ) + C
du 1 a+u
R
20. a2 −u2 = 2a ln a−u +C
R
21. u sin udu = sin u − u cos u + C
R
22. u cos udu = cos u + u sin u + C
h i
ln u 1
udu = un+1 n+1
R
23. − (n+1) 2 +C
sin2 udu = 12 u − 1
R
24. 4 sin 2u + C
746 Modern Engineering Mathematics
cos2 udu = 12 u + 1
R
25. 4 sin 2u + C
tan2 udu = tan u − u + C
R
26.
R R
27. udv = uv − vdu + C
Γ(x + 1) = xΓ(x).
R∞
Now, Γ(1) = 0 e−t dt = 1. Therefore, Γ(2) = 1Γ(1) = 1 and Γ(3) =
2Γ(2) = 2.1 and so on. √
We see that n is a positive integer, Γ(n + 1) = n! Γ 12 = π can be derived
from (A.27) by x = 1/2.
Remark A.1. Although the integral form (A.27) does not converge for x < 0,
it can be shown by means of alternative definitions that the gamma function
is defined for all real and complex numbers except x = −n, n = 0, 1, 2, . . . .
over rectangular regions Q. The exposition closely parallels that given in [6]
for the case of a single variable. We therefore shorten it somewhat and refer
the reader to [7] for more details.
Double integral
Let the function f be defined in a rectangular region Q = [a, b] × [c, d] of
the xy-plane. We partition Q into rectangular subregions as follows. Choose
Basic Concept of Calculus 747
Here, each point (ξi , ηj ) lies somewhere in the rectangle Qij . We define the
oscillation of f on the rectangles Qij as
Oij (f ) = oscQij (f ) = max |f (x) − f (z)| . (A.29)
x,z∈Qij
Again, we have to replace the maximum by the supremum if the former does
not exist. Next, we consider two different Riemannian sums for the same
partition ∆,
n X
X m
s∆ = f (ξi , ηj )(xi − xi−1 )(yj − yj−1 ) ,
i=1 j=1
n X m
(A.30)
X
s̃∆ = f (ξ˜i , η̃j )(xi − xi−1 )(yj − yj−1 ) .
i=1 j=1
|I − s∆ | ≤ V∆ (f ) (A.32)
where (ξ, ηj , ζk ) lies somewhere in Qijk . From this point onward, the definition
of ZZZ
f (x, y, z) dV
Q
749
750 Modern Engineering Mathematics
A + O = O + A = A.
AX = B
a11 a12 . . . a1n x1 b1
a21 a22 . . . a2n x2 b2
where A = . , X = . , B = . .
. . .
.. .. .. .. .. ..
AX = λX.
d(x, y) = |x − y|
x = (x1 , x2 , ....., xn )
Here, ∆ is a partition of the interval [a, b] of the form a = x0 < x1 < · · · < xn ,
and each point ξk lies somewhere in the subinterval [xk−1 , xk ]. Thus, the value
of s∆ depends on the choice of ∆ as well as on the choice of the points ξk .
One can estimate the influence of the latter choice through the notion of
oscillation of a bounded function. If I is a subset of the domain of f (here, of
the interval [a, b]), the oscillation of f on I is defined as the maximum possible
difference of two function values on I, that is,
We remark that it may happen that this maximum does not exist (that is,
there are no points x, z ∈ I where a maximum value is attained). One then
replaces the maximum by the so-called supremum, which in this case is equal
755
756 Modern Engineering Mathematics
|I − s∆ | ≤ V∆ (f ) (C.5)
Therefore f is integrable.
Let us remark that there are different ways of defining the integral. We
have chosen a method which can be generalized conveniently to the cases of
double and triple integrals.
Fundamental theorem
Let f be a continuous function on [a, b] then
Z x
F (x) = f (t)dt
a
d Rx
has derivative at every point in [a, b] and a
f (t)dt = f (x).
Rb dx
Also a f (x)dx = F (b) − F (a).
Proof. Since f is continuous, it attains its maximum and minimum on [a, b].
Let
M = max f (x) , m = min f (x) .
x∈[a,b] x∈[a,b]
F (x + h) − F (x)
lim = f (x) .
h→0 h
In order to prove this, fix x ∈ [a, b] and let h be any number such that
x + h ∈ [a, b]. Using the definition of F together with property of indefinite
integral yields
Z x+h Z x
F (x + h) − F (x) = f (t) dt − f (t) dt
a a
Z x Z x+h Z x
= f (t) dt + f (t) dt − f (t) dt
a x a
Z x+h
= f (t) dt .
x
Consequently, if h 6= 0, then
x+h
F (x + h) − F (x)
Z
1
= f (t) dt .
h h x
In the case h > 0, by the mean value theorem, we can find a number z = z(h)
in the open interval (x, x + h) such that
Z x+h
f (t) dt = f (z(h)) · (x + h − x) = f (z(h)) · h
x
and, therefore,
F (x + h) − F (x)
= f (z(h)) . (C.6)
h
Since x < z < x + h, we have limh→0+ z(h) = x. It then follows from the
continuity of f that
lim+ f (z(h)) = f (x) ,
h→0
Proof of Selected Theorems 759
Now we consider the double integral. The region D has the form
We compute the double integral on the right hand side of (C.9) for g = 0,
using the fundamental theorem of calculus,
ZZ Z b Z k(x)
∂f ∂f
− (x, y) dA = − (x, y) dy dx
D ∂y a h(x) ∂y
Z b
=− f (x, k(x)) − f (x, h(x)) dx .
a
An analogous proof for the case f = 0, taking F̃ = (0, g), shows that
I I ZZ
∂g
F̃ · dr = g(x, y) dy = (x, y) dA . (C.12)
C C D ∂x
(In that proof, we decompose the boundary C of D into a left part and a right
part, described by some functions x = h̃(y) and x = k̃(y), respectively.) For
the general case, we decompose (f, g) = (f, 0) + (0, g), and add (C.11) and
(C.12). The theorem is proved for domains D of the form in Figure C.1.
y =k(x)
y = h(x)
x=a x=b
For the third term on the right hand side we consider the partition of Σ as
described above,
ZZ ZZ ZZ ZZ
f3 n3 dσ = f3 n3 dσ + f3 n3 dσ + f3 n3 dσ . (C.17)
Σ Σ+ Σ− Σ0
The surface integral over Σ− is treated analogously; there, however, the outer
normal vector points downward, so we have n3 = −1/ν instead of (C.19).
Consequently,
ZZ ZZ
f3 n3 dσ = − f3 (x, y, k(x, y)) dx dy . (C.21)
Σ− Dz
764 Modern Engineering Mathematics
Adding the three equations in (C.22) and (C.23) yields (C.13). This completes
the proof for the special form of D as considered.
To prove the divergence theorem for domains D of general form, one em-
ploys so-called partitions of unity of the vector field F, which reduces
the general situation to a situation where similar computations can be done
as in the proof presented above. In addition, let us remark that while we
have treated the situation in three-dimensional space, more or less the same
proof works for the case of an n-dimensional region D bounded by an n − 1-
dimensional surface Σ, where n is an arbitrary number greater than or equal
to 2. Both these developments are, however, outside the scope of this book.
where F is a vector field, Σ is a surface with unit normal field n and boundary
curve C, suitably oriented.
Proof of Theorem 45.The strategy of the proof is to transform the situation
to the xy-plane, in order to apply the Green-Ostrogradski theorem. Let us first
consider the surface integral on the right hand side of (C.24). The surface Σ is
described as z = S(x, y), that is, as the graph of a function S defined on D, the
projection of Σ onto the xy-plane. The unit normal at a point (x, y, S(x, y))
of Σ is given by
1 q
n= − ∂x Si − ∂y Sj + k , ν = 1 + (∂x S)2 + (∂y S)2 , (C.25)
ν
Proof of Selected Theorems 765
Let us now consider the line integral on the left side of (C.24). If C is pa-
rameterized by r : [a, b] → R3 , then by the definition of the line integral we
have I Z b
F · dr = F(r(t)) · r0 (t) dt . (C.30)
C a
Since Σ is the graph of S defined on D, C is the graph of S restricted to
the boundary Γ of D. Let Γ be positively oriented by the parametrization
q : [a, b] → R2 , set
where ∂x S and ∂y S are evaluated at q(t) = (q1 (t), q2 (t)). Let us now define
the plane vector field F̃ by
Setting x = q1 (t) and y = q2 (t) we see in view of (C.31) and (C.32) that
for all t ∈ [a, b]. (This is the reason for defining F̃ by (C.33.))
The theorem of Green and Ostrogradski, Theorem C.1,asserts that
I ZZ
F̃ · dr = ∂x F̃2 − ∂y F̃1 dA . (C.35)
Γ D
We compute the partial derivatives from (C.33) with the aid of the chain rule,
Since the function S was assumed to have continuous second partial deriva-
tives, we can interchange their order, so we have ∂x ∂y S = ∂y ∂x S. Thus we
obtain from (C.36) that
∂x F̃2 −∂y F̃1 = (∂z f2 −∂y f3 )·∂x S +(∂x f3 −∂z f1 )·∂y S +(∂x f2 −∂y f1 ) . (C.37)
We compare this expression with the corresponding one in (C.29) and find
that ZZ ZZ
∂x F̃2 − ∂y F̃1 dA = (curl F) · n dσ . (C.38)
D Σ
We now put together the previous calculations and finally conclude that
I Z b Z b
0
F · dr = F(r(t)) · r (t) dt = F̃(q(t)) · q0 (t) dt
C a a
I ZZ
= F̃ · dr = ∂x F̃2 − ∂y F̃1 dA
ZΓZ D
= (curl F) · n dσ .
Σ
We compute
Z Z Z b
F · dr = F(r(t)) · r0 (t) dt = ∇ψ(r(t)) · r0 (t) dt
C C a
Z b
= g 0 (t) dt = g(b) − g(a) = ψ(r(b)) − ψ(r(a))
a
= ψ(B) − ψ(A) .
In the middle line of this computation we have used the fundamental theo-
rem of calculus. Indeed, one may view Theorem 36 as a generalization of the
fundamental theorem of calculus to line integrals.
Proof of Theorem 37.
Since we already know that every conservative vector field is circulation free,
it remains to prove that every vector field, which is circulation free, is con-
servative. Let F be circulation free. We first show that F has the property of
path independence. If C1 and C2 are two curves with initial point A and end
point B, let C denote the curve which first connects A to B via C1 and then
B to A via C2 in the opposite direction (the latter curve we denote by −C2 ).
Since F is circulation free,
I Z Z Z Z
0= F · dr = F · dr + F · dr = F · dr − F · dr ,
C C1 −C2 C1 C2
thus Z Z
F · dr = F · dr .
C1 C2
Therefore
h
ψ(x + he1 ) − ψ(x)
Z
1
= f1 (r(t)) dt .
h h 0
Since the limit of the right hand side exists as h → 0 and is equal to f1 (r(0)) =
f1 (x), we obtain
∂ψ
(x) = f1 (x) , x ∈ D .
∂x
An analogous argument works for the other coordinate directions, so we finally
conclude that ∇ψ = F. The proof is complete.
771
772 Modern Engineering Mathematics
forcement, and other fields that utilizes images to generate enhancement and
measurements.
EEG: An electroencephalogram (EEG) measures and records the electrical
activity of the brain. Special sensors (electrodes) are attached to head and
hooked by wires to a computer. The computer records brain electrical activity
on a screen or on paper as wavy lines. Certain conditions, such as seizures,
can be seen by the changes in the normal pattern of brain electrical activity.
ECG: The electrocardiogram (ECG) records the electrical activity of the
heart. Each heart beat is displayed as a series of electrical waves characterized
by peaks and valleys. An ECG reveals (1) the duration of the electrical waves
crossing the heart which determines whether the electrical activity is normal
or slow or irregular and (2) reveals is the amount of electrical activity passing
through the heart muscle which enables to find whether the parts of the heart
are too large or overworked. Normally, the frequency level of an ECG signal
is 0.05100 Hz and its dynamic level is 110 mV.
MRI: Magnetic resonance imaging (MRI) uses a magnetic field and pulses of
radio wave energy to depicts organs and structures inside the body. In many
cases MRI gives different information about the body that can be seen with
an X-ray, ultrasound, or computed tomography (CT) scan. For more details
we refer to [8] and [9].
One of the most important uses of core data is to help geologists identify
depositional environment as a function of distance along the wells. Well logs
record parameters such as spontaneous potential (SP), resistivity logs, dielec-
tric logs, passive gamma-ray logs, active gamma-ray logs, neutron logs, NMR
logs, sonic logs and dip meter logs. Thus, a well log is a set of time series data.
These logs are very well presented in Iske and Randen [1]. For detailed dis-
cussion one may refer to Luthi [2] and Selly [3]. An intensive review of reservoir
modelling is also given in [2] while [3] deals with the updated petroleum geol-
ogy.
The procedure of well logging involves sending a package of instruments
known as a sonde-suspended from a cable or wire, into the formation. Well
logging is used to determine the type and properties of any field in the rocks
and to determine the geometric and physical properties of the rock. A major
aim of well logging is to tie horizons observed in wells to horizons. Well logging
helps to locate and quantify potential depths zones containing hydrocarbons.
This appendix discusses applications of wavelet methods to well logs for de-
termination and interpretation of cyclicity, zonation and abrupt changes in
sedimentary successions. Stratigraphy is a branch of geology studying rock
layers and layering. Sequence stratigraphy is the study of cyclic sedimentary
deposits. Gamma rays, porosity and bulk density are the most important
properties in oil exploration.
Appendix E
Solution of Odd Number Exercises
E.1 Exercises
775
776 Modern Engineering Mathematics
dy (y − 1)(y + 1)
=
dx (x − 1)(x + 1)
dy dx
or =
(y − 1)(y + 1) (x − 1)(x + 1)
1 1 1 1 1 1
or dy − = − dx
y−1 y+1 2 2 yx − 1 x + 1
or ln |y − 1| − ln |y + 1| = ln |x − 1| − ln |x + 1| + ln c
y−1 c(x − 1)
or =
y+1 x+1
Solution of Odd Number Exercises 779
Using y(2) = 2 we find c = 1. the solution of the given initial value problem is
y−1 x−1
= or y = x
y+1 x+1
dy
2.17 (i) dx − xy = 2y, y(1) = 5.
2.19 (i) 4m2 + 8m + 16 = 0. this gives m1 = −4 and m2 = −4, that is,
m1 = m2 so the general solution is
y = c1 e−4x + c2 xe−4x .
P (2) = 2P0
dP (t)
2.35 = λ(P (t)) or P (t) = P (0)eλt
dt
3
P0 = P0 e λ
2
3
λ = ln
2
3P0 = P0 eλt
1
3 = eλt or t = ln 3
λ
1 3
or t = 3 ln 3 = ln 3 − ln 2
ln 2
t = ln 3 − ln 3 + ln 2.
P (t) = P0 e2 ln 2 = 50000e2 ln 2 .
y2 (x) = x + 16 x3 + 121 4
x + 1201
x5 + . . .
1 2 1 4 1 3 1 5 1 2 1 3
2.73 y = c0 (1− 2! x + 4! x +. . . ) +c1 (x− 3! x + 5! x +. . . ) + 2! x + 3! x +. . .
1 2 1 3 1 4
2.75 y1 (x) = 1 + 2 x + 6 x + 6 x . . .
y2 (x) = x + 12 x2 + 12 x3 + 14 x4 + . . .
2.77 y = x − 13 x3 + 15 x5 − 17 x7 + . . . y1 (x) = 1 + 12 x2 + 16 x3 + 24
1 4
x + 301 5
x ...
1 3 1 4 1 5
y2 (x) = x + 6 x + 12 x + 120 x + . . .
2.79 y = c1 y1 + c2 y2 , where
y1 = c0 (1 + 14 x2 + 1 4
48 x + . . . )
y2 = − 12 y1 ln x + y[− 2x1 2 + 96
7 2
x − 19
2304 x
4
+ ...]
2.83 Since v 2 = 0 in the Bessel equation the general solution is c1 J0 (x) +
c2 Y0 (x), See [13].
2.85 It is Legendre’s equation of order 0. It can be seen from solution of Leg-
endre’s equation in Section 2.8.5 that it is y0 (x) = c0 = 1.
(vi) Since
y1 y2 y3
x1 x2 x3
y.(x × y) =
y1 y2 y3
= y1 (x2 y3 − x3 y2 ) − y2 (x1 y3 − x3 y1 ) + y3 (x1 y2 − x2 y1 )
= y1 x2 y3 − x3 y2 y1 − y2 x1 y3 − x3 y1 y2 + y3 x1 y2 − x2 y1 y3
= 0,
y is perpendicular to x × y or x × y is perpendicular to y.
3.3 (a) (i) We have x0 = 4, y0 = 2, a = −1 and b = 5; therefore the parametric
equations are x = 4 − t, y = 2 + 5t.
(ii) x = 1 + 4t, y = 2 + 5t, z = −3 − 7t.
d dG dF
3.5 (a) We know that [F.G] = F + G. Taking F = G = γ yields
dt dt dt
d
(γ(t).γ 0 (t)) = γ(t).γ 0 (t) + γ 0 (t).γ(t)
dt
d
so [||γ(t)||2 ] = 2γ(t).γ 0 (t).
dt
Since ||γ(t)|| is constant, γ(t).γ 0 (t) = 0. Therefore, γ(t) is orthogonal to γ 0 (t).
or div(∇φ × ∇ψ) = 0.
Solution of Odd Number Exercises 783
3.9
Z C Z 0
F.dr = (i − xj + k).(− sin ti − cos tj)dt
Zππ
= (− sin t − x(t) cos t)dt, wherex(t) = sin t
Z0 π
1
= (− sin t −
sin 2t)dt
0 2
= [[cos t]π0 + [cos 2t]π0 = 0
H
3.11 The work done is equal to C F.dr. Let D be the disk enclosed by the
circle C and let A(D) denote its area. Then
I I
F.dr = (ex − y + x cosh x)dx + (y 3/2 + x)dy
C
Z Z
∂ 3/2 ∂ x
= (y + x) − (e − y + x cosh x dA
D ∂x ∂y
by the theorem of Green Ostrogradski
Z Z
= (1 + 1)dA = 2A(D) = 2π(12)2 = 288π
D
as C is constant, divC = 0.
H RR
3.17 By Storke’s theorem, C F.dr = Σ
(curlF ).ndσ, where n is suitably
orient normal to the disk x2 + y 2 ≤ 1. We have
i j k
curlF = ∂ ∂ = −zaj + (2xy + 1)k
∂
∂x ∂y ∂z
4.9 Part (a) is Bessel’s inequality and its proof can be found in [3] of Chapter
4.
(c) It is famous Riemann-Lebsegue lemma and its proof can be found in [1,3]
4.13 Follows from definition.
4.15 We find fˆ for f = χ[− 12 , 12 ]
2 5
fˆ = sin
s 2
∞ (2n−1)2 π 2 t
(2n−1)π−8(−1)n −
4
sin (2n−1)π
P
5.3 u(x, t) = π2 (2n−1)2
e 64
8 x
n=1
∞ nπ n2 π 2 t
1−cos
40
e− sin nπ
P
5.5 u(x, t) = π n
2 64
2 x
n=1
R∞
5.21 u(x, t) = 1
2 e−|ω| cos ωteiωx
−∞
" #
Rt x+c(t−τ
R )
1
5.23 u(x, t) = 2c f (s, τ ) ds dτ
0 x−c(t−τ )
∞
400 P sin (2n−1)π x h i
5.31 u (x, y) = 2
sinh (2n−1)π
2 (1 − y)
π n=1 (2n − 1) π
(2n − 1) sinh
2
∞
200 P 1
+ sinh nπx sin nπy
π n=1 n sinh 2nπ
1 ∞ 1 − (−1)n
2 P
5.35 u (x, y) = x+ 2 sinh nπx cos nπy
2 π n=1 n2 sinh nπ
786 Modern Engineering Mathematics
∞
nπ
Ra
cos nπ 1
P
5.37 u (x, y) = A0 y + An sinh a y a y; A0 = ab f (x) dx,
n=1 0
1
Ra
An = a sin nπ f (x) cos nπ
a xdx.
a b
0
2
Rπ sinh ω(2−y)
5.41 u (x, y) = π F (ω) (1+ω 2 ) sinh 2ω sin ωxdω
0
h i
1
5.43 u (x, y) = π arctan 1+x
y + arctan 1−x
y
6.6 The two iterations of Jacobi’s method give the following results.
a. (0.1428571, −0.3571429, 0.4285714)t
b. (0.97, 0.91, 0.94)t
c. (−0.65, 1.65, −0.4, −2.475)t
6.7 b. r0 = (1, 0, −2)t , x1 = (5, 0, −1)t ,
r1 = (−1, −2, −5)t , x2 = (.51814, −.72539, . − 1.94301)t
r2 = (1.28497, −.80311, .64249)t , x2 = (1, −1.4, −2.2)
7.1 xn yn
1.00 5.0000
1.10 3.9724
1.20 3.2284
1.30 2.6945
1.40 2.3163
Solution of Odd Number Exercises 787
1.50 2.0533
7.9 The figure shows the values of u(x, y) along the boundary.
We need to determine u11 and u21 . The system is
u21 + 2 + 0 + 0 − 4u11 = 0
1 + 2 + u11 + 0 − 4u21 = 0
or
−4u11 + u21 = −2
u11 − 4u21 = −3.
Solving we obtain u11 = 11/15 and u21 = 14/15.
7.11 The figure shows the values of u(x, y) along the boundary. We need to
determine u11 , u21 , u12 ,and u22 . By symmetry u11 = u21 and u12 = u22 . The
system is
7.15 The table in this section gives a selection of the total number of approx-
imations
where
Z 1
1, n = 1
an = 2 sin nπx sin nπxdx =
0 0, n = 2, 3, 4
and Z 1
2
bn = 0dx = 0.
nπ 0
Thus
u(x, t) = cos πt sin πx
where x = 54 , y = − 54
√
8.3 (b) 5 2(cos 7π 7π
4 + i sin 4 )
8.5 (a)
π π π π
z1 z2 = 8[cos(
+ 3 ) + i sin( + 3 )]
8 8 8 8
= 8i
z1 1 π π π π
= [cos( − 3 ) + i sin( − 3 )]
z2 2 8 8 8 8
√ √
2 2
= −i
4 4
8.9 (a) f (z) = (7x−9y−3)+i(7y−9x+2) where u(x, y) = 7x−9y−3, v(x, y) =
(7y − 9x + 2)
5z 2 −2z+2 5(1+i)2 −2(i+1)+2
8.11 limz→1+i z+1 = 1+i+1
5(1 + i2 + 2i) − 2i
=
2+i
(z+∆z)2 −z 2 2z∆z+(∆z)2
8.13 (a) f 0 (z) = lim∆z→0 ∆z = lim∆z→0 ∆z
1 z3 z5 z7
8.28 (a) f (z) = [z − (z − + − + . . . )]
z5 3! 5! 7!
1 1 z2 z4
= 2
− + − + ...
3!z 5! 7! 9!
3z − 1
8.29 (a) From f (z) =
[z − (−1 + 2i)][z − (−1 − 2i)]
−1 + 2i and −1 − 2i are simple poles.
(z − 4i)z
8.31 (a) Res(f (z), 4i) = lim
z→4i (z − 4i)(z + 4i)
z
= lim
z→4i (z + 4i)
1
=
2
z
Res(f (z), 4i) = lim
z→−4i (z − 4i)
1
=
2
Z 2π I
1 4 z
8.33(a) 2
dθ = dz
0 1 + 3 cos θ i c 3z 4
+ 10z 2 + 3
√ √
4 3i 3i
= ( )2πi[Res(f (z), ( ))] + [Res(f (z), −( ))]
i z z
=π
0.9 - - O(t/2) ~
O.B
0.7
0.6
0(21) 0.5
0.4
0.3
0.2
0.1
0
-4 -3 -2 -1
1.0
O.B
0.6
0.4
0.2
-4 -2
R∞
−∞
ψ(2t−k)ψ(2t−j)dt = 0 if j 6= k. See Figures E.1 to E.4. For more details,
see [31]
796 Modern Engineering Mathematics
1.0
0.8
0.6
0.4
0 .2
-4 -2
'
~
0.8
0.6
0.4
0.2
lj/(21)
-0.2
-0.4
-0 .6
-0 .8
'
Z ∞
10.3 (i) < ϕ, ψ > = ϕ(t)ψ(t)dt
−∞
1
Z 0 Z 2
= ϕ(t)ψ(t)dt + ϕ(t)ψ(t)dt
−∞ 0
Z 1 Z ∞
+ ϕ(t)ψ(t)dt + ϕ(t)ψ(t)dt
1
2 1
1
Z 2
Z 0
= 0+ 1dt − 1dt
1
0 2
= 1 − 1 = 0.
Solution of Odd Number Exercises 797
R 1 R1
= 2
0
1dt + 1 (−1)(−1)dt
2
= 1.
10.5 ϕ(x − k) is an orthonormal system, that is
||ϕ(x − k)||2L2 = 2k .
Infact, 2j ϕ(2j t − k) is an orthonormal system. ϕ(x − j) and ϕ(x − k) have
disjoint support.
10.7 Let f (t) denote a signal over interval [a, b] then its energy is defined
Rb 1 PN 1
as Ef = ( a |f (t)|2 dt) 2 . If f (t) is discrete signal then Ef = ( n=1 |f (tn )|) 2 .
Energy is the characteristic
PN Pof a signal . If a function is represented by a wavelet
M 1
series then Ef = ( J=1 k=1 |dj,k |2 ) 2 where dj,k are wavelet coefficients of
f and
1
Ef = (||Wf ||2 ) 2
where Wf is the continuous wavelet transform of f . For more details see [1],
[14], [18], [22], [28], [34] and [33] through [40].
FIGURE E.5
FIGURE E.6
FIGURE E.7
FIGURE E.8
11.13 A graph that has neither self-loops nor parallel edges is called simple
graph.
11.15 Choosing F = X 2 in Itô Lemma we get
dF = 2XdX + dt
Z t Z 1
X2 = F (X) = F (0) + 2XdX + 1dt
0 0
Z t
= 2XdX + t
0
Rt
Therefore 0
X(Γ)dX(Γ) = 12 X 2 (t) − 12 t
Solution of Odd Number Exercises 799
801
Computer Programs Used
Chapter 1
Section 1.8 (MATLAB and MATHEMATICA)
Chapter 2
(i) Visualization of scalar and vector valued function
Let us discuss an important feature of MATLAB that is handling powerful
graphics. MATLAB has capability of presenting our output and helping us
in interpreting the data graphically with help of various types of curves and
plots.
Plotting scalar functions with MATLAB
2D dimensional Plots: To plot a function, we have to create two arrays (vec-
tors), one containing abscissa, and the other corresponding function values.
Let us plot f (x) = sin(x). Type following commands on command prompt.
>> x=-2*pi:pi/100:2*pi; % range of x from −2p to +2p in steps of p/100.
>> fx=sin(x); % function sine() to compute sine of al x.
>> plot(x,fx) % plot function, which plot f x vs x.
>> grid % creates grid in plot.
After giving all these commands, and pressing Enter Key, we get following
curve window as shown in Figure E.9.
Let us plot now f (t) = et/10 . sin(t). Type the following commands on com-
mand prompt.
>> t=0:0.01:50;
>> ft=exp(-t/10).*sin(t);
>> plot(t,ft)
>> grid
In above two examples, we have used plot() function for plotting. Now, we
will see that how we can combine two or more plots in one window. Just for
simplicity, let us combine plot of function f1 (t) = et/10 . sin(t) with another
function f2 (t) = et/10 in same figure window.
803
804 Computer Programs Used
0.8
0.6
0.4
0.2
·0 2
-0.4
·0.6
' '
-o a -------- .'~ -------- ''~ ------- ---- ' '
--- ! -------- -------- ----- -- ! -------
' '
' ' ' '
' ' ' '
FIGURE E.9
0.8 ----:------:------
. ' -:------ '
-:-------:------
' . ~.----- - -:'------ -:-------
'
~ --- --
. . .
• I I I 0 0 I I
0 ' ' ' 0 0 ' '
0 I 0 I 0 0 I 0
.
I I I
. .
o o I I I
---- ------... --- --.,. ----- .,. ------ .y------ ,.. ---- -.,. --- ---.. ----- .,. -----
I I I 0 0 I I I
'o 'o 'o 'o '
.. ' .. ..
I o o •
' ' ' ' ' '
-0.6 ' ' ' ' '
' ' ' ' ' '
I
'
0
'
0
'
0 I I
'
0
'
0
'
0
FIGURE E.10
Computer Programs Used 805
___ -- .. - ---
---- _..' ----- -- ____,'_______ ..' ______ ..' ____________ ...' _______ ..' ______ -----
-0.2 ' ' ' ' ' '
' ' ' ' ' '
' ' ' ' ' '
' ' ' ' ' '
' ' ' ' ' '
-0.4 ---- -f -----1-------:-------:-------f------ -------:-------:------- -----
' ' ' ' ' ' '
' ' ' ' ' '
' ' ' ' ' '
' ' ' ' ' '
-0.6 ------,-------·-------.. ------ ------ -------.-------.. ------ -----
'
'
'
'
'
'
..'
'
'
'
'
'
.
'
' .'
' .'
'
'
.
' .
'
' .
'
'
-o.s ol__ _L__ o,----,Ls __2,.co_ _J25L - - ,3Lo_--,1
_,
, 35,---
4Lo __4.Ls- - -so
'
FIGURE E.11
Similarly, we can add several graphs in same window with the command
plot(x, f x, y, f y, z, f z, ..). Please see MATLAB help for more detail.
attributes such as colors, styles etc. here we are taking some more examples -
Function MATLAB Code
>> x = 0:0.1:10;
Stairs() >> y = exp(-x).*sin(x)
>> stairs(x,y)
>> x = 0:0.1:10;
Area() >> y = exp.*sin(x)
>> area(x,y)
>> x = -4*pi:pi/5:4*pi;
>> y =sin(x)./x;
Stem() >> y((length(y)-1)/2+1) =1;
>> stem(x,y)
>> x = -4*pi:pi/5:4*pi;
>> y = sin(x)./x;
Bar() >> y((length(y)-1)/2+1) =1;
>> bar(x,y)
>> x =0:0.1:10;
Semilogx() >> y =x.*exp(-x);
>> semilogx(x,y)
>> grid
>> x =0:0.1:10;
Semilogy >> y =x.*exp(-x);
>> semilogy(x,y)
>> grid
>> x =0:0.1:10;
Loglog() >> y =x.*exp(-x);
>> loglog(x,y)
>> grid
>> theta =0:pi/100:2*pi;
Polar() >> r = sqrt(abs(sin(4*theta)));
>> polar(theta, r)
Output graphs of these examples are shown in Figure E.12. There are several
other types of plots also, one can go through matlabs help.
Three dimensional (3D) plots: Let us have a look on 3D plots. Three
dimensional graphs, we can plot for functions of two variables such as z =
f (x, y). Here, we will plot x(t) = et/10 . sin(t) verses y(t) = et/10 . cos(t) along
with ‘t’ axis (see Figure E.13). Type the following commands
>> t=0:0.01:30;
>> x=exp(-0.1*t).*sin(t);
>> y=exp(-0.1*t).*cos(t);
>> plot3(x,y,t) % 3D plot command.
>> grid
We will get the following figure.
MATLAB has several specialized 3D plots. Just try this one-
Computer Programs Used 807
04 04
0.1
80 100
05
-10 ·5 0 5 10 15
S!em func!ion Barfunc!ion
170
Polar func!ion
10° "''''''"''"''"'''""'""''•'''""'""'"
:::::::!:: :: ::::::::: ::;:::: :::~: :: ::::; : ::::::
10"
1
,;:; ;:::;";;::!:;::;;;~;;;::;;~ :;:::;;::;;;;;;
:::::::i:::::::i:.·=:::x:::::::c:::::::t:::::::
·······l·······l····
....... ,.......,..... ···:········(·······f·······
.,. .......•.......,.......
1o' ,.,,..i,,.,.,:,.,,.,.,;,,,,.,.\,.,.,.,\,.,.,.,
::::: ::!::::::::::::::::: ::: · : ::~ ::: ::::!:: :::::
....... , ....... , ....... y···· ., ....... , ..... . .
.......,.......,....................... ,.......
................................................
.... .. ....................................... . . . 3
,~, ......;..,, ,,;'"'"'
~ ~
FIGURE E.12
808 Computer Programs Used
...
... .. , . ~.
'•I • ~•
0
30
'· •
' '•
I .. .,".,._J
20 ... "·
' '
... . ...
I
~"
•'•
..
I"" ...... .,:
-... :'
'•
,•"!
10
0
0
...
'• ..
·0.5
·1 ·1
FIGURE E.13
>> [x,y]=meshgrid(-8:0.5:8);
>> r=sqrt(x.^2+y.^2) + eps;
>> z=sin(r)./r;
>> mesh(x,y,z)
This will give, following result-
One can try following plot commands on same function code and see the re-
sults.
>> surf(x,y,z)
>> contour(z)
>> surfc(x,y,z)
>> surf1(x,y,z)
>> meshz(x,y,z)
>> waterfall(z)
Please see help on each command in MATLAB help
(ii) MATLAB and MATHEMATICA for Differential equation Section 2.3.
Chapter 3
Plots for vector valued functions in 2D and 3D
Computer Programs Used 809
0.5
-0.5
10
10
-10 ·10
FIGURE E.14
1.5
/
' ' \ I /
'
' ' I
I
/ -.....
----....-......"
I / .,-
0.5
~
' .....' \ /
/ /
' .....
....,.
---- ---/
..,.. ~__...__... I
'
- ,.
/ \
-0.5 ~
/ / I \ --...
/ ' \
'
' "
I
'
/
-1
''
~ /
-1.5
-2
·2 -1.5 -1 -0 5 0.5 1.5
Chapter 4
ansum=eval(suma);
plot(t, avg+bnsum+ansum) % plot of truncated harmonics function
hold on
% plotting actual function
t1=-pi:pi/1000:mp;
plot(t1,-1,‘r’)
t2= mp:pi/1000:pi;
plot(t2,1,‘r’)
grid on
% formatting plot
x label(‘Time’)
y label(’Amplitude’)
title(‘Fourier approximation plot for 15 harmonics for
square function’)
legend(‘Fourier Approximation’,‘Actual Function’)
The result is shown below
Here is another program to plot harmonics of the ramp function which is de-
fined as:
0 −1 ≤ x ≤ 0
f (x) =
x 0≤x≤1
One can see the required modifications in this program as compared with pre-
vious one.
% Fourier Analysis of ramp function for first 5 harmonics.
t0=-1; % initial time
t0\_ T=1; % final time
mp=0; % mid point
T=t0\_ T-t0; % time period
syms t; % sym variable declaration
ft=diff(diff(t)); % zero part of function
ftt=t; % t part of function
w0=2*pi/T; % frequency
n=1:5; % number of Harmonics
% computation of Trigonometric Fourier Series Coefficients
a0=1/T*(int(ft,-1,0)+int(ftt,0,1));
an=2/T*(int(ft*cos(n*w0*t),-1,0)+int(ftt*cos(n*w0*t),0,1));
bn=2/T*(int(ft*sin(n*w0*t),-1,0)+int(ftt*sin(n*w0*t),0,1));
Computer Programs Used 813
0 ..... . I I I I
r········ r········ r········ --------' --------, ········'
I I
Time
FIGURE E.15: Fourier Approximation Plot for 15 Harmonics for Square Func-
tion
ann=an.*cos(n*w0*t);
bnn=bn.*sin(n*w0*t);
avg=double(a0); % converting sym variable to value
t=-1.5:0.010:1.5;
suma=0; sumb=0;
for j=1:5 % First 5 harmonics
sumb=sumb+bnn(j);
suma=suma+ann(j);
end
bnsum=eval(sumb);
ansum=eval(suma);
plot(t, avg+bnsum+ansum) % plot of truncated harmonics function
hold on
% plotting actual function
t1=-1:0.010:0;
plot(t1,0,‘r’)
814 Computer Programs Used
t2=0:0.010:1;
plot(t2,t2,‘r’)
grid on
% formatting plot
x label(‘Time’)
y label(‘Amplitude’)
title(‘Fourier approximation plot for 5 harmonics for ramp
function’)
legend(‘Fourier Approximation’,‘Actual Function’)
1.2,-------,-r=======:;---,-------,
- - Fourier Approximation
--Actual Function
-----------------------------------------------------------------
' ' '
' ' '
' ' '
'' '' ''
' ' '
0.8 ----- --- - ----------- ~ -----------; -----------; -----
Time
FIGURE E.16: Fourier Approximation Plot for 5 Harmonics for Ramp Func-
tion
Chapter 5
Simulation of Heat Equation, Wave Equation and Laplace Equation by MAT-
LAB 404-423, Section 5-6.
Chapter 6
Algorithms in Section 6.9.1, 6.9.2, 6.9.3 and 6.9.5.
Computer Programs Used 815
Chapter 9
For MATLAB programme of computation in this chapter we refer to [19].
disp(headertext(1 , B(1,:)));
fid = fopen(‘test.txt’,‘r’);
fseek(fid, pos, ‘bof’); g1 = fgetl(fid);
disp(sprintf(’%s \n ’,g1));
pos=ftell(fid); fclose(fid);
disp(‘If the Variables are correct, Enter 1, else Enter 2’)
tt= input(‘ENTER YOUR CHOICE: ’);
if (tt==1)
C(:,i)= ndata(:,B(1,i));
elseif (tt==2)
fid = fopen(‘test.txt’,’r’);
fseek(fid, pos, ‘bof’); fgetl(fid)
pos=ftell(fid); fclose(fid);
Disp (‘Please Choose the Correct Column Numbers’);
else
error(‘INPUT CAN BE 1 OR 2 ONLY’);
end
end
end
x1=[]; y1=[];
for k = 1:length(B)
x1=[x1 seasonal(a4, ndata(:,B(1,k)), k)];
end
x =[x
S(:,i)
S(:,i+1)];
if (i==15)
break
end
end
WA = []; WB = [];
for i = 1:1:length(lmd)/365
WA = [WA lmd(365*(i-1)+1:a4+365*(i-1))];
WB = [WB lmd(365*(i-1)+a4+215:365*i )];
end
y = [];
for i =1:2:length(lmd)/365
y =[y
WA(:,i)
WB(:,i)
WA(:,i+1)
WB(:,i+1)];
if (i==15)
break
end
end
b2=1:50:8000;
x=1:1:length(fx1);
x=x’;
for k = 1:1:length(a)
for i =1:1:length(b1)
c1=b1(i).*ones(size(x));
c2=b2(i).*ones(size(x));
y1=abs(x-c1);
y1=(y1.*y1)./(a(k).^2);
y2=abs(x-c2);
y2=(y2.*y2)./(a(k).^2);
z1=exp(-y1./2)/(a(k)*(2*pi)^0.5);
z1=(2-y1).*z1;
z2=exp(-y2./2)/(a(k)*(2*pi)^0.5);
z2=(2-y2).*z2;
phi1=conj(z1);
phi2=conj(z2);
w1(k,:)=fx1.*phi1;
w2(k,:)=fx2.*phi2;
m1(k,:)=abs(w1(k,:)).^2;
m2(k,:)=abs(w2(k,:)).^2;
end
n1(k)=sum(m1(k,:))/a(k);
n2(k)=sum(m2(k,:))/a(k);
a(k)
end
figure; plot(a,n1,‘k:’,‘LineWidth’,2);hold on;
plot(a,n2,‘r-’,’LineWidth’,2);
xlabel(‘Scale’); ylabel(‘Wavelet Spectrum’);
set(gca, ‘XTick’, 0:5000:20000);
set(gca, ‘XTickLabel’, 0:5000:20000);
legend(STN1,STN2);
legend(STN3);
SUBPLOT(3,1,3), plot(__rain_HDt2,‘-r’);
hold on; plot(__rain_HDt3,‘-b’);
legend (‘coef6’,‘coef24’);
Title(‘DWT-based pointwise Holder exponent’, ‘FontSize’,8);
SUBPLOT(3,1,1), plot(__speed,‘-b’);
Title(‘Daily Speed (1990-2005)’,‘FontSize’,8);
SUBPLOT(3,1,2), plot(__speed_HDt0,‘-r’);
hold on; plot(__speed_HDt1,‘-b’);
legend (‘db2’,‘db20’);
Title(‘DWT-based pointwise Holder exponent’,‘FontSize’,8);
SUBPLOT(3,1,3), plot(__speed_HDt2,‘-r’);
hold on; plot(__speed_HDt3,‘-b’);
legend (‘coef6’,‘coef24’);
Title(‘DWT-based pointwise Holder exponent’, ‘FontSize’,8);
SUBPLOT(3,1,1), plot(__temp,‘-b’);
Title(‘Daily Temperature (1990-2005)’,‘FontSize’,8);”
SUBPLOT(3,1,2), plot(__temp_HDt0,‘-r’);
hold on; plot(__temp_HDt1,‘-b’);
legend (‘db2’,‘db20’);
Title(‘DWT-based pointwise Holder exponent’,‘FontSize’,8);
SUBPLOT(3,1,3), plot(__temp_HDt2,‘-r’);
hold on; plot(__temp_HDt3,‘-b’);
legend (‘coef6’,‘coef24’);
Title(‘DWT-based pointwise Holder exponent’, ‘FontSize’,8);
Index
823
824 Index
vector, 2, 252
components, 265
coplanar, 9
cross product, 5, 259
geometric interpretation, 260
directional derivative, 274
dot product, 5
eigenvalue, 47
eigenvector, 47, 48
linearly dependent, 9
linearly independent, 9
magnitude, 4
orthogonal, 7, 257
scalar product, 7, 256
geometric interpretation, 256
scalar triple product, 262
standard unit, 254
steady state, 72
unit, 254
unit coordinate, 254
vector field, 265
differentiation, 269
flux, 317
velocity, 308